You are on page 1of 893

SLE Collection Bank 2011-2012 (from August 2011 to Augest2012)

-8-2012
(it contains all 1st 2nd and 3rd edition Questions in addation to:

490606)

B) New Collections (from March to July2012).(pages607-699) -700893)

Writers

Dr.Afnan Rajeh Dr.Hten Al-Majed Al-Hazzani Dr.Raneem Alshareef Dr.Manal M AL-anazi Dr.Hind sindi Dr-Bayan Ahmad Dr.Ahmed Alfaqwi Dr.Mahmoud Hijazi

organization Team:
1)Dr.Ibraheem Al-Awadh 2)Dr. Asma Al- Qusibri 3) Dr.Ghazi AlMutari
4) Dr.Sumayah Althunyan
-organization

Dr.Azzam Alkhalifah Dr.Yasser Alashhab Dr. HASSAN ABDELRAHMAN Dr.Hassan Mortaja Dr.Rusha Dr.wadah khairi Dr.Ahmed Omer Alhebshi Dr.Eyed Alhudaithi Dr.Samia Khan

and Supervision :

Dr. Abdullah Saleh AlHudaib, Head of MD27 Interns Physicians groups,


Qassim , Md27interns@gmail.com in cooperation with : Study SLE Together Group on facebook

SLE OROMETRIC 26 /9/2011

DR. AHMAD A ALSHOMAR,Qassim College of Medicine Please dont forget to prey to me

) the most common cause of nipple discharge in non lactating women is ; a-prolactenoma ( my answer ) b-hypothyroidism c- breast CA

d-fibrocystic disease with ductal ectesia . please check the correct answer because I m not sure 2-seniro with patient has fear , SOB , sweating when he is in automobile DX

b-panic disorder

c-generalize anxiety disorder

d- post traumatic stress disorder

heavy bleeding 3dysfunctional uterine bleeding How to manage ? a-combined oral contraceptive pill ( the correct answer b-hospitalize and give blood transfusion

c-hysterectomy d-do D& 4- polymyalgia Rhematica case with elevated ESR , other feature ; a-proximal muscle weakness b-proximal muscle tenderness 100% (correct) cd-

5- senior female patient with hiatal hernia ; which of the following correct

a-it become more severe in pregnancy ( my answer ) bc-

6- regarding barret easophgitis which correct ? a-risk of adenocarcenoma 100% true b-risk of squmou cell CA c-

7- man change his job , he must in new job to talk in front of 50 that instead of him, who can you help him ? 1. propranolol 2. Biofeedback ( my answer ) true

persons , he feels that he can not do this and he send his friend to do

8- pregnant LADY GIVING HISTORY OF INCREASED BODY WEIGHT ABOUT 3 KG FROM THE LAST VISIT AND LOWER LIMB EDEMA TO CONFERM THAT SHE HAD PREECLAMPSIA what u want to cheek : a-measure BP ( my answer ) b-protein urea

9- pregnant lady with hyperthyroidism what you will give her : propylthiuouracil (my answer ) methamazole B blocker

Radioactive iodine

10-which of the following correct about positive predictive value :

11- Which heart condition is tolerable during pregnancy:??? a. Eisenmenger syndrome c. b. Aortic stenosis ( my answer )

d. Dilated cardiomyopathy with EF 20% my answer is b but I think the correct is c please cheek it

tolerable in pregnancy but in case of sever regure I dont know )

Severe mitral regureg ( I am not sure bcoz the regure can be

e. Mitral stenosis and the mitral area is 1 cm (or mm).

12-- 35 year old smoker , on examination shown white patch on the leucoplakia tongue, management: a. Antibiotics b. No ttt c.

d- excision biopsy ( may answer )

Close observation

13-6 yrs old child came to you he only had his BCG vaccine, HbsAg +ve (mother also +ve) wt to give:

-DTP,OPV,HiB,HepB,MMR -..........

-DTP,OPV,HiB,MMR ( my answer ) 14- what is the causative organism of infectious mononucleosis ? a-EBV ( true) correct ? 15- male singer with colon cancer stage B2 ; which of the following

c-2 ===

d-lymph node metastasis + distant metastasis the correct answer is a

Stage IIB

T4a, N0, M0: The cancer has grown through the wall of the colon or rectum but has not grown into other nearby tissues or organs (T4a). It has not yet spread to the nearby lymph nodes or distant sites

16- child swallowing battery in the esophagus management : -bronchoscope( correct answer) - insert fly catheter - observation 12hrs ( my answer) -Remove by endoscope

17- wound at end inflammatory phase which of the following correct : Epithelial tissue formation

Wound clear( my answer)

Wound eschar formation-Correct answer-

18- Greatest reversible risk of stroke: DM Elevated blood pressure ( my answer ) Family history of stroke Hyperlipedemia Smoking

18- colon cancer with stage 3 give the chemotherapy: As soon as possible ( my answer ) After psychological prepare -After 1 week

19- TTT of hallucination and delusion ? a-antipsychotic ( true ) b-

19- An elderly lady presented with chronic knee pain bilaterally diagnosis is: a) Osteoarthritis ( true ) b) Rheumatoid arthritis c) Septic arthritis d) ..

that increases with activity & decreases with rest. The most likely

20-A pregnant lady, 8 weeks gestation, came with Hx of bleeding for the last 12 hours with lower abdominal pain & she passed tissue. O/E the internal os was 1cm dilated. The diagnosis is: a) Complete abortion c) Missed abortion e) Threatened abortion

b) Incomplete abortion ( true ) d) Molar pregnancy

21- A 34 year old lady presented with pelvic pain and

uterus was of normal size & retroverted. She had multiple small tender nodules palpable in the uterosacral ligament. The most likely diagnsosis is: a) Fibroid

b) Endometriosis ( true ) c) Adenomyosis d) PID

22- similar case about endometriosis ; the beast way to investigate ? a-US b-repeated BHCG d- hysteroscopy

c-laparoscopy (my answer )

23- typical senior of acute cholycyctitis ; the beast way to investigate ? a-US ( my answer) b-x-ry c-

24- Treatment of herpes zoster in ophthalmic division:or how to prevent post herpitic neuralgia ; a) Oral acyclovir alone c) Prednisolone

b) Acyclovir & Prednisolone true 100% d) IV Acyclovir

25- Lichen planus most common site ? Scalp Knee ( my answer) Buttocks Mouth ???

26- Relation of indirect hernia to spermatic cord Superior medial

Superior lateral (like anteriolateral) ( my answer ) Inferior medial

27- Old pt complaining of back pain on walking on examination there was stiffness of the muscle and there was some finding on the X-Ray best effective ttt Physiotherapy ( my answer ) NSAID Surgery

28- Pt came with deep injury on the wrist site, the nerve that has high risk to be injured will manifest as? Claw hand Can not oppose thumb to the other finger ( my answer ) Drop hand

29- Pt work most of the time on the computer came with wrist pain , be in

positive tinel sign you will do cast for the hand so the hand position should A-Dorsxiflexion ( I think its correct answer

C-Ulnar deviation A or D

D-Extension ( my answer )

30- Patient after accident, there was a part on his left chest moving inward during inspiration and outward during expiration Dx Pneumothorax Rib fx

Flail chest ( my answer ) Rib dislocation

31- old male bedridden with ulcer in his buttock 2 *3 cm ; involve muscle Which is stage : pressure ulcer a-1 c-3 b-2 d- 4 correct answer

stage I : non-blanchable redness that NOT subside after relive of the pressure) -stage II : damage to epidermis & dermis but NOT deeper ) -stageIII : subcutaneous tissue involvement) -stageIV : deeper than subcutaneous tissue as muscles & bones)

32- male

st

a-maintains airway ( may answer ) b-give O2 cd-

step in management ;

33- young male patient present to ER due to RTA with poly trauma ; the beast way to maintains airway in responsive poly trauma patient is ; A-orophargenial airway b-nasophargenial airway

d-endotracheacheal intubations ( my answer ) 34- long case patient with RTA with Blount trauma to abdomen . patient after 6 month with chronic diarrhea , SOB , sign of anemia , CBC show megaloblastic anemia What the cause of anemia : A-folic acid deficiency bc-alcohole d-

undergo remove of distal small intestine and proximal colon , patient come

35-male patient present with swollen erythem , tender of lt knee and rt wrist , patient give history of international travel before 2 month , ; aspiration of joint ravel , gram ve diplococcic , what is most likely organism a-nesseria gonorrhea (my answer) b-staph coccus c-strepto coccus

36)the initial non pharmacolgical measurment in osteoartharitis is : a-steringth of qaudrcepc muscle ( the correct answer ) bc-

37- the beast exersice that increase muscle density and strengths of bone mass is : ????

38-lang senior , patient with Grenache nasal discharge , sinus pressure last 4 month , He ttt with broad spectrum antibiotics with no response , management now ; a-antihistamine

chronic sinusitis not response to antibotuc , what is the

d-antibiotic

e- observation Young patient with congested nose, sinus pressure, tenderness and green nasal discharge, has been treated three times with broad spectrum antibiotics previously, what is your action? a) Give antibiotic b) Nasal corticosteroid c) Give anti histamine d) Decongestant The correct answer is b

39- child pt. came with scenario of chest infection , first day of

admission he treated with cefotaxime , next day , pt state became bad with decrease perfusion and x-ray show complete rt. Side opcifaction + hydrothorax , causative organism : Strepto. Pnem( my answer ) Staph. Aureus true if pnumothorax Hemophilus influenza type b

40- case infant has genital rash ( the rash spares genital fold ) not response to antibiotics , most likely Dx; A-candida albicans

b-napkin dermitis ( my answer ) c-contact dermatitis d- atobic dermatitis

e- sebborich dermatitis

41- 13 years old child with typical history of nephritic syndrome ( present with an urea , cola color urine , edema , HTN ) what is the next step to DX . a-renal function test ( my answer ) b-urine sediments microscope c-US d-renal biobsy

42-long senior patient came with chest pain , burning in character , clinical examination normal DX GERD

retrsternal , increase when lying down , increase after eating hot food ,

b-peptic ulcer d-

c-GERD ( correct answer )

43-lady with - Breast feeding 10 month child ,the mother has convulsion ,,,he take phenoparbital : Weaning withen 3 weeks - stop - do not stop - after 8 hours

phenoparbital

Lactating mother newly diagnosed with epilepsy , taking for it phenobarbital you advice is: a. Discontinue breastfeeding immediately b. Breas@eed baby aAer 8 hours of the medica_on c . Continoue breastfeeding as tolerated the correct answer is c

-very vague question , some books avoid Phenobarbital during breast

possible. And in American academy of pediatric classified Phenobarbital as adrug to that cause major adverse effect in some nursing infant, and should be given nursing women with cation . 44-lady with big abscess in left arm , how to manage ; a-antibiotic cb-antibiotics and incion & dringe ( my answer) d45- female about 30y c/o abdominal pain related to menses (scenario going with endometriosis)next step in dx: Laparoscopy my answer U/S CT

46- about head and neck injury : Hoarsness of voice and stridor can occure with mild facial injury ( my

Tracheostomies contraindicated

Facial injury may cause upper air way injures

47- about fetal alcohol synderome ??

48-female pregnant has HIV +ve , what is the most accurate information to tell her about risk of transmition to baby ; A-likely transmtion through placenta b-through blood cord d-by breast feeding c-hand contamion of mother

most likely correct answer is b

49- the beast way to ttt pinged induce nervosa ( bullima nervosa ) a-interpersonal psychotherapy c-pharmacotherapy db-cognitive behavior therapy ( true )

50- 35 year old smoker , on examination shown white patch on the tongue, management: a. Antibiotics b. No ttt c. Close observation d. ---------

-This is a case of leukoplakia and the management includes:ask the pt. to or cancer in the biopsy ; surgical excision should be done. 51- about vareciall vaccine in adult , which is true ; a-2 vacceine abart of 1 month

stop smoking, do a biopsy for the lesion; if there is pre-cancerous changes

d-3 vaccine abart of 6 month the correct answer is Two doses are always recommended. In the first case years. For people older than 13 the two doses are administered 4 to 8 weeks apart.[17] 52- there is outbreak of diphtheria and tetanus in community , regarding to pregnant woman: a. contraindication to give DT vaccine b. if exposed , terminate pregnancy immediately c. if exposed , terminate after 72 hour d. give DT vaccine anyway d. give DT vaccine anyway -The correct answer is d.

-6

30 year woman with dysmenorrhea, menorrhagea, infertility, and on examination found immobile mass on uterosacral ligaments : a. uterine fibroid b. endometriosis c. ---

53- Most common symptoms or sign of renal cell carcinoma in adult is a. Hematuria b. Abdominal mass c. Flank pain d. -The correct answer is a.

54- pt taking bupropion to quit smoking what is SE a. Arrythmia b. Seizure c. xerostomia d. Headache?? The correct answer is b

b+ d

55- your advice to prevent plaque disease is a-hand washing b-rodent eradication ( ???) c-spry insect side 56- pregnant with insulin dependant with good control, so to decrease risk of congenital disease a-good metabolic control before pregnancy b-"""""""""""""""""""""""1st trimester d-""""""""""""""""""""""""3rd """"""" c-""""""""""""""""""""""""2nd """""

the correct answer is a (I'm not sure ) a-lithium ( true 100%) b-Na volabrate cd-

57-the drug used in maintance phase of biopolar is :

58- 6 years child was born to HBS positive mother is HBS positive , he was only vaccinated by BCG after birth , what you will give him now :

b. HBV + oral polio + dt + MMR +hib c. oral polio + Dtp + MMR+ hib ( true ) 59- The useful excurcise for osteoarthritis in old age to maintain muscle and bone a- Low resistance and high repetion weight training: a. Conditioning and low repetion weight training b. Walking and weight exercise the correct answer is b

Exercise is one of the best treatments for osteoarthritis. Ask your doctor for an exercise prescription based upon your particular situations. The best exercises for osteoarthritis suffers depend on what joints are affected.

Swimming, walking, and cycling are often the best exercises for people with osteoarthritis. Try to get thirty minutes of exercise five times per week. The key is to start slowly.

60-old male with stroke , after 9 day he loss left eye vision , what are the affect structure ; a-frontal lobe b-partial

c-occipital ( my answer ) d-temporal

61- a-somatization c-depression

b-malingerinc ( true ) 62- The best way to reduce the weight in children is: a. stop fat intake b. Decrease calories intake c. Drink a lot of water d- decrease CHO

e- multifactorial intervention with family ( my answer)

63- Patient came to you and you suspect pre eclampisa, which of the following will make it most likely: a. Elevated blood pressure b. Decrease fetal movement c. ?? The correct answer is a 64- regarding group A streptococcus infection , how lead to rheumatic fever; a-blood dissemination c- joint invasion d-

b-by causing pharngitis /tonsillitis ( my answer )

65- the most common regimen in TTT of uncomplicated community acquired pneumonia ; b-flouroqunlone c-penicilline d-gentmycine a-azithromycine ( my answer)

a. Anterior Pad sign b. Posterior pad sign (my answer ) c. Anterior line of humerous intersecting the cubilium d. Radial line forming 90 degree with cubilium 68- Which of the following medication if taken need to take the patient immediately to the hospital: a. Penicillin b. diphenhydramine c. OCPs d. Quinine or Quinidine ( my answer ) bleeding , cervix 2 cm How to manage ; a-CS ( my answer ) b-spontius delvery c-forceps delivery d- do amniotomy

69- case 38 wks pregnant lady with placent brevia marginal with mild

70- Infant born with hemangioma on the right eyelid what is appropriate time to operate to prevent amylopia: a. 1 day b. 1 week ( 1000% correct answer true ) c. 3 months d. 9 months difficult Q.

71- Patient was presented by ear pain , red tympanic membrane , apparent vessels , with limited mobility of the tympanic membrane , what the most likely diagnosis : A. Acute otitis media . B. Tympanic cellulitis . C. Mastoditis . The correct answer is a 72- Benign tumors of stomach represent almost :

C. 50 % D. 90 %

73- Patient was presented by back pain relieved by ambulation , what is the best initial treatment : A. Steroid injection in the back . B. Back bracing . C. Physical therapy . The correct answer is c

73- Child came to ophthalmology clinic did cover test, during eye cover , his left eye move spontaneously to left, the most complication is: a) Strabismus b) Glaucoma c) Myobloma d) ?

74- The most common active form of thyroid hormone is: a) T4 b) T3 ( my answer) c) TSH d) TRH e-T2 the active form is T3 but the highest level is T4 73- y old patient , farmer , coming complaining of dry eye , he is smoker for 20 years and smokes 2 packs/ day , your recommending : a. advise him to exercise b. stop smoking c. wear sunscreen The correct answer is b , smoking increase risk of dry eye (international study

74- most important point to predict a prognosis of SLE patient : ?? a. degree of renal involvement

c. leucocyte count the correct answer is a 75- what is the prophlaxis of mengiococcus meningitis a- rifimbcine\

76-how to prevent malaria 77- male old patient has S&S of facial palsy ( LMNL) ; which of the following correct about it ; A- almost most of the cases start to improve in 2ed weeks b- it need ttt by antibiotic and anti viral c- contraindicated to give corticosteroid d- usually about 25 % of the cases has permanent affection

correct answer is A

Please dont forget to prey to me

Q) in cachectic patient, the body utilize the proteins of the muscles >> to provide Amino acid and protein synthesis Q)-Thyroid cancer associated with: >> Euothyroid / hyper / hypo /graves gland it passes: >> Superficial to retromandibular vein and ext. carotid artery / deep to ex. Carotid / deep to R vein

Q) Facial nerve when it exits the tempromandibular joint and enter parotid

Q) patient with recurrent pneumonia and productive cough , foul smelling sputum increase with lying down + clubbing >>> bronchectasis / BA / pneumonia

Q) long constibation + painful defecation persist for 30 min + bleeding >>> anal fissure Q )pt has HTN come with pulstile abdomen swelling >> aortic aneurysm / renal cause / etc.. Q) pt with vomiting , constipation ,pain and distension past hx 7 month appendectomy dx ; Mechanical IO / ileus / ets

Q) flu like sx since to days and now has red eye ( pic ) Dx:

Viral conjunctivitis / bacterial conjunctivitis / uvitis / glaucoma

Q) according to hemorrhoid >> can be due to portal HTN & pregnancy

Q) young pt came to ER with dyspnia and productive tinged blood frothy sputum , he is known case of rheumatic heart dz , AF and his cheeks has dusky rash dx : Mitral stenosis / CHF / endocarditis

Q) Malaria : the most common cases is caused by Plasmodium falciparum. Q) 5 y.o child with h.o fever and swelling of the face ant to the both ears (parotid gland enlargement) what is the most common complication >>meningitis / labrynthitis / orchitis

Q) girl with band like headache increase with stress and periorbital , twice / week >> tension headache / migrin / cluster Q) self breast examination >> monthly for maintenance >> 1600 ml .

Q) You r supposed to keep a child NPO he's 25 kgs, how much you will give First 10 kg X 100ml >> 1000 ml

Total = 1600 ml

Q) old pt take hypercalcemic drugs and developed gout what is responsible drugs >> frosamide / thiazide Q) lady drive a car and can't see the traffic light ( which one test the distance ) >> snelln chart / tonometer Q)lactating women with mastitis >> continuo breastfeeding decongestion /Local steroid / Systemic antibiotic .

Q) 56 y old present with vasomotor rhinitis >> Local anti histamine /Local Q) status epileptics >> Continuous sizure activity more than 30 min without regaining consciousness ??? the choices wsa confusing BUT the new definition ;

Tonic-clonic seizure activity lasting > 5 to 10 min

Q) In pt with moderately sever acne valgarus best ttt >> Oral isotretinoin / topical Retinoids /Topical clindamycin / oral antibiotics

2 seizures between which patients do not fully regain consciousness

Q) which of the following TTT contraindication in asthmatic pt >> Nonselective B blocker Q) case with 60 years old male with RT upper quadrant pain after dinner , most likely DX gallstone ; What is most appropriate inx to DX gall stone ?

Q) another case , typical case acute cholcystits , What is most appropriate ivex to DX colycystits : Abdominal ultrasound / oral cholycystogram / isotope scan Q) long case , patient fall down from ladder , come to ER with labored breath , cynose , decrease breath sound on rt side + hyper resonse , management is ? O2 via mask / tube throctomy / endotrocheal tubation Q) long case , acute pancreatitis which is TRUE; ca , low sugar / Naso-jujenal tube infection ?

NB; there was NO needle decompression or chest tube in choices Total parental nutrition / Regular diet with low sugar /High protein ,high Q) human bite to hand , most common hand position that proposed to

Clenched hand / dependent / extended thump / extended fingers Q) TTT of opiod toxiacity >> Naloxin

Q) medication induce ovulation >> clomphine citrate

Doxcycline / Azithromycine / Metroniadizole ( also, doxcy used ) Cervix eroded + friable DX ;

Q) Cervictitis + strawberry cervix + mucopurelnt yellow disharge

Trachimonus vaginits / Chlamydia / Nesisseri gonnerhia Q) case infant , hepatospleanmgly , , jaundice , what is the dx ? Congenital CMV Q) most common cause of renal failure >>DM

Q) strongesrt factore for intracerebrah hemorrhage >> HTN antibiotic DX >>Candidiasis

Q)case cord like cheesy white adherent odour less vagina after use of

Q) case of nesseria gonnerha , the beast TTT ? Peniciline G / Ceftrixone (( make sure plz ))

Q) Relation of indirect hernia >> Antero lateral or supralateral Q) PTs complaint of loss of association and cirumstantionciality the defect in >>>> form NB;

CONTENT ; delusion , obsession and phobias . Kill the vectore and ..

Q)- malaria case , beside antiobtic how to prevent ?

Q) which of the following increase bone density and muscle strength >> Andurance and whigh exercise / high repetition . / low repetition . Q)Positive predicitive value : Definition ?

Answer was >> C " pt who has high Risk factor & +ev test " Q) Pt with HTN and multiple risk factors " obese +high sodium intake >> Obesity / High Na intake/ High K intake / alcohol And the most important action >> wt reduction

+alcohol intake + high potassium " which is most important RF for HTN ?

Q) Femal come to family physician ask about diet that decrease CVD , ( She has family hx) ? Increase fruit and vegetable / Decrease the intake of meat and dairy / Decrease the meat and bread .

Mass media / Group discussion / Internal talk (true) Q) Most difficult method to prevented in transmission: Person to person / Vector / Droplet /Air flow Q) Child with morbid obesity , what the best advice for him:

Decrease calories intake / Dec fat intake /Increase fiber / incr. water Q) The most active for of thyroid function test >> T3 Q) Case of hemangioma in the eye affecting vision , when you have to remove >> 1 week sure 100% inshallah

Q) Clear case of osteoarthritis ( bilateral knee pain incre with activity . Q) Case back pain on walking and stiffness on muscle radiology show spinal stenosis , best ttt ? Physiotherapy ( because its mild ) / NASID / surgery Q) In diabetic retinopathy , most related factors:

Q) Scoliosis, when to refer the patient to surgery >> 20 degree ( sure ) Q) Patient is known case of cervical spondylolysis , presented by

parasthesis of the little finger , with atrophy of the hypothenar muscles , action now?

EMG showed cubital tunnel compression of the ulnar nerve , what is your Ulnar nerve decompression / Steroid injection / CT scan of the spine Q) Newborn came with red-lump on left shoulder, it is >> Cavernous Hemangioma ( my answer) Q) 20 year old male had been stabbed on midtriceps , one week later show gram positive cocci in chain ? meningocemia valve ?

greenish discharge , On microscopic examination of this greenish fluid Streptococcal gangrene / Chlostrideal gangrene / Fourniers gangrene / Q) female young with dew tear vesicles on rose red base and painful on Syphilis / HSV /Chancroid

Q) 9 yrs pt come with ear pain , red tense tympanic membrane , -ve Rhine's test with + ve Weber test with lateralization ( conductive loss) for TOW

Otitis media / otosclerosis / cholestiatoma Q) The same case above BUT he said conductive hearing loss directly without those tests >> Otitis media ray, the management : Q) pt was PDD ve , know become + ve , there is no symptoms , normal x Reassure / Rifambicin and INH for 6 month / Streptomycine for 7 month / rifambicin for 6 months .

Q) pt with COPD, Which of one increase surveillance ? O2 home therapy /Steroid / ibratropium synchronized CD / Digoxin Q) pt come to ER with AF, BP 80/60 what it the management:

Q) old pt, bedridden , with bactermia , organism is enterococcus fecalis , what the source of infection: UTI / GIT

Q) 4y girl, decrse head growth, decrse social intraction, decrase in language etc:

Q) youg female become flushing face and tremors when she talk to any one what ttt: Beta blocker( there is no ssri in choices)

Q) pt a afraid to go outside >> agoraphobia Q) case of Raynaud's phenomenon it was direct >> pallor then cyanotic then Q) read about rebound hyperglycemia in DM ?? somogi and down phenomenon .

red finger without other clinical features .

My exam done in 18 October 2011 Hope this collection help you in your exam Very spical thanks for dr. Ahmad Alshomar and dr.hussam alsulmi because the help me in remembering the Qs and in writing Dr. Yasir Abdulmohsen Alrusayni,Qassim Co0llege of Medicine Wishing for you all the best

SLE-15/10/2012

1. PTS come with history infertility complaining of decrease period , 2. PTS with history of infertility the first line of investigation for this 3. PTS come with history of infertility of 6 months unprotective intercourse couple is >>> semen analysis acne,hirstusim diagnosis is >>>>>> PCOD

intercourse >>>you must complete 12 months of unprotective

4. Old PTS with depression and you prescribed SSRIs for him < counsel 5. PTS with depression manifestations , what is the mechanism of the 6. Why the SSRIs is the first line of treatment of depression >>> effective and tolerable for the PTS is >>>>> take 3-4 wks to produce action

drug that you will prescribe >>>> increase availability of serotonin

7. PTs inside his home catch and cover the TV , and when ask him why to do this , he said the government follow him by watching and >>>>>Schizophrenia in >>>> form listening to his actions, he said the God told him about this diagnosis 8. PTs complaint of loss of association and cirumstantionality the defect 9. loss of libido , loss

of concentration , wt gain since hot flush , affect marital state >>>>> estrogen???? / progesterone / fluxatine ?????? infarction adult PTS with history of anemia sickle cell , he at risk of >>>

10. 11.

Child with history of SCA and recently treated from acute crisis

12.

on examination absence of pedal puls , cold RT leg and normal LT leg diagnosis is >>>acute Arterial embolism . Old PTS with history of recent MI complaining of sever

Old PTS with history of recent MI complain of pain of RT leg ,

13.

abdominal pain , distention , bloody diarrhea, slightly raised serum amylase diagnosis is >>>> Ischemic colitis ??? Old pts with history of bilateral pain and crepitation of both

14.

knee for years now come with acute RT knee swelling , on

examination you find that there is edema over dorsum and tibia of RT leg ,what is the best investigation for this condition >>>>Rt limb 15. venogram

that increases with activity & decreases with rest. The most likely diagnosis is<<< osteoarthritis

An elderly lady presented with chronic knee pain bilaterally

16. 17. 18. 19. 20. 21.

registration of disease through a period of time ?????

What is the definition of epidemical curve >>>graphic Standard deviations >> measurement of central tendency.

for >>>>> make your skin sensitive for sun light ????? inhlation

PTS 18 yrs , you prescribe for him retinoid gel will counsel him In a flame burn what is the cause of acute death >>> Gas Sliver ??? ???? drug used in Burn , what is the side effect >>> Conscious poly trauma pts , what is the action >>> ABC

leuckopenia????

22.

120 , RR 40 , chest x-ray show, white lung field in the LT hemithorax , what is your action >>>> thoracoectomy. primary and sleep apnea.??????? sigmoid scopy? / colonscopy What is the name of questionnaire that differentiate b/w Old pts with positive occult blood in stool >>>> flexibale Young pts come with sever testicular pain , decrease in doplex

Pts hit on his chest , after 2 hours come with , BP 100 /70 , pulse

23. 24. 25.

supply to tests, what is your action >>>> refer to surgen / refer to urologist / more investigation >>>> chlymdia urethritis Female with dysuria on examination there is epithelial cell Post partum female with recurrent attack of hearing loss ,

26. 27.

which diagnosed as conductive hearing loss , on CT the is dehesion in the of semi circular canal diagnosis >>>> otosclerosis / miner's / Tuberus sclerosis . / 6 months ???

28. 29.

When the spinal length stop after menarche >>>>> 1 yr / 2 yrs Infant newly giving cow milk in 9 months old , closed posterior

fontanel, open anterior fontanel with recurrent wheezing and cough , what is your action >>>> diet free milk / corticosteroid / antibiotics

sputum examination reveal hemoptesis , x-ray show lung infiltration , ??????

30.

vomiting , constipation , abdominal pain , diagnosis by >>>>> rectal

One months Infant brings by his mother complain of bilious

31. 32. 33. 34.

deficiency >>> vit D deficiency. give >>>>OPV, DTP,MMR,Hib.

Child with posing head , bowing tibia ,,,, rickets ,,, what is the 6 yrs +ve hepatitis , no vaccination , only BCG >>> what you will Which of the following is contraindicated to breast feeding 80 year old man complain of sever itching mainly in the wrist

>>>> asymptomatic HIV / Hepatitis c.

and b/w fingers , with excoriation mark linear and superimposed by course of Antibiotics >>>>>> Monilia ???? eczema ??? icythiosis ???? change health behavior of PPls lupus lipura !!!! melasma ????? Strongest method to prevent the disease >>> immunization / Female take OCPs come with skin changes on the face >>>>

35. 36. 37.

dicloration, painless over the face , thers is history for exposure unproductive to sun rays >>>>>Sqamous cell carcinoma /decrease exceasise .

Live guard come to annual examination , no compliant , macular

38. 39.

Strongest risk factor to ostoprocesis is >>>> smoking / age Community problem of multiple chlymedia infection in the eye

, best prevention method is >>>> good water and good sanitation supply .

40. 41.

systemic AB/local AB/steroid

Purulent discharge from ear middle ear how to treat him>>>> Child with URTI then complained from ear pain on examination

benefit what is the best TTT>>>ugmentine/azythromycin / 42. ciprofloxacin/steroid Female pregnant 34w gestation complain from bleeding heaver

than normal period O/E US show per placental lucency ,placenta minute,CX 3cm, fetal HR170 what is your action >>>>CS deceleration

implant normally post. In the fundus , uterine contraction every 4 43. 44. 45. Which IS considered abnormal & indicate fetal distress<<<late What is the vector for leshmania disease <<<< sand fly

rhinitis , persistent cough +ve aglutenation test & the doctor treat him by ribavirin DX>>>pertusus/RSV <<<<tetracycline/penicillin/?????.. Aluminum salt & salt will decrease absorption of 40 yrs old male com with HX of smoking & alcohol intake for

Infant in respiratory distress ,hypercapnia , acidosis & have

46. 47.

long time complain of painless ulcer ,role out border on the lateral border of the tongue DX<<<<<SCC????/lukoplakia Genital herpes CCC by >>>>> painful ,vesicular ,ulcer

48. 49. 50. 51. 52.

clear , vginal secretion DX>>>>> foreign body

5yrs child have congested throat 2 day , complain of painless , Old PTNs with osteoporosis TTT for HTN with diuretic that

prevent Ca loss complain of severe pain in big toe DX>>>>>thizide initial therapy>>>>digoxin/frosamide/debutamine

Old male come with CHF & pulmonary edema what is the best

53. 54. 55. 56. 57. 58. 59. 60.

Most effective way to stop smoking is >>>> PTNs desire Painless penile ulcer what is next step>>>> dark filed

>>>>> family HX of early IHD/BMI>30/mother worry

microscopy

cause is>>>> chemical/ mechanical irritation of retena renal/essential

Patient with HX of URTI & flash of light when he sneeze the Adult with HTN what is the most common cause >>>>> Adult with unilateral headache pulsetile increase with activity What is the antiviral drug that cause fever ,chills &muscle pain All of the following exaggerate the gastric ulcer Pregnant in 5 month gestation &on iron supplementation since

& light >>> migraine <<< interfiron

except<<<<decrease gastric empty time

that time &now com with dyspenia ,weakness &easy fatigability lap DX<<<<<thalasemia B/IDA second dose only

61. 62. 63.

Adult only taken first dose of varcella vaccine >>>> give him Carpal tunnel syndrome<<<<< dorseflexion

parasthesis of the little finger , with atrophy of the hypothenar what is your action now :

Patient is known case of cervical spondylolysis , presented by

muscles , EMG showed Ulnar tunnel compression of the ulnar nerve ,

B. Steroid injection .

C. CT scan of the spine

64.

what is the best method to TTT HTN>>>>>>Decrease weight????/decrease win/decrease salte????

Adult with essential HTN BMI30-40,drink red win,high salt dite

65. 66.

Sever pain in anatomical snaph pox >>>>> scavoid fracture

relative &iron conc. In blood 700ml???? what is the best intervention>>>gastric lavage/charcoal oil/

Child come to ER after ingestion of multiple iron tablet of his iv defrroxamin

67.

on throat exam showed enlarged tonsils with petechi on palate and uvula , mild tenderness of spleen and liver :DX<<<<< infectious mononucleosis

Patient presented with sore throat, anorexia, loss of appetite ,

68. 69. 70. 71.

pnemothorax<<<-ve pressure ventilation/lung damage antidote<<<<atropine&pralodexam intake

Male patient have ARDS &on ventilation have manifestation of Patient with lacremation ,salivation, diarrhea, what is Child with recurrent UTI how to counsel him>>> increase fluid Female com with lump in breast which one of the following

make you leave him without appointment () Blood on aspiration Solid

Cystic lesion with seruse fluid that not refill again??

72.

prolactenoma breast CA

lactating women

What is the most common cause of nipple discharge in non

hypothyroidism fibrocystic disease with ductal ectesia intraductal papilloma 73. 74. 75. 76. 77. 78.

patient with DVT what is the cast effective TTT>>>>> LMWH

&control>>>case control study DM1<<<<<< HLA DR4

group of patient with lung cancer divided In tow group disease

exposure to sun light<<<< eyes drops 4 daily Gouty arthritis -ve pirfringes

Patient with contact lens 10 yrs complain of tearing on Alzheimer caused by <<<<< brain cell death?????? crystal

mechanism>>>>deposition of uric acid crystal in synovial fluid due to over saturation Wavy like abdominal contraction>>>> obstruction

what

is

the

79. 80. 81.

oppose the thump senior

Patient with injury to ant. Surface of wrist >>>>>unable to female patient with hiatal hernia ; which of the

following correct <<<<it become more severe in pregnancy

Old pt complaining of back pain on walking on examination there was stiffness of the muscle and there was some finding on the X-Ray best effective ttt>>>Physiotherapy 82. 83.

+ve puss cell but ve for N.M, chlymedia>>>>RA/riters disease /gonococcal Regarding group A strept pharyngitise what is true<<<early Pulled Elbow scenario the TTT>>>immediate reduction Most active form of thyroid hormone is <<<<< T3

Patient have of urethritise now com with lt knee urethral swap

relation of indirect hernia to spermatic cord>>>>superiolateral

84. 85. 86. 87. 88. 89. 90. 91. 92. 93.

TTT decrease incidence of post strept GN supination

vitamin D

One of the following have anti cancer activity>>>> fiber / Child with asthma take steroid inhalation SE>>>>glaucoma Child with mild viral gastritis & you decide to give ORS how Which of the following indicate benign thyroid

/growth retardation

lesion>>>>lymphadenitis

BMI 24.5 >>>>> normal weight

incomplete defecation>>>hemorrhoid

Patient with pain full defecation ,fresh blood at the end ,sense of The best way to eliminate brucellosis is>>> milk pasteurization

SLE OROMETRIC nov 2011

1- 35 yr old with painful eye movement and decrease visual acuity is having: a. Optic neuritis b. Retinintis pigmentosa c. Central retinal artery occulusion d. Central retinal vein occlusion 2- 80 yr old in his normal state of health presented with decrease visual acuity bilaterally without any defect in visual eld his VA Rt eye= 20/100 VA Lt eye=20/160 fundoscopic exam showed early signs of cataract and drusen with irregular pigmentations. No macular edema or neovasculirization. The appropriate action beside antioxidants and Zn is: a. Refer the pt for emergency laser therapy b. Refere the pt for cataract surgery c. See the patient next month d. No need to do anything 3This pt has this painful lesion. The Dx: a. Herpes zoster b. Folliculitis c. Cellulitis 4- Hypertensive patient on Thiazide presented at night with severe lt foot pain involving the first toe with redniss extending to the mid leg. The Dx: a. Cellulitis b. Septic arthritis c. Gouty arthritis 5-

c. Treatment of ilius

6- Pt with abdominal pain and distension with vomiting and constipation. He has mild symptoms of dehydration. There is evidence of air in the rectum. The Rx: a. Rectal decompression with IV antibiotics b. Nasogastric tube with IV isotonic fluid c. Systemic antibiotics 7- Patient with symptoms of blephritis and acne rosacea the best Rx is: a. Doxacyclin b. Erythromycin c. Cephtriaxone 8- Primigravida with whitish discharge the microscopic finding showed pseudohyphae the treatment is: a. Meconazole cream applied locally b. Tetracycline c. Metronidazole d. Cephtriaxone 9- Female with monilial vaginal discharge the treatment is: a. Meconazole cream for 7 days b. Fluconazole orally for one day c. Metronisazole orally for 7 days 10- A 56 yr old his CBC showed, Hb=11, MCV= 93 Ret= 0.25% the cause is: a. Chronic renal failure b. Liver disease c. Sickle cell anemia d. G6P dehydrogenase deciency 11enduration in the child this suggest: a. Inconclusive result b. Weak positive result c. Strong positive result 12- An outbreak of TB as a prophylaxis you should give : a. Give BCG vaccine b. Rifampicine c. Tetracycline d. H. influenza vaccine hr you nd a 10mm

b. c. Give antithyroid drug d. Follow up the patient 14- Pathological result of cervical lymph node showed well differentiated thyroid tissue without any masses in the thyroid gland the best management is: a. Total thyroidectomy with radical resection b. Total thyroidectomy with modified resection c. lobectomy with radical resection d. lobectomy + isthmectomy with resection of the enlarged LN

15- which is true about gastric lavage: a. It is safer than ipecac if the patient is semiconscious b. It is done to the pt in Rt Decubitus position c. 16a. Gastric lavage b. Activated charcoal c. Total bowel irrigation 17- The most dangerous condition in menopause is: a. Ovarian cancer b. Endometrial cancer c. Osteoporosis 18- Female pt with irregural menstrual cycle it comes every other month and lasts 7-8 days with a very heavy bleeding making her to put double pads yet these pads will be soaked completely. The best description is: a. Menorrhagia b. Polymenorrhea c. Metrorrhagia d. Metromenorrhagia 19- Null hypothesis : a. The effect is not attributed to chance b. There is significant difference between the tested populations c. There is no significant difference between the tested populations 20- The specificity is: a. When the person does have the disease with +ve test b. When the person does have the disease with -ve test c. When the person does not have the disease with +ve test d. When the person does not have the disease with -ve test 21- The important risk factor for Stroke is: a. DM b. HTN c. Dyslipidemia

b. Increase in peripheral vascular resistance c. Salt and water retention 23a. Respiratory alkalosis b. Hypoxia c. Hypothermia 24like to do: a. Renal function test b. Urine microscopic sedimentation c. Renal ultrasound 25- 65 yr old presented with acute hematuria with passage of clots and lt loin and scrotal pain . the Dx a. Prostitis b. Cystitis c. Testicular cancer d. Renal cancer 26- Which of the following radiosensitive: a. Choriocarcinoma b. Seminoma c. Yolk sac tumor d. Embryonic cell carcinoma 27female genital organs. The underlying process is: a. Prolactin over secretion b. Androgen over secretion c. 28- 3 yr old with symptoms of acute urinary tract infection which of the following you would like to do in this acute state: a. Renal U/S b. Folly catheter c. VSUG 29a. Drink a lot of fluid b. Do daily exercise c. 30- Pt with sudden cardiac arrest the ECG showed no electrical activities with oscillation of QRS with different shapes. The underlying process is: a. Atrial dysfunction b. Ventricular dysfunction c. Toxic ingestion d. Metabolic cause

b. Lidocaine c. Qindine d. Quanin e. Metoprolol 32- Pt with sudden Lt leg pain after attack of MI, the leg is cold and pale: a. Acute atherosclerosis of the leg b. Acute embolism of the leg c. Acute cellulitis 33- 44 yr old a mother of 3 presented with bouts of shortness of breath fatique dizziniss chest discomfort. She thinks about her jobe and children a lot . she is doing well at her job: a. Depression b. Panic attack c. Generalized anxiety disorder d. Social phobia 34- Which personality disorder is associated with inflexibility and perfectionism: a. Narcissistic personality disorder b. Borderline personality disorder c. Obsessive compulsive personality disorder d. Histrionic personality disorder 35- 5 yr old adopted child their recently parents brought him to you with white nasal discharge. He is known case of SCA. What you will do to him: a. Give prophylactic penicillin b. c. 36- Which of the following antipsychotic associated with weight gain: a. Respiridone b. Quitapine c. Olanzipine d. Ziprasidone 37- Which of the follwing antidepressant is not given in erectile dysfunction: a. Sertraline b. Amytriptaline c. Butriptyline d. 38- Pt presented with nausea and vomiting and nystagmus with tinnitus and inability to walk unless he concentrate well on a target object. His Cerebellar function is intact: a. Benign positional vertigo b. meniere's disease c. vestibular neuritis 39week ago but he didnt took the medication. Now he has this pain with some inflammation inside the ear with loss of light reflex: a. Lt Otitis media

40need to take the antibiotic prophylaxis for how long: a. 5 months b. 6 years c. 15 years 41- Pt with difficulty getting air. Nasal exam showed unilateral swelling inside the nose. What is the initial treatment for this pt: a. Decongestant b. Sympathomimitics c. Corticosteroid d. 42- Nasal decongestant can cause: a. Rhinitis sicca b. Rebound phenomena c. Nasal septal perforation 43- The antibiotic prophylaxis for endocarditis is: a. 2 g amoxicillin before procedure 1 h b. c. 2 g clindamycine before procedure 1 h d. 44- Child with fever and runny nose, conjunctivitis and cough then he developed Maculopapular rash started in his face and descend to involve the rest of the body: a. EBV b. Cocxaci virus c. Rubella virus d. Vaccini virus 45- 19 yr old girl with URTI and splenomegaly. The cause: a. Infectious mononucleosis b. Streptococcus pharyngitis c. Malaria 46- Child with leukemia he has septicemia from the venous line the organism is: a. E coli b. GBS c. Pseudomonas 47showed Gram +ve cocci the organism is: a. Clostridium b. Streptococcus c. Pseudomonas

48- Pt after surgery the wound opened and secrete fluid this complication is: a. Colistridium infection

49stool leukocyte. The organism: a. C. perfernges b. C. difficele c. Salmonella 50-

acute watarry diarrhea with

previous history, the stool sowed high fat he is a known somker: a. Acute pancreatitis b. Chronic pancreatitis c. Pancreatic carcinoma 51- When showed you refer a pt with scoliosis: a. 10 degree b. 15 degree c. 20 degree 52- What is the best way of health education: a. Mass media b. Internal talk c. Individual approach 53- The best way to prevent house mite: a. Cover the pillows with impermeable cover b. Wash the clothes in hot water c. Remove the old carpets 54- Pt with bilateral knee pain without signs of inflammation: a. RA b. OA c. Septic arthritis 55b. Exercise alone is not enough c. 56- Pt with DM-II has conservative managenet still complaing of wight gain and polyuria a. Insulin short acting b. Metformin c. Long acting insulin -139/80-85: a. Wight reduction and physical activity

57- A father of twin want to ask you about the puberty of his children: a. Girls enter puberty 6-12 months before boys b. Girls enter puberty 2-3 years before boys c. Girls enter puberty 1d. Girls enter puberty as the same age of boys

c. cloxacillin with continue breast feeding 59should take: a. Td, Hib,MMR,OPV b. DTB,Hib,MMR,HBV,OPV c. DTB,Hib,MMR, OPV d. Td, Hib,MMR,OPV,HBV 60- Pt with good health presented with sudden shortness of breath with rt side hyperresonant the Dx: a. Spontaneous PTX b. Plural effusion c. Severe asthma d. Pulmonary embolism 61- 43 yr old sustained trauma to the chest present with severe short of breath with cyanosis, his rt lung is silent with hyperresonance. The FIRST step to treat this pt: a. O2 mask b. Tube thoracostomy c. CXR 62a. Needle decompression in the 2nd intercostal space midaxillary line b. Needle decompression in the 2nd intercostal space anterior-axillary line c. Needle decompression in the 5th intercostal space midaxillary line d. Needle decompression in the 5th intercostal space anterior-axillary line 63- Pt sustained a major trauma presented to ER the first thing to do: a. Open the air way give 2 breath b. Open the airway remove foreign bodies c. Give 2 breath followed by chest compression d. Chest compression after feeling the pulse 64- Pt has pelvic inflammatory disease received ceftraixone but no improvement the propable organism is: a. Syphilis b. HSV c. Chlamydia 65any attack since. She want to stop taking the drug due to facial hair growth: a. It is reasonable to stop it now b. Stop it c. d. Dont stop it 66a. Cluster headache b. Migraine c. Tension headache

b. c. It is unilateral pounding headache 68- Pt want to get pregnant so you gave her iron. Then presented to you with the following labs: Hb=9 MCV=60, the Dx is: a. Thalassemia b. Iron deficiency anemia c. Sidroblastic anemia 69- Pt with elbow trauma presented to yoy wich of the follwing is abnormal sign: a. Humeral head in continuation with the capitillum b. Posterior fad pad c. Anterior fat pad 70- Pt with 3 weeks history of shortness of breath with hemoptysis the appropriate investigation is: a. CXR,AFB,ABG b. CXR,PPD,AFB. c. CT,AFB,ABG 71- Pt with celiac sprue he should take: a. Carbohydrate free diet b. Protein free diet c. Gluten free diet 72- Pseudogout is Ca: a. Pyrophosphate b. Sulfate c. Uriate 73- First sign of MgSO4 overdose: a. Loss of deep tendon refelex b. Flaccid paralysis c. Respiratory failure

74- A 43 yr old with bloody nipple discharge: a. Duct ectasia b. Duct papilloma c. Fibrocystic disease 75- Pregnant lady underwent U/S which showed anteriolateral placenta. Vaginal exam the examiners finger cant reach the placenta: a. Low lying placenta b. Placenta previa totalis c. Placenta previa marginalis d. Placenta previa partialis e. Normal placenta 76- Athlete who jogs on daily basis presented with groin rash with erythema, the Rx: a. Topical antibiotic

77- Premature ventricular contraction is due to: a. Decrease O2 requirement by the heart b. Decrease blood supply to the heart c. Decrease O2 delivery to the heart 78- A mother brought her 9 days neonate with jaundice he is doing well with Icteric discoloration the cause is: a. Glucurenyl dehydrogenase deficiency b. Breast milk jaundice c. Fetal maternal blood transfusion 79- A pregnant lady full term presented with agitation and change level of consciousness she is having difficulty breathing: a. Pulmonary embolism b. Pulmonary edema c. Amniotic fluid embolism 80- Which of the following is a treatment for giardiasis: a. Prazequantil b. Mebendazole c. Metronidazole d. Albendazole 81- Which of the following is appropriate method to prevent brucellosis: a. Killing the vectors b. Prophylactic antibiotics c. pasteurization of the milk

82- pt after trauma to the knee present with knee swelling of bloody content , the probable mechanism is: a. platelet deficiency b. clotting factor deficiency c. platelet dysfunction d. blood vessels dysfunction 83- computer programmer presented with wrist pain and +ve tinnel test. The splint should be applies in: a. dorsiflexion position b. palmarflexion position c. extension position 84- proven effective method of improving the patients with osteoarthritis is by: a. walking and weight exercise b. c. 85- Treatment of community acquired pneumonia:

d. Tetracycline 86- Infant born with hemangiomas obstructing his vision to prevent Amblyopia the child need to undergo the surgery within: a. b. c. 87- Epidemiology is: a. b. c. 88- Pt with Rt arm tenderness with red streak line the axillary lymph node is palpable: a. Cellulitis b. Carcinoma c. Lymphangitis 89- Which is true about DM in KSA: a. Mostly are IDDM b. Most NIDDM are obese c. 90appropriate management is: a. Give oral labetalol b. Give MgSO4 c. Admit her till she deliver her baby 91- The most common cause of non traumatic subarachnoid hemorrhage is: a. Middle meningeal artery hemorrhage b. Bridging vein hemorrhage c. Rupture of previously present aneurysm 92- Pt with DKA the pH=7.2 HCO3=5 K=3.4 the treatment: a. Insulin 10 U b. 2 L NS c. 2 L NS with insulin infusion 0.1 U/kg/hr 93- Treatment of severe depression whixh is resistant to treatment is by: a. TCA b. Electroencephalographic therapy c. Electroconvulsive therapy 94- A 45 yr old his lab showed Hb=11 MCV=93 a. Chronic renal failure b. Liver disease c. Thalassemia d. G6P dehydrogenase defeciency 959697-

: \ \ \ \ \ \

My SLE exam Nov. 2011 In the name of allah

1. Compelete
a) Frontal b) Parital c) Cortex

loss of vision Lt eye, in pt with recent infarction?

d) Occipital

2. Female with Hx of
discharge ??

recent IUD ,complaining of watery brownish

a) BID >>> mostly correct one b) Bacterial vaginosis

Plz remember there was no ectopic pregnancy

3. Most accurate test for CTS ? carpal tennel syndrome

c) Nerve tapping pumbness

There was no nerve conduction velocity.

4. Drug induced urticaria ?


a) Cemitidine b) Hydroxizia ?? ):

Drugs induce urticaria are : Penicillin , sulph , NSAID , Aspirin , antihistamine , codine , dextrose , rantidine , tetracycline , vaccine , etc

5. Hx of Child has itching in his RT hand


a) Scbias >> im not sure

described as linear fissures at their top there is blacknish ? b)

which increase in the night

6. Old male pt with diarrhea about 1 liter , how much he will loose from
his weight? a) ,5 Kg b) 1 Kg > mostly

7. Which of the following method is rapid and best for complete gastric
evacuation ? ...!!!! a) G lavage c) Syrupe b) Manual induce V d) Active charcoal

8. Polygonal skin lesion DX ??

a) linchea planus

remember it is pathognomonic .

9. old male pt with hx of IHD, DM , HTN , dyslipidemia ,


family hx of heart disease ,,,, lab showed :- LDL : 199 ,,,, HDL : 37 so , in this pt what is most dangerous Risk factor ? b) Decreased HDL

a) Increased LDL >> correct one inshallah

10.

abd pain , increase in frequency , voiding small amount of utine , HTN , ttted with multi ABx but no improvement . Lab : CBC , metabolic tests all are normal , wht is DX ??

Verrry loooong scenario , in short : Female has dysuria, lower

a) Candidia albican c) DM

b) Intrstital cystitis ????

... (: ...

11.

of memory ,Living without care , urinate on him self , what is next step to do for him ? a) Give antipsychotic

Old man psych pt , has halosination , aggressive bebaviour ,loss

b) Admit him at care center for elderly . mostly

12.

a) Iv theiophilen ??? my answer I am not sure

Pt with sever asthma , silent chest what is next step ?

b) Neb salbtamol or sulmatrol >> is it work in silent chest

13.

a) Picin cell ???

Most commonest type of parotid gland tumor ?

b) Sorry I forgot

Plzz red about it

14.

shows soft prostate with multiple nodularity & no hard masses , the a) Sit with the pt to discuss the cons & rods in PSA test ????

60 y/o male known to have ( BPH) digital rectal examination

pt request for ( PSA) for screening for prostatic ca what will you do ? b) Do trans-rectal US because it is better than PSA in detection Plzzzzzzz c) Do multiple biopsies for different sites to detect prostatic ca

15.

withdrawal appear after a) 1-2 days ????? b) 2-4 days c) 5-7 days d) 8- 10 days

pt want to quit smoking you till him that symptoms of nicotine

16.

prolapse clinically by cardiologist, he had never done echo before what is appropriate action a. Do echo ????

pt want to do dental procedure , he was dx to have mitral valve

b. No need for prophelaxis c. give ampicillin d. Give amoxicillin calvulinic

17.

afraid from vertebral stress fracture, best way to prevent: a) VD supplementation b) Diet

Female postmenaupusal came with sign of osteoporosis,she

c) excersise.

18.

make you leave him without appointment ?

Female com with lump in breast, which one of the following

Cystic lesion with seruse fluid that not refill again?? Blood on aspiration Solid

Fibrocystic change on histological examination

19.

fontanel, open anterior fontanel with recurrent wheezing and cough , what is your action ? a) diet free milk

Infant newly giving cow milk in 9 months old , closed posterior

sputum examination reveal hemoptesis , x-ray show lung infiltration , ):

c) antibiotics ??????

20.

In battered women which is true: a) mostly they come from poor socioeconomic area b) usually they marry a second violent man c) mostly they come to the E/R c/o..

d) mostly they think that the husband respond like this because they still have strong feeling for them.

21.

you do:

by investigation :microcytic hypochromic anemia .

Child has pallor , eats little meat ,

what will

a) Trial of iron therary b) Multivitamin with iron daily

.. malnutrition !!!!!

22.

is called ??

If we take Number 10 from all families to do a study ..this type

b) Stratified Rand. Study

23.

Child with hemagioma in the back ?? a) Leave without TTT c) Evacuation

b) Intar-lesional steroid >> but Im not suuure =(

24.

redness, tearing , photophopia , So, what is TTT ?

pt with recent Hx of URTI , develop sever conj. Injection with

a)Topical ABx b) d)

Topical acyclovire >> mostly

25.

c) Oral acyclovire

a) Vit C >> correct b) Ca

Which of the following increase absorption of iron ??

Topical steroid

(( )) .. .. . . . .. .

The End

.. .. . . ..

.. .. .

Bader S. AL-Beshri ,Qassim COllege of Medicine :

.. .

SLE Exam , Sat 29-10-2011 Done by : Sami Hillel AlHarbi ,Qassim college of Medicine

1-20yrs old age sexual active suffer from pain during intercourse and when do urine analysis was gram negative diplococci intracellular diagnosis is : -Genococcal sexual transmitted disease -measurement of variity 2-what is the defintion of standard deviation

4-Atrritable risk-: have the disease

-measurement of exposed and not have the disease mius those exposed and 5- 30 years female have abdominal pain non cyclic , no menorrhagia on the diagnosis? -

examination have abdominal tenderness and painful cervix motion what is pelvic inflammatory disease 6-what is the side effect of fast correction of hypernatremia -? brain edema 7-contraidicated to use Iprofen which of the following conditions is -? -peptic ulcer disease 8-schizophrenia best indicator for prognosis-? family history of schizophrenia 9-female come with manfestations of hypothyrodism ---

sleeping,myxedema, cold intelorance , now she suffer from diffuclty in ALP = normal Ca =decrease , phosphorus = normal what is your diagnosis? - secondary hypoparathyrodisim

he put a table show TSH=normal , T4 normal, and... breathing , wheezing

10-old female came with scales aroud the areola ,she took steroid but no -

A-Antibiotics

B-anti-fungal

C- Mammography

mammography ANTI-fungal 11- child come with hypetension , oligourea, cola urine .... diagnosis is nephritic syndrome what is the best next step : 12-best treatment of bulimia nervosa cognitive behavior therapy central venous line - urine analysisurine sedmentation microscopy

13-patient have ARDS now devolpe pneumothorax what is the cause 14-patient with Ischemic heart disease what it is best diet increase vegtable and fruit Vit K Injection 15-newborn given an injection to stop bleeding 16-best treatment of otits media senario Amoxcillin for sure Insha'Allah which ? 17-patient come with diarrhea , confusion , muscle weakness he suffer from A-hypokalemia hypercalcemia

B-hyperkalemia A

19-Patient come with jundice , three days after the color of jundice change to greenish what is the cause? oxidation of bilrubin

20-which of the following represent inferior MI I,II and AVF 21-Patient with Rhumatoid Arthritis he did an X-Ray for his fingers and underlying process?

show permanant lesion that may lead to premnant dysfunction , what is the substance the secreted by synovial 100% correct

-paget's best treatment for patient with HTN and DM is

ACEIs

23-best treatment for female with migraine and HTN propnalol mortality ACEIs 24-which of the following drugs used in ischemic heart disease to decrease

25-defintion of case control study

26-60 year old male come with depressed mood , loss of interest , sleep disturbance after dying of his son 3 months back after long period of suffering of disease >>>what is your diagnosis Breavment

boiling

28- prevention of lyme disease , what is best advice to parents insect invdelent 29-5year child obese best mangement family intervention

29- 12 year obese best way to reduce weight is decrease calorie intake

30-female suffer from post-partum physcosis best step after medication include family intervention on the mangement of disease best prevention is

31-52 old female have ostopenia and she fear from getting back fracture Vit D supplement daily

32-mid clavical fracture in neonate best descriptiom is usually heal without complications wrist drop 33-fracure in the humerus affecting radial nerve lead to 34-spondloysis and ulnar nerve comprssion mangement is ulnar nerve decompression 35-injury to the anterior of hand lead to can not opposition the thumb 36-best treatment of 4th degree hemorroids is

stop warfarin and re check next day

38-which of the following associated with high risk of colone cancer high alchol , smoking , granunder disease is 39-2 year old child come with brocholitis and cynosis best intial treatment A-O2 B-antibiotics C-corticosteroids

39-best way to secure airway in responsive multi-injured patient is 40 child with moderate asthma and he on b2agnosit what you will add to decrease the recurence of asthma attacks corticosteroids inhaler diagnosis is endotracheal intubation

41-30-40 year male suffer from tetnus, vertigo , sensorconductal hear loss Miner's disease complexes and pluse is 300 bpm what is the true Amidarone should included in the mangement

41-patient 20 year old come with palptations ECG show narrow QRS

MY SLE, 29/10/2011 40 years female complaining of thinking a lot in his children future,

Best wishes for all

make sure that doors in her home are closed, in spite of doors already closed, provotional Dx: OCD, GAD?????, schizo

Long scenario of restless leg syndrome(he didnt mention Dx in

scenario), 85 old male many times awake from his sleep bcz leg pain, this pain relieved by just if he move his foot, but it recure, atsetra,best management:

Colazpin, haloperidol, lorazepam, one drug from dopamine agonist group forgot its name, its the right answer.

best drug for von willbrand disease is: vasopressin in choices).

fresh frozen plasma, cryoprecipitate????, steroids,(he didnt mention

Best fast management of acute hyperCalcemia is: Iv fluids, frusamide????, dialysis,

Scenario of digitalis toxcisity, old age male have CHF and he is on light.

digitalis for 3 years, sudden decrease of visual acuity, he see yellow flash

Bad prognostic Factor for schizo is : early onset???, family Hx, mood dis,

Null hypothesis definition>>> No ..No. Difenition of specificity:

Without disease and _ve test. Long scenario of a pt with melanoma in back, he is afraid of malignat change, which one of the following indicate malignant melanoma: >6mm, irregular and invade the skin,

Best inv to visualize the cystic breast masses is: MRI, CT, Mammogram, US

Regarding of DM in KSA:

Most of NIDDM are obese,

Female pt new diagnosed as HTN pt, BMI 28, U sld advise her by: and exercise may benefit him,

Body wight reduction ALONE doesnt benefit her HTN, wt reduction

Long scenario for pt smokes for 35 y with 2 packets daily, before 3 tinged sputum, X ray show opacification and filtration of rt hemithorax, DX: Bronchogenic CA, acute bronchitis???, lobar pneumonia

days develop cough with yellow sputum, since 3 hours became blood

Young male pt with dysuria fever and leukocutosis, PR indicate soft boggy tender prostate, Dx : Ac prostites, Ch. Prostites, prostatic CA

Pt with hypercholestrelemia, he sld avoid: Organ meat, Avocado, chicken, white egg

5 y child diagnosed as UTI, best inv to exclude UTI comp: Kidney US, CT, MCUG, IVU

6 y old boy, eat the paper and soil, best initial ttt is: Fluxitin, behavioral therapy,.

Baby said baba mama, pincer grasp, creeping well, sitting wth support, estimate age: 6m,7m,8m,10m

Overcrowded area, contaminated water, type of hepatitis will be epidemic: A,B,C,D

One of these not live vaccine: HBV,OPV,MMR

Most lethal infection for a pregnant woman: Toxoplasmosis, HIV???, rubella, measles

Drug of choice for gonorrhea: Ceftriaxone, ampicillin, cipro

Drug of choice for a schistosomaisis is:

Praziquanetil, oxaminiquine, artemether (sure)

Pt with acute Periumblical pain with +ve obturator sign, -ve psoas sign, Dx: Ac appendicitis, Ac cholycystites, Ac pancreatites Old male pt, smoker, alchoholic, fatigue, debilitated, back abd pain (scenario didnt mention to jaundice or lab findings) Dx: Ac pancreatitis, ch pancreatitis, pancreatic CArcinoma???, insillinoma ( Bcz he didnt mention to jaundice or decurvesier low, Im against CA ).

55 ys old male pt, presented with just mild hoarsness, on exam, there was a mid cervical mass, best inv is: Indirect laryngioscopy, CT brain, CT neck

Varecilla vaccine: 2 dose 6 weeks apart

Pt with +ve tinnle`s test, best position for a hand fixation: planterflexion, dorsiflexion, extension. ( suuuuuure )

45 years male, presented with back pain radiate to the lower, on exam just a paraspinal muscles were contracted and tender, on MRI,

is(didnt say the best) : physiotherapy, cervical laminotomy,. (suuuure).

Female postmenaupusal came with sign of osteoporosis,she afraid from vertebral stress fracture, best way to prevent: VD supplementation, diet, excersise.

( didnt mention to HRT, if mention; it will be the answer ). Verrryy long scenario of old age pt with DM, HTN, hx of multiple cardiac attack, CVA, came for routine check up in PHC, u found u will: bilateral opacification in both lenses, with decreasing of visual acuity, Refer to lazer therapist, refer to cataract surgeon, refer to ophthalmologist, follow up

Best preventive method for lyme disease: Insect repellent, wear fiber clothes, (suuuure)

45 male pt, will undergo dental procedure, he had a Hx of LBBB, but range, no hx of arrhythmia, the good next step: for prophylaxis???.

there is no structural abnormality, and ejection friction rate in normal Avoid hard dental maneuvers, give Abs before, give Ab after, no need

4 years old child, was diagnosed as SCD,so many times came to best strategy for prolonged therapy is: tertiary haem center.

hospitals with, dyspnia, dactylites , ( he put sign of acute crises ), the IV hydration fluids with analgesia???, follow in Out pt clinic, refer to ( bcz he said STRSTEGY, im going with last one). 80 ys old male pt, come with some behavioral abnormalities,

annoying,(he mention sm dysinhibitory effect symptoms) , most

postulated lobe to be involved: frontal, parietal, occipital, temporal. Long scenario for a pt came to ER after RTA, splenic rupture was clear, accurate sintences describe long term management: We give pneumococcal vaccine for high risky people just, we sld give ABs prophylaxis if there Hx of contact even with vaccination against pneumococcal, pneumoccal vaccinw sld not be given at same time with MMR

40 ys female, multigravida, no sexual intercourse for 1 year bcz her husband going abroad, C/O was intermenistrual bleeding with menorrhgia, provotional Dx:

Female pregnant, 32weeks of gestational age, diabetic, and she has a show no fetal distress,best management:

Hx of full term fetal demise, but her DM now well controlled, and BPP Wait for SVD, report a CS in 36 weeks?????, Teminate pregnancy

Couples with 6 month of unprotictive sex, no pregnancy: Wait , do semen analysis, US

Pt with a long Hx of conjuntavites, red eyes, corneal ulceration, most common cause: (suure) Ultraviolet rays, dust and pollens, viruses

Typical scenario of osteoarthritis. Triad of heart block, uveites and sacroileatis,Dx:

Ankylosing spondylites, lumbar stenosis, multiple myeloma

Dr. Mohammad Sa`d Alhomoud Ur praaaays

Best wishes for alllll <3

SLE- exam - Abdulaziz ALGharis,Qassim College of Medicine 22-11-2011 1. Patient with hypertension , DM, somking, which the following are most important to be deal with : a. obesity and HTN b. smoking and obesity c. smoking and HTN

2. Best treatment for depression>>> ssri 3. Mechanism of action of SSRI is ? i a. Increase availability b. Block receptor

c. Decrease availability 4. Contraindication to use in Migraine : Buprobion, lithium, valium ,

5. Patient with come decrease hearing, ear examination lost tympanic OM, acute OM

membrane normal looking , which one is most likely cause : serous

6. Patient has decrease visual acuity bilateral, but more in rt side , visual field is not affected , in funds there is irregular pigmentations and early cataract formation . what you will do a. Refer to ophthalmologist for laser therapy

b. Refer to ophthalmologist for cataract surgery ( true) 7. Female come to with 3 UTIs history in last 6 months,, what is your advise? a. wipe frome behinde to front after defecation b. take a bath insted of shaower. c. increse flude intake

8. Heavy smoker came to you asking about other cancer, not Lung cancer, that smoking increase its risk: a. Colon c. Liver b. Bladder

9. Carbl tunnel syndrome , in which postion would u fix patient's hand>>> dorsiflexion ( sure)

10.

11. 12. 13. 14.

a. Cone biopsy ( I think it's wrong ) / Direct biopsy / adenocarcinoma, barrot esophegous, SCC comp.

atypical squmous , what it is the next step:

Pt . 32y- have 2 children ,done a pap smear that showed

Pt. has columnar change in esophagus + finger projection >> Breast examination to young girl : Mammography and self Positive prognostic factor in schizophrenia >> prominent of Pain near eye precided by tingling and parathesia ocure mant

affect symptoms

times a week in the same time , also there is nasal congestion and eye lid edema dx? a. (Cluster headache)/migraine with aura/ tension headache / withdrawal headache SCA with hip joint pain 4 weeks >> AVN

15. 16. 17.

True about peritonitis>>> chemical erosion A case of how to mange the enuresis : ?? I don't know

18. surgery the discharge is greenish showed Gram +ve cocci the organism is: a. Clostridium

b. Streptococcus

19.

43 yr old sustained trauma to the chest present with severe

short of breath with cyanosis, his rt lung is silent with hyperresonance. The FIRST step to treat this pt: a. O2 mask c. CXR

b. Tube thoracostomy 20. computer programmer presented with wrist pain and +ve

tinnel test. The splint should be applies in: a. dorsiflexion position c. extension position

b. palmarflexion position 21. Infant born with hemangiomas obstructing his vision to

prevent Amblyopia the child need to undergo the surgery within: a. After 1 day

b. After 1 week 22.

c. After 1 month

a. A diastolic high pitch b. B - systolic low pitch c. C- diastolic low pitch

17 mitral stenosis :

23.

posetion?

human bite to the hand .. greatest risk of infection in which

a. dpendent

b. clenched

24.

infertile pt. with 3 previous d/c .. otherwise healthy .. Dx??.. a- PCOS b- Asherman syndrome

25.

mentiod ddx:

female infertile, hirsutism, male hair, plvic exam normal ,no US

a. hyperprolactin, PCOS,

26.

tricky quastion

-13healthy child with pRBC in urin 15 cells/hpf .. what to do ??..

a. repeat urine analysis for blood and proten. b. cytology ( I think wrong ) c. renal biopsy

d. serm parathyroid hormone 27. 28. SE of steroid : DVT , osteomalacia a. _ doxycyclin c. _ ceftriaxon Lactational mastitis..Rx:

b. _ ciprofloxacin d. _ gentamyecin e. _ cephalexin

29. 30. 31. 32. 33. 34.

Gout : sodium urate or Calcium pyrophosphate Snellen chart : 6 meter

What to adjust ? Insulin chart : long night, short day Athelis + groin rash >> topical antifungal, Abx ? Which drug contra indication in cluster headache ? buperbion, Pt daily asthma , nothing at night, using herbal for 2 months

lithum, valium,

with no improvement : inhalation salbutamol, high dose steroid inhaler, ipratrubium answer

35. 36. 37. 38. 39. 40. 41. 42. 43.

How to manage golf elbow : golf refine (x) , I don't remember Children has bitten patient in back, child immunization up to Stroke >>> HTN

date, Rx: suture, tetanus toxoid ?

Length of spinal cord post puberty ? 1,2,3 yr Burn @ face and neck ? admit hospital ?? L4/5 disc prolapse will cause : knee parasthesia, ankle jerk, Vitamine cause loss sensation and momvet disorder: Vit B12, MMR and lactation is :live attenuated virus

groin sensation B1,B2 , niacin

Endometritis, Polyp

Pt complain of Dysmenorrhea + Amenorrhea >> Endometriosis,

45.

early osterprosis , she is not on estrogen medication , what to give ? VIt D + Ca+Bisphonate , Floic acid and ., others stress, urge, flow ,

Post menopause Female with lower back pain , doctor said it's

46. 47. 48. 49. 50. 51. 52. 53. 54. 55. 56. 57. 58. 59. 60.

Old man with unrine incontunance, he feel distended bladder : Elbow truma >> posterior pad sign

topical steroid predicted !

Firgen body fully removed from eye, what to give ? Topical Abx, Risk assessment of ve true !!! ?? I choose not predict of Lichnus planus ( photo) flat top papule, purple.

sign>> Iron intoxication

Boy eaten family medication come with black vomit and other Case of spontnus pnumothorax

What is sign of high aldosterone? Hypo-K

DDx ? primary bilary cirosis

Pt has high cholesterol, Jaundince, xanthoma, xantholesema , 10 Kg boy, maintaince is 1000 mL

HBV, St.viredans

Young, drug abuser, asymptomatic . What to investigate ? HIV, Varicolla vaccine , 2 dose, 6 weeks apart

verbtebra and ostephyte : Ankylosisng spndiolytis Boy wet bed , how to treat ?

Pt complaining of pain at neck, Xray show narrow space of

Sexually active multigravida, joint pain, swollen and red >>

61. 62. 63.

biopsy, direct biobsy, .

Woman with tumor at pelvic examination, what next ? cone What vitamin given to neonate to prevent hemorrhage : Vit K Hypercholesteremia should avoid : Avocado

SLE- exam - Muzoen ALMohamid ,Qassim College of Medicine 26-11-2011

- c-20 b-15 a-10

-1typical presentation of diabetic nephropathy , when

2-: copd patient not responding to bronchodilators .. what u will add to him a- aminophylline I.V b- methylprednisone I.V ( ?? cord prolapse

3- : instrumental delivery , what u should take care about child with feve , anorexia develop papuls then became vesicles then Varicella -4pustules in hourse

a-screening for breast cancer decrease

-5: what is most true about eduacation

b- screening for colorectal cancer still low d-screening of lung cancer decrease

physician screening fo smooking is satisfactory

6-patien complain of painfull knee ,, (before 4 months the RT big toe was swollen and panifull )the aspiration from knee was negative for crystals a- ca pyrophsphtase crystals after acute episode b- uric acid after acute episode

c- uric acis after perciptation of supersaturation state d ca pyro. After perci. Superaturation st

7-female c/o colorless itching vagina , her partner c/p urethral disch. . : Cervical examination shows strawbery spots meconazole cream estrogen cream douch

progesteron cream

8-? ttt of tremor and sweatinf in front of audiance Propranolol ssri

9-four y old child c/o bleeding for 4 m, intermittent painless bowel UC

: movement , good appeite , after ex. The examiner found blood at his digits meckles diverticulus juvenile polype

10-? fever , rigor for 3 days .painfull hepatosplenomegally Malaria vesiral leishmania toxoplasmosis

11-: stable non bleending hemhrroid increase fibre intake

diabetic pstient with ulcer in foot , not healing , not infected , high blood ? glucose a- high blood glucose stimulate bacteria to grow b- decrease phagocytosis c- dec. Immune system

12-: appropriate way to prevent spread of diseases change human behavior screening tests

13-: to prevent trachoma !! water and eradication m boy with dental caries at icisors and central tooth .. what is the cause 18 14Milk caries water and sanitation

excess fluride

m12 m18 m36

SLE- EXAM FOR hussam alsulmi and abdulrahman almoklef,Qassim college of Medicine Written by : hussam

Atherosclerosis in the coronary arteries Atherosclerosis in the arteries

: What is the Pathology of coronary heart disease

Deposition of fat in the outer layer of aretery

Batterred ( same choices in qassim collection In battered women which is true: a. mostly they come from poor socioeconomic area b. usually they marry a second violent man c. mostly they come to the E/R c/o..

d. mostly they think that the husband respond like this because

Female patient presented with tender red swelling in the axilla with a. Immidate surgery b. Topical antibiotic c. Cold compressor d. Oral antibiotic

history of repeated black head and large pore skin in same area: ttt is

Which of the following oral contraceptive drugs cause hyperkalemia: "" Norethadion ..etc ..

Ehinyle seradiole Patient with hypertension , DM, somking, which the following are most important to be deal with : obesity and HTN smoking and obesity smoking and HTN Children while he was playing a football , the ball hit his hand

from lateral fingers, after a while the children complaing pain and swelling on those fingers and painful middle finger with

hyperextension of interphalengial joint, swelling was more in the DIP and IP Joints , also , there was pain on his palm, what is the

Rupture of superfacial ms Extra articular..

Rupture of profound ms in hand( 100% true)

Intra articular interphlengial joint fracture Patient was presented by bollus in his foot , biopsy showed sub dermal lysis , fluorescent stain showed IgG , what is the most likely diagnosis : A. Bolus epidermolysis . C. Herpetic multiform .

B. Pemphigoid vulgaris . D. Bullous pemphigoid . The correct answer is d Female patients come with Black maculae in forehead and cheeks. She is Not pregnant what is the most likely DX: ( I forgot if it was associated with sun exposure or not) Lentigo PeutzJeghers syndrome, Chloasma. Dermatitis herpitiforms

someone for 6 wk ( long scenario made me hesitated in pick up the ---bereavement , adjustment with depression . Depression correct answer)

Case of old man presented with Sing s of depression after died of

( a very short scenario)Female come to the clinic with her baby of 6 month , she had tremor and other sign I forgot it, which of the following is most likely dx

hypertyrodism hypothriodism

Postpartum thyroiditis

...Hashimoto

sub acute tyroditis Why do we take a prophylaxis in asthma, is IT to prevent:

To treat Allergic sensitivity To Decrease inflammation etc

Old pt with pneumonia , H. influenza .. Which of the following is the best in Ttt:

Cefurxim , cipro, Doxo

. Penicillin

ceftriaxone

VSD

Left ventricle hypo plastic , Gardener complaing of Watery discharge from his eyes , and he can not avoid his job , how would you solve this problem :

Cool compress ( sure) saline venger lenses.

Female had history of severe depression, many episodes, she got pregnant .. your advise

her remission for three months with Paroxitine ( SSRIs) .. now she is a. Stop SSRi's because it cause fetal malformation b. Stop SSRi's because it cause premature labor c. Continue and monitor her depression d. Stop SSRIs

The correct answer is c or a In general, SSRI have the least side effects then other classes of antidepressants. Some SE: Sleep disturbance, dizziness, sexual dysfunction, Nervousness, and diarrhea

a) Fluxetine (Prozac) ( very short scenario) Femal after delivery come with tearfull and Postpartume blues ,pp pychosis pp depression irritability , the mood is normal

Buprobion >> side effects ( same question in Q- collection) stenosis , what most likely will happen to the heart:

Patient with rheumatic hear disease and he developed mitral

RT -ventricular hypertrophic and dec pul pressure Lt atruim hyper and dilatation( ithink true) Rt ventricl hypertrophic with constrict champer Lt atrium hypertrophy with constrict champer Wich of the Following true about child spell

Picture of graphical presentation chart" simple columen" in which the X= Osteioporsis AND Y= Age,, chose the best statement that explain the chart"

Patient with Sever hypothyriodism and hyponatremia (108= Na ), high TSH and not respond to painfull stimuli, how would you treat him :

Oral intubation , Thyroid replacemet , Steroid and 3% Na Same above but Without steroid Thyroid and fluid and%3 Na Thyroid and fluid replacements only

Bcg ( Pneumocool 94.

In close contact with asymptomatic TB what you will give

make you leave him without appointment () Cystic lesion with seruse fluid that not refill again?? Blood on aspiration Solid

Female com with lump in breast which one of the following

Fibrocystic change on histological examination ???

95.

one is most likey cause :

Patient with come decrease hearing , which

serous om.( I think true ) Steriod ( I think true) anti hisatmin Decongestant 96.

Pat with perennial allergic rhinitis. Tttt

Patient has decrease visual acuity bilateral , but more in rt side , visual field is not affected , in fundus there is irregular a) Refer to ophthalmologist for laser therapy pigmentations and early cataract formation . what you will do b) Refer to ophthalmologist for cataract surgery ( true)

Patient is a known case of gout presented with frequent Stones .. would you decrease the frequency of attacks

Increased creat and urea .. The time btw attacks is decreased , how

Increse fluid

Allupurinol ( I think true) Propenside Old patient presented with Ear pain ,headache , hem paresis>> most likely cause:

intakeclaries

Subdural hematoma Which of the following is true regarding gastric lavage :

A. Patient should be in the right lateral position .

B. It is not effective after 8 hours of aspirin ingestion . In holding breath holding which of the following True: b. Increase Risk of epilepsy

a. Mostly occurs between age of 5 and 10 c.Aknownprecipitant cause of generalized convulsion

patient presented with excruciating headache and blurred vision: acute angle glaucoma

d. Diazepam may decrease the attack

Mother worry about radiation from microwave if exposed to her child. What you tell her: a. Not all radiation are dangerous and microwave one of them b. Microwave is dangerous on children c. Microwave is dangerous on adult d. ----

Woman with cyclic bilateral nodularity in her breast since 6 breast : what you will do next

month, on examination there is 3 cm tender mobile mass in her

f. Mammogram g. Biopsy h. Follow up for next cycle Male patient have ARDS &on ventilation have manifestation of line insertion i. Observation

pnemothorax<<<-ve pressure ventilation/lung damage / central

Heavy smoker came to you asking about other cancer, not Lung cancer, that smoking increase its risk: a. Colon c. Liver b. Bladder How to prevent lyme disease:

Insect repillant/ change the cloth to the natural clothes Wound at end inflammatory phase when: a. Epithelial tissue formation b. Angiogenisis c. when the wound clean d. Scar formation

8y with weight and height > 95%and BMI= 30 .. What u will do for him:

Observation for 12 month /refer for surgical intervention/ life What it is the most common congenital heart disease come with rheumatic heart disease: VSD/ASD/coartication of aorta lower sternum ..DX atresia style modification/

Child with failure to thrive , pink, O/E thereis pansystolic on the VSD/ TOF/TGA/ pulmonary stenosis with left ventric../ tricusp

Patient 57 y-o, somker for 28 y , presented with bleedind per COLON CA/ IDA rectum and positive guiaic test , also he has IDA:

Pt, presented with sever ingestion of eye and watery discharge ,, no history of itching , truma and pain ,, vision is normal , what most appropriate ttt:

Antihistamine( 100 % sure)

Pat presented with constricted pupile, ciliary flushing and cloudy anti. Chamber there is no abnormality in eye lid, vision and lacrimal duct :

Uvietis ( sure 100%)

Pt he did not see the traffic light clearly , what u will do for him >> Pat know case of asthma , and allergic rhinitis , he presented with with excuriations , dry skin and pruritis most likely DX >>> ATOPIC DERMATITI ( SURE) snelling chart test ( sure

PATIENT he can not opposite his thumb >>> median nerve ( sure) hand>>> dorsiflexion ( sure) Carbl tunnel syndrome , in which postion would u fix patient's

Pat presented with sharp sever chest pain increase with also there was a pic of CXR)>>> Pericardial effusion with pain)

movement and supine postion and decrase in leaning forward (

A clear case of primary dysmenorrheal ( 19 y-o female presented Patent with amenrrhea for 2 month , on exam there is tender invest .? >>>> US ??

pelvic, prolactine was normal ,, what it is the most appropriate

Pt . 32y- have 2 chidlren ,done a pap smear that showed atypical squmous , what it is the next step: Hysterectomy / repeat after 1y/ loop elec / coploscopy

Pt . complaine that he has a palpitation , sweating and panicky well in the job,, dx?

feeling when there are talking in there job ,,, but there are doing Agoraphobia/ GAD/performance anxity ( true)

Best ttt for depression>>> ssri

Indication of ECT >>> Major depression with psychomotr Positive prognostic factor in schizophrenia >> promenant of affect Loss of smell >>> temprol lesion spoon? Lesion Cerebellum ?? symptoms symptoms( sure)

Pt . had a closed head injury after that he can not eat by using

Chlesteatoma/ BPPV/Vestibular neuritis( true)

Patient had URTI then he developed vertigo what it dx?? Pat has snoring in sleeping and and on exam there is larg tonsile >> what u will do for him :

Weight reduction/ adenoidectomy

Pain near eye precided by tingling and parathesia ocure mant eye lid edema dx?

times a week in the same time , also there is nasal congestion and

Cluster headache/migraine with aura/ tension headache / withdrawal headache What is the most appropriate investigation to visualize the cystic mass :

US/ MRI/ mammogram

Pt with hx of SCA.. he admitted many times to hospital due to crisis attacks : and know he caome with abdomenial pain and neck , body and arm pain ,, what u will do for him:

Hospitalization and pain manegment and observation Give him narcoyics

Outpatient manegment hydration , pain mangment and observation True about peritonitis>>> chemical erosion common CML:

What is true about appendicitis in elderly patients>>> rupture is Which of the following is the most important prognostic factors in Stage/ age/ lymphocytic dubling time/ involvement of bone Pt has pharyngitis rthen he developed high grade fever then normal and no shift tp left: dx( organism) / clhlamydia scenario: marrow degree

cough then blateral pulmonary infiltration in CXR ,,, WBC was Satphylococcous aureous/ staphylococcous pneumonia/ legonilla

PATIENT has RTA and membranous uretheral disruption . Long

Suprpubic catrheter ( may be ) / pubic repaire/ trans uretheral Patient with testicular mass . non tender and growing on daily catheter/ abdominal repaire

Refer pt to do open biobsy or percutaneous biobsy / refer him to Newborne with white creamy leasion on the mouth after taking course of antibiotic ,, ttt: Oral nystatine ( t)/ steroid/ AB/ antiviral Imipramine and vasssopressine( I think true)/ clonodine and vassopressine/ clonodine and guanfacine population do US and surgical opening ( I think true)

A case of how to mange the enuresis :

PPV ; Proportion of people at high risk which have target To prevent infection in neonate >> wash hand before and in between patients examenations Patient is known case of cervical spondylolysis , presented by

parasthesis of the little finger , with atrophy of the hypothenar nerve , what is your action now :

muscles , EMG showed cubital tunnel compression of the ulnar C. Ulnar nerve decompression . (100 %sure) D. Steroid injection . E. CT scan of the spine .

Will you give him prophylaxis before procedure/ u will not give him 2g amoxicilline before( true) / 2g after

Patient with subaortic hypertrophy:

Patient with hx of endocarditis and he will do an oral surgery :

Early pregnant come to your clinic, which of the following is most CBC/ urine preg test/ US/MRI/blood groubing and Rh benefial to do :

A pregnant lady came to you to in second trimester asking to do screening to detect down syndrome, what is the best method: Triple screening( I think true)/ amniocentsis/

Ttt of trichomoniasis >>> metronidazole

Patient with gunshot and he developed dypnea , rasied JVP , Deviated trachea ,>>> ttt >>> needle decompression

and RR= 18 ,and there is part of omentum protruded,, what u will do : Exploration of the wound/ schedule for laparotomy( I thing is Picture of slightly red swelling just above the nail bed of finger , painfull, patient is what you wiil do: true) / DPL" diagnostic peritoneal lavage/ FAST

Incision and drainage with general anestheasia/ I and D with local Pt known case of hypothyrodisim , and you start levothyroxine THS INREASED ; anestheasia/ change AB/ Complete augmentine for 1 wk

but she come after 1 wk with cold intolerance, and bradycardia,

Continue and check after 1-2 month/ decrese the dose/ stope until tsh is become normal

Incrase dos and after 3 wk/ incresea and follow after 6 wk immunization

Most potentional prevention to decrase the disease>>> 70 y-o pt , come with investigations showed osteolytic lesion in skull, monoclonal spike, roleahex formation>>>> multipul myeloma

by SLE : 27/11/2011 Done by : thamir als3eed

1- normal puriparium () mother start breast feeding after 24 hours

lochia

2-indication of adenoidectomy sleep apnea abscess

3-

ulnar nerve injury ---> wrist drop radial nerve injury --> claw hand median nerve injury --> atrophy of interossus muscle tarsal n injury --> tarsal tunnel syndrome(correct) : delsion-4 positive menngiocoal tb -6 - girdasis-5

3-single dose IM ceftriaxone 7- about ocb :

a- decrease breast cancer

b-decrease ovarian cancer (my answer) c-increase ectopic pregnancy d-don't take by diabetic patient

e-dnt take by healthy women over 30

8- ulcerative colitis in compare to chron disease a-fistula b-risk of cancer

9- old age , smoker ( ) obese , intermittent diarrhea , bleeding per rectum , positive Stool guaiac test , IDA ( ) 10-about ecg colorectal ca ( )

,drolling , , , - 5 years , :, pharynx is congested and filled with mucus : croup acute epiglottis , compressible , no , 13- adult , , erythema, 1-

ventricular dysfunction

aspiration biopsy -

synovial analysis

http://en.wikipedia.org/wiki/Synovial_fluid 15- Proctalgia fugax ?which nerve affected facial nerve 17-

anterior 2/3 , , base skull fracture , 16-

, ct mri

, , no change in conscious ,

observation

refer to neurologist 18- 1-oral ab 2-local ab

3-oral steroid

4-local steroid

ab

watery) , ( discharge conjctivits , ( [: discomfort sensation , 1-bactrial conj 2-virl conj ? , visual acuity

3-chlymdia conj 4-glucoma kawsaky 20- strawberry tongue

21- inferior infarction :

changes in leads II, III, aVF tempro-frontal 22- frontal )androgenic alopecia (sure

23- scale !!

outbreak for tb , -

rifampicin

-bcg vaccine - - DKA

1-starvation cause increase of amino acids and fatty acids which utilize by the body Ketone body which excreted in urine

2-decrease in insulin lead to > fattyacid > ketone body 26- tight band like , stress dka

, Bursting throbbing

1-migrane

2-tension headache

-27 trauma in chest present sob with cyanosis, his rt lung is silent with hyperresonance. The first step to treat this pt:

b. Tube thoracostomy c. CXR d-needle decompression specfity : -

29- DM1: DR4

single palmar crease ) (trisomy 21

SLE- EXAM FOR Moyad ALKhalifa ,Qassim college of Medicine

# which of the following true about headache :" -increase ICP at last of day -normal CT may exclude subarachnoid hemorrhage . ???? -amnursus fugax never come with temporal arteritis . - neurological sign may exclude migran

#patient came with cervical carcinoma next investigation : -cone biopsy -pap smear - Direct biopsy

# patient with typical sign of infections mononucleosis come with

-abdominal CT and IV fluid ????

-antibiotic and IV fluid and and observation #patient with hypersensitivity skin at back take paracetamol and develop vesicle at back extend to abdomen Dx : Herpes zoster

#infant (28 week gestation ) 900 gram go to NICU after resuscitation ABG increase of Pa co2 with normal PH next step : -give IV acyclovir -give IV bicarb - give IV glucose #picture of viral warts -pleural calcification

#pt with asbestoses what is the specific sign : #dust mite how to prevent :

-wash clothes with hot water

#pt with bilateral infiltration in lower lobe (pneumonia )which organism is suspected : -klibsella -ligonella ( my answer )

# pt in crowded area and has pneumonia which vaccine you will give (long scenario) -hemophilus influenza (my answer ) -menigococcal vaccine

#pt in burn wll die due to : -smoke inhalation .

#what vitamin you will give to prevent hemorrhagic disease of newborn : Vit k

#pregnant with thyroid function test and it is completely normal except high TSH ..what diagnosis :

#holding breath :

-risk for generalize convulsion

#elderly patient bedridden for long time what will you do : -include family support (my amswer ) -IV valum

#you have difficulty to get information from patient ..next step: -direct question

#young patient with anxity ,,TTT : -fluxitn (SSRI)

#which drug contraindication in peptic ulcer :

-drug related to (NSAID) but I forget exactly ^_^

#degree of scoliosis to refer to orthopedic : -20 degree

-motor neuron disease ??? -polyneuropathy

#typical scenario of osteoarthritis

#typical scenario of rheumatoid arthritis #pt with polymyalgia rheumatic treatment : ( I advise you to read about it ) -prednisolone -acyclovir -antibiotic #typical scenario of bacteria vaginosis : -fish odor discharge , clue cells .

#treatment of thyroid carcinoma : -surgical resection (my answer ) . -radiotherapy -antithyroid drug

#pt discharge with menigiococcal meningitis and now asymptomatic ..what is next step: -ceftriaxon -rifampin (my answer ) not sure until now ..correct or not -no vaccine #high sensitive and specific test for renal stone : -CT scan

#pt with nodule near to eyes (dacryocystitis ),,TTT: -oral antibiotic

SLE- EXAM FOR Mohammed Al-Anazi,Qassim college of Medicine 20-11-2011

TTT of refractory hiccup?

*Best TTT of somatization? Multiple appointment ,multiple telephone calling, antideppresant,send him to chronic pain clinic. *carpel tunnel syndrome >> hand position antigenic drift .

*why influenza vaccine given annually? Bacterial resistance ,viral *visual acuity >> snellen chart nerve palsy

*cover one eye another eye go laterally >> strabismus , amyblobia,3rd *TTT of miagrine? Sumatriptan

*most common cause of intracerebral Hg >> HTN paranoid,grandiosity

*most common psychiatric condition come with mania ? *most common cause of female precocious puberty? Idiopathic

male,same age of puberty ,male earlier than female.

Female puberty 6-12 months earlier to male ,2-3 years before

*patient come with difficulty breathing in one nostril sympathomimetic

..O/E:erethematous structure best TTT: decongestant, antihistamine, *patient take seldenafil which drug must be avoided? Nitrate ovarian ca, thrombophelebitis.

*estrogen containing pills associated with ? breast ca, endometrial ca, *pregnant want to take antibiotic ,not known to sensitive to any drug , which antibiotic safe to given to pregnant? vesicle and pustule ?HSV1,HSV2, varicilla *patient with fever and fatigue prior to develop maculopapular then *contraindication of breastfeeding ?HIV, hep C *best stimulus for lactation? Breastfeeding!! osteoporosis. *Diagram and answer was >>patient above 80 years at high risk of *tibial tubercle pain?osgood shlutter, perthes disease. *typical scenario about osteoporosis. *which is the following true about chronic fatigue syndrome? Give him antidepressant, rarely resolve with TTT. *which is of the following true about pathophysiology of HTN? fibrinoid change of the vessles.

Decrease sensitivity of baroreceptor, peripheral vascular resistance , *10 year old had an episode of rheumatic fever without any defect to the heart. The patient need to take the antibiotic prophylaxis for how

a. 5 months b. 6 years c. 15 years

*pt discharge with menigiococcal meningitis and now asymptomatic ..what is next step: -rifampin -ceftriaxon

-no vaccine

*postmenopausal women at high risk of: osteoporosis *why SSRI best TTT? Effective and tolerable *compliance of prophylactive antiasthmatic drugs important to reduce :airway inflammation, reduce esinophil

*patient blood group A, they gave him blood group B and developed *quick TTT for SVT? Adenosine!!

limper pain, dyspnea and hypotention why? Q was about mechanism *case of appendicitis do resection what you will find if you examine appendix histologically?normal, lymphoid in muscular layer, neutrophil in muscular layer

*Pt. Taking bupropion to quit smoking, whats the side effect? A. Arrhythmia B. Seizure C. Xerostomia D. Headache

*Mother has baby with cleft palate and asks you what is the chance of having a second baby with cleft palate or cleft lip , 25% 50% %1 4%

*Which of the following dietary factors considered protective against cancer Vit D Fiber

Calcium

*Q about Buchard's nodule in PIP

*2 months amnorrhea refuse examination because she is tense and anxious what will do for her : FSH and LH, US pelvis exploration, scheduled lapototomy *stab wound of abdomen and omentum is seen what will do : *vit C in wound healing? Collagen synthesis

*60 years old patient has only HTN best drug to start with: *most common cause of nephropathy : diabetic nephropathy *baby with clavicle FX? Figure 8 !! *drug induce lupus ? hydralazine *self examination of breast every? Weekly, monthly, 6months, yearly *typical case of Wegener with saddle nose and other feature. ACEI, ARB, diuretics, beta blocker, alpha blocker

*pregnant I forget GA but in second trimester with uterine fibroid come with abd. Pain what will do for her : myomectomy, drugs, terminate pregnancy *Battered women.

*structure normally not palpable? LN

*case AF best TTT? Digoxin, synchronized DC the age? 48months, 36months

*child know 4 color and can wear clothes with little difficulty what is *Q about female urine incontinence ? SLE 26 nov 2011 Abdulrahman Al-Darrab King Saud University

1- How to prevent malaria:

a- Kill the vector and avoid mosquito bites b- Kill the vector and spray your clothes c- Avoid and spray d- Something

I searched it ..and most likely the answer is A

a- Bilateral fibrosis

b- Plueral calcefication

The most likely answer is A

3- Surgery in c3 colon cancer : a- Curative b- Palliative

c- Diagnostic

The answer is A

4- Weiiiiird psychiatric scenario .. patient with echolalia, echopraxia, a- Lithium

poor hyegine, insomnia, and weird postures what's the treatment?

5- (picture) showing huge mass in the Rt side of the neck with normal skin color .. no other masses in the body and some signs : a- Tb b- Infecious mononeoclusis c- Lymphoma The most likely answer is C

6- Long scenario for a lady suffer from vulvar itching .. remember that there's "bubles" in the scenario .. what's the dx:
a- Lichen simplex chronicus

This is the correct answer

7- There is interaction between Carvedilol and : a- Warfarin b- Digoxin c- Thiazide

d- Something 8-

and some other symptoms .. what's the dx: a- Mesentric ischemia b- Renal colic

The most likely answer is B

9- Scenario .. pt. suffering from wheezing and cough after exercise .. not on medications .. what's the prphylactic medication ? a- Inhaled b2 agonist c- Oral theophilline 10b- Inhaled anticholinergic

a- Pharingitis

Which of the following doesn't cause ear pain ?

c- Dental caries

d- Vestibular neuritis The correct is D

11-

lateral incisors .. what's the cause of this caries ? a- Tetracycline exposure c- Milk bottle

Scenario .. 18 months has dental decay in the upper central and

b- The family doesn't brush his teeth (something like this)

12-

breathing .. pulse of 60 ) .. what's the best thing to do? a- Warming and drying b- Ventilation

Baby with apgar score 3 in 1 minute ( cyanosis .. weak

13-

shortness of breath after exercise but no cough and there was a table Fev1=71% Fvc=61%

Old patient .. stopped smoking since 10 years suffering from

Tlc=58%

What's the dx? b- Asthma

a- Restrictive lung disease c- Bronchitis

d- Emphysema

e- Obstructive with restrictive

14-

x-ray showed dilated loop specialy the transverse .. what's the dx? a- Acute pancreatitis c- Bacterial enteritis b- Acute cholecystitis

Scenario .. diffuse abdominal pain, diminished bowel sounds ..

15-

pericarditis .. what's the dx? a- Kawasaki b- Still's disease Most likely A

Scenaria .. child sweats at night .. myalgia . arthralgia ..

16-

symptoms .. +ve family Hx .. what's the dx?

Scenario .. patient with multiple pigmented spots and other

Most likely A

17-

a- 30-40% of pregnancies end with misscariage b- Most of them happen in the second trimaster c- Cervical assessment must be done

Question about spontenuos abortion :

18-

a- Inhibits release of thyroid hormone fortha gland

Propythiouarcil mechanism of action :

b- Inhibits releaseor hormone for thyroid globuline I thimk the correct is C

c- Something about inhibt coupling and something about tyrosine

19-

a- Enalapril

Cause hypertensive crisis:

b- Lorsartan

c- Hydralazine

The correct is C Question about ptrygium

20-

This is the correct 21Long scenario about h pylori .. what's the regimen?

a- Omeprazole, clarithromycin, ampicillin This is the correct

22-

a- Schezophrenia b- Depression c- Enuresis

Colzapine is used in which childhood psychiatric disease?

23-

and ascites .. what to use ? a- Thiazide b- Hydralazine c- Something

Hypertensive patient with liver cirrhosis , lower limb edema

24-

a- TPN

Scenario about Acute pancreatitis .. how to feed ?

Correct

25-

a- Cevix

What's the most common area in women gonorrhea affects ?

b- Urethra

c- Poterior fornix of vagina The correct is A

26-

a- Placenta inhibit the passage of alcohol

Qustion about fetal alcohol syndrome:

b- Will cause fetla retardation and facial fetures and other symptoms c- It's safe to drink wine and hard something once a week Most likely B

27-

do ?

8 year old boy weight and height above 95th centile what to

a- Surgery

b- Medications

c- Observation for 12 months d- Life style midification 28-

a- Ibs

defecation .. 3-4 times a day

Abdominal pain mucus diarrhea .. no blood .. relief after

b- Ulcerative colitis

29-

a- Somatedin Correct

Acromegaly .. the cause

30-

fatigue generalaize pruritis and mild cough .. dx a- Hogkin's b- Lyme

Boy with presented with painless neck mass .. hx for 5 weeks of

c- Infectious mono Maybe A

31-

a- Hemophilia a

Fresh frozen plasma in what case ?

b- Hemophilia b d- Dic

c- Von willbrand e- Coagulopathy form liver disease Maybe D

32-

a- Decrease protien b- Increase glucose

In aseptic meningitis .. in the initial 24 hours what will happen?

e- Something

Most likely C Patient with small congenital inguinal hernia

33-

a- It will cause infertility

b- Surgey to be dona at 35 years

c- Elective surgery if it's reducible 34Mass in the upper back .. with punctum and releasing white

frothy material anything

a- It's likely to be infected and Antibiotic must be given before b- Steroid will decrease its size

c- It can be treated with cryotherapy

d- It must be removed as a whole to keep the dermis intact 35Pt. after stroke or trauma I don't remember .. he lost his smell

sensation.. which part is affected a- Frontal b- Temporal c- Occipital The correct B

a- Penicillin 1200 iv When meningococcal disease is suspected,

treatment must be started immediately and should not be delayed involves prompt intramuscular administration of benzylpenicillin SLE 26 nov 2011 Khaild Jebrin King Saud University

while waiting for investigations. Treatment in primary care usually

a-zollibdon

1- Ssri Pt after 2 month post MI cannot sleep what to give him

b- diazepam

2- obese, HTN cardiac pt with hyperlipidemia, sedentary life style and unhealthy food What are the 3 most correctable risk factor?

ac-

bd-

HTN, obesity, low HDL

High TAG, unhealthy food, sedentary life High cholesterol, HTN, obesity

High cholesterol, unhealthy food, sedentary life

ac-

b-

Ranitidine Heparin Atrovastsin Pt with 3 month hx of feeling like depression, recently become

4-

will do: acTricyclic Psychotherapy support Moi

bd5-

Epidemiology curve is: a- Plotting number of cases on time line b- Geographical places c- Case with similar diagnosis

6-

Painful vision loss:

a- Central vein thrombosis

b- Central artery embolism

c- Acute angle closure glaucoma

7- Neonate with mass on his eye : a-Neuroblastoma

8- 15 years old with palpitation and fatigue. Investigation showed RT ventricularhypertrophy, RT ventricular overload and right branch block what is the diagnosis : acASD VSD Coartaction of aorta Young pt with vague central abdominal pain then shifted to RLQ

b-

9-

with tenderness what is the diagnosis: aAppendicitis

b-

Diverticulitis

10- Middle age pt come complaining of abdominal pain and he think he has gastric cancer he went to 6 gastroenterologist did 1 CT 1 barium enema and series of investigation all are normal what is the diagnosis: acHypochondriasis Conversion Somatization

b-

11- 75 y/o female c/o hip pain after walking and busy day also prevent her from sleeping and continue in the morning for several

ac-

b-

Osteoarthritis Depression

Rheumatoidarthritis

12- 70 y/o male pt c/o knee pain after walking imaging showed narrow joint space with hypochondral sclrosing what is the diagnosis: acOsteoarthritis

b-

Rheumatoidarthritis Reactive arthritis

13- Similar hx to Q withsimilar choices 14 pt with HTN on diuretic he developed painful big toe what kind of medication he was on: aHyderocholrathiazid Furosemide

b-

15- Pt with HTN on thiazide came to ER shouting from pain in LT big toe O/E the whole left leg is swollen and tender no fever what is the diagnosis: aCellulitis

b-

Gout attack

16- Table with investigation Na 112

Osmolality 311 low What is the diagnosis? acb-

Connssyndrome SIADH

Cushingsyndrome d-

Diabetes insipidus

17- Repeated Q about nephrogenic Diabetes insipidus 18- A 5-year-old child came with earache on examination there is fluid in middle ear and adenoid hypertrophy. Beside adenoidectomy on management, which also you should do: a. c. Myringotomy

b.

Grommet tube insertion Mastidectomy

d- Tonsillectomy

19- What best explain coronary artery disease: acNoatherosclerosis

b-

Fatty deposition with widening of artery Atherosclerosis with widening of artery

20-Mechanism of vitamin C in wound healing: a. c. Epithiliazation

b. d. e. 21 a-

Aerobic fibroblast synthesis Collagen synthesis ---Enhance vascularization - Pt on long term steroid what are the main complication Osteoporosis DVT 22- pt with asthma on daily steroid inhaler and short acting B2 agonist what category:

b-

ac-

bd-

Mildintermittent Mildpersistent Moderate Sever

23- about head and neck injury : a. Hoarsness of voice and stridor can occure with mid facial i njury b. Tracheostomies contraindicated c. Facial injury may cause upper air way injures

24- treatment of trigeminal neuralgia aCarbamazepine

25- skin lesion pale pink on the leg of young boy not responding to 6 week miconazol what is the diagnosis: aTineacorpora

b-

Erythema nodusm

26- Mother worries about radiation from microwave if exposed to her child. What you tell her: j. Not all radiation are dangerous and microwave one of them Microwave is dangerous on children Microwave is dangerous on adult ----

k. l. m.

27- in pediatric what is the most common for failure to thrive : acProtein& milk intolerance Psychosocial Cystic fibrosis

b-

28- female with dysuria urine analysis showed ve gram diplococcic what is the organism : i. gonnrhia

ii.

Chalymide

29- Similar diagram about window period: iv. vi. HBc

v.

HBs ag

HBs antibody

30-This diagram came the Q was typical onset of nephrotic in diabetes: ac10 years 15 20 25 31 varicellavaccineyou will give: ac2 doses in 2 weeks 2 doses in 6 week 2 doses in 1 year 2 doses in 2 month

bd-

bd-

32- varicellavaccine in women wants to get pregnant acIs not contraindicate in pregnancy Before get pregnant 3 months Its live attenuated BACTERIA

bd-

Terminate pregnancy immediately

33- there is outbreak of diphtheria and tetanus in community,regarding to pregnant woman: n. o. p. q. Contraindication to give DT vaccine Ifexposed, terminate after 72 hour Give DT vaccine anyway

Ifexposed, terminate pregnancy immediately

34- All of the following are live vaccine except: a. MMR

b. Oral plio c. d.

Varicella BCG

e.

Hepatitis B vaccine

35- Patient with retrosternal chest pain, barium swallow show corkscrew appearance: a. c. Achalasia GERD

b. d.

Esophagitis Diffuse esophageal spasm 36- newly married woman complain of no pregnancy for 3 month

Withunprotective sexual intercourse:

37- teacher with vomiting and jaundice and 2 of his student no blood contact what is the best investigation: acHeb A IgG Heb B

b-

Heb A IgM

38-20 year old male had been stabbed on midtriceps, . On microscopic examination of this greenish fluid show grampositivecocci in chains: f. Streptococcal gangrene Chlostrideal gangrene Fourniers gangrene Meningocemia

g. i.

h.

39- old lady with skin changes near areola according to her because new detergent she used, if it didnt resolve after 2 weeks of steroid cream what you willdo: acMammography Cbc US

b-

40- pt with sever vomiting and diarrhea in ER when he stand he feel

a- insulin something

b- dehydration something

41 pt diagnosed with EBV and discharged a few days later he came to ER and when taking hx he become tachycardia and hypotensive what you will do: acFluid management

b-

Urgent abdomen CT

IV antibiotic with fluid 42- most common symptoms of soft tissue sarcoma:

j.

k. l.

Paralysis Pain

Ongrowing mass (painless)(slow-growing) 43- what is the dangerous symptom during pregnancy?

a-

b-

Vaginal bleeding 43- pregnant lady wants to know if her baby has Down syndrome Contractions

a-

b-

Amniocentesis Triple 44- Patient came to you complaining of hearing voices, later he

started to complain of thought gets into his mind and can be taken out a. c. SCZ

b. d.

Mood

Mania

Agoraphobia 45- Best sentence to describe specifity of screening test, is the population of people who:

a. c.

b. d. e.

Are negative of disease, and test is negative Are positive of disease, and test is negative Negative disease, positive test Positive disease, negative test Are positive comparing to total other people

46- mid clavcular fracture in neonate acMust be intraoperative fixation Figure of 8 Non union is common

bde-

Most of them heal by it self

a-

48- pt with hypothenar muscle atrophy numbness on little finger EMG showed ulnar entrapment what you would do acPhysiotherapy Observation Surgicalrelease Oral antibiotic Oral steroid Topicalantibiotic

b-

Allopurinol Prbencide

b-

49-newborn with eye infection ac-

b-

50-your advice to prevent plaque disease is: a-hand washing

b-rodent eradication c-spry insect side

51-female with inflammatory acne not responding to doxycycline and topical vitA .want to use oral vit A what you should tell her: aIt cause birth defect

m.

52- infant 48hs in ICU with jaundice mother healthy with previous term pregnancy what is the most likely the cause acSickle cell diseas Thalassemia Maternal fetal blood mismatch Hereditary genetic disease Croup

bda-

53- child with barking cough and fever 38

b-

Epiglottitis

54- what is the drug that comparable to laparoscopy in ectopic pregnancy?

a-

55- Mother who is breast feeding and she want to take MMR vaccine what is your advice: a. can be given safely during lactation

Methotrexate

b. contain live bacteria that will be transmitted to the baby 56- Major depression management: a. Intial therapy even severs

c. stop breast feeding for 72 hrs after taking the vaccine

57

15 y/o boy with +ve occult blood in stool .what is the best

investigation: a. isotope

58-56y old present with vasomotor rhinitis a. Local anti histamine b. Local decongestion c. Local steroid d. Systemic antibiotic

59- Patient after accident, the left ribcage move inward during inspiration and outward during expiration: a. Flial chest a. DM

60-Greatest risk of stroke: b. Elevated blood pressure

d. Hyperlipedemia e. Smooking

c. Family history of stroke

61- Child with large periorbitalhemangioma, if this hemangioma causes obstruction to vision, when will be permenant decrease in visual acuity a. After obstruction by one day

c. by 3 months

62- pt with renal stone what is the best invesatgation aCT

d. By 6 months

63- increase survival in COPD aO2 supplementation Smoking cessation

b-

64- cause of non-traumatic subarachnoid hemorrhage aMiddlemeningeal artery Bridging vein

b-

65- what's the organism responsible for psuedomembranouscolitis: a. c. Pseudomonas Colisteridum E.coli

b. d.

66- another Q as above

Enterococcus fecalis

67-18 months old came with bite by her brother, what you will do: a. Give augminten

b.

Give titunus toxoid

68- what cause epistaxis in children: aSelf induced

69- pt on anti Tb medication with hear loss what is the cause: A-pyrenzmaid b-

70- you r supposed to keep a child NPO he's 25 kgs, how much you will give:

Streptomycin

a. c.

b. d.

1300 1400 1500 1600

71- Deep jaundice wit palpable gallbladder? a. Cancer head of pancreas

72- Lactating mother newly diagnosed with epilepsy , taking for it phenobarbital you advice is: a. Discontinue breastfeeding immediately

c . Continoue breastfeeding as tolerated

73-Pregnant women has fibroid with of the following is True: a. Presented with severe anemia c. Surgery immediately b. Likely to regress after Pregnancy d. Presented with Antepartum He

74- Patient with GERD has barretesophagus , this metaplasia increase risk of : a. Adenocarcinoma

b. Squmaou cell carcinoma

75-In holding breath holding which of the following True: b. Increase Risk of epilepsy

a. Mostly occurs between age of 5 and 10 c. A known precipitant cause

76-All can cause gastric ulcer except:

d. Diazepam may decrease the attack

of generalized convulsion

c-

de-

Sepsis.

Salicylates.

77-about specifity:

Gastric outlet incompetent

b. test is +ve in healthy poplation

a. test is +ve in disese population

c. test is -ve in diseased population 78- Patient with high output fistula , for which TPN was ordered a few comatose and unresponsive , what is the most likely cause : A. Septic shock . d. test is -ve in healthy population

weeks ,2 unit of blood given and after 2 hours , the patient become

B. Electrolytes imbalance . D. Hypoglycemia .

C. Delayed response of blood mismatch . E. Hypernatremia

A. Main complication is hypoglycemia . B. Can lead to weight gain .

79- Which of the following is true regarding metformin :

C. It suppress the hepatic glauconeogenesis .

a) Intracellular c) Interstitial

b)Extracellular 81- Female patient did urine analysis shows epithelial cells in urine, it d)Vulva

comes from:

e) Cervix

f) Urethra g) Ureter

82- Child patient after swimming in pool came complaining of right ear

tenderness on examination patient has external auditory canal redness, tender, and discharge the management is: h)Antibiotics drops j) Steroid drops

i) Systemic antibiotics

83- regarding weight gain in pregnancy what is true : a- Pregnant woman should consume an average calorie 300-500 per day b- Regardless her BMI or body weight she should gain from 1.5 3

84-) 1st step in epidemic study is : 85-most common site of malignancy in paranasalsinuses : a. verifying diagnosis

a. Maxillary sinus

86- best indicator for labor progress is : a. frequency of contractions b. strength of contractions c. descent of the presenting part

98, BP 140/80, RR 18. A part of omentum was protruding through the wound. What is the most appropriate next step: a. FAST Ultrasound

87-A patient with penetrating abdominal stab wound. Vitals are: HR

b. DPL (Diagnostic peritoneal lavage) c. Explore the wound d. Arrange for a CT Scan

e. Exploratory laparotomy

88-A female patient with history of cyclic abdominal pain, inability to conceive, heavy menses, and examination showed tenderness & nodularity in uterosacral ligaments. What is the diagnosis? a. Endometriosis

89-A burn patient is treated with Silver Sulfadiazine, the toxicity of this a) Lycosytosis

drug can cause:

b) Neutropenia

c) Electrolyte disbalance d) Hypokalemia e) ..

SLE1 b King Saud University

Patient with CML taking imatinib mesylate and odansetron for nausea and vomiting presented with tachycardia,fever Diphoresis and hyperreflexia Dx: _ imatinib toxicity _ neuroleptic malignant syndrome ?? _ odansetron toxicity

P3 with hx of D,C after 2nd delivery complaining now of amenorrhea with high( FSH,LH) low estrogen..Dx: _ turner syndrome _ ovarin failure _ asherman syndrome

Waking up from sleep..cant talk, no fever, can cough, normal vocal cordsDx: Functional aphonia +ve leichman test: ACL injury

All are 1ry prevention of anemia except: _ iron fortified food in childhood

_ health education about food rish in iron _ limitation of cow milk before 12 month of age _ genetic screening for hereditary anemia _ iron,folic acid supp. In pregnancy and postnatal?? Methylergonevine is # in: Maternal HTN

OCP that causes hyperkalemia:

Estradiol p

Sever blepheritis with rosacea..Rx: _ topical ABX _ oral doxycyclin Lactational mastitis..Rx: _ doxycyclin _ ceftriaxon _ ciprofloxacin _ gentamyecin _ cephalexin

Patient with ARDS on ventilation developed pnemothorax..cause: _ -ve pressure ventilation _ central line _ 100% O2

( note : answer is +ve pressure or lung injury ) Recurrent watery discharge of eye, pain, sensitivity to light..on exam.> inflammation,ulceration of eye..cause: _u/v light _ dust&pollens

_ night accommodation

2 month infant with white plenched papules in the face what to do: _ reassurance _ topical steroids _ abx

Young suddenly develops ear pain, facial dropping..what to do: _ mostly will resolve spontaneously _ 25% will have permenant paralysis _ no role of steroids

40 y/o with mild epigastric pain and nausea for 6 months..endoscopy>loss of rugeal folds, biopsy> infiltration of B lymphocytes..treated with abx..cause: _ H.pylori _ salmonella

Ashmatic child taking beclomethason that mostly cause: _ increase activity _ intraocular HTN

_ growth retardation 16 wk pregnant not known to have illness before has high BP..DX:

_ chronic HTN _ gestational HTN

Patient with continous seizures for 35 min. despite taking 20 mg Iv diazepam..what to do?? _ give IV phenytoin _ give 40 mg IV diazepam _ give IV Phenobarbital

Pt after swimming pool(clear Dx of otaitis externa) Rx: _ nothing _ steroid _ amphotericin B _ ciprofloxacin drops

Which of following favor Dx of SLE?? _ joint deformity _ lung cavitations

_ sever rayaniod phenomen _ cytoid body in retina In patient with rheumatoid arthritis: _ cold app. Over joint is good

_ exercise will decrease postinflammatory contractures Before vaginal delivery, obstetrician should rule out: _ cord prolapsed?? _ cephalopelvic disproportion

Pt. with 1st and 2nd degree burn involving face and neck: the burn involves the face During heart contraction,heart receive more blood by: _ coronary artery dilatation _ IVC dilatation

All choices with no hospital admission except one which I choosed as

_ pulmonary vein constriction :

1- Fick method in determining cardiac output ; 1 BP 2 o2 saturation in blood 2- PVC caused by;

a decrease o2 supplement to heart b increase co2 to heart

3_ old pt with 2 years bone pain , lethargy , fatigue, wedding gait , came with table show high calcium and high phosphorus ; A_ osteoporosis B_ osteomalacia

C_ paget disease of bone

D_ metastases prostate cancer E_ paraneoplastic syndrome

4- child with inferior thigh swelling and pain but with normal movment of knee , no effusion on knee what the important thing to do ; A_blood culture b-ESR c_ASO titer

d-aspirate from knee joint d-plain film on thigh

5-propylthiouracil drug mechanism of action:

6-

a-Indwelling foley cather drain b voiding cytctogram c- cystoscopy

7- drug contraindication hypertrophic obstructive cardiomyopathy; A_ digoxin B_ one of b-blocker

alpha blocker 9- cold utritcaria treatmemt .-_-

10- chronic use of vasoconstrictive result in ; A_ rhenoid phenomenon B_rhinits sicca C_vasomotor rhinitis 11- posterior hip dislocation : ( from reconstruction) A flexion , adduction b- flexion abduction

c- extension, adduction 12- action of ocp : ( from reconstruction) A - inhibition of estrogen then ovulation B inhibition of prolactin then ovulation

c- inhibition of mid cycle gonadotropin then ovulation

13- thyroid cancer associated with : : ( from reconstruction) A hyperthyroidism B hypothyroidism C euothyroid

14- young fall high absent sound in right side and resounce percution first thing to do oxygen mank : 1 oxygen mask 2 tube thoractomy 15 -

scissors or nail cut which cause sharp pain at the base of the finger in (mean flexed DIPj) and cause pain when try to extend the finger.. 1-trigger finger 2-tendon nodule 4- mullet finger

matacarpophlyngeal joint and the finger become directed downward in

3-dupetren contracure

16- itching scale in pack of knee . face and ant elbow : A scapis

c- contact dermitis 17 mitral stenosis : B - systolic low pitch

A diastolic high pitch C- diastolic low pitch 18 chylmedia non pregent treatment : A doxycylene

19- randomized controlled trail : B both group treated simalary C open allocation

A systemic assiment predictably by particpint

20 difference between unstable and stable angina : A necrosis of heart muscle B - ..

21- mechanism of destruction of joint in RA :

B anti inflamtory cytokines attacking the joint c- . A - .. 22- most common malignant parotid tumor in children:

SLE,my new MCQs Exam Tue 20-9-2011 ( :) today with pt. with 10 days history of MI discharged yestarday .. present - sudne pianful left lim by exam limb is cold and pale .. Dx

sure it's correct % my answer ) I'M ) c- arterial embolism ( a- dpendent (my answer finger extended -

posetion human bite to the hand .. greatest risk of infection in which - (clenched ( correct answer - obsruction b/c lf case) << diffuse abd. pain with hestory of sursegy - adhesions ),m next invX a- barium enema

sure b- barium follow throuw ( my answer) not c and d other barium : caus what the .. pt. with ARDS in hospitl .. he devolop tension pnemothorax - i'm sure about positeve pressure .. <<< .a- negative pressure ventilation !!!!!!!!!! but negative b- 100% o2

c and d were irrelevent

there is small pt. with PID there is lower abd. tenderness.. on pevic exam - mass in xxxxxx ligamente.. Rx a- colpotomy 100% sure b- laprotomy c- laproscopy a- PCOS

.. infertile pt. with 3 previous d/c .. otherwise healthy .. Dx - b- ........ syndrome

in ob/gyn section ) :) c- Txxxxxxxxx syndrome. 100% sure (i got 10/10

d- ....... syndrome

e- shehann syndrome

: diagnosis is pregnant with heptitis .. best blood test to confirm the - a- alkaline phosphatase b- wbc d- ESR c- STOG it's negative TB outbreak ..and one pt. come to doing tubercalin test and - .. what to do a- BCG b- isonized

all of the following is extrapyramidal Sx exept - a- dyskinisia b- akathesia c- xxxxx esia

d- clonic - tonic convulsion advise

months,, what is your female come to with 3 UTIs history in last - defecation a- wipe frome behinde to front after .b- take a bath insted of shaower c- increse flude intake a- hydrocele c- CHD

in newborn exam .. what is more dangerous - b- abcent femoral pulses

what to .. gestation in NICU 900 gram wighet .. otherwise normal - <<<< do a- give hem milk orally b- glucose infusion

c- broad spectren antibiotic quastion

tricky .. healthy child with pRBC in urin 15 cells/hpf .. what to do - . a- repeat urin analysis for blood and proten ..

<<<< having growth spurt ...Dx tebial tubercle pain .. in 13 y/o boy -

( ) a- osgood fracture b- strees fracture ......... -c

multible old dibetic man with sudden unilateral visual loss .. thare is -

..pigmentation in retena with macular edema .. Dx

( a- retenal detachment ( wrong .. come with floters

( b- retinal artry occlosion ( wrong .. no chirry red spots ( d- dibetic retinopathy ( no macular edema :she was 30.. Dx

( fit with all data givine c- retinal vien thrombosis ( my answer .. it's when y/o female become deaf suddenly.. her mother become deaf - a- otosclerosis 100% sure b- acostic neuroma c- tympanic perforation

sure i'm 90% << a- ostioprosis b- hot flush c- deppresion

: define epidemlogy-

d- pelvic floor weakness

.deases the scince of distrebution and determnant of <<<<<<< a- metoprlol b- atenalol c- mesoprolol d- xxxxxx lol e- yyyyyy lol

: an alpha blocking effect which of the folloing b- blocker .. have -

: miliary TB caractarized by - a- spare lung apical (b- septal line (not sure

c- multible lung nudules

to recognize primary snoring fros OSAS Questionnaire Screening the - .. : is Questionnaire a- otowa ......... -c ........ -d ADHD <<<<< : ADHD Rx - a- olanzpin Questionnaire Horchover -b

-c : epidemic curve is - read about it :case control study is-

read about it cuz they will give very long defintion mortality digoxin- .......-

decrese his patient with congestive heart failure , which medicaion will - forsumide -

what is the patient with congistive heart failure and pulmonary edema - :best treatment spronalctone- .forsumide- ....

the post partum women when she went back to work ,, she exposed to - sun and started to have brown discolortion in her face .. what is the :diagnosis uritcariA pigementosa?? not sure - : regarding hiatal hernia - (sympotmes increase with pregnacy (i choosed it - sympotmes increased with lying down - ( (: N-acetylcystine (correct :child took 20 pills of paracetamol .. what u will give - :regarding murmer of mitral stenosis

holosystolic - mid systolic- new scool .. child moved with his family to new city n he started to go to - any in the school he had low mood n doesn't want to interactive with :activity .. this a cas of depression- hypomania ! i think it is not correct- palapitation ,, years old girl failed in math exam .. then she had - : tachypnea and paracethesia .. this is conversion- areobic- vit d- hyperventilation syndrome? i think this is the answer- : patient with IHD ,, best exercise is- :whic one will reduce colon cancer - zenc-

:symptomes of open angle glacome- will do

!! no fiber no vit c

investigation u child with syomptomes of intestinal obstruction .. what- barium enema-

barium follow throgh- GD LUCK

------------------------------

()

what is the most common malignant parotid tumor in children: a) Mucoepiptheloild carcinoma b) Adenocarcinoma c) Undifferentiated CA

2- Female with dysurea and cervical motion tenderness: a) Cervicitis b) PID c) Cystitis

d) Undifferentiated sarcoma

3- What is the best diagnostic test for maxillary sinusitis: a) CT scan b) X ray d) MRI e) Us

d) Pyelonephritis

c) Torch examination

4- A man is brought to the ER after having seizure for more than 30 min the most initial drug you will start with: a) IV lorazepam b) IV phenobarbital

5- A 40 year old man who become sweaty with palpitation before giving a speech in public otherwise he does very good at his job, he is having: a) generalizes anxiety disorder b) performance anxiety c) agoraphobia d) depression

d) IV haloperidol

6- A women who lost her husband 2 weeks ago she is unable to sleep at all you will give her: a) floxitine b) diazepam

c) halperidol

d) amytriptaline

7- A diabetic pregnant with HX of fetal demise .. now is having a fetus who is healthy and her DM is very well controlled .. you will allow her for: a) C/S at 38 weeks c) allow SVD b) induction at 36 weeks 8- A picture of mid line swelling that moves with degilution:

c) Cystic hygroma

9- a man who is having a severe pain on his big toe with knee pain and examination revealed negative perferingent crystals: a) uric acid deposit secondary to synovial fluid over saturation

d) Thyroid carcinoma

10- Secondary dysmenorrhea is: a) due to anovulation. b) due to gonadal agenesis

b) Ca pyrophosphate secondary to synovial fluid over saturation

11- Patient with symptoms of blephritis and acne rosacea the best Rx is: a) Doxacyclin b) Erythromycin 12- Female with dysurea, urgency and small amount of urine passed .. she c) Cephtriaxone

c) always pathological

received several courses of AB over the last months but no improvement .. should consider: b) DM

all investigations done urine analysis and culture with cbc are normal .. you a) interstitial cystitis c) Cervical erosion

d) Candida albicans

a) Give BCG vaccine b) Rifampicine c) Tetracycline

d) H. influenza vaccine 14- A boy who was bitten by his brother .. and received tetanus shot 6 will:

month ago and his laceration was 1 cm and you cleaned his wound next you a) give augmentin

b) suture the wound c) give tetanus shot d) send home with close observation and return in 48 hr a) liver enzyme will peak within 3-4 hr

15- in aspirin overdose:

b) first signs include peripheral neuropathy and loss of reflexes 16- A 42 year old man with cushing syndrome and had a fracture .. you should investigate a) osteomylitis c) 150 mg/kg of aspirin will not result in aspirin toxicity

b) osteoarthritis 17- The specificity is: c) osteoporosis

b) When the person does have the disease with -ve test

c) When the person does not have the disease with +ve test 18- The important risk factor for Stroke is: a) DM b) HTN d) When the person does not have the disease with -ve test

19- a man who has had MI you will follow the next enzyme a) CPK b) ALP c) AST

c) Dyslipidemia

20- an old man who had stable angina the following is correct except: a) angina will last less than 10 min b) occur on exertion c) no enzymes will be elevated

d) Amylase

d) will be associated with loss of consciousness

21- Which of the following is given as prophylactic ant arrhythmic after MI: a) Procainamide

d) Quanin

22- 5 yr old adopted child their recently parents brought him to you with white nasal discharge. He is known case of SCA. What you will do to him: 23- The antibiotic prophylaxis for endocarditis is: b) 2 g amoxicillin before procedure 1 h c) 1 g amoxicillin after procedure a) Give prophylactic penicillin

e) Metoprolo

d) 2 g clindamycine before procedure 1 h 24- 19 yr old girl with URTI and splenomegaly. The cause: a) Infectious mononucleosis c) Malaria b) Streptococcus pharyngitis 25- Child with leukemia he has septicemia from the venous line the organism is: a) E coli b) GBS 26e) 1 g clindamycine after procedure

c) Pseudomonas

discharge is greenish showed Gram +ve cocci the organism is:

urgery the

27- a man travelled to Indonesia and had rice and cold water and ice cream .. he is now having severe watery diarrhea and severely dehydrated .. most likely he has: a) vibrio cholerae b) C difficele d) Dysentry e) Shigella c) C perferngins

c) Pseudomonas

28- When showed you refer a pt with scoliosis: a) 10 degree b) 15 degree 29- What is the best way of health education: a) Mass media b) Internal talk c) 20 degree

30- Pt with bilateral knee pain without signs of inflammation: a) RA b) OA

c) Individual approach

c) Septic arthritis

a) assess airway b) assess GCS c) Establish IV lines

32- an old man 65 years with Hemoglobin 9 .. you will: a) Assess Iron levels b) Assess LDH

33- computer programmer presented with wrist pain and +ve tinnel test. The splint should be applies in: a) dorsiflexion position c) extension position b) palmarflexion position 34- The most common cause of non traumatic subarachnoid hemorrhage is: a) Middle meningeal artery hemorrhage b) Bridging vein hemorrhage 35- ECT is good for those:

c) Arrange for endoscopy

c) Rupture of previously present aneurysm a) severe agoraphobia

b) Severe major depression

c) acute otitis media d) cholesteatoma e) Eustachian tube dysfunction

37- A man who is having severe vomiting and diarrhea and now developed leg cramps after receiving 3 liters of dextrose .. he is having: a) hypokalemia b) hyponatremia c) hyperkalemia d) hypernatremia

38- A man who bought a cat and now developed watery discharge from his eyes he is having: a) Allergic conjuctivitis b) Atopic dermatitis c) cat scratch disease a) 24h

39- Facial injury suturing remove after? b) 3 5 days ( most likely ) c) 7 10 days d) 14 days

40- Acute angle glaucoma , with COPD and DM you give? a) Metoprolol b) Something with lol

41- Child 9 month hx of congenital heart disease .. central and peripheral cyanosis Dx? a) Tetralogy of fallot

d) Steroids

b) Coarctation of aorta c) Truncus arteriosus d) ASD

42-Buprion use to quit smoking most common side effect is ? a) Seizure b) cough

c) weight loss 43- And old man with low hemoglobin and IDA he is most likely having: a) Hemorrhoids b) colorectal cancer

44- A female with dysurea invx showed presence of epithelial cells a) chlamydia urthitis b) cervicitis

c) ulcerative ****** not cloitis

45- a 3 year old with low hemoglobing eats lots of milk and very little red meat you will give :

46- Old pt complaining of back pain on walking on examination there was stiffness of the muscle and there was some finding on the X-Ray of spondyloarthropathy best effective ttt: a) Physiotherapy b) NSAID c) Surgery

c) Send home with observation

47- A child with congestive heart failure and several hemangioma on the body .. the most likely place for the hemangioma is: a) liver b) spleen

c) intestine

48- Old pt with knee pain increase with walking , crepitus and stiffness for several hrs on waking Dx? a) Osteoarthritis

d) pancreas

49- Newborn with clavicle fx:

b) Rheumatoid arthritis

a) Mostly brachial injury occur c)

b) Mostly heal without complication d) Occur in the premature infant commonly

50- a patient with acne of several appearances open .. closed .. red .. it is most likely: a) obstructive

51- a patient with a large nodule in the nose which is painful and talangectasia on the face you will give: a) deoxycycline b) clindamycin c) retenoid

b) inflammatory

52- a man who received blood transfusion back in 1975 developed jaundice most likely has: a) Hep A b) Hep C

c) Hep D d) Hep E e) Autoimmune hep

53- A woman wants to take MMR she is breast feeding you tell her: a) may be given in breast feeding c) contraindicated in pregnancy b) it contains live virus which will be transmitted to the baby d) stop breast feeding for 72 hrs

54- Best method to prevent plague is:

b) Kill rodent

c) spray pesticide

55- Dental caries to prevent it .. mix the water with: a) Vit A c) Zinc b) Fluoride d) Calcium

d) give prophylactic AB

56- a man with high fever, petechial rash and CSF decrease glucose .. he has: a) N menengditis b) N gonorrhea c) H influenzae

57- High senstive & specific for urolithasis the man had severe pain for one day and you suspect kidney stones : a) CT scan b) X ray c) MRI d) IVP 58e) US

about 1 y ago what you'll do:

a) give her double dose vaccine

59- In a certine study they are selecting the 10th family in each group,ahat is the type of study: a) systemic study

c) see if she has antibody and act accordingly

b) non randomized study 60- the separation of chromatid occur in: a) anaphase b) metaphase 61- a patient old with WBC 17000 and left iliac fossa tenderness and fever most likely has: a) diverticulistis b) colon cancer c) crohn disease c) telophase c) stratified study

62- a DM HTN patient with MI receiving metformin and diltiazem and other medication his creatine clearance is high .. you will do: a) add ACE II inhbitor b) remove metformin

63 - 70 yr old presented with wt loss, fatigue, anemia , upper quadtrant pain without any previous history, the stool sowed high fat he is a known

c) continue same medication

a) Acute pancreatitis

b) Chronic pancreatitis

64- a man after defecation finds blood on toilet paper he been having difficulties with defecation: a) colon cancer c) anal fissure b) hemorrhoids 65- a baby with blood in the stool and bought of crying and x ray shows obstructive pattern.. looks like intosusseption you will do: a) surgery b) Barium enema c) observation d) giv e IV fluids and let obstruction solve itself

c) Pancreatic carcinoma

66- a child 3 years old fell from the bed vomited twice and has mild headache and no loss of consiousness .. you will: a) call for neurlogist c) CT scan d) MRI b) send home with close observation

67- Mother has baby with cleft palate and asks you what is the chance of having a second baby with cleft palate or cleft lip:

b) 50% c) 1% d) 4%

68- over oxygenation with 100% O will not result in: 69- Which of the following with antipsychotic medication have rapid onset of action? a) sublingual b) oral c) IM d) IV a) depression

70- which of the following contraceptive method is contraindicated in lactation: a) OCP

b) Progesterone only 71- the following is not a risk factor for coronary heart disease: a) High HDL b) HTN c) DM c) IUCD

d) Hypercholestrolemia

72- pregnant lady 16 wks presented with vaginal bleeding ,enlarged snow storm appearance on US: b) Partial hydatiform mole c) Endometriosis d) Fibroids

abdomen, vomiting ,her uterus is smaller than expected for the gestational a) Complete hydatiform mole

73 - the drug which is used in seizures of eclamptic origin ( pre eclampsia ) a) Mg sulphate b) Diazepam c) Phenytoin

74- Female presented with thirst and polyurea.. all medical history is

d) Phenobarbital

negative and she is not know to have medical issues.. .she gave history of What is the cause of her presentation a) Nephrogenic DI b) Central DI

being diagnosed as Bipolar and on Lithium but her Cr and BUN is normal.

75- child swallowing battery in the esophagus management : a) bronchoscope b) insert foley catheter

c) observation 12hrs ( my answer)

76- Strongest method to prevent the disease: a) immunization b) change health behavior of PPls a) warfarin b) heparin c) aspirin

77- a female pregnant previously she have DVT you will now give her:

78- ibuprofen is contraindicated in: a) peptic ulcer b) seizures

d) enoxparin

79- In holding breath holding which of the following True: a) Mostly occurs between age of 5 and 10 b) Increase Risk of epilepsy

c) A known precipitant cause of generalized convulsion 80- the best to give for DVT patients initially which is cost effective: a) LMWH b) Unfractioned Heparin d) Diazepam may decrease the attack

81- Facial nerve when it exits the tempromandibular joint and enter parotid gland it passes: a) Superficial to retromandibular vein and ext. carotid artery b) deep to ex. Carotid c) deep to R vein 82- In hiatal hernia:

d) between retrmandibular vein and external carotid artery a) It will increase with pregnancy

83- a patient come to ER with constricted pupil and respiratory compromise you will suspect: b) opiates d) ectasy c) cocaine 84- the treatment of trichomonas vaginalis: a) mteronidazole b) deoxycycline d) Amoxacillin c) Ciprofloxacin 85-

develop same condition in the next 2 week , what is the attack rate a) 40% b) 60%

d) 20%

86- Contraindication of breast feeding: a) Asymptomatic HIV b) Active hepatitis C c) Veneral wart

87- Case about a child with drooling, fever, barking cough in sitting position, dx: a) Croup ( acute treacheolaryngiobronchitis ) b) Broncholities c) Pneumonia

d) TB treated for 3 months

88- a boy with nocturnal enuresis .. psychotherapy failed to show result you will start with: a) Imipramine and vasssopressine b) clonodine and vassopressine c) clonodine and guanfacine d) Imipramine and guafacine

d) Acute epiglotitis

89- a patient who thinks that he has a brain tumor with a long list of symptoms:

b) hypchondraisis d) depression

c) generalized anxiety disorder 90- a picture of raised skin with black dot in the middle a) mulluscon contagisom b) viral wrats c) erythema nodusm 91- Assymptomatic woman with trichomonas: a) Treat if symptomatic c) Treat her anyway e) follow up b) Treat if she is pregnant d) tell her to come in one month if she developed symptoms 92- Pregnant women has fibroid with of the following is True: a) Presented with severe anemia c) Surgery immediately b) Likely to regress after Pregnancy d) Presented with Antepartum Hemorrhage a) repeated elbow trauma b) Autoimmune disease d) chicken pox

93- Olecranon Bursitis of the elbow joint caused by:

94- an old patient with the following labs Na was low and plasma osmolality or urine was low I don't recall it: a) Cushing syndrome c) Conn syndrome a) Sleep apnea b) Addison syndrome 95- Which of the following is an indication for tonsillectomy? b) Asymptomatic large tonsils c) Peripharygeal abscess d) Retropharyngeal abscess a) Fistula formation

d) rupture of bursa

96- Which of the following features is related to crohns disease: b) Superficial layer involvement SLE Exam dec 2011 ()

P MITRAL .. P TALLER THAN 2.5

-1REGARDING ECG:

4-CHILD SEVERLY ILL AND FEVER FOR 2 DAYS ANOREXIA NAUSEA BODY ??

-3TTT OF SCABIES :PERMETHIN

VOMITING THEN PETECHIA RASH APEAR IN TRUNK AND SPREAD IN THE MEASELS- MENINGOCOCCAL MENINGITIS MOUNTAIN FEVER 5-WHICH DRUG WE GIVE WITH ANALGISIC TO PREVENT S.E ? CIMITIDINE METOCLOPRAMID -6BEST DIAGNOSTIC IN ACUTE DIVERTICULITIS ?

CT BARIUM ENEMA-COLONOSCOPY-SIGMOIDOSCOPY 7- RECOGNISED FEATURE OF HIATUS HERNIA ? LEUCOPINIA- SKIN PIGMENTATION ANOREXIA MORNING VOMTING INCREASE WITH PREGNANCY8-PT CAME WITH RAPID BREATHING ACETON SMELL GLUCOSE 500 ? HYPOGLYCEMIAL CASE LACTIC ACIDOSIS AND M.ACIDOSIS ACIDOSIS UNCONTROLLED HYPERGLYCEMIAL CASE L.ACIDOSIS AND METABOLIC STARVATION BREAK PROTEIN ANF FORM KETONS 9- ANEMIA OF CHRONIC DISEASE ? 10?RECURRENT VERTIGO-TINNITUS HEARING LOSS ?

MEINERES CHOLESTEATOMA VESTIBULAR NEURITIS

SURGICAL DECOMPRESSION CAT SCAN STEROID AND ANTIINFLAMMATORY

CHEXEN BOX SCABES-ATROFIC DERMATITIS REFER TO OPHTHALMOLOGIST DIET RESTRICTION ?PROPHYLAXIS

WRIST WITH PAPULOVESICULAR RASH 12

ON RETINA EXAMINATION NEOVASCULARIZATION 20 13-

14 PT WITH SEASONAL NASAL IRRITATION AND SNEEZING ANTIHISTAMINE-STEROID-DECONGESTED 15- DRUG USED IN ATTENTION DEFICIT NITROFUNTON-CEPHLAXIN-SMT DRUG? 16- PT WITH UTI ALLERGIC TO SULFA AND PENICILLIN ? 17- PT HAS ASTHMATIC ATTAK WITH EXERSISE AND COLD ?INITIAL SHORT B 2 AGONIST STEROID INHALER THIOPHYLINE 18- PT WITH MENINGO MENINGITIS DRUD OF CHOICE ? PENICILLINE DOXACILIN

20- RAYNOD SYNDROME PASSIVE SMOKER BLUISH HAND AT COLD WHAT U WILL ADVICE HER ? PASSIVE SMOKER DOES NOT AFFECT HER USE ANTI VIBRILATION GLOVES WARM HER CORE BODY TEMP AT COLD DAYS

21-GOLD STANDERS TEST IN RENAL FUNCTION ?

CREATININE CLEARANCE 24H CREATIN- ??RATIO

HIV 22!!!! PNUMOCYCTIC CARNII-KAPOSI SARCOMA CAUSE ? 23-PT WITH LOIN PAIN ON US HYDRONEPHROSIS OF BOTH URETER ? BLADDER CANCER PROSTATE ENLARGMENT- PELVIC CANCERFIBROSIS 24- PT DYSMENORRIA DYSPARUNIA INVERTED UTERUS ??????? ADENOMYOSIS-ENDOMETRIOSIS-UTERINE LIOMATA-UTERINE CARCINOMA

25-which drug C/I in case of obstructive hypertrophic cardiomyopathy ? B blocker

26-Child with mild Truman develop hemoarthrosis , in past hx similar episode Dx THROMPOCYTOPENIA- FACTOR 8 DIDICIENCY-

27- PT GO TO PHC FOR HIS MIGRANE MEDICATION HE RECENTLY

DEVELOP EASY FLUSHING OF THE FACE AND LARGE NOSE !! BEFORE 2 DAYS OPHTHALMOLOGIST DISCRIBE TO HIM FOR KERATITIS AND BLEPHARITIS WHAT IS THE DIAGNOSIS??? ATOPIC DERMATITIS ROSACEA.. 28-1RY OSTEOPROSIS CASED BY ?

CA INTAKE VIT D INSUFFICINCY-AGING PROCESS-EXERCISE 29- PT HAS COLON CANCER STAGE C2 ROLE OF SURGERY ? CURATIVE PALLIATIVE-EXPLORATORY-DIAGNOSTIC HYPOTENTION ?INITIAL ACTION? 31- RT LUNG :

30-PT IN HER 4TH DAY AFTER C SECTION WE FOUND HER PROFOUNDLY GIVE 0.9 NS WITH NACL ALBUMIN- DO SEPTIC WORKUP AND START ABX 1 FISSURE-2 PULMONARY VEIN 7 SEGMENT

SLE Exam 2011 by :Omar Alharbi King Saud University

SLE Exam 2011

regarding anticaogulation therapy .. vit k reverses the action of warfarin * long scenarion of MI , the q is ,, inappropriate management: IV ca++ channel blocker nitro paste iv morphine beta blocker * chronic diarrhea is a feature of ... typical men malzamat alqaeem * ARDS , cause of pneumothorax, central line insertion damaged lung

* pt is taking tx for glaucoma, now having SOB and Cough .. what medication is he on ?

pilocarpine timiolol

* first indication of pre-eclampsia is high Bp abruption Pv bleed * pt GA 37 week , painful pv bleed, dx ,,, abruption * dysuria + yellowish greenish discharge.. trachoma candiada

* pt, GA 34 breach presentation , what to do? wait and reassess by 36 weeks * pt have cheesy pv material ... candia

trachoma B. vaginosis * photo of skin rash, dx, sebbrhoic dermatitis don't worry typical description * graph , women and osteoprosis can't remember the q , bas ka2annaha feeh fe malzamat alqaseem * red tympanic membrane, + hemorhagic vesicle !

H influenza * bluging tympanic membrane: tx amoxicillin * basic science regarding IHD ,

all answers are crappy except for, HTN is a known risk * antipsychotic , least to cause tardive dyskinesia .. clozapine * pt has diarhhia and occult blood and colonoscopy is showing friable mucosa , bipposy is showing g crypt abcess .... chrons UC * pt with UC and tender nodule ... erythema nodosum * tx of pyodrma gangrenosum .... oral anbx

local anbx * pt on antidepressant medication, having insomina and restlessness what is he on .... SSRI MAOI

* pt has EBV, during abdomen exam., became pale with tender LUQ ..... IVF / Urgent CT

rush him to OR * sutured triceps post trauma, greenish material gram +ve in chains staph strept * MCC of physiologic hypoxemia .... don't know, don't care :P

* DM, tired and blood glucose is 60 ... he is on sulphonylurea * type 1 DM ... HLA DR4 *doesn't go with head engagment ... 3/5 of head in abdomen

* urinalysis, epitheilal cells ..... contamination,same as alqaseem malzama scenario of hypothyroididm

* mother hep B positive , son is also positive .... vaccine ,,, same as alqaseem malzama SLE Exam2011 Dr.Amani Alahamri King Khaled University

Patient complain of infertility obese hirtsuim acne . Ultrasound .. multiple cyst What is most likly DX Polycystic ovary sydrom and ask patient to decrease the weight pateint complain of RUQ pain radiated to shoulder and fever ( -2 picture of acute cholycystits) what is most usful test in DX Ultrasound

Age Fever High WBC High sedmentation rate paient suspected to have connective tissue disease what is most -4 favurable to SLE Cystoid body in fundoscopy Cavitaion in lung ve anti RNP+ Sever Ryundoe phenomena Child w -5

examination there is congested thorat and pharynx and white to papule on erthymatus base in mouth and lip what is most yellowish likly DX Coxsacki virus Herps simplix virus Child with high grade fever .. Cough drooling of saliva and Stridor-6 TTT ( ( Paracetamol Admitted to ICU snd call the ENT psteint fall from ladder .. Breathing labored and pt cynosed on -7 examination silent chest over right lung with resonant on percussion What first step Chest x ray Cricothyrodtomy

Oxygen mask Oral endotracheal tube patient with breast cancer and metastasis came complain of -8 tachycardia hypotension , engorged neck vein and SOB what is most action next D-dimer Ventlation prefusion scan Give fursamide and refer to do echo -9 child came with fever and ear pain on examination ( the same -10 picture of otitis media) ttt Amoxacillin Cefruxime Which drug can not be use in acute cholysystits-11 Naproxen Morphine Mepriden Acetamenophin Perdoxyphen Which drug can use in acute back pain-12 Diazepam Alprozam Metoxelen old patient came with fever . left LQ pain and tenderness but no -13

Sigmoid volvuls Diverticulitis Intestinal obestraction -14 abdominal pain and bloody diarrhea DX Ischemic colitis 15 - pateint with decrease lipido and weak erection ( or ejaculation ( In investigation prolactin high , LH and FSH normal what is next step Brain MRI Abdomen and pelvic CT all live vaccine except-16 Hepatits B ttt of non inflamatory acne -17 Clindamycin Retonic acid Isoretinon picture of bulls in food ... In biosy there is epidermal lysis and on -18 immunoflurescen: deposition of IgG DX Bulls pemphigoid Pemphigoid valgarius

Can be presented with painless limp It alawys unilateral ... 20-Athelet man came complain of pain in foot wiyh walking on excamination there is tenderness in planter of foot what is DX Planter faciitis Halux vagus Hallux rigidus ( ) pateint complain of left elbow pain -21 lateral epichondylitis relative risk -22 paient -23 dysmeanorhea) DX Endometriosis Pelvic congestion Endometritis and pelvic pain .all investigatin -24

Mafnemic Compined pill ... : withdrwal symptom of stop smoking -25 day 2-1 day 4-2 pateint came with cough ,SOB snd wheezing he had past hx of -26 glucoma what is most likly drug cause this Betoxelol Timolol prinetal mortality rate -27 ) patient with PPD test positive -28 Isonized and rifampcin for 6 month Isonized for 6 month Isonized and rifampcin and streptomycin for 12 month ): ..

) Induration more than 10 mm in philpine man .... .. ): old patient C/o stifness in knee and bilatral enlargment in knee -30

Osteoarthritis you want to give varcilla vaccine in one no have vaccin before -31 Two dose and 6 weak between pregnant not vaccinated against measls and mumps and rublla .. -32 She exposed to rublla 3 day ago what you do No treatment Immunoglobin Tell her no affected on her pregnancy if she take the vaccine 33- in -ludwig angina most common complication Asyphaxiation patient with vertigo ... Progressive hearing loss. 34 What you find in MRI Acustic neuroma 35- Pateint complian of diplopia , weakness , and frequant aspiration pnumonia in last 2 month ... In examination there is spascity and fasciculation DX Mythenia gravis Mythenia syndrom

36- child with low grade fever , sore thorat in examination there is lymph node enlarment but not tender and no exudate on phrynx DX It is most likly streptococcal than viral It is viral more than bactrial Most likly EBV 37- what is the. symptom Most likly occure with hiatus hernia Skin pigmentation The symptom increase with pregnancy ...

>> 38- patient with bilateral eye redness . Discharge and tearing on examination cornea , lens all normal Nd tere is conactival follicle DX Acute conjactivitis 39- child C/O fever , sore thorat all examination was normal What is the ttt : Cefruxime Ceftriaxone Give paracetamol and take pharenx swap 40- pateint C/o ictrus in skin and eye on investigation WBC 2500 plt 70,000 HG 7 lekocytosis 17% total bilirubin 51 and direct

+ve coomb's test In US obestructive billiary duct jhemolysis 41- Child not complain of any thing all investigation and examination was normal except the pharengeal swap +ve for meningiococal ... What ttt No need for ttt IM one dose of ceftriaxone 42- non pharmalogical ttt of osteaoartritis Analgsic cream Strenthing muscle excerise 43- kawshirkor Low protin and high carbohydrate Low protien and low carbohydrate 44... Because the organsim develop resistant Develop new antigenic drift 45- unfavurable risk factor of schezophrenia Family hx 46- infant with high grade fever .. Irritable .. Look sick .. Complain of

anuria 4 hour with multiple petechiea and purpura on body .. He was tachycardic and hypotensive DX Renal fauiler Septic shock 47-patient c/o low self steam and fatigue .. Lack of intersted and concentration loss of sleaping , depressed mood for last 2 years what DX Dysthymic 48- patient with bed sore involve skin and extend to fascia what a grade Grade1 Grade 2 Grade 3 Grade 4 49-lacteting mother complain of fever and breast tenderness and redness diagnosed as bactrial mastitis what is ttt : Continoue breastfeeding and hot compresser and antibiotic Discontinue breast feeding and give antibiotic to mother and baby 50- patient with rhumatic heart disease and had mitral valve stenosis Mitral vave diameter less than 1 mm

Left atrial hypertrophy and decrease pulmonary prusser Left atrial hypertrophy and champer dilitation RV hypertrophy and decrease pulmonary prusser RV hypertrophy and champer dilatation

...

SLE Exam,26/12/2011 By:dr.Jehid alhajaj

In the name of allah My SLE exam (19-12-2011) Dr.Rawn Alkhalif 1back, insomnia, 8kg loss O/E there is mild jauindice, and he is fit the criteria of depression what is the cause of his depression: MDD Dysthimic disorder Adjustment disorder depression secondary to medical condition 2every thing normal except decrease in visual acuity 20/100 in rt. Eye, 20/160 in lt. eye ,, cornea, lens, visual eld all within normal ,, on fundoscopy you find early cataract formation in both eye what you will do: Refer to cataract specialist for cataract surgery Refer to ,,,,,,,,,,,, ,,,,,,,,,,, ,,,,,, laser correction Unergent referral to ophthalmologist 3-post-partumbreastfeeding lady complain of breast engorgement & tenderness what you will do: Warm compressor before feedinf Cold ,,,,,,,,,,,,,,,, ,,,,,,,,,, ,,,,,,,,,,, They mention some antibiotics 4- pt. complaining of retro-sternal chest pain more in morning, after food when laying down..Dx: GERD 5-most common cause of epitasis in children: Polyp Self induced injury 6- typical scenario of closed angle glaucoma, Rx: IV ..+% eye drop Other choices not accepted (IM, oral) 7-child waking from sleep with crustations what is Dx; Bacterial conjunctivitis 8| P a g e

coming with new complain which is parasthesia & numbness in Lt. side of face what is the Dx: Aquistic neoroma Mennier disease Laberyinthitis 9-typical case of noncomplicated UTI, Dx: TM+. 3days 10- filling defect in renal pelvis not opaque, on US echo.. (they prescribe the apperence of this filling defect but I forget it) what is this filling defect: Uric acid stone Blood clot Epith. Cells Vascular 11- old patient you suspect cognitive dysfunction what to do before sending the pat. To home: Do brief IQ test Asses for hearing loss Click test (not sure for name) . Chart 12- true regarding syncope: Most common cause of fall in elderly If without warning sign it suspect vasovagal attack 13Hypothermia Hypoxemia Carbon monoxide poisoning Hypokalemia 14-Rx. Of scabies in pregnant women: 15-what is the mechanism of action of aspirin: Inhibit cyclooxygenase Inhibit lipooxegenase 16- what is the drug that will preserve the histology in primary liver cirrhosis: I couldn`t remember the choices but one of them is interferon. 17-young lady every thing within normal regarding her menses but there is 7cm mass in ovary, what is it: Benign ovary teratoma
| P a g e

Luteal mass Follicular mass 18- what is the most economic twice daily prescribe NSAID: They put a list of nonsteroidal!! one of them ibuoprophen!? 19- child on supplementation, coming with nausea, vomiting & diarrhea with black emesis, you suspect a toxicity of: May be iron because the stool is black 20- acute loss of body fluid in abdomen will cause: Sepsis Hypovolemic shock 21- contraindication of breast feeding: Asymptomatic HIV 22- Which one of the following need to add to water to decrease the incidence of dental caries: Fluride 23- all are risks for IHD except: High HDL 24- Strongest risk factor for stroke: HTN 25- Postpartum lady with post partum psychosis, which of the following is an important part in her management: Family support 26- case of tension headache (Dx.) 27- most common cause of nont-raumatic subarachnoid hem.: Rupture aneurysm 28- which of the following will indicate recurrent breast cancer: This Q. is repeated so you can search for the choices cuz I can`t remember them. 29- pat. Complain of occipital & neck pain DX: Cervical spond.. 30- true about UC: Increase risk of malignancy 31- true about crohn`s disease: Fistula formation 32- typical case of acute cholecystitis 33- role of surgery in stage C2 colon cancer: Curative, palliative, diagnostic , exploratory 34- epidemic curve:
| P a g e

35- pt. with exertional dyspnea, PND & mild pedal edema (not sure for Ch. HTN & mild pedal edema (not sure for Ch. HTN (no Hx. of HTN in the scenario!!) Valvular heart disease ((no HF in the choices!!)) 36- child with enuresis which investigation is important: 37- indication of tonsillectomy: 38- best Rx. Of sleep apnea: I couldn`t remember the choices but one of them is surgery (vulvo. Surgery) 39- pt. with LBBB on ECG, otherwise normal regarding examination &echo, he will do dental procedure, what is your advice: Endocarditis prophylaxis before procedure ,,,,,,,,,,,,,,,,,,, ,,,,,,,,,,,,,,,,,, after ,,,,,,,,,,,,,,,, No need for prophylaxis 40- what is common CHD associated with endocarditis VSD ASD PDA Tetralogy 41- till birth ,3 died within months ,2 died before their 1st birthday , with 750 p come out & 250 come in what is the birth mortality rate in this city : a. 4 b. 6 c. 8 d. 9 42-old pat. Complaining of abdominal pain , vomiting O/E there is long longitudinal scar in abdomen, on abdominal X-ray there is air fluid level, what is the next step: Nasogastric decompression & IV fluid 43- what is the best investigation regarding renal function: Serum creatinine level Inulin!! Level 44- 3w old neonate with projectile vomiting Dx. : Pyloic stenosis
| P a g e

Duodenal atresia Hirschsprung disease 45- female with positive urine pregnancy test at home what next to do: Sreum beta HCG CBC 46- Pict. For zoster, with scenario asked about Rx: Acyclovir Famciclovir 47- other case of zoster (old pat. With back pain then develop vesicular eruption at the same area band like with tenderness) DX. 48- case of prenicious anemia 49- which true regarding pregnancy: Use of anti-thyroid medication increase the incidence of congenital anomaly GERD increase the incidence of IDA I forget the other choices 50- all are primary prevention of IDA except: 51- typical case on ankylosing spondylitis ask about Rx. : 52- old pat. With diffuse hair thinning & loss of eye lashes( he mention at the end of scenario that this pat. Admit to pull out his hair in stress) ask about Dx: Trichotillomania (the pattern of hair loss will not be diffuse) Other choices !!?? Vague Q!! 53- typical case of pertussis (ask about Dx.) 54- pat. On (name of drug may be diazepam ) complaining of muscle pain & spasm, also he complain of cubital spasm after removing b.p cuff the cause of his problem is : Hyponatrema Hpokalemia 55- old pat. With tremor , mask face. Dx: Parkinsonism 56- which of the following is true regarding alzahimer: Brain atrophy will be more in (he means not diffuse atrophy) Sulci widening more in frontal than occibital There is plaque 57- LONG case of SCA at the end he ask about what of the following is best to give:
| P a g e

Penicillin 58- why flu vaccine need to give annually: Viral drift 59- the commonest presentation in abrupto placenta is: Vaginal bleeding 60- 43y old female with irregular menses 3m back & 1next to do: US Human chorionic gonadotropin Placental ,,,,,,,,, ,,,,,,,,,,,,,,,,,,, FSH LH 61- newly married couple (6m ) not conceive what to do: 62- case of depressed man after death of his son, he can`t sleep at all for 2days, which drug will for short term: Lorazepam Imipramin 63- child already he is on supplementation, he develop nausea, vomiting & diarrhea. The emesis color is black , what toxicity he suffer from it: Iron 64- boutonniere deformity: PIP flexion with DIP hyperextension). 65- case of perth`s disease what is the appropriate management: Physiotherapy Surgery Non weight bearing for 6m 66- Patient is known case of cervical spondylolysis , presented by parasthesis of the little finger , with atrophy of the hypothenar muscles , EMG showed cubital tunnel compression of the ulnar nerve , what is your action now : cubital decompression . 67- true regarding trichomoniasis: Green , frothy discharge 68- Using the following classification : Risk factor Case (disease) Non case total Present A B A+b Absent C D C+d Total A+C B+D
| P a g e

Relative risk of those with risk factor to those without risk factor is:

r. A/A+B , C/C+D

s. A/A+B t. C/C+D 69- null hypothesis: 70- Rx. Of trichomoniasis 71- adult pat. With mod. Persistant asthma on short acting bronchodilator & small dose inhaled steroid (the rest of scenario I didn`t understand it, but he mention that pat. Need to take drug twice daily!! Increase the dose of steroid inhaler Theiphylin + steroid + steroid 72- diabetic with arterial insufficiency (ask about Dx.) 73- typical case of OA 74- child with congested throught & tonsil with white plaque on erythematus base on tongue & lips , also there is gingivitis (Dx.) 75- case of epiglotitis ask about best next step regarding the management of this case: ICU with . other choices not appropriate for this emergency case 76- other case of epiglotitis ask abo 77- case of flail chest (ask about Dx.) 78- post. Pad sign 79- case of acute pancreatitis next step: Total parentral nutrition Jejuna nutrition 80- treatment of anaphylaxis: Epinephrine 81- Prolong use of vasoconstrictive nasal drop will cause: Vasomotor rhinitis Rebound phenomena Good luck

SLE Exam 24/12/2011 By: dr. Rawan Alshreem

This is my exam in saturday 24-12-2011 ( sorry i chouldn't remember all Q but these will help ) ---------------------------------------------------------| P a g e

1Q)difination of case control study ? 2Q)regarding prevention of plaque : rodent eridication 3Q-pregnant with HIV , the most accurate statment regarding risk of transmission of HIV to the baby : Placenta Through cord blood Contamination of the hands Breast feeding ?!!!! 4Q-pregnant with uterine fibroid , has no symptoms only abd. Pain , US showed live fetus ,,,,, What is the appropriate action to do: Myomectomy Hysteroectomy Pain management Pregnancy termination 5Q)10 months old baby came to the clinic with his mother , she breastfeed him 3 times a day ,, she is known cace of epilepsy on phenobarbital,,,,,, What u going to tell her : Stope breastfeeding immediately Weaning over 2 weeks period Breastfeed after 8 h from taking the drug Respond to what the mother and child wish 6Q-Elbow fx , on lateral xray : Post. Fat pad sign 7Q-Pt came with eye pain, watery discharge and light sinsitivity Eye examination showed corneal ulceration. Her symptoms are frequently repeated . Which of the folowing is triggring for recurrence of her symptoms: Dusts Hypertension and hyperglycemia Dark and driving at night Ultraviolet light and stres( 100% sure ) 8Q- p.t taking a medication , came to the ER suspecting she has overdose of her medication, her symptoms ( convulsion, dilated pupil, hyperreflexia and strabismus) the medication is: TCA ( 100% sure ) SSRI Hypervitaminosis
| P a g e

9Q-Pt complain of hearing voices from the microwave and refrigerator Visual hallucination Auditory hallucination ( 100 % sure ) 10Q)Old retired man having ansomnia only . Has no symptoms related to anxity or depression .. U will give him : Diazepam ( sure 100%) 11Q-pt take cephalexin after tooth extraction for days After that he develop profusre , green foul smilling diarrhea with low grad fever . He has tachycardia and mild abdominal dist. Sigmoioscopy showed white mucosal patches , what is the most ttt for this condition? Clarythromycine Vancomycine Cephalosporine Lineozides 12Q-pt with COPD came with couph , wheezing and greenish sputum The causative organism: H.influanza Strep.pneumonia Chlaymedia Mycoplasma pneum. 13Q-what is the most effective measure to limting the complications in COPD: Pnumococcal vaccination Smoking cescation 14Q-pt with hypertrophic subaortic stenosis ,, want to do tooth extraction ,, regarding to development of endocarditis : High risk 50% no need for prophylatcic antibiotics Post procedure antibiotics are sufficient Low risk 12% 15Q)25 years old female came complaining of difficult hearing , she mentioned that their a family history of early oncet hearing loss ( her grandmother) Oto. Exam was normal .. Weber and rinne tests result in ( bone conduction is greater than air conduction ) ... Next action is : Refer her for aid hearing Tell her there is no avalible ttt Refer her to otolaryngologist 16Q)Old man came complaing of progressive hearing loss , it is mostly profounded when he listining to the radio, he does not has any symptoms
| P a g e

like that before Weber and rinne tests result in bilateral sensorineural hearig loss.. Diagnosis: meniere's disease Otoscelerosis Noise induced deffnese Hereditary hearing loss 17Q)Prophylactic antibiotics after appendectomy Cephatrixone Metronidazole 18Q)Pt came with cough , wheezing , his chest ascultation revealed monophonic sound , on xray ther is patchy shadows in the upper lobe+ low volum wirh fibrosis ,, he lives in a crowded place .. What is the injection shuold be given to the pateint's contacts : hemophe.influanza type b Immunoglobuline Menngioc. Conjugated C Basil calament .... !!? 19Q)The most common cause for chronic irrigular rectal bleeding is: Diverticulitis Hemorrohids Colon cancer UC 20Q)What is the term used to describe the increase of the frequency of the menstrual cycle: Ammenorrhea Dysmenorrhea Menorratogia Hypetmenorrhea Polymenorrhea 21Q)45 years old female came to ER with acutely swollen knee + ballotment patella .. The most important to do is: MRI of the knee Aspiration Complete blood count Rhumatoid factor 22Q)Pt came with a history of about 12 dayes duration severly red , swollen painful first metatarsophalangeal joint.. He is hypertensive with inverted T wave on ECG ... The most appropriate meaure for diagnosis: CBC
| P a g e

Uric acid level Troponin level C-reactive protein 23Q)Adult P.t recived a vaccine ( i don't remeber the name ) After that he complain of itching , tachycardia and SOB What is the ttt? IV hydrocortizone 500 mg SC epinephrine 24Q)Pregnant on iron supplementation throughout her pregnancy for her anemia , now she come complaining of weakness and easy fatigability Her Hemoglubin 7 , MCV 60 .... What is the diagnosis? Iron def. Anemia Hypothyrodism Vit B12 def. Beta thalassemia 25Q)Picture ( diagram related to BMI ... So clear ) With a scenario of a female with BMI 32.6 Normal Overweight Obese Morbid obesity 26Q)Picture ( xray for intestinal obstruction ) With very clear scenario and descreption .. The Q about what to do ? Remove the obstruction ilues management Intest. Decopression 27Q)Pt has a scaly hypopigmented macules on the chest and arms They seem even lighter under the sunlight,,, what is the ttt? Topical steroid Na selinum Topical antibiotics Oral antibiotics 28Q)Repeated Q about baby who can name 4 colors .... His Age : 48 months ( 4 years ) 29Q)Pt came with left lower quadrant pain + fever and vomiting On examination there is left lower quadrant tenderness with localized rebound WBC 17.000 .... What is most likely diagnosis? Diverticulitis Granulomatouse lesion of crohns
| P a g e

Intestinal ischemia Sigmoid vulvolus 30Q)Pt came after fight ( gunshot ) there is a pice of the omentum coming out from the wound . Vital signs ( HR 98 , BP 130/80, RR 18 ) .. What is the best action to do ? CT DPL Fast us Wound exploration Scheduled laprotomy 31Q)Which of the following breast mass is bilateral : Paget disease Lobular carcinoma Mucinouse carcinoma 32Another Q about which breast mass present with bloody discharge ? ( i didn't remember the choices , sorry ) but u should read about it 33Q)Most Dangerouse sign during pregnancy? Vaginal bleeding 34Q- recurrence if clef lip in next pregnancy : 4% 35Q- regarding perotinitis which is true : chemical erosion 36Q- Why influenza vaccine given annually : viral antigenic drift 37Q- ECT Indicated in : sever depression with psycho motor retardation 38Q- clear scenario about appendicitis 39Q- repeated Q regarding newborn clavicular fx 40Q- repeated Q one statement is true regarding head and neck injury 41Q) Child came with his parents to the clinic , their parents said that their son looks bigger than the other children on his same age His BMI 34 ... His w.t and h.t on the growth chart is greater than his age Your advice will be: Life style modification Decrease fat intake 42Q) repeated Q about the puberty of the females earlier than the males By 2-3 years earlier ( true ) 43Q) the most common cause of epistaxis in children is: Nasal polyps
| P a g e

Self induced 44Q)female patient presented with migraine headache which is pulsatile, unilateral , increase with activity . Dosn't want to take daily medication. Which of the following is appropriate: Bio feedback Triptan BB CCB 45Q)old man with bilateral knee pain and tenderness that increase with walking and crepitation relieved by rest: OA 46Q)lady c/o headche bandlike pain tension headache 47Q) child came with generalized body swelling, fever , dark urine with decrease urine output ,,, what is the most useful investigation for diagnosis: CBC Renal function test Abd. US Urine sedmintation test 48Q) one of the folowing manifest. As croup: Forigne body Pneumonia Common cold Asthma 49Q) all live vaccine except : Hepatits B 50Q)COPD patient with emphysema has low oxygen prolonged chronic high CO2, the respiratory drive is maintained in this patient by: Hypoxemia 51Q) clear scenario of varicocele ( bag of worms scrotum ) 52Q) repeated Q about refering pt with scoliosis at which degree 20 53Q) relation of indirect inguinal hernia to the spermatic cord anteromedial 54Q) scenario of glucoma in old pt ,, what is the best ttt? Acetazolamide + pilocarpine ( sure 100% ) 55Q) clear scenario of keratitis .. on examination there is dendritic ulcer: Herpes simplex keratitis -------------------------| P a g e

Good luck .. wish u all the best :) SLE Exam 20/12/2011 By: 1- What is the true about obstructed labor : a- Increase with antroposterior position b- Related to polyhadramnes . 2- What is the true about antepartum hemorrhage : a- Indication to vaginal examination . 3- What is the correct about unstable angina : a- Same drug that use in stable angina . b4- What is the side effect of steroid on the eye ? a- Glaucoma . b- Cattarect . c- Keratoconus . 5- What is the main side effect of silver salazin in burn : a- Acidosis . b- Skin discleration . 6- Scenario for COPD 7- Scenario for TB . 8- The antidote for organophospherous is : Atropine . 9- What is the true about appendicitis in elderly : 10- The best investigation for acute diveticolitis is : a- US b- Barium enema . c- CT d- Colonscopy e- Sigmidscopy 11- Degree of scilosis that referral to orthopedic clinic : 20 12- In infant with bleeding problem give him : Vit K 13- Defenciy of B1 called : BeriBeri 14- In DM : a- DR4 b- DR5 c- DR7
| P a g e

d- DR8 15- Pap smear 16- Patient with history of AF + MI , the best prevetion for stroke is : ? a- Warfirin b- Surgery procedure c- Shunt 17- Profeational player came with history of truma on the lateral side of left knee , on examination there is swelling in the medial aspect of left knee , the diagnosis is : a- Medial collateral ligament spasm . b- Lateral collateral ligament spasm . c- Medial meniscus tear . d- Lateral meniscus tear 18- Food poisoning , group of people came with diarrhea and vomiting diagnosis is: a- Staphiloccous aureus poisoning b- Salmonella poisoning 19- Gram - organism : klepseilla 20- Yellow green discharge from vagina is : Trichomonus 21- In the burn , role of : nine 22- The best description of Case Study is : 23- The best description of epidemolgy is : 24- In Acute pancreaitis there is : a- Psedocust . b- Fistula ( my ejabah ) 25- Typical case for epiglotitis 26- The drug case optic neuritis is : Ethambutol 27- Pain in breast spechaily above the aurola , most common cause is : a- Fibrocystic disease 28- Pregnant devlope sudden left leg sweeling , best manegment is : a- Dopplex Rest Heparin 29- Absoulute contraindication of lactation is : HIV 30- When start lactation : as soon as possible 31- Typical case for specific phobia 32- Typical case for social phobia
| P a g e

33- Agrophobia case 34- Tretment of rash I forget the spell "diliak rash" : a- Typical steroid b- Typical antifungeal 35- Baby with maculopapular rash : Rubela 36- Rhumatoid fever "Typical " 37- In blindeness what is the aerea that affected : Occiptal lobe 38- In polycythemia cause of anemia is : Hypoviscosity 39- In anemia of chronic disease there is : decrese iron and decrease TIBC 40- SSRI 41- Children eat the paper , what is the initial ttt : behivor ttt 42- Case of ectopic pregnancy , site of pregnancy is : FT 43- The abnormal sign in elbow X-Ray is : Posterior Pad sign 44- Case " Ulnar compression " ttt : cubetus decompression 45- Man with hand work by hummer came with pain on elbow diagnosis is : Lateral epicondlitit 46- In pre-eclampsis the main sign is : elvated blood pressure 47- Duration of drug in Rhumatoid fever is : a- 6 years b- 15 years 48- Typical case of "Migrine" 49- Typical case of "Uveitis " 50- th degree of hemorride : heamoridectomy 51- What is the food should avoid hyperlipedia patient : a- Avocado b- Organic meat 52- What is the true for fracture of head and nech 53- Regarding the CPR what is the true : a- 40 % recovery b- Provide 2 breath 54- Patient he had multiple problem in his chest and he lives in crowded area what your action:
| P a g e

a- Immunoglobulin b- H.influnza 55- Patient with Rhumatoid arthritis on hand X-Ray there is swelling what you will do for him : a- NSAID b- Injection steroid 56- Patient came to you with small swelling under his eye , on examination he have inflammation in lacrimal duct , you refer him to ophthalmologist before that what you will give him ? a- Topical steroid b- Topical antibiotic c- General antibiotic 57- The best investigation for kidney function : a- 24 h collect urine b- Creatinine clearance

| P a g e

SLE Exam 28/11/2011


1-Pt . have right stab trauma in his chest on right side .. he came to er councious .. oriented but tackepnic....trachea shifted to the other side.. what is the next step in management? a-Order cxr b-Insert large needle in 2nd intercostal space mid clavicular c-Insert neddle in 5th intercostals

2)Pt with adult respiratory distress syndrome.. he got tension pneomothorax.. what is the propable cause: a-Distruction of lung paranchema b-Negative pressure . c-Oxegen 100%

everything is normal except palpable tip of the spleen.. positive monospot test .. whats your action: a-Send him home b-Empiric antibiotic c-Antivirul d-Observation e-Supportive ttt

| P a g e

of sudden right lower limp pain with numbness and pallor .. ur diagnosis a-Acute thrombus b-Embolus

5)Eldarly pt presented with faver.. by blood culture there is enteroccus facsus ,.. what is the propable sourse of this bacteria 1-Skin 2-Urinary tract 3-Upper respiratory tract

a-Spreding in the blood stream b-Cousing pharyngitis and tonsillitis

7)Elderly pt presented with talangectasia on face with painful nodule of the nose.. what is ur action a-Antiviral b-Antibiotic

8)According to the weight of the pregnant lady: a-The lady shoud get an extra 300 to 500 calorie a day to be normal b-Regardless the habit of the lady, she has to gain 30 lb in her pregnancy and this is the weight of the baby c-In the third trimester the lady will gain wight rapidly and this indicate the the rapidly growing of the baby that may indicate an intervention d-Normaly the lady will gain about 50 lb during pregnancy 9)Pt presented with orthopnea and pnd .. he have a history of mitral stenosis .. there is bilateral basal crepitation what is the dx a-Rt sided heart failure | P a g e

b-Lt sided 10)Adult respiratory distress syndrome Aortic stenosis

SLE November 2011

Best description for Mitral stenosis murmur on auscultation Systolic crescendo decrescendo

Pansystolic Diastolic decrescendo

The heart increase its blood supply by Pulmonary resistnace Dilate coronary artery Constrict aortic artery Dilate IVC

Which most common condition associated with endocarditis VSD ASD PDA TOF
| P a g e

Which condition least common associated with endocarditis

VSD ASD PDA TOF

How you can confirm Factitious fever CBC RhF CXR Heart rate

Treatment of peritonitis(the organism is Bacterioid fragile Clindamycin Mitronidazole .

Long case about migraine (Pt want to treat the headache but she don't want to take daily medication):

Biofeedback CCB b blocker

Migraine case (How to confirm the diagnosis)? MRI

Careful history and examination .


| P a g e

..

Pt. with MCP joint swelling+HTN+ECG show inverted T wave:

Case of gout: Ca pyrophosphate

Na urate

Pneumothorax management:

Insert neddle in 2nd ICS medclavicular line .. ..

Pt. take antibiotic and at end of course he develop diarrhea,what's the cause:

Clostridium.. Clostridium defficile .

HIV pt. with skin lesion show (spindle cells) Diagnosis: Kaposi sarcoma ..

Pt. with headache and vertebral lesion (Moth-eaten),Investigation? Bone scan

Healthy pt. with +ve PPD, no symptoms, normal CXR, Management: isoniazide 6 mon
| P a g e

INH + rafimpecine 6 mon .. .

Elderly pt. with Hx of forceful; vomiting ,Hx of antacid use. diagnosis:

Gastric outlet obstruction

Smoking cessation program most effective with:

Pt. desire Pharmacological Tx Dr. advise pt. to quit Change lifestyle

Side effect of steroid eye drops: Cataract

Pt. with leg pain aggravated by walking and relived by rest Ex: hair loss +cold: Chronic leg ischemia

DVT

Venous insufficiency

Infertility case G3P0 with Hx of pregnancy termination + D&C: Asherman syndrome Sheehan syndrome PCOS

Child with SCA treated completely from UTI, Ex: normal except runny nose, Management:
| P a g e

Prophylactic penicillin

What is true about Pneumococcal vaccination in sikler 11 mon baby: 23 valent should be take

Pt. need prophylactic antibiotic even if vaccinated .

Hx of wheezing and subcostal retraction for 2 days on salbutamol:

Add corticosteroid Thiophilin

Pt. with moderate asthma on b-agonist: Add inhaler corticosteroid .

Child with whitish plaque on teeth, Hx of milk bottle in mouth during night, Dx: Herpitic gengivostomatitis Milk cries Congenital syphilis

..

Child with Hx of URTI present with large epiglottis, On Ex: respiratory distress, management: Endotracheal intubation Endotrachectomy

| P a g e

Child with Hx of malaise, congunctivitis,,whooping cough for 2days: - pertussis - .

Hx of trauma in DIP(finger hyperextention)with palm pain: (incomplateQ)

Extraarticular fracture in DIP Intraarticular fracture in PID Superficial tendon tears Tendon profundus tear Tx of cystic acne and scaring Isotretinoin Retinod

Pt. take one breathe then stop for 10 seconds then take another breathe(I forget the description exactly), type?

Cheyne-stokes Kussmaul's

Pt. with ear pain, congested nose, Ex: red,loss cone reflex, Management: Antibiotic Decongestant .

Hx of hearing loss, tinnitus, vertigo for 2 years, reecent numbness in face: Acuistic neuroma Vestibular neuritis
| P a g e

Common complication of pancreatitis: Psudocyst Phlegmon

SLE exam 14-12-2011 By: Dr.Arwa


Basic science: (2/3)
1) Mechanism of vitamin C in wound healing : a. Epithiliazation b. Aerobic fibroblast synthesis c. Collagen synthesis (my answer) d. Enhance vascularization
| P a g e

e. --- 2) At which stage sapration of chromatoids occur : )a- Metaphase (my answer b- Telophase c- . d-

)Medicine: (12/25
1) The most common side effect of long use of systemic corticosteroids: a.Asthma b.Weakness in pelvic muscles I forget my answer 2) female pt c/o sever migraine that affecting her work,she mentioned that she was improved in her last pregnancy,to prevent that: a. biofeedback )b. propranolol (my answer 3) what drug that improve the survival in CHF 1. digoxin 2. Hydralazin 3.ACEI (name of drug) my answer 4) old man with bilateral knee pain and tenderness that increase with walking and ;crepitation relieved by rest a. RA )b. OA (my answer 5) Patient with dysphagia to solid and liquid , and regurg , by barium there is non peristalsis dilatation of osophagus and air-fluid level and tapering end.diagnosis is a. Osophageal spasm )b. Achalasia cardia (my answer c. Osophageal cancer
| P a g e

_________________________________________________________ 6) The useful excurcise for osteoarthritis in old age to maintain muscle and bone Low resistance and high repetion weight training: a. Conditioning and low repetion weight training )b. Walking and weight exercise (my answer 7) child with low grade fever and congested throat, negative ASO and positive EBV. He has )a. infectous mononucleosis (my answer b. URTI 8) one of the following food is known to reduce cancer )a. fibers (my answer 9) Business man went to Pakistan, came with bloody diarrhea, stool examination showed trophozoite with RBC inclusion, Dx: ) a. Amebic desyntry (entamoebahistlolytica 10) Young patient with pharyngitis, inflammation of oral mucosa and lips that has whitish cover and erythmatous base, febrile, splenomegaly. Dx: (this is infectious )mono a. Scarlet fever )b. EBV (my answer c. HZV 11) lady c/o headchebandlike pain )a. tension headache (my answer 12) Used for treatment of pseudomembranous colitis: )a.Metronidazole (my answer unilateral , increase with activity . Dosn't want to take daily medication. Which of the following is appropriate: )a. Bio feedback (my answer
| P a g e

b. TCA c. BB )14 the respiratory drive is maintained in this patient by: )a. Hypoxemia (my answer b. Hypercapnemia c. Patient effort voluntary ? d. 15) most effective ttt of cluster headach: a. Ergotamine nebulizer b. S/C Sumatriptan )c. 100% O2 (my answer d. IV Verapamil 16) The most common regimen in ttt of uncomplicated community acquired pneumonia: )a.azythromycin.(my answer b.fluroquinlone c.penicillin d.gentamycin 17) pt taking digitalis he developed sudden disturbance in vision yellow )discoloration and light flashes (thats what I remember from the question a.digitalis toxicity b.retinal detachment )(I dont know 18) Female patient complaining of urinary symptoms since one year she took many antibiotics with no improvement on examination mild tenderness on the base of the bladder CT and MRI normal what u suspect: a.Interstial cystitis b.DM c.Candidaalbican d.Urethral injury ___________________________________________________________
| P a g e

19) Pt has ca pt a.Asprin daily b.Angiography ?20) Question about pathophysiology of DKA how the ketones are formed 21) What is the most specific test for syphilis: a.TPI b.FAAT 2)adolescent had pharyngitis then he developed pneumonia what is the most likely organism: Strept. Pneumonia Staph arues 3) pt had history of hypertension and no medication taken he eats a lot of meat with no cholesterol ,high trigelcride, low HDL in which category u well put the pt for risk of IHD: A I forgot B I forgot C High cholesterol Sedentary life d High cholesterol obese High BP

24) pt came with PND and orthopnea an examination he has bilateral basal crepitation and pulmonary edema what is the diagnosis: a.left heart failure
| P a g e

b.right heart failure.

)Family Medicine: (12/15


1) About DM in KSA: a. about < 10 % b. Most of the pt of insulin dependant type c. female more aected with type 2 DM )d. most of NIDDM are obese (my answer 2) 17 y.o,he missed her second dose of varecilavaccine,the rst one about 1 y ago what you'll do: a. give him double dose vaccine )b. give him the second dose only (my answer c. revaccinate from start d. see if she has antibody and act accordingly 3) there is outbreak of difteria and tetanus in community , regaring to pregnant woman: a. contraindication to give DT vaccine b. if exposed , terminate pregnancy immediately c. )d. give DT vaccine anyway (my answer 4) Flu vaccine not given to the baby who is allergic to: )a. egg (my answer 5) pt want to quit smoking you till him that symptoms of nicotine withdrawal appear after a. 1-2 days )b. 2-4 days (my answer c. 5-7 days d. 8- 10 days )6)Patient with DM II with good vision, to prevent eye disease (Retinal back ground to develop is to avoid: a. HTN, Smoking
| P a g e

b. Obesity, Smoking c. HTN, Obesity(my answer) d. ? 7)most effective measure to prevent spread of infection among health care workers &pts in a nursery: a. wash hand befor and after examining each pt .(my answer) b. wear gown and gloves before entering the nursery c. wear shoe cover 8) I forgot the answers ____________________________________________________________ 9) I study done on 10,000 people for about 3 years in the beginning of the study 3,000 developed the disease and 1,000 on the end of the study what is the incidence: a. 10.3% my answer but im not sure ____________________________________________________________ 10)Which of the following is the recommended diet to prevent IHD : A. Decrease the intake of meat and dairy . B. Decrease the meat and bread . C. Increase the intake of fruit and vegetables . (my answer)

Obstetric: (2/5)
1)Patient came to you and you suspect pre eclampisa, which of the following will make it most likely: a. Elevated blood pressure (my answer) b. Decrease fetal movement c. ??
| P a g e

2 uterus is larger than suspected , B-hcg is very high , the doctor diagnosed her as having tumor which is chemo sensitive , what is the diagnosis : A. Ovarian cancer B. Endometrial cancer . )C. Gestational trophoblastic . (my answer 3)the most useful test to detect early pregnancy: a.urine pregnancy test (my answer) there was no serum BHCG in the chooses b.ultrasound 4)pregnant lady in the 7 month on iron therapy came with HB: 7.8 and MCV:60 what is the ?diagnosis a.Iron deficiency anemia b.Megaloblastic anemia 5)Pregnant lady which is hypertensive regarding methyldopa what well u tell her Methyl dopa better then lisnopril )(I couldnt remember the other chooses

)Gynecology: (5/5
1) 30 year old women with cyclic pain ,pain with coitus on examination there was mild tenderness in pelvic examination: )Endometriosis (my answer 2)treatment of gonrehea: )Ceftriaxone (my answer
| P a g e

3)44 lady has previous history of DVT her husband doesnt want to use condom what well u advice her: a.OCP doesnt increase the risk. )b.IUD is preferred in this case.(my answer c.she is unlikely to become pregnant. ?what well u do a.Gastric lavage b.Give her charcot c.only observation 5) what is true about puerperium: a.lekoria stays red for 4 weeks b.epidural analgesia cause urinary retention c.abdominal uterus is not felt after one week

)Psychiatry: (4/5
1) Patient has Alzheimer agitative and aggressive ttt: )Haloperidol (my answer 2)the antidepressant used for secondary depression that cause sexual dysfunction: )A.sertatline(SSRI) (my answer 3)Before giving bipolar patient lithium you will do all of the following except: a. TFT )b. LFT (my answer c. RFT d. Pregnancy test 4)ECT therapy used in : )a.Major depression (my answer
| P a g e

5) Pt covers the tv because he says that they see hem and well split on his face diagnosis: a.SCZ(my answer) 6) A man has excessive worry form germs on his hand a. Specific phobia b.Agrophopia c.OCD 7)Scenario about premenstrual dysphoric disorder. (straight forward and they asked about the diagnosis

Pediatric: (12/15)
1) Infant presented with hemangioma on the back .your management is: a. Intralesional injection of corticosteroids (my answer) b. Topical corticosteroids c. Excise of the lesion 2) kwashikor disease usually associated with : a. decrease protein intake, decrease carbohydrate b. increase protein , increase carbo c. decrease protein , increase carbo( my answer)

the hospital: a. Penicillin b. diphenhydramine


| P a g e

c. OCPs )d. Quinine or Quinidine (my answer e.arabiccoffee

)Maximum spinal height is reached after menarche by how many years ? A. Months . )B. Two years .(this is my answer but I think it is wrong )C. Three years .(according to alqasim questions

a.blood dissemination )b.by causing pharyngitis, tonsillitis.(my answer c.joint invasion.

)a.Muscle biopsy(my answer ? what well u give him )a.prophlacticpencillin(my answer 8)child has allergy to dust what well u advice the family a.keep humidity of the house about.. b.cover his pillow with. c.clean his clothe with warm water 9)child came with history of arthralgia and fever he had past history of URTI with fever On examination there was enlarged liver what is the diagnosis: )a. Rheumatic fever(my answer
| P a g e

)child with congenital; heart disease his parents doesnt know the name of the disease he has peripheral and central cyanosis: a.PDA b.tetrolgy of fallot.

Gen. surgery: (11/15)


1) 20 year old male had been stabbed on midtriceps , . On microscopic examination of this greenish fluid show gram positivecocci in chains: a. Streptococcal gangrene (my answer) b. Chlostrideal gangrene c. Fourniers gangrene d. meningocemia e.

2) One of the following decrease chance of colon cancer : a. b. c. d. Zinc Vit. E Beta carotene Folic acid (my answer) im not sure that its right or not

3) 36 y female with breast mass mobile and change with menstrual cycle , no skin dimple or fathering mammogram is not diagnostic. Your advice is a(my answer) but im not sure b-make biopsy c-fine needle aspiration (there is NO singe of breast cancer. It is Fibroadenoma. Just do FNA to exclude cancer & relive the pt) d-oral contraception 4) Old male with abdominal pain , nausea , WBC 7. What is true about appendicitis in elderly?
| P a g e

a. Ct not usefull for diagnosis. b. WBC is often normal. (my answer) not sure c. Rupture is common d. If there is no fever the diagnosis of appendicitis is unlikely e. Anemia is common BPH. ur next step in management is a. transurethral retrograde prostatectomy b. start on medication (alpha blocker) (my answer) c. open prostatectomy urinary dripping and hesitencyurDx is mild

a. Testicular Ca b. RCC ( renal cell carcinoma ) my answer c. Cystitis 7)bilateral breast mass diagnosis a. ductal carcinoma b. pagets disease c. lobular (my answer) 8) 30 yrspt c/o feeling heaviness in the lower abdomen having pulgepapable at the top scrotum that was reducible and icreasing in valsalva maneuver diagnosis a. hydrocele b. variocele c. indirect inguinal hernia (my answer) d. direct inginal hernia

9) Patient came after deep laceration at the anterior part of the wrist: a. Wrist drop b. Sensory loss only c. Claw hand
| P a g e

)d.Unable to do thumb opposition (my answer )Pt with thyroid mass , rm ,2x2 cm what is most appropriate for Dx : )a- Neck US (my answer but im not sure it could be b )b- FNA (this could be the right answer c- Neck CT d- Surgery )11)Treatment of papillary thyroid cancer: (read about it a. radioactive iodine uptake scan b. surgery 12)senioro about old man came with jaundice in skin and eye , all investigations were normal except for bilurbin and gave value for direct and indirect the direct was high )a. extrahepatic biliary obstruction(my answer b. st step: )a. Secure air way (my answer b. Tourniquet on the arm )a. Lumbar lordosis (my answer b. Parasthesis ___________________________________

)Dermatology: (2/3
1) 15y boy appear patch in rt lower leg these patch is clear center , red in peripheral, no fever no other complain so diagnosis (there was a picture with lesion in the )groin area a-contact dermatitis b-tinea corpora
| P a g e

c- lyme disease )d-psiorosis (my answer which is wrong 2) main ttt of non inflammatory acne is )a-ritonic acid (my answer ;b-clindmycin c-azalic acid d-erythromycin )3) picture of herpes zoster (the same picture

)Orthopedic: (3/3
1) Young adult presented with pain on lateral elbow, tingeling of lateral arm, he plays Squash: a. carbel tunnel )b. lateral epicondylitis.(tenis elbow) (my answer

2) A boy felt down on his elbow , the lateral x-ray shows: a. Anterior Pad sign
| P a g e

b. Posterior pad sign (my answer) c. Anterior line of humerous intersecting the cubilium d. Radial line forming 90 degree with cubilium

little finger , with atrophy of the hypothenar muscles , EMG showed cubital tunnel compression of the ulnar nerve , what is your action now : A. Ulnar nerve decompression . surgical decompression (my answer) B. Steroid injection . C. CT scan of the spine .

ENT: (3/3)
) child with epistaxis.. management: a. Compression on nose and leaning forward (my answer) b. .backward

Ophthalmology: (2/3)
1) Man is complaining that he doesnt see the traffic signs well what is the best way to measure the distance vision: 1 Snellin chart (my answer) ) Infant born with hemangioma on the rt eyelid what is appropriate time to operate to prevent amylopia: a. 1 day b. 1 week (my answer) c. 3 months d. 9 months 3)question about pt had pterygium what well you tell the pt:
| P a g e

-it is malignant I forgot the choices read about the topic

SLE OROMETRIC 14/12/2011 By: dr. Ahmed Alabdulmohsen

alsalam alikum this is new prometric questions, it contain some mistakes hope to solve it together

I only wrote the new questions that I could remember, most of the other questions were repeated Q's, sorry I couldn't remember all the choices.

1)Patient on Lisinopril complaining of cough, what's a drug that has the same action without the side effect: Losartan

2)Patient with stage 1 hypertension (BP: 140/85) and overweight (BMI= 28) , how would you treat him? a-Exercise and weight reduction.
| P a g e

b-Weight reduction alone is not sufficient. c-Dietary pills. d-Antihypertensives.

3)A woman G1P0, 13-week pregnant came to you with a blood pressure of 145/100, she hasn't visited her doctor for years and doesn't know if she has previous Hx. Of HTN, the next visit her BP is 142/98, no protein urea, She exercises regularly 3 to 4 times per week. What's most likely? a-Pre-eclampsia b-Chronic Hypertension c-Pregnancy-Induced hypertension (I chose this one because of the regular exercise but I'm not sure)

4)A patient presented with pain in the index finger, he feels severe pain when holding scissors in the base of his finger on the palmar side, the finger is locked and there is also pain on full extension of the finger.. a-Trigger finger ? (my answer) b-Mallet finger c-Dupuytrens contracture d-Tendon cyst

5)Patient with painful rectal spasms associated with diaphoresis and tachycardia lasting for a few minutes and occurs mostly during the night, what's the cause? a-Ulcerative Colitis b-IBS
| P a g e

c-Proctalgia fugax ? (my answer) d-Gay bowel syndrome (huh?!)

6)An athlete presented with a well demarcated rash in the groin area, how would you treat? a-Cortisone cream b-Antifungal cream d-Antibiotics

7)A boy with rheumatic fever: a-Antibiotic prophylaxis before future dental procedures. b- 2 Blood cultures and presence of Osler nodes are diagnostic according to Duke's criteria. c-Duke's criteria isn't dependable for the diagnosis. d- 1 blood culture + new murmur are diagnostic.

8- patient with a suspected corneal ulcer: a-Cotton debridment and systemic antibiotics. b-Cotton debridment and cycloplegics. c-Burr debridment and (forgot the choices) d-Topical antibiotic, cycloplegic and refere to ophthalmologist. (I chose this)

| P a g e

9-A middle aged woman with multiple sclerosis , complaining of urinary incontinence..she doesn't feel the urge to empty her bladder but urine incontinence occurs.. a-Reflex incontinence b- Stress incontinence c-Overflow incontinence d-Urge incontinence

10)Old male with acute pancreatitis, (high glucose, low Ca)the appropriate nutrition : a-TPN b-Regular diet with low sugar c-High protein ,high ca , low sugar d-Naso-jujenal tube

11)A man notices blood on toilet paper during defecation, persistent rectal discomfort: a-Ulcerative proctitis b-Crohn's disease of the rectum c-Hemorrhoids d-Abscess

12)An old man with heart failure, likes to eat outside, what would you advice him regarding his diet? a-Full 3 meals every day b-Don't add salt to food c-4 g sodium diet .
| P a g e

13)Patient with a scenario going with liver cirrhosis with acsites, diet instructions: a-High carbs, low protein (I chose this one but it could be Na restriction too no?) b-Sodium restriction 14)An old patient undergone aortic femoral poplitial bypass surgery had multiple symptoms after surgery (goes with dementia), what's the Dx? a-Alzehiemer's dementia b-Multiple infarct dementia ? c-Delirium

15)Patient newly-diagnosed with depression, TTT? a-SSRI's b-MAOI's c-TCA's d-Atypical Antidepressants

16)Hopelessness is an early warning sign for: a-Suicide b-Learning disorder c-Bla blab la d-Bla bla bla

| P a g e

17)Patient on Amitriptyline 30 mg before bed time, wakes up with severe headache and confusion, what's the appropriate action? a-Shift him to SSRI's b-Change the dose to 10 mg 3 times daily .. .

18)A parent complaining that his 6 year old boy eats paper and clay, what would you do? a-Behavioral therapy b-Heat CT c-Fluoxetine . 19)An old woman complaining of hip pain that increases by walking and is peaks by the end of the day and keeps her awake at night, also morning stiffness: a-Osteoporosis b-Osteoarthritis ..

20)Child with picture of pneumonia treated with cefotaxime but he got worse with cyanosisintercostals retraction and shifting of the trachea and hemothorax on x-ray, the organism: a-Pneumocystis carnii b-Strep pneuomonia c-Staph aureus d-Pseudomonos
| P a g e

21)A lady 2 weeks post-partum comes to you with her baby, she this he's deformed and evil and that he won't survive for long, what's the Dx.? -Post-partum psychosis

22)A placenta that's positioned on the antero-lateral wall of the uterus, can't be reached by finger through cervical examination: a-Low lying placenta b-Normal lying placenta c-Marginal placenta previa d-Partial placenta previa

23)Baby with emesis, bloody mucoid discharge per rectum, constipated, loud bowel sounds and obstructive picture, your action: a-Barium follow through. ? (my answer) b-Double contrast. . .. 24)4 year old kid keeps spitting his food: a-Reassure b-Endoscopy .

| P a g e

25)Old man with left lower abdominal pain with fever and constipatin, imaging showed decreased the fatty shadows around distal colon, your next step: -Double contrast

26)A young lady with cyclical metromenorrhagia and pain, she has never used any kind of contraceptives before, your TTT? a-NSAIDs b-OCP (my answer) ?? c-Danazol .

27)Patient presented with periumbilical pain , +ve psoas sign: -Acute appendicitis

28)Middle aged man with hematuria and uremia, Rt. And Lt. Quadrant masses palpable "what quadrants?" what's the Dx? a-Hepatorenal syndrome b-Suprahepatoma "what now?" c-Polycystic Kidney disease (my answer) ..

29)Young patient with decreased hearing and family history of hearing loss, ear examination was normal Rene and Weber test revealed that bone conduction is more than air conduction, what would you do? a-Tell him it's only temporary and it will go back to normal. b-Tell him there is no treatment for his condition.
| P a g e

c-Refer to audiometry. d-Refer to otolaryngeologist

30)Patient with pain in the anatomical snuffbox, he most likely has: a-Boxer's fracture b-Colle's fracture c-Scaphoid fracture

31)Old patient with asthma and urine retention due to prostatic enlargement, hypertensive (BP: 180/100) what's the most appropriate drug to control hypertension? a-Labetalol b-Phenalamine c-Propanolol . ( I forgot the name 9ra7a)

32)Patient called his doctor complaining of right back pain and the doctor adviced him to take analgesics, he came to the clinic the next morning saying that the pain wasn't relieved and that he noticed skin changes over the back "vesicles" forming a tight chain like pattern from the back to the abdomen, what's the DX? Herpes Zoster

33)An 18-month old child brought to you for delayed speech, he can only say "baba, mama" what's your first step in evaluating him? a-Physical examination b-Delevelopmental assessment.
| P a g e

c-Head CT d-Hearing test. 34)Old patient with recurrent DVT and SVC obstruction, most likely due to: a-Christmas disease b-Lung cancer ..

35)Most common site of tumor in sinuses: a-Maxillary sinus b-Frontal Sinus c-Ethmoid Sinus d-Sphenoidal sinus

36)Goodpasture syndrome is associated with: a-Osteoporosis. b-Multiple fractures and nephrolithiasis c-Lung beeding and glumerulonephritis ..

37)The most common cause of immediate death in flame burn victims: a-Inhalation of smoke. ? b-Associated injures. ? (most likely) c-Hypovolemic shock.
| P a g e

d-Septic shock. 38)Patient complaining of torso pain after using tan bed, on examination skin on the chest was red, reblenchable and painful: a-1st degree burn b-2nd degree burn c-3rd degree burn

39)In city with population of 15000 people & 105 births per year , 4 stillbirths , 3 died within months ,2 died before their 1st birthday , with 750 moved out of the city and 250 came in.. the perinatal mortality rate in this city : 4 6 8 9

I hope this will help even if a little Best of luck to all of you.kick some *** Best regards,

| P a g e

SLE OROMETRIC 24/12/2011

alsalamo alikom everybody these r some of my exam questions which was on 24th of dec hopefully u can get the benifit :)

1) what is the most common cause of death in patients with Ludwig's angina? a-sepsis b-Sudden asphyxiation c-rupture of the wall

2) 6 yr old with HBsAg his mother has HBV he did not receive any vaccination except BCG he should take: a. Td, Hib,MMR,OPV b. DTB,Hib,MMR,HBV,OPV c. DTB,Hib,MMR, OPV d. Td, Hib,MMR,OPV,HBV

3)which of the following is not a feature of normal ECG:


| P a g e

-P wave is the repolarization of the atria

4)80 yr old in his normal state of health presented with decrease visual acuity bilaterally without any defect in visual field his VA Rt eye= 20/100 VA Lt eye=20/160 fundoscopic exam showed early signs of cataract and drusen with irregular pigmentations. No macular edema or neovasculirization. The appropriate action beside antioxidants and Zn is: a. Refer the pt for emergency laser therapy b. Refere the pt for cataract surgery c. See the patient next month d. No need to do anything

5)Old age female, deep aching pain in the hip, increased early morning and by walking a- Osteoporosis b- Osteoarthritis c- rh arthritis

6) normal puerperium.. a-it lasts for up to 4 weeks b-the uterus can't be felt after the 1st week

7)best test to detect age of gestation is a-LMP b-U.S.

8)a scenario of a patient undergone gastrectomy 1 day back..what's the cause of fever
| P a g e

a-wound infection b-inflammatory mediators

9)a young pt with osteoarthitis..intial management a-strength muscles b-steroid

10)ibuprofen is contraindicated in -HTN -DM -peptic ulcer

11)a pt is complaining of vomiting..in ex there was wavy movement..so most likely dx is -intestinal ob.

12)a pt with AF came with black stool (and i think hypotenstion)..dx is: -ischemic mesntry

13)a short scenario of a pt taking antidepressant and c/o insomnia (and two other things) what r the expected drug he's taking: a-SSRI b-MOA c-TCA

| P a g e

14)An old woman complaining of hip pain that increases by walking and is peaks by the end of the day and keeps her awake at night, also morning stiffness: a-Osteoporosis b-Osteoarthritis

15)old pt c/o bilateral knee pain with mild joint enlargement ESR and CRP normal dx a- Osteoarthritis b- Rheumatoid arthritis c- Gout d- Osteoporosis

15)Patient is known case of cervical spondylolysis , presented by parasthesis of the little finger , with atrophy of the hypothenar muscles , EMG showed Ulnar nerve compression of the ulnar nerve , what is your action now : a- cubital tunnel decompression . b- Steroid injection . c- CT scan of the spine .

16)The initial management for osteoarthritis in a young age pt Strenthining of the quadriceps

| P a g e

17)31 year old Women with cyclic bilateral modularity in her breast since 6 months on ex there is 3 cm tender mobile mass wt u will do next a-FNA with cytology b-mammogram c- biopsy d- follow up for next cycle e-observation

18)a pt c/o deep jaundice which has a progressive course..o/e: the gall bladder was palpable a-pancreatic ca b-acute cholecystitis 19)Well known case of DM was presented to the ER with drowsiness , in the investigations : Blood sugar = 400 mg/dl , pH = 7.05 , what is your management ? a-. 10 units insulin + 400 cc of dextrose . b- 0.1 unit/kg of insulin , subcutaneous . c-. NaHCO . d-One liter of normal saline

20)Female patient with candida most likely has : a- DM . b-SLE

21)Pt with painless thyroid mass..what is most appropriate for Mx :


| P a g e

a- Neck US b- FNA c- Neck CT d- Surgery

2)2Pregnant lady , 34 wk GA , presented with vaginal bleeding more than her menstruation. On examination , cervix is dilated 3 cm with bulging of the membrane, fetal heart rate = 170 bpm . The fetus lies transverse with back facing down . us done and shows that placenta is attached to posterior fundus and sonotranulence behind placenta. Your management is : a.C/S b.Oxytocin c.Tocolytics d.Amniotomy

23)If diabetic mother blood sugar is always high despite of insulin, neonate complication will mostly be: a. Maternal hyperglycemia b. Maternal hypoglycemia c. Neonatal hypoglycemia d. Neonatal hyperglycemia

24)Female with greenish vaginal discharge, red cervix. under the microscope it was a protozoa..Dx: a. Trchimoniosis

25)clomiphene citrate:
| P a g e

a. induce ovulation

26)a case of an old man feels that he's inforced to count the things and he doesn't want to do so.. -obsession -compulsion

27)newborn with fracture mid clavicle what is true: a. Most cases cause serious complication b. Arm sling or figure 8 sling used c. Most patient heal without complications

SLE POROMETRIC 12/2011

1-In active increase transaminase which of the following drugs contraindicated a-rinatidine b-infidipine c-vastatin ????? not sure my answer statin 2- which of the hollowing drug cause , hpertennsive crisis ?? clonidine 3- side effect of silver sulphazidine : a-leukopenia | P a g e

b-skin pigmentation c-acidosis d- electrolyte impalnce answer d 3-side eect of prolong 100% oxygen except : a-retrosternal chest painting b-sizure c- depression ????? answer c 4-prenatal mortality mean : ab- number of stillbirth + first week neonate c-numer of deaths /1000 ???? idont know may answer b 5-stage IIIcolon cancer start chemo therapy : a-As soon as possible b-if lab reasult normalize c-according the pt psychology d- if pt >60 y age my answer a ???? 6- Eating diorder mangement : a-cognetive and behavioral therapy b-pharmacology ???? my answer a>>>??? 7- which of the following treatment should be give in mentinance bipolar : a- valporate b-lithium c-olanzapine ??? other medicatiob i forget it | P a g e

my answer B 8- scenario for child transfer from city to anoter city >>>>and he go to shool he is not good psychology (i miss what he have ) what is the DX : Adjustment disorder 9-pt came with painful rectal spasm , diaphorisis , tachycardia especially at night , DX : a-thrombosed hemorrhoid b- proctalgia fagux (answer c- ???syndrome the answer b 10 - what is the finding in anemia of chronic illnes : a- decrease iron and increase TIBS b-decrease iron and decrease TIBS C-increase iron ????? my answer b 11- secondary amenorrhea a-due to gonadal agenesis -b-sheehan's syndrome c-It is always pathological answer b seehan's syndrome 12-polythcemia vera also associated with : a-ms weakness b- splenomegaly c-?? the answer b 13-case scenarion pt came with chest pain , radiate to jaw , increase with exersize ,decrease with rest DX : a-unstable angina b-stable angina c-prenzmetal angina | P a g e

answer b 14- child ,found meningits in blood >>>i think and he is assymptomatic ,what well you do : a-oral penicillin b-oral rifampacin c-IV ceftriaxone ???? i am not sure answer >>>c 15- female pt , with RTA ,she has bilateral femur fracture >>>like this scenarion , systolic blood pressure 70 >>>what will you do: a-Iv fluid b- blood tranfusion c-??? ??????? not sure answer b

16-female pt ,KCO rheumatic heart , diastolic murmur ,complain of aphasia and hemiplegia , what will you do to find the >>>etiology<<< of this stroke: a-MR angiography b-Non-contrast CT c-ECHO D-ECG E-carotid doppler ??????? im not sure between a,e my answer a 17-patinet improve with antidepressant , suicide risk : | P a g e

a-great b- less c- same ?????d !!!! not sure bet a or b my answer a 18- pt diagnose papillary carcinoma , Mangement : a- surgical resection b???? 19 - which of the following take wiht analgesic to decrease side effect ?? .. a- cimitidine b-psudoephidrine c!!!!!!!! NOT SURE answer A may be H2 blocker can protect stomach from analgesic like NSAID

20 -child with hematuria 15 RBC /hPF , all examination normal ,what is next : a-urine cytology b-cystoscopy c-renal biopsy d- repeat urine for RBCs and protein !!!!! not sure my answer d 21 - scenario i think for TB pt , with upper lung fibrosis , he live in crowded area , : what will give to the contacts a-Himophilus influenza vaccine | P a g e

b-immunoglubulin c-meinongicoccal vccine d- BCG !!!!!! IDONT KNOW MY answer D & i think it is wrong 22-PT, with herpes in vagina , what is ture : a-pap smear every 3 year b-CS delivary if infe >>>>> answer B 23- ORTHOPEDIC : child came withor Toeing-In , set in W shape , this """ what the Dx : a- metatarsus adductus b- femoral anteversion (femoral torsion ) c-??? answer B

NOTE: femoralantiversion Epidemiology Femoral Anteversion is most common cause for Toeing-In More common in girls (2:1) Most common onset ages 3-5 years : Definitions Anteverted hip (Femoral Anteversion) Femoral head significantly anterior to Femoral neck Associated with Toeing-In (normal in young child) Antetorsion used to describe abnormal anteversion Normal hip | P a g e

Femoral head slightly anterior to Femoral neck Retroverted hip Femoral head posterior to Femoral neck Associated with Toeing-Out Mechanism Excessive medial rotation of the femur Normal Femoral Neck Anteversion angles Adults: 15-25 degrees Children Age 3-12 months: 39 degrees Age 1-2 years: 31 degrees Symptoms Standing appearance: "Kissing patellae" Clumsy gait Running appearance: "Egg-Beater" In-Toeing feet ("Pigeon-Toed") Sitting position: "Inverted W" Sitting with hips flexed and internally rotated Does not worsen Femoral Anteversion

24- -1- -SENARION : old pt , e hx of MI 2 weeks back and discharge from hospital <<<came with sudden lower limb pain and numbness ,on ex the limb pale , cold >>the other limb normal what is the DX : a- Acute artery thrombosis b- acute artery embolus c- DVT D - ??? NOT sure bet b,c my anwer is c but i think wrong 25-sickler pt came with painful crisis what is the RX : a- mangement outpatient + analgesic | P a g e

b-hospitalization +analgesic c-reer to 3ry center ????? not sure my answer b 26- PTH high ,Ca low ,creatinine high ,vit d nomal DX : a- vit d deficiency b-chronic renal failure answer b 27- pt with leg or knee swelling >>>>i think , last month have big toe swelling and recive NSAID , and improved Q About gout : a-due to septic deosite b- deopsite due to highsaturated ?????? answer b >>>>>>

28- Allopurinol : a- use in acute phase b-it is uricosuric c-contraindication in chronic renal disease d-decrese uric acid renal stone NOT SURE answer d 29-With asprin overdose a- metabolic acidosis with respiratory alkalosis 30 -parent came with child vomit ahter every feed , normal growth parameter ,what will y do : reassure the parent 31-case scenario plural effusion , cardiac effucsion e low pretein, LDH , >>>>>>i forget THE nomner>>> | P a g e

what is the cause ? a-Tuberculosis b-heart failure answer b 32- normal child ,he want to walking , he have brother dead after walking , what of the following must be excluded before walking ??? a-PDA b-VSD C-hypertrophic cardiomyopathy !!! D-!!! :( !!!! Vague question !!!!! IDONT KNOW aanswer VSD

33- old pt ,she have MI and complicated with ventricula tachycardia , , then from that time recive Buspirone what INX must to be done a- thyroid function b- liver and thyroid c- ??? !!! :( also vage Q iforget the rset of the choises i dont know what the pt have and how investigate 34- at which age child sopke few words a-12m | P a g e

b-24m c-36m

..

SLE POROMETRIC 26/12/2011 Dr.Mohammed Alsharif


- Carvidelot drug is interacted with ..

2 it is correct about alloprinol

3 regarding head and neck injury | P a g e

4 good prognostic factor in schizophrenia

5 null hypothesis

6 metabolic acidosis is

7 statistic about osteoposis in family ( graph )

8 child with cough drooling fever what is ttt

9 abdominal pain for 6-months , constipation , diarrhea

10 high risk for hypertension

11 GERD and diagnosed as paretts esophagitis , complication

12 child in amitryptalline 15-mg , the potential ADR may develope ..

13 child patient that prescribed clozapine by a psychiatrist , which disease you expect ..

14 45-year irritable , excessive warry for 8-months with low apetite and decreased concentration , Dx

15

| P a g e

16 most commo cause of intracerebral hemorrhage

17 correct about kawashoki disease

18 white bleeding per vagina with itching ttt

19 patient with vit. D defiency , which part of epithelium formation will be affected

20 vitamin that should be given for newly born neonate

21 correct about newly born with scapula fracture

22 patient from crowded area came with CXR show apical infiltration , wht you should give to prevent relatives

23 pt with chronic lung diseas , with new pleural effusion , wht is the cause of PE

24 posterior hip fracture , to which site

25 fracture of humerus related to which nerve injury

26 pt child start with waddling gait , wht is appropriate investigation

27 - which carcinomas come in bilateral breast

28 pt come to doctor with genetic hx of colorectal carcinoma , and he want to prevent himself from the disease , what is the best you advise for him | P a g e

29 high risk factor of colorectal carcinoma

30 pt with COPD and DM , newly diagnosed with open angle glaucoma , ttt

31 61-year with depression during 6-months , new diagnosed with IBS , low apetite , less weight , less concentration , Dx

32 pt come to hospital due to feeling of lump in neck without ang thing else , Dx

33 wt is not palpable in the normal neck

34 - related to blunt abdominal trauma

35 unconscious patient in ER , your action during wait your senior

36 pt come only with gasping , do

37 pt come with diplopia , dysphagia , ptosis , Dx

38 correct about ECG

39 graph about female young with 32 kg m2

40 6-year with positive HSsAg , only with BCG , give him

| P a g e

41 question about case control study

42 pt with splenomegaly, lymphadenopthy , positive EBV

43 pt with viral infection , gingivitis

44 correct about infectious monoclonosis

45 regardibg breast carcinoma

46 young with anovulation , hirstuism , dx

47

48 large female, obese , big hand and jaw , which hormone cause this problem

49 correct about DM in Saudi Arabia

50 32-year young with thyroid carcinoma , it is related to

51 - pt with DM2 and she want to dietary change , wht is your advice

52 infant with erythema in diaper site , ttt

53 1st sexual developed in male

| P a g e

54 chromose in polycystic ovary

55 dx for PCOD

56 breast tenderness , nodule , multiple , best diagnosis

57 correct abiut hemorrhoids

58 pt with tender braest , nodule , not related to menses , dx

59 urine spicemen show RBC and cast , dx

60 urgency , dysurea , on flank pain , dx

61 pt with painful 1st metatarsopharangeal , tenderness, swelling, fever, dx

62 pt with scaling non topped in lower limb non scaling , dx

63 female with hirstusim , normal estrogen and abnormal FSH an LH , dx

64 child come to ER with ingestion toxic drug , wht is antidote

65 iron poisoning antidote

66 correct about multiple abdominal trauma

| P a g e

67 - child pt with sore throat , ear pain , fever , with nodule , wht is organism cause this manifestations

68 abdominal pain for 6-monthe related to menses , 2worsen , dx

69 study about lung cancer , take the person according to sex, resident, income, and then divided to tow group depend on smoking , which study is this

70 female with dysfunctional vaginal bleeding , best ttt

71 ttt of herpes simplex virus

72 most of vaccine sored in degree of

73 -

Thanks

SLE Exam,12/2011
| P a g e

By : Dr.Hind
1-Drug use in CHF with systolic dysfunction? a-Nifidepine b-deltiazm c and d -and two drugs from ACEI I forget the name e-the forth choice is one of B blocke

2-Young male c/o pleurisy pain at rt side On EX there is only decrease breath sound tachypnia other wise normal and there is CXR I dont know if it is normal or not But it seems to me normal what will you do? a-discharge pt bez it is only viral plurzy b-discharge him on Augmentine C- I think refer him to pulmonologist the other choice I can't remember 3-Pathology in HSP: arteriole venule capillary 4-Pt 60 yo collapse brought to ER then he awake before collapse he felt epigastic discomfort Now tachycardia BP 100/80 a-leak aortic aneurysm b-perforated peptic ulcer A Neuro 5-aseptic meningitis early will found: a-lymphocytosis I can't remember the other choices sorry but you must read about the CSF analysis in aseptic meningitis and which cells present Definition 6-Kernig's sign:

Psychiatry

| P a g e

7-wt is non hormonal drug use to decrease hot flush in postmenopausal women: paroxitine

8-Battered women: came c/o unrelated symptoms (the correct) 9-Pt fear that alien will land on her backyard and she feel that she will be crazy she knows that this idea is silly: a-Obsessive b-compulsive c-delusion 10-Def of delusion Surgery

11-Diagnosting peritoneal lavage positive when 1000 RBC 50WBC

EPid 12-attributable Risk definition 13-Pt diabetic he has wound in his leg with poor healing, Exudates, no sign of inflammation the hyperglycemia cause poor wound healing by affecting: a-phagocytosis B-stimulate bacterial growth c-decrease immunity OB 14-adolescent female till about the spinal cord will stop after menarche by 24m 38m 15-female with irregular cycle month and absent for two month with heavy bleeding: a-metroohaia | P a g e

b-menorraghe c-menometrogia d-polymenorrhagia 16his dehydration is: 10% 20% 5%

Hind

My SLE exam in 12/12/2011 SIAM 7 Salman Almansour

DoNe By :

with internal rotation .. ur DX ?!!? a- Inferior dislocation d- subclavicle ant dislocation b- subacromal post Dislocation c- subglenoid ant dislocation

e- sub.. ant dislocation

I think with epilepsy it`s posterior, so most likely it`s B .. make sure plz Q2) Mx of somatization ?

| P a g e

a-Multiple phone call c- refer to pain clinic

b- multiple clinic appointment d- antidepressant e- ?????

Q3) pt with severe depression and now he shows some improvement with therapy , the risk of suicide now is : a- No risk b- become greater c- lower d- no change ( same B4 Mx )

" " make sure plz !! same condition ? a- Malignant melanoma d- measles e- nevi b- neurofibromatosis most likely c- hemochromatosis

* NF-2 accounts for only 10 percent of all cases and is characterized by bilateral acoustic neuromas
(masses around the eight cranial nerve in the brain) which causes hearing loss, facial weakness, headache, or unsteadiness, Caf-au-lait spots.

Q5) 2 years old child with hair loss in the temporal area and boggy swelling I think was ? a- Trichotillomania c- Kerion d- favus !! b- Aplasia cutis congenital !!

*Aplasia cutis congenita "Congenital absence of skin," and "Congenital scars") is the most common congenital cicatricial alopecia, and is a congenital focal absence of epidermis with or without evidence of other layers of the skin. * Kerion is the result of the host's response to a fungal ringworm infection of the hair follicles of the scalp and beard accompanied by secondary bacterial infection(s). It usually presents itself as raised, spongy lesions. This honeycomb is severely painful inflammatory reaction with deep suppurative lesion on the scalp. The follicle may be seen discharging pus. * Favus (Latin for "honeycomb") is a disease usually affecting the scalp but occurring occasionally on any part of the skin, and even at times on mucous membranes.

a- Anorexia Sorry .. not sure about it !!

b- digitalis

c- clomiphine citrate

d- OCP

| P a g e

Q7) female her height is 10th percentile of population , what u will tell her about when spinal length completed ,after menarche ? a- 6m b- 12 m c- 24 m d- 36 m Mmm ?! I think it`s from 2 to 3 years so, both C and D are correct , I chose d bcz he said completed so I chose the , but I think it`s C .. upper range Q8) female middle age with multiple sclerosis , complaining of urinary incontinence .. a-Reflex incontinence b- stress incontinence

c- overflow incontinence d- urge incontinence I think it`s A but i`m not sure read about urinary incontinence in multiple sclerosis .

a- Sodium chloride b- I.V heparin & spiral CT for pulmonary embolism c- albumin 20 % d- ..

a-Antibiotics

b- H2 blocker c- depend on the pt situation d- total gastroectomy

e- sucralfate

I think it`s C but i`m not sure I asked specialist, he said I will go with but omeprazole is effective !! . Bacteroides fragilis, so u will give : a- Amoxicillin Sorry .. not sure about it !! SaLm@n7 Q12) which of the following is Fick method to assess cardiac output ? a-B.P I think it`s B diaphoresis ? a-IBS c- UC b- gay bowel syndrome d- Proctalgia fugax | P a g e b- o2 uptake d- carbon dioxide in venous blood c- carbon monoxide in blood e- pco2 b- erythromycin c- doxycycline d- gentamicin

Proctalgia fugax (or levator syndrome) is a severe, episodic, rectal and sacrococcygeal pain. It can be caused by cramp of the pubococcygeus or levator ani muscles. It most often occurs in the middle of the night and lasts from seconds to minutes, an indicator for the differential diagnosis of levator ani syndrome, which presents as pain and aching lasting twenty minutes or longer.
think the scenario is going with delirium . MX ? a-Valum b- call one of the family to stay with him c- put him in quite and dark room d- haldol till the symptoms abate e- another drug start with Re.. till the symptoms abate . Sorry .. not sure about it !! Q15) one of the following is component of TOF ? a- ASD d- aortic stenosis b- VSD e- tricuspid stenosis c- Lt ventricular hypertrophy

s brother died suddenly while he is walking to his work bcz heart problem , everything in the examination of this boy is normal no murmurs , equal pulses in all extremities what u should exclude in this pt before he participate in this activity ? a- ASD c- VSD b- bicuspid valve d- hypertrophic cardiomyopathy

a- Caffeine b- OCP most likely c- tamoxifen but i`m not sure if it was one of the choices but danazol, bromocriptine were not of the choices d-.. *The most comprehensively researched endocrine treatments for mastalgia are danazol, bromocriptine and tamoxifen .. Q18) 5 years , febrile , develop tonic clonic convulsion what u will give him ? a- Phenytoin c- carbamazepine Sorry .. not sure about it !! Q19) treatment of acne rosacea ? a- Doxycycline b- erythromycin c- gentamicin b- phenobarbitol d- diazepam not sure if it was one of the choices

a- Hydralazine

b- magnesium sulphate

c- other drugs | P a g e

Q21) pregnant lady develop HTN , drug of choice of HTN in pregnancy is ? a- a-methyl dopa c- thiazide b- Hydralazine d- b-blocker

Q22) pregnant lady drink alcohol , what u will tell her ? a- Can cause fetal alcohol syndrome which include mental retardation , hyperactivity , abnormal facial feature b- just tell her to decrease the amount c- no effect of alcohol on baby Q23) chemoprophylaxis of vibrio cholera ? a- penicillin V b- gentamicin c- tetracycline

a- 2 h

b- 18 h

c- 2 d

d- 6 d

e- 8 d !!! From Wikipedia

Bioavailability 72% (peak at 6-8 hours) Half-life 1-3 days (acute) 4-6 days (chronic) Q25) which of the following is true about allopurinol ? a- It`s contra indicated in acute gouty arthritis b- increase incidence of uric acid stones c- xanthine . Inhibitor MOA d- u can`t use it with colchicine I think it`s C Q26) child fall down from bu normal, mental state not change no signs of skull fracture, what u will do next ? a- CT of brain d- observation I think it`s A b- MRI e- skull x-ray c) neurosurgical consultation

Q27) what is the cause of death in ludwig angina ? a- Dysrythmia c- pneumonia Q28) a- Dilatation in the atrium with chamber hypertrophy b- Dilatation in the ventricle with chamber hypertrophy b- asphyxia d- wall rupture

| P a g e

c- atrium dilatation with decrease pressure of contraction d- ventricle dilatation with decrease pressure of contraction . I think it`s A

Q29) at term of pregnancy which of the following change ? a- Tidal volume b- total lung capacity c-.

eveloped blanchable tender rash .. i`m not sure if there is blister or not ? a- Prodromal b- 1st degree I think if there is blister 2nd degree , but if there is no blister 1st degree Q3 a- HBS ag I think it`s D Q32) pregnant lady healthy except swilling lip with bleeding I think from lips what is it ? a- ITP !! b- tumor c- b- Hbc ag c- anti HBe d- anti Hbc ab c- 2nd degree d- 3rd degree burn .

a- Hypervitaminosis I think it`s B !! not sure SaLm@n7

b- iron overdose

c-.

Q34) the strongest reversible risk factor for stroke ? Q35) the strongest risk factor for stroke ?

a- HTN

b- smoking

c- AF i`m not sure if it was one of the choices ?

: Q36) the wound stay in its primary inflammation until ? a/ epithelial tissue formation | P a g e

b/angiogenesis c/when the wound clean d/ eschar formation correct insh allah .

. .. . ... SIAM 7 DoNe By : Salman Almansour

#..everything..will be {OK} in the end..{if it's not ok , it's not the {END}

SLE EXAM 25-12-2011 By : Dr.Reem A. AL-Shammery


I will try my best to remember the Qs :)
| P a g e

Q-Breast feeding mother she said I did not take my MMR vaccine what your advice ? Q-old man having pelvic pain worse by movement even at night when want to sleep still . OA having the pain ?-Q-child I forget how old is he but i am sure he is less than 2yrs he came with !!!!!! The peripheral blood film shows ..cresend shape cells .. What is the ongoing management ??
Q-child come with...O/E swelling epiglottis

Q-best non pharmacological ttt of OS ?---quadriceps ms exercise


The answer of the second q ---was wt exercise Q-old man post MI best drug for antiarrythmia ? Q-best management for epistaxis ?--press on the soft part of the nose and leaning forward Q-about elderly with appendicitis ? --less risk for perforation than young pts (that was my answer make sure plz ) -usually normal WBCs -no fever exclude appendicitis in elderly
-

-Appendicitis in elderly mask intestinal obstruction Q-pts alcoholic and smoker having white patches on the tunge non painful but when touch bleeds ?-- lukoplakia ( this was my answer ) Q-male pts having penial ulcer. ... I forgut the rest !the q was how to investigate -take biobcy -dark filed microscopy (this was my answer )

| P a g e

Q-femal having vaginal discharge white and microscopically revels sphudhyphe what is the best drug ?---metronadazole Q-19yrs old female having an infant 4mon old and dose not want to become pregnant soon , she is breast feeding him and pregnancy test b-hcg was negative? --reassure and aske for her contraceptive counseldation .( I hope it is the correct answer ) Q-female pts she tells that she hear some one talking to her ?--auditory hallucination Q-old man diabetic dose not having any significant medical problem before but by repeated BP measurements in his visits it was 138/ .. What you will do ? -nothing -add ACEI (this was my answer ) Q- in newborn what the most important drug to give to prevent bleeding ?--vit.K Q-pts having from months pain pre defecation that releaved by defecation he is having loose , ther is no bleeding not bloating <<<<?.. --IBS (This was my answer make sure plz) -UC Qpart of the proximal large intestine and connect them now he is complaining from SOB and fatigue what you are thinking of ?

-folate deficiency -vit B12 deciency .. Q-rt lunge anatomy ? --2pulmonary vanes (this was my answe) -2 sser -8segments -...
| P a g e

Q-indication for fetal distress ? -- late deceleration (this was my answer ) -early deceleration -.... Q-30yrs old male with diculty in breathing hyper resonant...?- spontanuse pnumothorax (this was my answer ) Q-the best for breast milk feeding ?---is breast feeding
Q-Gualine-Barrie syndrome<<.

-asending paralysis -descending paralysis Q-in female the N-Gonnorea in which part of the reproductive system ? -the cervix Q-which of the fowling indicate progress of labor ? -frequency of the contraction -strength of the contraction -descent of the presenting part(this was my answer and I am sure it is correct ) -I forgut the last one but there was nothing about the dilation of the cervix .
Q-typical clinical feature of EBV infection at the end they said by investegation result EBV +ve what is the dx? -infectious mononucleosis. Q-Polycystic kidney mode of inheritance?--Autosomal dominant

Q-about Rubella infection...?--arthritis. Q-pt history of IHD and he told you about his diet and he describe what diet he take What you will advice him ?---diet contain vegetables and fruit. Dr.Reem A. AL-Shammery

| P a g e

SLE Exam 12/2011 By: dr.Saher


1/Gonorrhea aect : a. Cervix b. urethra c. rectum d. post fornix

2/least effect on tardia dyskinesia : a. halipridol b.respridol

Subcutenous epinephrine

one week ) complaining of pale, cool diagnosis : a. Acute arterial thrombosis b. acute arterial embolism c. DVT

5/ Treatment of chronic acne : Ritonic acid

a. acyclovir for 3-5 days b. acyclovir and refer for ophthalmology | P a g e

Clomid

a. inhibits the enzyme thyroperoxidase b. inhibit the action of the sodium-dependent iodide transporter

dism ttt: a.b blocker b.propylthiouracil

a.surgical is better than medical ttt b.contraindication to do surgery in reducible hernia c.can cause hypoinfertilty

Physiotherapy 13/ which is true about perths dz : a.unilateral always b.painless

a. GERD

a. RT upper lobe | P a g e

b. Rt middle lobe c.Rt lower lobe d. Lt upper lobe e. Lt lower lobe 16/scenario about ectopic pregnancy Ba.observation b.medical. c.laproscopy d.laprotomy. 17/eect of niacin is : a.decrease uric acid . b.hypoglycemia c.increase LDL d.increase HDL e.increaase triglyceride 18/pt with pneumococcal what u will give : a.Pneumococcal & influenza now b.pnemococcal now c.influenza now

: a.US b.non contrast CT of abd & pelvis 20/RT lung anatomy : a.2 veins b segments | P a g e

21/pt with DM and obese ,plane to reduce his wt is : a.decrease calori intake in day time b.decrease calori and increase fat c.decrease by 500 kcal/kg per week d.decrease 800 per day 22/Romberg sign lesion in : a. dorsal column b. cerebellum c. visual cortex 23/RA nodule in proximal interpharengeal joint : Bouchards nodules

a. US b .LMP l Hb in sickle cell disease : Hydroxurea a. most common in postmeneposal women b.adolesent 27/which of the following is normally not palpable : a. thyroid gland b. parotid gland c. sublingual gland d. cervical node e. hyoid bone

| P a g e

US

a. drainage and oral Abx b.oral Abx 30/cholesterol increase In : a.egg white b.chicken c.avocado d.orange meat 31/target lesion are found in erythema: a.multiform b.annular. c.nodosum d.marginatom inx : TSH normal , T4 normal . ca low , pho high the cause is : a . uncontrol hypothrodism b. primary hypoparathyrodism c. secondary hypoparathrodism 33/angioedema due to use of : B blocker 34/fetal distrees in : a.early deceleration b.late deceleration

| P a g e

SLE Exams,January 2012


\o diplopia RT side eye lis ptosis and loss of adduction of the eyes and up word and out word gauz !! reacting pupil no loss of visual field -Something like that !! The option : Faisal palsy Oculomator palsy of the rt side Masynia gravies !! 2 )The coz of high mortality in pregnant female !: Sphillis Toxoplasmosis Phenocchromctoma -------- forgat ! ANSWER : toxo but am not sure ! -3)Ummm a child have drink corrosive material and came to the er look not well drooling What u r management : Give 2 cup of milk Lavage Establish airway | P a g e

Ask about the crosive material it alkli or acidic ! ANSWEr : NO FREAKIN IDEA !!

Immunization Teaching individual how to protect them self Shi zy keda !! :s Answer : -_Bed reidden pt he have confusion and fever blood culter shows enerocoucas From where : Pneumonia Uti I forgot the rest wallah ! I answer : uti not sure 5)Which tocxisiaty u will rash to the baby to hospital A.S.A.P : Tac toxisoaty Qunini toxsiaty W other I dont recall it :s

using ! but still she have this irration It was waxy with grayish "something" : Atopic dermtisist Conact dermtisiis Linch sipmplex Linch . Something :'\ Answer : who knows !!!!!!!!! | P a g e

7)There is also other derama q was as hell !! I dont know any of them ?

Lucman test : in ortho Testing for what ligment ! Ttt of h. pylory : I pick Omoprazol , clinadmycin , w other :s I dont recall the answer wallahi ! 8)Pt have travel to kinnia and he resived blood transfusion there now he c\o sore throt and generalized lymphonapathy and tender spleen and hairly lucoplika ! HIV Lypomna No EBV from the answers And I think its hiv !

But over all u should read all the QS from qassim uni collection and check the answers

| P a g e

SLE Exam 2012


.. .. finger}, atrophy of hypothenar 1-patient has numbness over distribution of ulnar nerve { little and ring muscles ..ttt a-Release of ulnar tension

2--Mallory-Weiss Tear common in pregnancy <<<% i dont remmeber answer 3-all tr ab Ecg ex <<vit d+ca<<< 4- -osteoprosis wt vaccin give <<< 5-child for hepatitis mother he also has hsa +ve HSAg wt antibiotic<<<<6-red eye long hx then dirrhea diagram <<<7-diabetes+nephropath .<<<.<<<<<8-when neuropath start?5year 9-cushing syndrome easy 10-steo arthritis bilateral cribitation oa.or..ra<<11-knee swelling cripitation <<<decrease air entry <<<12-pnemothorax 13bilateral breast lesion Holding bearth spell or holding ..wich of the following is true -14A)mostly occurs between age 5-10 B)increase risk of epilepy C)a known precipitant cuz of generalized convulsion my answer D)diazepam may decrease the attack ears (parotid gland y.o child with h.o fever and swelling of the face ant to the both -15) 15 meningitis / labrynthitis / orchitis<< enlargement) what is the most common complication tension headache << 16)girl with band like headache increase with stress and periorbital , twice / week / migrin / cluster organism<<<-cat bit 17)apendicitis gun shot throy abdome wt prophlax antbiotics Antero lateral or supralatera << 18)Relation of indirect hernia | P a g e

OTITIS MEDIA CASE<<<<

19)child with ear dischage ectopic pregnancy !! hiv -ve 2nd secreeniCHGng for cx cancer<<20)vaginal discharge :Young female with whitish grey vaginal discharge KOH test ?? smell fish like diagnosis is a. Gonorrhea b. Bacterial Vaginosis c. Trachomanous Vaginalis 21)treatment major depresssion in child for child wet her bed 5 y/o<<<treatment stwk,3month,1 year<<22)clopoma oberation 23)otitis externa treatment 24)fall dowen on out streach hand <<<<colis!!fracture or smith or 25)Abnormal radiological sign in lateral x ray of elbow after falling Posterior pad sign - ADH my answer 27) ve productive value + ve peple become +ve+ ve peple becomy +ve+ ve peple become really -ve+ 28)Positive predicitive value : Definition " ev test+ & C " pt who has high Risk factor << Answer was 29)Most effective method for health education Mass media / Group discussion / Internal talk (true

HX+INVESTIGATION +VE<<<QUSTION ABOUT GOUT ) :

| P a g e

A copy of dr- Mona Edwan Exam that was posted in the group 8-1-2012 1- In active increase transaminase which of the following drugs contraindicated rinatidine infidipine vastatin not sure my answer statin

-which of the hollowing drug cause , hpertennsive crisis Clonidine

-side effect of silver sulphazidine: a-leukopenia b-skin pigmentation c-acidosis d- electrolyte impalnce answer d

-side eect of prolong 100% oxygen except: a-retrosternal chest painting


| P a g e

b-sizure c- depression answer c

-prenatal mortality mean: a-number of stll birth<20 WEEK gestational age b- number of stillbirth + first week neonate c-numer of deaths /1000

idont know may answer b

-stage IIIcolon cancer start chemo therapy: a-As soon as possible b-if lab reasult normalize c-according the pt psychology d- if pt >60 y age

my answer a

-Eating diorder mangement:


| P a g e

a-cognetive and behavioral therapy b-pharmacology my answer a<<<

-which of the following treatment should be give in mentinance bipolar: a- valporate b-lithium c-olanzapine other medicatiob i forget it my answer B

-scenario for child transfer from city to anoter city >>>>and he go to shool he is not good psychology (i miss what he have ( what is the DX: Adjustment disorder

-pt came with painful rectal spasm , diaphorisis , tachycardia especially at night , DX: a-thrombosed hemorrhoid b- proctalgia fagux (answer c- ???syndrome
| P a g e

the answer b

- what is the finding in anemia of chronic illnes: a- decrease iron and increase TIBS b-decrease iron and decrease TIBS C-increase iron

my answer b

-secondary amenorrhea

a-due to gonadal agenesis -b-sheehan's syndrome c-It is always pathological

answer b seehan's syndrome

-polythcemia vera also associated with: a-ms weakness b- splenomegaly c| P a g e

the answer b

-case scenarion pt came with chest pain , radiate to jaw , increase with exersize ,decrease with rest DX: a-unstable angina b-stable angina c-prenzmetal angina

answer b

-child ,found meningits in blood >>>i think and he is assymptomatic ,what well you do: a-oral penicillin b-oral rifampacin c-IV ceftriaxone

i am not sure answer >>>c

-female pt , with RTA ,she has bilateral femur fracture >>>like this scenarion , systolic blood pressure 70 >>>what will you do: a-Iv fluid
| P a g e

b- blood tranfusion c-

not sure answer b

-female pt ,KCO rheumatic heart , diastolic murmur ,complain of aphasia and hemiplegia what will you do to find the >>>etiology<<< of this stroke: a-MR angiography b-Non-contrast CT c-ECHO D-ECG E-carotid doppler

im not sure between a,e my answer a

-patinet improve with antidepressant , suicide risk: a-great b- less


| P a g e

c- same d

!!!! not sure bet a or b my answer a

-pt diagnose papillary carcinoma , Mangement: a- surgical resection b

- which of the following take wiht analgesic to decrease side effect .. a- cimitidine b-psudoephidrine c-

!!!!!!!! NOT SURE answer A may be H2 blocker can protect stomach from analgesic like NSAID

| P a g e

- a-urine cytology b-cystoscopy c-renal biopsy d- repeat urine for RBCs and protein

!!!!! not sure my answer d

- scenario i think for TB pt , with upper lung fibrosis , he live in crowded area :what will give to the contacts a-Himophilus influenza vaccine b-immunoglubulin c-meinongicoccal vccine d- BCG !!!!!! IDONT KNOW MY answer D & i think it is wrong

-PT, with herpes in vagina , what is ture: a-pap smear every 3 year
| P a g e

b-

<<<<< answer B

-ORTHOPEDIC : child came withor Toeing-In , set in W shape when walk both feet and knee inward with 20 degree , both femur inwarr """ what the Dx : a- metatarsus adductus b- femoral anteversion (femoral torsion( c-

answer B

NOTE : femoralantiversion Epidemiology Femoral Anteversion is most common cause for Toeing-In More common in girls (2:1( Most common onset ages 3-5 years

:Definitions
| P a g e

Anteverted hip (Femoral Anteversion( Femoral head significantly anterior to Femoral neck Associated with Toeing-In (normal in young child( Antetorsion used to describe abnormal anteversion Normal hip Femoral head slightly anterior to Femoral neck Retroverted hip Femoral head posterior to Femoral neck Associated with Toeing-Out

Mechanism Excessive medial rotation of the femur Normal Femoral Neck Anteversion angles Adults: 15-25 degrees Children Age 3-12 months: 39 degrees Age 1-2 years: 31 degrees

Symptoms Standing appearance: "Kissing patellae" Clumsy gait Running appearance: "Egg-Beater"
| P a g e

In-Toeing feet ("Pigeon-Toed(" Sitting position: "Inverted W" Sitting with hips flexed and internally rotated Does not worsen Femoral Anteversion

- - - -SENARION : old pt , e hx of MI 2 weeks back and discharge from hospital hrs prior to his presentation >>>came with sudden lower limb pain and numbness ,on ex the limb pale cold >>the other limb normal what is the DX: a- Acute artery thrombosis b- acute artery embolus c- DVT D -

NOT sure bet b,c my anwer is c but i think wrong

-sickler pt came with painful crisis what is the RX: a- mangement outpatient + analgesic b-hospitalization +analgesic c-reer to 3ry center
| P a g e

not sure my answer b

-PTH high ,Ca low ,creatinine high ,vit d nomal DX: a- vit d deficiency b-chronic renal failure

answer b

-pt with leg or knee swelling >>>>i think last month have big toe swelling and recive NSAID , and improved Q About gout: a-due to septic deosite b- deopsite due to highsaturated

answer b <<<<<<

-Allopurinol :
| P a g e

a- use in acute phase b-it is uricosuric c-contraindication in chronic renal disease d-decrese uric acid renal stone

NOT SURE answer d

-With asprin overdose a- metabolic acidosis with respiratory alkalosis

- parent came with child vomit ahter every feed , normal growth parameter ,what will y do: reassure the parent

-case scenario plural effusion , cardiac effucsion e low pretein, LDH <<<<<< i forget THE nomner<<< what is the cause a-Tuberculosis b-heart failure

answer b
| P a g e

-normal child ,he want to walking , he have brother dead after walking what of the following must be excluded before walking

a-PDA b-VSD C-hypertrophic cardiomyopathy!!! D!!!-

):

!!!! Vague question !!!!! IDONT KNOW aanswer VSD

-old pt ,she have MI and complicated with ventricula tachycardia then from that time recive Buspirone

what INX must to be done

a- thyroid function b- liver and thyroid


| P a g e

c -

): !!!

also vage Q

iforget the rset of the choises i dont know what the pt have and how investigate

-at which age child sopke few words a-12m b-24m c-36m

Another Exam Posted by dr- Ibrahim Qaid : ( )


(1/6) The best ttt for binge eating disorder: - cognitive - behavioral therapy - problem - solving therapy - interpersonal therapy

| P a g e

(2/6) prophylaxis: - inhaled short acting B agonist - inhaled steroid

(3/6) drug useful for pts with idiopathic anovulation: - chlomophine

(4/6) bad breath smell with seek like structure, no dental caries & Ix are normal, what's the likely cause: - cryptic tonsillitis - Sojreen's synd.

(5/6) old aged female with atypical squamous cells of undetermined significance (ASCUS) on pap on pap smear, what's next: - vaginal biopsy - endometrial biopsy - syphilis serology

(6/6) 24 y. Female with new Dx of DM2, she weared glasses for 10 years, you will advice her to follow ophthalmic clinic every: - 6 months - 12 months - 5 years - 10 years
| P a g e

SLE Exam January 2012 Posted By dr- Sana Sohail

| P a g e

salam to all.my exam was today.it went fine.most of questions were new and unseen.i am going to share some new qustions.hope they will be helpful for future candidates.... 1.patient with hypopigmented macules.loss of sensation.thickend nerves.diagnosis was leprosy.which type tuberculoid lepromatous borderline 2.female presented to er with HCL burn on her face there was partial thickness burn.management irrigation with water irrigation with soda bi carb immidiate debridement 3.young female with left sided abdominal pain.no dysuria or change in bowel habit.history of hysterectomy 4yrs back but ovaries and tubes were preserved.on ex abd tender but no guarding.inv show leucocytosis and few pus cells in urine.there was also history of unprotected coitus with multiple partners. i didnot get the scenario well but i think it was salpingitis. management consult surgeon oral antibiotics diagnose as ulcerative colitis 4.chickpeas.kidney beans and lentils contain which element of following bromide chromium iron selenium 5.a case of clamydial eye infection 6.a scenario of otosclrosis 7.a picture of JVP graph to diagnose.patient had low vol pulse,low resting bp.no murmr.pedal edema. constrictive pericarditis tricuspid regurg tricuspid stenosis pulmonary hypertension 8.treatment of psoriasis

| P a g e

9.picture of pelvic x ray what is diagnosis normal paget disease spondylitis osteoporosis 10.case of viral gastroenteritis 11.16 yr old female with primary ammenorrhea.scarse pubic and axillary hair but proper breast dovelopment diagnosis

| P a g e

New Collections for 3 Edition (January, Febrary and March )


rd

| P a g e

//
:

Q1- 4 y/o child awake from sleep because a croup , which one shuld be in you DDx ; A/ foreign body ( my answer ) B/ broncholitis C/cystic fibrosis D/ congenital heart diseas there was no B. A in MCQs

Q2- 18 Y/o girl NOT sexually active . came with vaginal bleeding ,the doctors cant exam her due to the pain , what is the NEXT step ? a/ reassure her that it is normal in her age , and follow after three month if bleeding dont stop . b/ urine pregnancy test c/ ultrasound d/ refrer to OBGyne I dont know

| P a g e

Q3- female patient came with fatigue and Jaundice. her CBC shows WBC =9 HGB= 9.5 ,PLT= 200 and his LFT show totoalbilirubint =3 , direct = 0,9 ... wahat is the most liklyDx : a/Dubin Johnson syndrome b/Gilberts syndrome C/primary scelerosing cholangitis D/criglernajjar syndrome type 1 Q-4- 8 y/o child with BMI= 30 and his height is more than 95 % for his age ... the next step ?senario not complete because the rest not important ? A/ observation and follow after 12 month b/ surgical intervention c/ obesity medication d/ life style modification( my answer but not sure ) Q5- which one of the following is the best managment for 1 degree burn A/ debdidment b/warm wash and remove the material C/ water and ice d/ keep the affected area in cool area

| P a g e

Q6- what is the most reliable laboratory to estabilishe diagnosis of Acute gloerulonephritis ? A/ RBC cast in urine B/ increase WBC in urine C/low HGB with normal RBC D/ small shrunk kidney by ultrasound Q7- which one of the following anti TB medication is consider as drug induce SLE A/ ethambutol b/INH ( my answer ) c/streptomycin d/rifampin Q8- endemic means a/ spread of disesas in incidence all the time b/it cause by virulent pathologic organism c/ spread of disesas from country to country by carrier d/rapid spread of disesase e/ there is very low incidence Q9 - before 14 d the child was bite ,now develope lip swelling eryrhema ... , what type of hypersensitivity ? a/type 1 b/type 2 c/thype 3 d/type 4 Q10- 46 y/o male came to ER with abdominal pain but not that sever. He is hypedyslipidemic ,smoking ,HTN , not follow his medication very well , vitaly stable ,, o/E tall obese pt . mid line abdomen tendernes , DX A/ marfan syndrome B/aortic anyrsm

| P a g e

Q11- 17 y/o boy came with andominal pain and vomiting and leg cramp blood test was done and randome glucose = 23 {{ pic. of DKA , what is the most important next step a/ abdominal ulratasound b/ABG ( my answer ) c/ urine analysis by dipstick d/chest x- ray Q12- elderly patient K/c of HTN and BPH , which one of the following drug Is potentially recommended as such case : ... a/ atenolol b/terazosin ( my answer ) c/ losartan Q13- Gualine-Barrie syndrome is closlyassocited with which one of the following : a/ descending paralysis start from upper limb b/normal CSF c/ascending paralysis start from the lower limb ( my answer ) d/need ECG

Q14- the physicain will look in patient with idiopathic autonomic insufficiency for which one of the followig ... A/ absent sinus tachycardia B/ muscle wasting c/ orthostatic hypotension( my answer ) D/horner syndrome Q15- student in college .complin of sever itching in ankle and between finger ,, it was first attak ,, well demarcated .. Dx scabis( my answer not sure ) tiaene

| P a g e

Q16- pt sustaine RTA his bp 70/90 HR=140 RR=40 cold skin}} sign of hypvolemic shock ,, clinically there is bilateral pelvic fracture , Wht is the Appropriate NEXT step ... A/ IV replacement( correct ) b/ blood transfusion c/ splint fracture as ATLS guidline maximum of 2 litter IVF then blood transfusion shoud be consider , Q17- which on of the folowwing is a MINOR crteria for rheumatic fever ? a/ arthritis b/erytherma marginutum c/chorea d/ fever ( the answer is correct ) Q18- regarding mitral stenosis which one is true the answer is mid diastolic murmer low pith 100 % suuure

| P a g e

Q19- 24 y/o healthy male complain of RT testes swelling , O/E there is larger rt mass not tender ,and the patient told you that its growin by the time , What is the most Approprite step in this situation : A/ refere to open scrotm and take a biobsy B/ refere to geneal surgey for ultrasound and take opinion ( my answer not sure ) C/ observation and foloow up next month

Q20- healthy femal came to your office complain of lesion in her vagina that stared sice just 24 h . O/E there is cystic mass lesion non tener measure 3 cm on her labia , what is the the most likely Dx : ... A/ bartholin cysy B/Vaginal adenosis C/ schic cyst ( some thing like that ) D / hygroma Q21- which one of the following factors MOSTLY determiner the recurrence of colorectal cancer : A/ age B/stage ( my answer ) C/family history D/gender

| P a g e

Q22- adult healthy male came with tender red swelling on right hand up to forarm and you found black head and large pore skin , he said it happen after trauma to his hand 1 week back , the management shuld be : A/ topcical antibiotic B/ topical Antfungal C/cryosurger D/ Oral antibiotic acyually I dagnose this case as Hidra adenitis suppurativa but I am not suure and as i know the ttt= mild = cold compressor ( was not in MCQ) moderater =AB sever= surgery Q23- male, presented with pain in the posterior aspect of the thigh, he was running long distance felt a pop in his thigh, on exam, tenderness, erythema, and swelling, no defect what is the best treatment: a- Surgery b- Ice, rest, bandages, and elevation of the limb.( my answer not sure but most likely )) c- Bandages only. d- Splint. Q24-What is the most ACCURATE diagnosis for Ectopic pregnancy : ... A/ serial B-HCG B/ ultrasound C/laproscopy D/progesteron

Q25- what is the best management for binge eating disorder: A/ cognitive behavioral theraby( my answer ) B/ problem solving theraby C/ interpersonal theraby Q26- Which congenital heart condition is the most common associated with endocarditis A/ VSD B/ ASD C/PDA D/TOF ( most likely )
| P a g e

Q27- A man who is thinking that there is Aliens in his yard although that he knows that Aliens are not existing but he's still having these thoughts especially when he is out of home he is afraid to be die due to that ..Dx A/obsession B/ delusion C/ hallucination D/ illusion Confuse ith him the plane to reduce his weight , you will told him to : A/decrease calori intake in day time B/decrease calori and increase fat C/decrease by 500 kcal/kg per week( my answer correct ) D/ decrease 800 per day

Q29- Old patient with asthma and urine retention due to prostatic enlargement, hypertensive (BP: 180/100) what's the most appropriate drug to control hypertension? A/ Labetalol B/Phenalamine C/Propanolol Same Q before but her with BA and different drug

Q30 Which one of the following dug consider as drug induce urticarial A/ Azithroycin B/ hydralazine C/ cortison
| P a g e

D/ Penicilline( my answer ) Q31 - regarding Boutonnere deformity which one is true A/ flexion of PIP &hyperextension of DIP.( my answer most likely ) . B) flexion of PIP & flexion of DIP C) extenion of PIP & flexion of DIP. D) extension of PIP & extension of DIP Q32 preganant lady 38 wks GA with placenta previa marginal with mild bleeding , the cevix is dilated cervix 2 cm How to manage ; A/ CS B/spontiusdelvery C/forceps delivery( most likely ) D/ do amniotomy Q33 -whaich one of the following is true regarding the weight gain in pregnancy: A/Pregnant woman should consume an average calorie 300-500 per day ( my answer most likely ) B/ Regardless her BMI or body weight she should gain from 1.5 3 lb which represent the baby's growth

Q34- 9 years old female presented to ER after ingestion almost 20 tablets of OCP and 3 tablets of another medication..She is clinically stable and there was no signs and symptoms...What will you do: a.refer her to gynecologist. b.refer her to psychiatrist. c.toxicology study.( my answer ,, because we dont know which other drug even if she stable now and then we can referee ) confuse d.no need for intervention.

Lichen planus most commonly found in :

Q35| P a g e

A/Scalp B/ Knee C/ Buttocks D/Mouth( my answer and most likely ) Q36- regardingright lung anatomy which one is true : A/one fissure B/ pulmonary segment C/ no relation with azygus vein D/ 2 pulmonary veins ( my answer ) E/ no sibson's fascia Q37-Patient on Amitriptyline 30 mg before bed time, wakes up with severe headache and confusion, what's the appropriate action? A/Shift him to SSRI's( my answer , not sure ) B/Change the dose to 10 mg 3 times dail C/ continue on the same

Q38- Old pt complaining of back pain on walking on examination there was stiffness of the muscle and there was some finding on the X-Ray of MODERATE spondyloarthropathy best effective ttt: A/ Physiotherapy ( my answer , not sure if Moderate different ) B/ NSAID C/ Surgery D/ bed rest Q39-17 years old with type I DM,he is mostly has association with HLA: A/ DR 4.( my answer correct) B/ DR 5. C/ DR 7. D/ DR 9.

| P a g e

Q40 - 28 years old diabetic female who is married and wants to become pregnant.her blood glucose is well controlled and she is asking about when she must control her metabolic state to decrease risk of having congenital anomalies: A/ before conception.( my answer ) B/ 1st trimester. C/ 2nd trimester. D/ .3rd trimester. Q41-All can cause gastric ulcer except: A/ Tricyclic antidepressant.( my answer ) B/ Delay gastric emptying. C/ Sepsis. D/ Salicylates. E/Gastric outlet incompetent Q42-Likelihood ratio of a disease incidence is 0.3 mean: A/.large increase B/small increase C/no change D/ small decrease ( my answer ) E/large decrease Q43-8 months child with 3 days fever 40 , vomiting , convulsion , poor feeding & sleep , OE dehydrated , depressed ant frontanell, red ears ,no neck stiffness , his 3 year old sibling asymptomatic , whichof the following will give the defenitiveDx : a- CXR b- CBC with deferential c- blood culture ( my answer not sure , I think about sepsis ) d- CSF analysis e- suprapubic urine analysis Q44-in PHC, from 50 child 10 got the disease on the 1st week, another 30 on the subsequent 2 weeks, what is the incidence of the disease in that PHC? a- 20% b- 40% c- 60% d- 80% ( my answer ) , 10+30 /50 e- 90% Q45-the name of Questionnaire that differenctite between the primary and secondryapnea : Straing name BUT the correct answer is horchover
| P a g e

Q46-which one is true Regarding appendicitis in the elderle: A/ If the patient is afebrile this rules out appendicitis B/WBC is often normal( my answer not sure but was most likely for me ) C/Rupture is not common D/anemia is coming finding

Q47- A middle age man presented with severe headache after heavy lifting objects. His BP was high. He was fully conscious. Examination was otherwise normal. the most likely diagnosis is: a) Subarachnoid hemorrhage b) Central HTN c) Tension headache d) Migraine e) Intracerebral hemorrhage Q48-patient HIV have white patch in oral cavity and how could you manage A/oral antibiotic B/loacal antibiotic C/local steroid D/chemo & radio theraby ( my answer , there was no antifungal so not candida, I diagnose it as Kaposi ) Q49- middle aged pt with ataxia , multiple skin pigmentation and decrease hearing , one of the family member has the same condition , what is the most likely DX : a- Malignant melanoma b- neurofibromatosis ( my answer ) c- hemochromatosis d- measles e- nevi Q50 -5 y.o child with history of fever and swelling of the face ant to the both ears (parotid gland enlargement) what is the most common complication at this age group :
| P a g e

A/meningitis B/ labrynthitis C/ orchitis

Q51- what is the treatment for common mastalgia : A/ tamoxifen( my answer ) l B/ caffeine C/ OCP NO in MCQ danazol nor cromocriptine Q52-4 y/o child with diarrhea for 2 days is complaining of anal discomfort. Your advice to the mother is: A/Wash with soap and water after each episode of diarrhea. B/Wash with cotton in warm water.- ( my answer ) C/Put a clean napkin in the underwear.D/Change the underwear to a highly absorbent diaper Q53-pt taking lasix having CHF and his electrolytes showed hypokalmia3 ,hyonatrima 123, hyperglycemia , hypchloerima and high urea and he had muscle cramps and weakness u will give A/ NS with 5 KCl In 20cc/hr -B/ NS with 40 KCL in 80cc/hr ( my answer ) C//2Ns with 5kcl in 20cc/hr D/2 NS with 40 kcl in 80cc/ hr Q54 4 Y/O Baby with scenario of ADHD, what is the best treatment in addition to behavioral therapy: a- Atomoxetine( my answer correct ) b- Imiramine Q55- 9- The most cause of tinnitus: A/ vitiligo b- Sensory neural deafness c- acute otits media d- noising induce tinnitus. ( my answer ) Q56- which one of the following anti TB medication consider drug induce SLE
| P a g e

The answer is INH Q57- which on of the following OCP cause hyperkalemia : The answer is ethynyl estradiol / :

ALI AL Rassi , best wish for u

Dr. thamer al-sa3ed exam


| P a g e

My 2nd sle good luck

1-Regarding to Mallory wiss syndrome what is true /

surgery mostly needed the hemorrhage stop spontaneously

2-About hepatoma (hepatocellular carcinoma ) what is true mostly associated with chronic liver liver disease Smoking is a risk factor 10% in africa and asia 3-diagnostic lavage wt is diagnostic 1000 rbs is diagnostic 500 wbs is diagnostic 2 ml of blood initial aspiration 2 ml of blood in pregnant

3-sckil cell anemia complication related to cns Infarction I dnt remember other choices 4- Child with vomiting (nt sure billous or nt ), abdominal dissension He passed stool immediately after birth : hirschsprung's dis Mid gut volvus 5-Elderly pat with dementia and change in his behavior (many things including agitations ) which lobe in brain affected : | P a g e

Frontal Occipital Temporal Partial Cerebellar

6-after aspiration of a breast cyst, which of the following indicate that the cyst is benign: Aspiration is clear & the cyst not refill Cytology study shows fibrocystic disease 7-27 weeks pregnant , glycosuria, she did ogtt which prove dm wt u will do ? -diet control - ogtt again - random blood sugar

8-40 years old , thin , k/o premature ejaculation , loss of libido ,he look sad , his wife is obese , money expender ,unorganized , claims their marriage is alright , the : examination prove no organ pathology Wts ur action 1-ssri 2-weekly testesrone injection

9-Guy take 20 pills of ocp and 2 other pills (didnt mention their type) he is alright , didnt vomit , wts ur action ? Gastric lavage Toxin screen Refer to psychiatry | P a g e

10-Old age female , with history of excision of breast tumor with radiation therapy , now the blood film and bone marrow biopsy prove CML , wts the most risk factor responsible for her condition? age previous cancer radition -45 years female with discharge contain blood Wts the comments cause -Duct papilloma Ductal ectcsia Abscess

12-Phenobarbital

Continue As mother child wish Stp breast feeding continue as tolerated

-About large uncomplicated pneumothorax wts true : There is deviation of trachea There is decrease in percussion of the affected side

14-70 years old male , he is newly diagnosed with htn his bp is 170/105 , history of dm since 20 years old , no history if MI or any vascular disease , wts the most appropriate anti htn to give ? Acei Thiazide | P a g e

CCB B blocker

15-Middle age Malepresented to the ER comatose and his skin lok reddish , wts the most likely diagnosis ? Carbon monoxide poisoning (my answer) High dose of insulin Septicemia minigo 16-Acute onset of disorientation , change level of conscious, decrease of concentration , , tremo ,he mention that he saw monkey ! Wts the diagnosis: Parkinson dementia Schizo Delirium (my answer ) Delusion disorder

-First step in conducting epidemic investigation Verifying diagnosis( my answer) Account the number of new cases

18-Newborn 32 week , cyanosed , grunting , flaring of nostrils , the x-ray show diffuse air bronchogram , his mother is diabetic , wts the diagnosis ? Insufficient surfactant (my answer ) | P a g e

Trechoesphgeal fistla

19xray show infiltration of lower lope , wt u will give him ?

Cefuroxime Amoxicillin Cipro. 20-speration of chromatids occur in late stage of ? -metaphase -anaphase tricky Question late stage 20-stop of smoking , the peak of symptoms occur : 1-2 days 2-4 days 21- diabetic patient for long time came after car accident complain flashes of light in the left eye or, blurred vision, shadows ? ddx Retinal detachment (my anser) Cataract Other choices 22-relative risk

| P a g e

Risk factor Present Absent Total

Case (disease) A C A+C

Non case B D B+D

total A+b C+d

Relative risk of those with risk factor to those without risk factor is: a. A/A+B , C/C+D this the answer

23.treatment of scabies ? Permethrin

24. ttt of t.vaginalis ? Metronidazol

25-Pt with barret esophagitis , risk of malignancy: _ adenocarcinoma (my answer) 26-Injury of arterial blood v -red bright, spurting (my answer)

| P a g e

SLE on 6th of Feb 2012 Written by Noura Al-mutlaq

1-patient with past of hx of endocarditis came to dental to do dental procedure , what antiobiotic u will give as prophylaxis : -amoxcilline 2 mg before the surgery -clindamycine 2 mg before surgery 2- the marker for ovarian ca : CA 125
| P a g e

3- TRUE about congenital hip dislocation : -abducting an infants flexed hip causes palpable click ortalani test 4-patient known to have CHF , came with fatigue & muscle weakness , Na :123 , K: 3 , how ull manage ? -normal saline with rate 80 /hour +40 mic kcl -normal saline with rate 20/hour +5mic kcl -1/2normal saline with rate 80 /hour +40 mic kcl -1/2normal saline with rate 20/hour +5mic kcl 5-what is the attributable risk : -number of cases of a disease attributable to one risk factor

6- patient came with retrosternalchest pain , increase with laying dawn &sleeping , ECG and cardiac enzyme were within NL -give PPI 7-what is the most specific diagnostic for PE : -V/Q scan -pulmonary angiogram -chest x-ray 7-child with DM type 1 ass with -HLA DR4 8-most common cause of conductive hearing loss : -acute otistis media
| P a g e

-Meniere disease - perforated tympanic membrane 9to manage ? Repeated qs but the choices werent the same :l 10-SE of sulfodizene : Leucopenia 11- 40 year old male , not known to have any medical illnesses , complaining of central obesity, acne , weakness , buffalo hump , hypertension : -cushings disease -psuedocushing induced by alcohol intake -adrenal adenoma -adrenal ca ng the face and neck , how

12- elderly patient complaining of LLQ abdominal pain with fever , diarrhea -diverticulitis 13-most common feature ass with chronic diarrhea : -metabolic alkalosis 14-typical scenario about primary bleary cirrhosis 15-acute fluid loss in the abdomen cavity what it will cause : -cardiogenic shock -neurogenic shock -septic shock
| P a g e

-hypovolemic shock 16-scenario about fibroid in pregnancy whats true : -fibroid will regress after pregnancy 17conceive , what ull do : -continue to try -prolactine level -TSH 18- drug for induction of ovulation : Clomophine 19-42 pt came with DUP what ull do : -OCP -D & C -hysterectomy

20-pt came with hx of 3 weeks amnerhea , with abdominal pain , laproscopy done and found to have blood in the boutch of duglas : Rupture of ectopic pregnancy 21-qs about vesicocele U should know if its repair of upper ant of the vaginal wall so the defect is vesicocele , if its lower anterior urethrocele , if upper post enterocele , if lower post rectocele

| P a g e

22-pt asking u why instead of doing self breast exam. Every month not to do mamogaraphy yearly , what ull say : -mamography only detect deep tumor -mamography and self exam. Are complementary -self breast exam are better bcz it detect early tumor - mammography are only for palpable masses 23-child came with sever anemia ,theyre suspecting thalacemia , whats the best diagnostic to confirm : -genetic test -iron study 24- scenario about heamophilia , whats the defect : - Clotting factor 25-typical scenario about essential HTN 26-20 yr male ptn came with couph , chest pain , fever , what antibiotic u should prescripe : -amoxicilline -ceftriaxone

27- pt came with caf au late spots , what other things ull look for : -axially freckling 28-qs about obstructed labor 29-drug are contraindicated in ttt of glaucoma :
| P a g e

Timolol Pilocarpine 2 new drugs I was confused , I dont remember their names :/ 30-qs abt osteoarthritis 31-pt came unable to do thump opposition : Median nerve injury 32-pt with scoliosis u will refer if degree more than : -20 33- baby at 6 months , what he can do ? -sitting without support -role from supine to prone position - role from prone position to supine 34-What feature of schizophrenia suggest good prognosis ? -family hx of scz -no precipitating factors -presence of affecting symptoms -early onset 35-qs about etiology of gout , read about it :D , all the choises were not easy

36-less expensive
| P a g e

-most tolerable and effective -to differentiate between psychosis and depression 37-elderly pt came with hx of coma and hypotension , before the coma she complained of epigastric pain , most likely due to -AAA small leakage 38-von well brand disease how to treat: -fresh frozen plasma -factor VIII replacement 39-drug used in smoking scessation c/I in pt : -hx of seizure 40-ptn with breast mass after FNA , u will leave it alone if : -clear fluid and not refill again 41-prophylactic of plague 42- 243-pt female with sever hip pain , increase with walking , after busy day , awake her almost all the night , ass with morning stiffness : -osteoartheritis -osteoprosis 44-pt with 32 BMI : Obese 45- qs abut EBV 46-qs about COPD mx 47-snellen chart , same qs in malzamah / 20/70
| P a g e

48- qs about pneumococcal vaccine 49-the most effective way in health education : Mass media Group discussion Individual approach 50- child with white yellow mouth lip n erythrematous base with gingivitis : HSV EBV CMV 51-do ECV 52-qs ptn with arthiritis , urethral discharge , culture of discharge came ve for gonnorhea and chalamedia : -Rieters syndrome -gonorhea 53- pt with skin rash , diarrhea , dementia Plegra 54- qs about asthma 55- typical scenario abt retinal detachment , what ull do :

| P a g e

SLE Exam 24-Jan-2012 SLE Exam Fuad Almalki


Most common complication of hysterectomy is : Hemorrhage PID urethral injury Orher options i forgot === Old female pt, c/o polyurea, polydypsia, dysurea for one year, she received many courses of antibiotics but no improvement, the physician should now think of : Traumatic urethritis Interstitial pyelonephritis Diabetes mellitus Other options i forgot == a pt presented with DKA & hyperkalemia & hypotension, best initial treatment : 2 liters NS with insulin infusion at rate of 0.1/kg 2 liters NS with KCl 20 meq desxtrose with insulin give NaHCO3
| P a g e

== most common cause of sleeping in daytime is : narcolepsy mood disturbance general anxiety disorder i forgot other options == hepatitis can be confirmed in pregnant lady by elevation of : ESR ALP WBC SGOT == about appendicitis in elderly : WBC is often normal low risk of rupture CT scan is not benifitial to make the diagnosis another MCQs about fever == a man fell down from the ladder, c/o SOB ( and i think cyanosis ), on exam breath sounds are decreased even in the right side ( this is how they wrote it !! ), u will do : needle thoracotomy insert endotracheal tube other options i forgot == a long scenario about head trauma presented with preiorbital swelling, the doctor suspected blowout fracture, what's true : an air-fluid level in the CT will exclude blowout fx globe injury is rare others options i forgot, just read about blowout fx == pediatric pt come with fever and inspiratory stridor, u will: give amoxicillin and go home admit him to ICU and call ENT do cricothyrotomy == a placenta that is anterior and close to OS but can't be reached by examiner is : low lying previa marginal total partial
| P a g e

7 March 2012 my SLE


: ! ... many thanx to study group for your help ,support and encouragement..all of you ,, i ddnt had the chance to thank you may GOD bless you Abo Yasser
| P a g e

)))))There is only one way to do any exam and that is once!

((((

6 month child , difficulty in breast feeding , active pericardium, pansystolic murmer s1 , loud s2 ASD large VSD MR AR PDA

1. 46 Y male , c/o early ejaculation , unability to sustained eriction , he belive his 26 yr marige is alright , his wife ok but unorganized , obese . doctor confirm no organic cause. He look thin .sad face , whats ttt: SSRI Sublingual nitrate 6 h befor testesteron injection 2. DM obese lady , newly discovered type 2 , compliance with diet and exercise , when start medication she felt dizziness ,dry mouth , which drug cause her symptoms: sulfonurea 3. Lady with 2 day hx of fever , lower abd and suprapubic tenderness , viginal discharge & tenderness Dx: acute salpingitis chronic salpingities acute appendicitis 4. Last trimester pregnant lady develop sudden left leg swilling . extend from left inguinal down to whole left leg , ttt venogame,bed rest,heparin duplex,bed rest ,heparin pleosongraphy,bed rest, caval felter duplex ,bed rest ,warfarine

5. Secondary prevention: seat bealt influenza vaccine for elderly


| P a g e

DPT vaccine for children coronary bypass 6. Child with iron toxcicity several hours ago , investigation show iron conc. 700 mg/dl ,ttt: gastric lavage activated charcoal iv dexoframin

7. Pt osteopnia in femure with increase serum alkaline phosphatase , normal serum calcium, normal phosphate ,normal vit d: ttt estrogen receptor modulator calcium regulator bisphosophnate

8. Pt respiratory problem , foul smelling , CXR bilateral lesion at base of lung, heamoptosis , finger clubbing: bronchactasis

9. Child with ear congested , opacity , recureent URTI , o/e NEED adenectomy , beside adenectomy u must do: tosilectomy maryingotomy mostly) government tube 10. Middle aged 23 y/o not known to have any medical illness apart of annular lesion in mouth with painless ulcer ,presented with fever, arthritis, and rash mainly in the palms and soles ,,, he gave hx of illegal relationship ,,, mostly he is having?? 2ndry syphilis 11. 60 y/o pt. presented with decrease vision bilt , specially to bright light on exam he was having cupping with wedge shaped opacities ... he is having?? a.lens sublexation b.catract (my answer and i got 3/3)

| P a g e

c.open angle glaucoma

12. Which of the following not a live vaccine: BCG HB OPV

Pt came with deep injury on the wrist site, the nerve that has high risk to be injured will manifest as? _ Can not oppose thumb to the other finger?median nerve( my 13. answer ) 14. Pt work most of the time on the computer came with

wrist pain , positive tinel sign you will do cast for the hand so the hand position should be in _ A-Dorsxiflexion 15. An elderly lady presented with Swilling knee pain bilaterally that increases with activity & decreases with no history of trauma .The most likely diagnosis is: Osteoarthritis ( true 5 y.o child with h.o fever and swelling of the face ant the both ears (parotid gland enlargement) what is the most common complication >>meningitis / labrynthitis / orchitis 16. 17. PT case of CHF , loved to eat outdoor 2-3 time weekly u advice him: - eat without any salt -eat 4 gm salt -low fat,high protein known case of cervical spondylolysis , presented by parasthesis of the little finger , with atrophy of the hypothenar muscles , EMG showed cubital tunnel 18. compression of the ulnar nerve , what is your action now? Cubital tunnel relase Female with dysuria on examination there is
| P a g e

19. epithelial cell >>>> chlymdia urethritis

20. hx of travel abroad , best in elimination of entameba histolytica>> - boiling water - freezing 21. child 6 yo with R.FEVER , continue AB for>>> 15years 22. pt known smoker 10 cigar for last 10yr, present with oral ulcer , recived antibiotic with no improvement ?? - biopsy -reassure -staining 23. newborn given injection to reduce bleeding>> vit. K 24. definition of epidemic curve >>>??????????

25. commonest cause of intracerebral haeg.: aneuresym trauma 26. headach , which trure: - normal CT can exclude subarachnoid haeg. -cluster headach more in male . 27. testicular fullness ,like bag of worm , positive valsava: varicocele 28. testicular mass at tip , positive valsava: indirect inguinal hernia

29. female 25yo , ask you about breast self examination when should be done: -6-7 day after cycle -5 day befor -7-10 day after -14-16 day after - after 2 day ????????????????????????/

30. teacher ,complain of panic , this after mistake in class room, he know it must be useful in future day , co sweting , tachycardia , tightness>
| P a g e

- benzodiazepam -ssri -social plhobia 31. pt told you the refregator told him that all food inside poisoning: - audiatory halluscination -dellusion -illeusion

32. pt underwent colectomy dignose as stage B2: - NO lymph node involve -2to4 l.n - one l.n 33. mastalgia ttt: OCP

34. Fibroid : regress after pregnancy 35. Ttt for menpusal women ,c/o bleeding , not ass with intercourse: -estrogen -progesteron 36. Picture ,Child with skin lesion at elbow , seen positive wood lamp: -fungal -bacterial 37. Skin rash in buttock, hamturia : HSP

38. Indication for tonsillectomy is: a. Pharyngeal abscess b. Sleep apnea c. Recurrent infection d. Asymmetric tonsillar hypertrophy

39. Most common problem present in primary care : coryza 40. Child with duodenal atresia, characterstic sign in imigaing: -double bubble 41. Firest sign in increase intracraineal pressure: -vomiting -nausea
| P a g e

-ipsilateral pupil constrict -cotralateral pupil constrict 42. Pt with barret esophagus , risk of get malignancy: _ adenocarcinoma -sequamous 43. Most common cause of CVA, Mostly embolic resource - AF -VSD 44. Old female ,fear from desk compession and fracture : - vit d,calcium -wt. reduction -progestrone 45. Known alcoholic chronic for long time, present with lymph node in mid cervical , your action: -larygoscop -excitional biopsy -needle biopsy 46. Old lady ,outcome baby with Clinical feature of down , single palmer creases , epicanthic fold, wide palepral fissure - trisomy 21

47. Pt elderly , with unilateral headach , chronic shoulder and limb pain , positive Rheuatoid factor ,and +ve ANA ttt\: - asprine -indomethacine -corticosteroid 48. Asthematic child , how to decrease the allergy: - cover pillow and bed with impermeable material -throw the ruge frome house 49. Yong male with 3 day of dusuria, anal pain , O/E perrectum boggy mass : - acute prosatities 50. Outbreak of TB , person found negative TUBECLIN : - rifimpcin -vaccination

| P a g e

51. Kwashikower: - low protein,high CHO 52. PICTURE CXR of pericardial effusion,TYPICAL presentation S&S 53. 80 y/o male CASE HTN on ttt with mild begnine proatatic enlargement , causes feelling of incomplete voiding - alpha blockers -surgery

Computer programmer, a case of carpet tunnel syndrome,positive tinnel test , how to splint: 54. Dorsiflexion( sure)

55. Chronic gastric ulcer , pt intake a lot of antiacid , no still complian:ttt> - H 2 antagonist -proton pump inhibitor 56. Most reliable test to diagnose Acut glomerular nephrities: - red cast in urine. 57. City with 1500persons, no of 105 birth , 5 are still birth , 4 die at first month,2 die befor age of one year , perinatal mortality? -4 -5 -6 -8 -9 58. Pt. chronic depression ,now u start tt.Paroxetin (paxil) you told the pt: - need 3 or4 week to act -side effect ??? idont remember

59. Phayngitis treated with oral penciling you should : - 7 day -10 day -14 day 60. Syncope due heart : - rapid recovery

| P a g e

61. Most specific for diagnosis of pulmonary embolism: -EKG -Ventilation perfusion ratio (V/Q scan). -pulmonary angiogram ???? 62. Elderly pt . fever and infection by enterocucus fecalies, source of infection: - urinary -lung 63. 12 y/o Child overweight BMI=31 , +ve family history of hyperlipedemia , parents fear of child get dyslipidemic, when you should request lipid profile?????????? - upon parent request - COZ overweight

64. Pt with high total chloestrol 265mg/dl , LDL 150 , triglecride 325 , HDL 100 most single risk factor??? - low LDL -High LD -High HDL -low HDL -high total cholesterol 65. Best secreening test for liver malignancy: -us+ liver biopsy -CT scan + Liver BIOPSY -CEA + AFP

66. FEMALE , analysis of urine tet ,epithelial cells:indicate - vulvar contamination -cervical tear -renal stone -UTI

67. Young female always eat fast food , you advice supplement of: -zinc +vit. C -vit. C+ folic -vit.d+ zinc -folic acid+ CA 68. Exercise recommended for osteoporosis pt.

| P a g e

69. Male with collosion bicyclic motor bike , closed head injury . cant direct spoon to his muoth , site of lesion: -cerbellum -parital lobe -frontal 70. patient has history of parotid and salivary gland enlargment complains of dry eye . mouth and skin ,, lab results HLA-B8 and DR3 ANA +ve rheumatoid factoe +ve what is the course of treatment a-physostigmin b-eye drops with saliva replacemnt c-NSAID D-plenty of oral fluid 71. LACERATION IN ANTERIOR ASPECT OF WRIST: - wrist drop -median nerve injury -claw hand 72. battered women which is true: a. mostly they come from poor socioeconomic area b. usually they marry a second violent man c. mostly they come to the E/R c/o.. d. mostly they think that the husband respond like this because they still have strong feeling for them 73. most common physiological cause of hypoxemia - shunt -Ventilation perfusion mismatch -hypoventilation 74. arterial bleeding after injury: - red blood ,continous -red bright , spruting -dark blood 75. teacher c/o malise fever , right upper abd tenderness , two student develop same condition , eye become icterus, best CONFIRM dignose: -HBA IgG - HBA IgM - HBA core AB

76. Apgar score: Heart rate is significant


| P a g e

My SLE in 5th of March 2012 Wish you all of the best Gray zone

.. .. 1- 26 yo psychotic patient presented to the hospital after 3 houres of ingestion of 3 pins, PE : unremarkable, X ray showed 3 pins in small intestine but no intestinal dilation or air fluid level. You action will be : a- Admit the patient to the hospital for serial x-rays and abdominal examination. b- Send the patient home and give follow up appointment. c- Start antibiotics and send home. d- Admit the patient and start antibiotics. 2- Most effective antibiotic to treat gonorrhea is : a- Ceftrixone b- Penicillin G.

| P a g e

3- Which of the following true regarding apgar score : a- Total score 12 b- Discoloration is not important c- Heart rate significant d- Assessed in the 2nd day of life.

c- Pipracilline. d- Gentamycin. e- Vancomycin

4- Patient with hx of recurrant eosphagitis in the last 5 years , biopsy shows that presence of glandular islet an columnar cells in squamus cellular zone , eosphagescopy showed finger like projection upward the squamo-columnar area . most likely diagnosis ; a- Adenocarcinoma of the esophagus b- Squamus carcinoma of the esophagus c- Barret esophagus d- Normal picture of esophagitis

5- Football player injured in the lateral side of his LT knee, presented to you with sever knee pain, PE there is swelling in the medial aspect of the lt knee ,valgus test showed free mobility but lachman test and McMurray's test are negative . what's your diagnosis: a- Lateral collateral ligament injury b- Medial collateral ligament injury c- Patellar fracture d- Medial menisci injury e- Lateral menisci injury 6- Regarding lung cancer: a- It's the leading cause of death in females b- Adenocarcinoma common in the proximal part 7- Which one of the following is a strong indicator to do diagnostic peritoneal lavage: a- Comatose patient due to sever head trauma . b- Patient with pelvic fracture c- Patient with sever abdominal pain and distention

| P a g e

8- Patient with rheumatoid arthritis came to came to you and asking about the most effective way to decrease joint disability in the future, your advice will be: a- Cold application over joint will reduce the morning stiffness symptoms b- Disease modifying antirheumatic drugs are sufficient alone 9- Most common cause of recurrent tonsillitis : a- Group B streptococcus b- EBV c- Bacteriod d- Rhino virus e- Parainflunza virus.

d- Patient with BP 80/56 with abdominal distention e- should be done to every patient had RTA.

10- A 10 YO was diagnosed with rheumatic fever without any defect to the heart. You will tell his parents that he needs to take prophylactic antibiotics for how many years? a- 5 months b- 3 years c- 6 years d- 15 years 11- Child with hx of URTI 1 week ago now he c/o arthalgia , fever and fatigability , what's your diagnosis: a- Rheumatoid arthritis. b- Rheumatic fever. c- .

12- While you do head and neck exam , which one of the following is NOT palpable normally: A- Thyroid gland B- Submandibular gland C- Parotid gland. D- Lymph nodes E- Hyoid bone 13- Which of the following finding suggesting anemia of chronic disease: a- Increase serum iron and increase TIBC. b- Decrease serum iron and increase TIBC. c- Decrease serum iron and decrease TIBC.

| P a g e

14- Yong man predict that he is going to have a seizure , then he became rigid for 15 sec the developed generalized tonic clonic convulsion for 45 sec. you initial ER action in future attacks will be : a- insert airway device. b- Apply physical splint or protection. c15- about vareciall vaccine in adult , which is true ; a- 2 vacceine abart of 1 month b- 2vaccine abart of 6 month c- 2 vacceine abart of 2 month d- 3 vaccine abart of 6 month 16- The useful excurcise for osteoarthritis in old age to maintain muscle and bone: a- Low resistance and high repetion weight training: b- Conditioning and low repetion weight training c- Walking and weight exercise 17- Pregnant lady 18 wks, her TFT showed : high TBG, high level of activated T4 , normal T4 and TSH . what is the most common cause of this result: a- Pregnancy. b- Compensated euthyroidism. c- Subacute thyroiditis. d- . 18- Patient with disc prolaps will have: a- Loss of ankle jerk b- Fasciculation of posterior calf muscles. c- Loss of Dorsiflexion compartment of the foot. d- Loss of the sensation of the groin and anterior aspect of the thigh. 19- Child presented to the ER after bee sting with SOB, anxiety and wheezing. PE : BP 75/54 , HR 120 and RR 20. Your action will be: a- Start IVF , IM epinephrine and antihistamine. b- Reassure the patient and tell him that everything gonna be OK after antihistamine injection. c- . 20- Patient loss his wife in the last 4 months , he looks sad cannot sleep in the last 2 days, which medication can help him: a- Lorazepam ( sure .. b/c it's short acting ) b- Diazepam c- SSRI.
| P a g e

d- Increase serum iron and decrease TIBC.

d- . 21- Wha's true about Malaria : the most common cases is caused by Plasmodium falciparum. 22- Pt work most of the time on the computer came with wrist pain , positive tinel sign you will do cast for the hand so the hand position should be in a- Dorsxiflexion sure 100% b- Planter flexion c- Ulnar deviation d- Extension

Wish you all of the best Gray zone

Maryam Abdulaziz Dammam university SLE

1-Female with Hx of PID and treated with ABs she came later with fever and pain on examination there was a mass, fluectuent (they mean abcess ) in a cul-de sac !!
| P a g e

what is ur next step? colpotomy laparotomy laparoscopy Pelvic US

2-a young girl who become very stressed during exams and she pull her hair till a patches of alopecia appear how to ttt: a) Olanzepin b) Fluxitin I don't remember the rest

3-18 weeks pregnant women her blood pressure was 160/..(high) a week after her BP was 150/..(high also) what is the Dx: a) Gestation HTN b) Preeclampsia

4-an Old man , not known to have any medical illness who presen ted with mid back pain , he's taking only aspirin , Ca, multivitamines. He's not taking dairy products and on examination he have tendrness in the mid back with mild kyphosis! and X-ray show compression Fracture in the vertebra in ?? and ?? levels what is your Dx?? Osteopenia Osteoporosis Osteomalacia

5-pt with HTN presented with edema, azotemia,GFR: 44 (not sure about the digits ) what is the cause of her Kidney diseae: a) bilateral renal artery stenoss b) diabetic nephropathy c) Reflux...?? d) Renal tubular acidosis

6-what's true about antipsychotics ?


| P a g e

a) predominantely metabloized in the liver b) Carbamazepin as a single dose os better than divided doses

7-a baby who fall down from stairs and came with multipe contusions some of them were old and X-ray show fracture in radius how to manage : a) Splinter for his hand b) Hospitalization and call social worker

8-In the Time of TB outbreak what will you give as a prophylaxis a) BCG b) Rifampicin .. mg PO

9-45 yr old female G0P0 not know to have any medical illness presented to ER with sever viginal bleeeding on examination there was blood in the viginal os her Pulse was 90 and BP 110 / 80 and on ) How to manage : standing her P: 100 , BP :122/90 ( a) 2 units of blood b) US .. 10-Al hydroxide+ Mg hydroxide inhibits the intestinal absorption of which drug? a) Chloramphinicol sorry I don't remember the rest 11-in epidemiological investigation best thing to do 1st: a) good sample b) count those who have the disease

12-A man who is thinking that there is Aliens in his yard although that he knows that Aliens are not existing but he's still having these thoughts .. Dx a) Obessions b)Dellusions

| P a g e

13-if the liklihood ratio is 0.3 what does that mean??


| P a g e

salma abubaker ,, alkhobar my SLE exam on 27/2/2012

1-Child presented with gum and nose bleeding and bruising all over the body after an episode of URTI. Dx: 1) 2) 3) 4) Henoch Scholein Purpura Idiopathic thrombocytopenic purpura(sure) Vitamin K deficiency Hemophilia

| P a g e

2-the most common side effect of antipsycotic a.alopecia b. wt gain( correct ,, got 5/5) c.hypotention d.constipation

3-year female has atypical squamous cells of undetermined significance on pap smear, past hx reveald 3 -ve smears, last one was 7 years ago she also geve hx of viginal wart, next step is: colposcopy hystrectomy follow up after 1 year excision by

4-which drug economical use twice a day a.ibuprofen b.Piroxicam C.Indomethacin d.Naproxen

5- 2 y/o child presented with painful swelling on the dorsum of both hands and feet,, he was jaundiced with Tbilli 3 Dbilli .9 ,, hgb 9 and retics 7,, what u will do as ongoing managment a.steroid b.NSAID c.penicillin and immunization (sure) d.paracetmol

| P a g e

6- 26 y/o pt. k/c of depression taking (citalopram)for depression ,, presented with ingestion of unknown drug ,,, on investigation she was found to have metabolic acidosis and anion gab 18 ,,, what is the most likely drug she ingested?? a. paracetamol b. asprin c. citalopram d. amitriptyline

7- 19y/o not known to have any medical illness ,presented with fever, arthritis, and rash mainly in the palms and soles ,,, he gave hx of illegal relationship ,,, mostly he is having?? a.chancroid b. 2ndry syphilis c. chalmedia trachomatis

8- 60 y/o pt. presented with decrease vision bilt , specially to bright light on exam he was having cupping with wedge shaped opacities ... he is having?? a.lentis sublexation b.catract (my answer and i got 3/3) c.open angle glucoma

9- 65 y/o pt. presented with hepatosplenomegaly and lymphadenopathy ...bone marrow bx confirm dx of CLL,, the pt gave hx of breast cancer 5 yrs ago and was treated with chemotherapy since then ,, the pt is also smoker what is greatest risk for developing CLL?? a. hx of radiation b. smooking c. previous cancer d. age
| P a g e

10- 25 y/o presented with ear pain and hearing loss in the rt. ear ,, on exam there was ear drum swelling and obscured tympanic membrane with fluid behind the membrane dx is?? a, otitis media b, tympanic cellulitis c,Chondrodermatitis

11-female pt c/o sever migraine that affecting HER twice weekly, she dont want regular medication best ttt you give - triptan - beta bloker - amitrptalyin - bio feedback

12-A child is about to be given flu vaccine, what allergy should be excluded before giving the vaccine? 1) Chicken 2) Egg(sure) 3) Fish 13-. Patient with greenish nasal discharge, was treated before with antibiotic but with no benefit. Management: A. Steroids B. abx

14-Young patient with unremarkable medical history presented with SOB, wheeze, long expiratory phase. Initial management: 1) Short acting B agonist inhaler 2) Ipratropium
| P a g e

3) Steroids 4) Diuretic

15- 38 week pregnant lady came to ER in labor, cervix 4.5 cm dilated, marginal placenta previa. Management: 1) Wait and evaluate fetus 2) SVD 3) C/S 4) Forceps 5) Rupture membrane

16- mother gave bitrh of baby with cleft lip and palate, she want to get pergnant again what is the percentage of recurrence a. 1% b. 4% c. 15%

17- In the appendisits the histology is: a. leukocyte in muscle b. layer of lymphoid c. tumor d. plasma cell

| P a g e

18-old Patient wil LLQ pain, vomiting, fever, high WBC (17.000), tenderness and rebound tenderness a. Diverticulitis ( my ans) b. Sigmoid volvulus c. Appendicitis d. Toxic enteritis

19-Young patient with pharyngitis, inflammation of oral mucosa and lips that has whitish cover and erythmatous base, febrile, splenomegaly. Dx: a. more common in children less than 14 yrs b. EBV c. HZV

20- 10 years old child with rheumatic fever treated early, no cardiaccomplication. Best to advice the family to continue prophylaxis for: a) 1 month b) 3 ys c) 6 ys d) 15 ys

| P a g e

21-The most important factor for smoker to quit is : a) Patient desire b) Give nicotine pills c) Give programmed plan d) Change life style

22-Best method for eradication of entameba histolytica: a. Boiling of water b. Freezing c. Using chlor

23-Old pt, knee pain, mild tenderness and swelling, on xray, narrow space joint with sclerosis : a. Osteoarthritis. b. RA c. Septic arthritis

| P a g e

24- 50 y/o female, operated for ovarian cancer, come to clinic for follow up , abdominal xray show scissor, what to do: a. Inform and refer to surgical. b. Inform and tell her it will resolve alone. c. Call attorney. d. Dont inform.

25- Pic of psoriasis, pink scaly lesion on the elbow,knees and scalp how to prevent flares: a. Avoid sun exposure. b. Avoid trauma c. use steroid

26- Computer programmer, a case of carpet tunnel syndrome, how to splint: a. Dorsiflexion( sure)

| P a g e

27- Gastrictomy post-op 1 day. He have temperature 38.8 & pulse 112. What is the most common cause ? a. wound infection. b. inflammatory mediator in the circulation. c. UTI d. normal 28-Child had chest tightness and cough when exposed to cold and excersise, what to give for prophylaxis : a. B2 inhaled agonist, b. Steroid inhaler. c. Tehyophillin. d. Oral steroid

29- ttt of acute gouty arthritis b. c. d. e. Allopurinol Indometathin(sure) Pencillamin Steroid

30-Most common cause of immediate death in flam burn: a. Inhalational injury. b. Septic shock. c. Hypovolemic shock. d. Other injury.
| P a g e

31- what is the most reliable laboratory to estabilishe diagnosis of Acute gloerulonephritis ? A/ RBC cast in urine B/ increase WBC in urine C/low HGB with normal RBC D/ small shrunk kidney by ultrasound

32- Gualine-Barrie syndrome is closlyassocited with which one of the following : a/ descending paralysis start from upper limb b/normal CSF c/ascending paralysis start from the lower limb ( sure ) d/need ECG

34- man walking in street and saying bad words to stranger , he is not aware of his conditiond , what is the description : flight of idea Deprivation(got 5/5) insertion of idea loosening of association

34-OCP increase risk of which of the following?? 1) Ovarian cancer 2) Breast cancer
| P a g e

3) Endometrial cancer 4) Thrompoembolism

35- Female com with lump in breast which one of the following make you leave him without appointment Cystic lesion with seruse fluid that not refill again?? Blood on aspiration Solid Fibrocystic change on histological

36-Holding breath spell or holding ..which of the following is true A)mostly occurs between age 5-10 B)increase risk of epilepsy C)a known precipitant cuz of generalized convulsion my answer D)diazepam may decrease the attack e) can occur in absence of emotional upset

37-An old woman complaining of hip pain that increases by walking and is peaks by the end of the day and keeps her awake at night, also morning stiffness: Osteoporosis Osteoarthritis

38-old pt c/o bilateral knee pain with mild joint enlargement ESR and CRP normal dx - Osteoarthritis
| P a g e

- Rheumatoid arthritis - Gout - Osteoporosis

39- Most common organism causing cellulitis in the age 6-24 month A. B. C. D. Strepto coccus Heamophilus influ Staph ,,,,, 40-pregnant lady with hyperthyroidism what you will give her : propylthiuouracil methamazole B blocker Radioactive iodine

41-The most common cause of immediate death in flame burn victims: Inhalation of smoke. Associated injures Hypovolemic shock. Septic shock.

42- Pt dx to have aortic stenosis ,,, he is a teacher ,, while he was in the class he fainted,,, what is the cause?? A. B. C. D. Cardiac syncope Hypotention Neurogenic syncope ,,,,,
| P a g e

best of luck salma abubaker

sle - Dr- reem al ahmari


-

Bacteria tricomosis prevent:

-a

Water eradication and -b


| P a g e

Water eradication and

-c

-2-What is the most effective method to prevent the brucellosis infection: -a-Treat the infected people b-Immunize the farmers & those who deal with the animals c -Get rid of all the infected animals -d-Pastralization of the diary products (my answer -3- Which drug conta indication in pregnant women in uti: Floro qinelon -4-Pregnancy 36 w her blood pressure 140/90 , no lower limp edema rst thing :: -aRepeat measure of blood pressure-( my ans) b-cs c-give anti hypertension medication

-5-Child with large periorbital hemangioma , if this hemangioma cause obstruction to vision , when will be permenant decrease in visual acuity a. After obstruction by one day b. By 1 week( my answer c. By 3 months d. By 6 months 6-a Corneal Ulcer, Abrasion other investigation a-fisial felid measurement b-slit limp c- florescen .day ( my answer)

-7-Drug induce urticarea


| P a g e

Hydralizin ( my answer_)

8- old female complain from rash then developed disne and lethargy What is the cause Subheretic dermatitis Urea depositin ( my answer) >>

9- pat dig hypertension obese high NA intak .the cause of hypertention in this pat.. a-high sodiam intak b-obesity

10- child obese what is adevice a-decrease kaloic intak -b-multi factorial interaction-( my answer)

11-patient has atreal fevrelation(af) risk: a-cva( my ans) b-mi

12- patient complain of headache for long time best for treatment: a-betablocher b-biofedback

13- Adult Polycystic kidney mode of inheritance:


| P a g e

Autosomal dominant

14-old patint with depression take ant sycatric medication Take time 2 week 3-4 week( mu ans)

-15-Burnt death du to a-Gas inhalation( my ans) b-Septic shock

16- old female ( ostoprosis) Fear from desk compression best treatment: -aDecreas the wight -b-Take vitamin d and calcium ( my answer)

18- breat feding contra indication in; -a-Tp for 3 month -b-Asyptiomatic hiv ( my ans) 19- verecella ( give second dose only)
| P a g e

19- hal dm type 1 on Dr3 Dr4( my answer)

20 -

picture of large nodule in neck, O/E moce with degilution, what is the dx:

a-lymphoma b-goiter c-thyoglaal cyst ( my answer) -21-Patient with COPD, Which of the following increase surveillance? a) O2 home therapy( my answer) b) Steroids c) Ipratropium

-22- After removing foreign body from the eye apply local: A) Antibiotics B) Steroids( my answer) I thing is wrong

-23intercourse : Try more ) -24-New marred female has vaginal discharge colorless no order no painful: What is this discharge?? Normal after intercore ( my answer)_
| P a g e

breech presentation Down syndrome( my ans)

Q26) pregnant never did check up before , her baby born with hepatosplenomegaly and jaundice : a-Rubella b-CMV( my ans) c-HSV Toxoplasmosis Q27-) old pt complaining of back pain on walking on examination there was stiffness of the muscle and there was some finding on the x-ray , best effective ttt is : Physiotherapy NSAID Surgery

-28-pregnant pt want to take varicella vaccine, what you will tell her ? That is a live vaccine It is ok to take it

-29-Female pt came to you post ovarian cancer surgery one month ago, you did X-Ray for her and you found metallic piece, what you will do ?

Call the surgeon and ask him what to do Tell her and refer her to surgery Call attorney and ask about legal action
| P a g e

Tell her that is one of possible complications of operation Don't tell her what you found

-30-hypertensive pt using sildenafil , in his case it is contraindication to take :

CCB B blocker Nitrate Diuretics

-31-pt K/C of uncontrolled asthma moderate persistent on bronchodilator came with exacerbation and he is now ok, what you will give him to control his asthma :

Systemic steroid Inhaler steroid(my answer) Ipratropium

32-patient epegastric mass show by upper gi ++


| P a g e

Investigation: Endoscopy( my ans) Fall blood test Paruim enema

- -Case of hypothirodism ( cold intleranse _+ wight gain

34-_osteoporosis depend on a-age ( my answer) b-stage Gender

-35-after aspiration of abreast cyst, which of the following indicate that the cyst is benign: Aspiration is clear & the cyst not refill The aspiration is bloody Cytology study shows hyperchromatic changes Cytology study shows fibrocystic disease -36-diagram about osteoporosis shows that from age 70-74 10% has osteoporosis, from age 75-79 40% has osteoporosis, age above 80 70% has osteoporosis, which is true : Women over 80 y has the highest risk for the osteoporosis( my ans) Women from age 70-74 , 10% will develop osteoporosis

| P a g e

37-pt with tingling of the little finger, atrophy of the hypothener, limitation of the neck movement, X-ray shows degenerative cervicitis, EMG study shows ulnar nerve compression, what will you do: Surgical cubital decompression(my ans) Cervical CT scan Nsad Phisotherapy

38-child with drowling saleva, stridor, what is the dx: a-croup b-epigoltites ( my ans)

croup 39--witch is true regarding peretonieal levarge aRbc 1000 b-wbc 50 d blood 2 ml in pregnancy

-40-Testicular pain pain in groin region in examination ther is tenderness no organo megaly A-reffer to surgry B-referr to urologu C- do parim enema
| P a g e

-41-Complication of apendecites A-smoll bowel obstruction B- illus paralytic true 42-* most commen of bleeding on postmenoposal women A-carvical plup B- utrine atony tru 100 % -Bmi 29.5 A- obese B- over Wight 44--- wart from 7 years and now Atyoical Squamus tissue groing Next step ab-Hiv smear c-Resection loop d-hestroctomy

-45-Women have unilateral syst

-46A- increase the dose of diazepam to 40mg B- add phyintion


| P a g e

my ans

C- add phenoparpital C- add carpamizepam

47-patent plunt on hes chest coplane of cyanosis and resonance on one side First step a-o2 ( my answer) b-intunation c-needle

.. -48-Patient with pnemothoax and respiratory distreas Firest do: ab-needile

-49-In cpr: Open the ear way and give to breath Give to breath for 2min and then chest compression -50-cpr CPR; a-2breath increase the chest ( rise chest) b--30-40% c-_ do DC 3 TIME
| P a g e

-51-BREAST feading in neonate A- as soon as possible B-

-52-Phudo goute: a-Phosphateab-Calssum c -Floride

53-Case of ostoartherites!!

-54-Nesseria goneria treatment: Ceftroxone true

55-Pre anal absses treatment: Incision and drange

56-Chronic fatige syndrome: Anti sycatric treatment( my ans) Releve by rest 57-Headech thromping in the eye band like
| P a g e

Sinisites headech Strees Migraine

58-plast AML ALL CML CLL

-59-PATENT ppd test positive for t.b before anti tb treatment

a-repate ppd test b-do mantox test( my ans) -60- smoking with drowel 1-3 2-4 d( my answer)

| P a g e

61-Major depression disorder treatment A- escitalprim true

!!!

62-:Secondary to A-disneses( my ans) B- phopia C-apdominal pain D-tachecardia Chest pain

63-Mechanism of DKA -Hop glucose increase insulin increase keton

64-Appendcites proflaxx Metronidazole Ceftroxone( my answer) Cefroxon

65| P a g e

Aspirin (my answer) t- PA colpidogril heparin

66-Case control study:

67 Decrease weight and exercise benefit

68-If we draw a line through the the long axis of the radius it will pass through the capitalum Anterior pad signs Posterior pad signs(sure )

-69-anal fissure most commen site _ Posterior( my ans) Anterior

| P a g e

70-patient prolonged period defecation painful + blood--A- anal fissure( my answer) B-hemoroid

71-Female dx recently with epilepsy & you gave her phenoparp., she lactate her 10 month old stop lactation immediately (my answer) stop lactation over three weeks

Continue the feeding

72-Qs about apgar score: Heart rate is significant (my answer)

73-Lactating mother with mastitis trratment: Doxcycyclin Ceftroxone Cefoxine Metronidazle


| P a g e

74-child malnutrition low protin+ no edema a- kwasherker b-muresmus 75-Heavy smoker came to you asking about other cancer, not Lung cancer, that smoking increase its risk: Colon Bladder( my answer) Liver

76adenoid hypertrophy. Beside adenoidectomy on management, which also you should do: Myringotomy Grommet tube insertion( MY answer) Mastidectomy Tonsillectomy

77-there is outbreak of difteria and tetanus in community , regaring to pregnant woman: contraindication to give DT vaccine if exposed , terminate pregnancy immediately

| P a g e

give DT vaccine anyway 78-Pt with scoliosis, you need to refer him to the ortho when the degree is: 5 10 15 20( my ans)

79-Most common cause of intra cerebral hemorrhage Av malformation( my ans) Pre exicting anurezem

-80-All are primary prevention of anemia exept: A-iron and folic acid in pregnancy and postnasal( my answer) B-iron food in children C-limitation of caw milk D- genetic screen for herdateriy anemia

81-Old female with itching of vulva , by examination there is pale and thin vagina , no discharge . what is management a. Estrogen cream

| P a g e

82-Rebound phenomenon definition??? ...

83-Holding breath holding Generalized convultion

-84-serious otites media

85-otites externa or media ( tympanic membrane intact and discharge in urical)

86-female have itching in valve and thigh Contact dermatitis And other not remember

87-All in hypokalemia exept: Hyper osmolar coma Phention toxicity Musle paralysis

| P a g e

88-In paracetamol toxicity: Pencelinemia N acetyl>>>true 100% K intake Dexoamin..

89-_ chilled with throat congestion and mutable Ebv Hsv Adenovirus

90-Child rach spreed qiuqly +fever+drwesnes fever..... Rubella Measles .... fever

91-pericardites Case sudden Pain in chest increase with movement.. Best investigation
| P a g e

Ecg my answer Cardiac enzyme

92crepetation on both lung a-pulmonary embolism b-pnemonia( my answer) c-mi reg d-orta reg

case cardiac 93-canion a wave fistula:)( BaD Q

94-Uric acid in body how the body removed by ab-increase metabolism of uric acid in liver c-execretion of uric acid by lung Q kwashecor syndrom 95-Child runny then developed pain in thigh no redness or tenderness
| P a g e

Best thing to do A- elevated the leg and cold compression( my anse) B- splint C-surgry -96 ) ( a-obsession b-post partum sycosis( my answer)

97-Kawasaky syndrome; Sterbery tong

98-Coarregation of orta all true except Skeletal deformity -a Upper limp hypertion -b Systolic murmur on all pericardium -c Obe 3-5, Gyna2-5 Optha 2-3 ortho 3-3 Ent 2-3
| P a g e

Derma1-3 Sycatric 4-5 Family 12-15 Pedia 12-15 Medicin 17-25 Basic 2-3

My SLE Exam Foad Bukhari 31-January-2012

1-In DPL is strongly positive if the result show; a- RBCs is 1000 HPF/CC b- presence of blood in the drainage tube c-I forgot the rest of choices

2-pt is pregnant in 6 m brought to u with sever abd. Pain, in US u find a life birth with fibroid, what will u do; a- pain managmets b- Myomectomy c- Hystrectomy d- Pregnancy termination

3- 17 yr old male pt withhx of multiple drug injection, otherwise healthy , came to ur clinic . what is the appropriate investigation that u have to do for him, a-ViralHep B
| P a g e

b- HIV c- Strep. Viredance d-MRSA

4- pt had sever itching with circular wheals and scar in the middle of them, then had swelling in his mouth and lips a- Dermatographia b- Solar urticaria c- coldurticaria e- coldurticaria and angiodema

5- hemangioma in the back with 2 cm diameter what is ttt; a- excision b- biobsy c-observation d-i forgot

6- hypocalemia will be with all of the following except; a- ATN (acute tubular necrosis) b- Metabolic acidosis c- Chronic diarrhea d- Addison disease

7-which of the following choises is true about DM in KSA; a-IDDM is about 75% b-Most of DM pt are obese
| P a g e

8- pt with DM that is controlled, want to be pregnant what u have to advice her about DM a- to be controlled prior to conception b- in 1st trimester c- in 2nd trimester d- in 3rd trimester 9- pt with preeclambsia what is true a- DM is risk factor b- present with headache and seizure c- mostly and rapidly become eclambsia d- come with multigravida rather than paragravida.

10 female pt, pregnant in 38 wk , come with bleeding and abdominal pain , what is the Dx? a- placetaabrubtion b- placentaprevia c- fibroid d- I forgot

11- pt obese, hirsutesm, insulin resistant, skin hyperpigmentation, US showed small multiblepolucystic ovary; a- Klinfelter syndrome b- kallman syndrome c-stein-leventhal syndrome

12-pt with LBBB, but has normal heart structure with good rate and rhythim, will go under dwntal procedure
| P a g e

a- give abx before b- giveabx after c- no need to give

13-20 year old male had been stabbed on midtriceps , one week later greenish discharge , On microscopic examination of this greenish fluid show gram positive cocci in chain ? Streptococcal gangrene Chlostrideal gangrene Fourniers Gangrene Meningocemia

14-pt with sudden SOB , had postrioinferior MI, what is the cause; a- pulm.Embolesum b- acute MR c- aute AS d- Arrythmia

15- best prevention of dust mites , Cooling clothes Humid house with 80 % humidity
| P a g e

Boiling cloths and linens

16-side effect of prolong 100% oxygen a-retrosternal chest painting b-sizure c- depression

17- -sicklerpt came with painful crisis what is the RX: a- mangement outpatient + analgesic b-hospitalization +analgesic c-reffer to 3ry center

18-drug useful for pts with idiopathic anovulation: a- chlomophine 19- Proctalgiafugax

| P a g e

SLE Exam January 2012 Ali Atwa


1. ??Difference between keratitis and uvietis:?? Photophobia, cilliary flush, constricted pupil 2. OCP cause hyperkalemia: I dont remember the choices Drospirenone in the internet 3. ??Lactating women with mastitis: Continue lactation Clean with alcohol Surgical drainage
| P a g e

Compression 4. Major risk for stroke is HTN DM Smoking 5. Women with IDDM advised to make schedule for glucose level FBG: 283 after lunch: 95 3pm: 184 Increase short acting insulin dose Decrease short acting insulin dose Increase long acting insulin dose Decrease long acting insulin dose 6. ??Women with mild preeclampsia : Monitorin Labetlol Diuretic 7. EBV: Infectouis monocliosis 8. ??Patient with sensinueral hearing loss and vertigo then develop numbness MRI showed mass in cerrbellopontine : Acoustic neuroma Meningioblastoma 9. Patient with earpain and congested nose O/E red tympanic membrane +ve insuflation reflex otometry showed peaked wave Antiobiotic Myrigtomy for indication and diagnosis 10. ??Child with vomiting and diarrhea Mild dehydrated child : ORS Antiemetic + ORS 11. Duedonal obstruction in child show: Double bubble sighn 12. ??Aspirin mechanism COX inhibitor lipo oxigenase 13. Increase survival rate in HF Enalpril Isosordil Fuersmide Spironolactone 14. ??Mother hebatitis b brought her child +ve HBV vaccination: DTP Hib pneumcoccal
| P a g e

15. ??Pt heavy smoker and have emphysema presented with pneumonia regarding vaccination Pnumococcal and influenza vaccine after 2 wks Pnumococcal and influenza vaccine now Pnumococcal after 2 wks and influenza vaccine now Pnumococcal alone influenza vaccine alone 16. ??25 year old presented painless scrotal mass with progressive increase in size on exam no transimulination Intraoperative and percutanous biopsy Refer to urology Reassure and appointment ater 1 month Testicular tumor is radiosensitive embryonal carcinoma, (2) teratoma, (3) choriocarcinoma, and (4) yolk sac tumor. (5)seminoma 17. ??65 year old pt with hallucination disorganization disorientation 2 days after femoral bybass surgery symptom been flactuant in 2 next day : Multinfarct dementia Alzehimer dementia delirium 18. ??Child with dark urine generalized body swelling hypertension HTN ur next step: urin analysis RFT Renal biopsy Urine sedmintation US 19. Case of osreoarthritis increase with moce , decrease with rest. 20. Treatment of rosacea : tropical steroids or topical metrendozoidz plus systemic abx just in sever form and sys metrendizol just in extremely sever form Child with rash lock like honey : Impetigo 21. Relation of indirect hernia and spermatic cord Superiolaterl, medio lateral 22. ??Chid with clavicle fracture Monitor Internal fixation Union is unlikely
| P a g e

23. Patient with parathesia in little fingerand decrease motion in neck radiology show degeneratve changes in cervical region EMG showed entrapment in the elbow: Spondolysis Brachial injury Surgical relive for entrapment NB I am not sure about Mcq 24. Pneumothorax: needle insertion 25. Pneumothorax: thoracotomy 26. Child with beriberi : vit b1 , vit b2 , vit b12 27. Patinent had unprotected coitus presented with joint pain culture showed Gve diplococcic: Gonerheal arthritis Non gonerheal artheritis RA 28. Moderate persistant asthma on b agonist inhaler: Add corticosteroid inhaler 29. Female with discharge microscopy showed clue cell + KOH test Bacterial vaginosis Treatment of Acne Valgares is 1 -topical tretnoin an clindamycin or erythro and sys tetracycline or erythromycin and syst Isotretenoin. 30. Medication increase reflux esophgitis Theophyline, ranitine , plasil, ampiciliin 31. +ve lachman test indicate injury in: ACL tear, PCL tear, meniscus tear, medial CL, lateralCL 32. Best way to eradicate Plaque: Rodent eradication , spray insect side hand washing 33. Patient exaggrat his symptom when people around : Somatization , malingering, depression 34. Child with HX URTI presented with bilateral knee pain Rheumatic fever, glomerulonephritis, 35. Patient with band like headache associated throbbing pain in eye : tension headach, migraine, cluster 36. 14 year old female with bilateral breast mass no family hx of cancer: fibroadenoma, fibrocyctic changes, breast cancer 37. Benign feature of breast lumb Bloody discharge Solid mass Cystic not appear after aspiration With fibryocystic changes
| P a g e

38. Human bite the most common position propose to infection : clenched, dependant, 39. Patient with terminal overy cancer after surgery radiology found clamp in her abdomen : Dont inform her bcz she is terminal Inform her and refer her surgery

My exam 18-2-2012 Sakinah Al-saeed


The new Qs: 1-What is contraindication for giving welbutrin in smoking sessation ? History of seizure (my answer) Hemolytic anemia

2-Acyanotic middle age man radiologycally come with prominent pulmonary arteries and vascular marking ,most likely Dx? VSD ASD Coarticarion of the aorta Truncus arteriosis Pulmonary valvular stenosis

| P a g e

3-Patient with eye pain not relieved by patching when he came you find red eye with sclera injection with cloudy anterior chamber,DX Retinitis Uvietis( my answer) 4-Patient come you find perforated tympanic membrane with foul withish discharge dX? Otoseclerosis Otitis externa Cholestitoma 5-Patient with nose truma with pain ,x-ray shows undisplaced nasal fracture ,your management Refer to ENT surgon Give analgesic Anterior nasal packing 6information)dx? Erythema mutiforme Fixed drug eruption Pytriasis rosea 7-Toddler with sever skin itching involving the abdomen hand and face papulvesicular Chicken pox Dermatitis herpitiform 8-Child shows spiral fracture of arm management Refer to orthopedic Open reduction and internal fixation(my answer) 9-Eye screening in DMI Now and annually
| P a g e

hand and soles (no other

Now and every 10 years

10-Sodium content in normal saline (0.9) 50 70 90 155 (the correct answer) 200

11-

th

month had vaginal bleeding .past history of hypertention

Come now with abdominal pain dx Placenta previa Ectopic pregnancy Abrubtioplacenta(my answer) 12-patient complain of irregular periodand excessive fasial hair .her mother had the same. BMI 36 normal estrogen increase testerone increase LH and decreased FSH,and her urine shows 17 hydroxysteroid Dx Chushing Polycystic ovary Adrenal adenoma(the correct answer) 13-The most important sign the physician should look in primary autonomic insufficiency ? Orthostatic hypotension (the correct answer) Sinus arrhythmia Horner

| P a g e

14anagment Oral electrolyte replacement (my answer) Internal cold water Warm intravenous fluid tepid water 15-patien work in hot weather come with clammy cold skin ,hypotensive tachycardiac heat stroke(my answer) heat exhusion

16-patient with red blood cell disorder ,with family hx of thalassimia to confirm the dx increase the level of A2 gentic 17-patient came with osteoprotic thorasic vertebral fracture t score for vertebra -2.6 z score the hip osteopenia osteoporosis established osteoporosis(my answer) normal bone mass atient has

WHO classification :
| P a g e

(Normal boneT-score greater than -1 OsteopeniaT-score between -1 and -2.5O steoporosisT-score less than -2.5 Severe (established) osteoporosisT-score less than -

18With anterolaterl lead shows st elevation, providing no tPCI in the hospital Management Streptokinase ,nitroglysrin ,ASA,beta blocker(my ans wer) Nitroglysren ,ASA ,heparin beta blocker Nitroglysren ,ASA,beta blocker Alteplase , Nitroglysren , ,heparin betablocker 19-Patient came with dysphagia interferer with daily life ,past history of hymphoma treated back and he did not follow in the last year,Face congested dx : Thorasic aortic anuresm Abdominal aortic aneurism Svc obstruction (my answer) IVC obstruction 20-Increase the survival in COPD patient Contious oxgen inhaled brochodilater steroid 21- most common organism causing pneumonia in adult streptococcus pneumonia
| P a g e

legionella hemophillus inflanza type b

22-what is questioneer used to diffrentiet between sleep apnea and snoring? Mitchigan Epworth (the correct ) cooner

SLE Exam 1/2/2012 By : dr Ohoud M Al-k


1-Diagnostic peritoneal lavage : 2 ml of blood initial aspiration 2 ml of blood in pregnant WBC in cc/ Can't remember 2 - varicilla vaccine what true Not given in first trimester preg Contain live attenuated bacteria | P a g e

3- painless lump in neck in child Hodgkin lymphoma Pharhngitis Infectious mononucleosis 4- surgery should done immediately in croons dis when : Fistula Intestinal obs Abdominal mass Intes bacterial overgrowth 5- child with bilious vomiting with yellow stool ,, abdominal dissension He passed stool immediately after birth . Harsh sprung dis Mid gut volvus Can't remember 6 - 3 months infant with tachpnea . Resp distress . x- ray shows lower and mid lobe infiltration , opaque right lung and shifted trachea to left .. Responsible organism : H inf Pneumococcus ... 7 - acute diarrhea with epithelial infiltration E- coli Salmonella Cholera Rota virus Shigella 8 - old pat with bilatral hydronephrosis and loin pain : Pelvic cancer Prostatic hyper trophy Bladder .. Tumor ?? 9- old pat with pain after walking no edema Claudication 10- old pat with tAchcardia pulse 150 otherwise normal TSH Stress ECG 11- to deffrentiate bet sinus tachycardia from atrial futter | P a g e

Carotid massage ... Artery massage Can't recall 12 - mitral stenosis LA hyper trophy with decrease plum .. LA hyper trophy with .. 13- Patient with panic attack .. Something related to secondary mechanism not symptom Epigastric pain Chest pain Dizziness TAchcardia 14- child with vaginal discharge green .. Bad odor , pelvic exam normal Foreign body Trichomonas 15- emprical treatment of peptic ulcer h. Pylori Omperazole , clarithro , antihero 16- anterio- latral located placenta not palpated by pv Marginal Low lying Normal lying Partial low lying 17- first sign of increase ICP hTN Decrease Level of consciousness 18 - increase igG in CSf Multiple sclerosis Duchine dystrophy 19 - pregnant 38 gestation bp 140/90 Cs Induction Observe bp reading Give anti hypertensive 20- patient was treated for glaucoma now presented with SOB , ... The drug reasons able for these symptoms : | P a g e

Timlol Pilocarpine 21 - pt live near industries came with attack of SOB the prophylactic B agonist. Oral steroid 22- polygonal rash flat topped : Lichen planus - sure 23 -Most important to instruct pt about Lyme dis : Kill ?? Insect Wear long fiber clothes ... 24- child with ear pain red tympanic mem and -ve insulation test Acute otitis media TRUE Secretory otitis media Chronic om 25- scenario of panic attack .. Treatment Benzodizepine SSRI .. 26 - young lady with emphysema A1 anti-trypsin def 27- pt with hemoptysis , night sweat . Loss appetite .. X- ray apical cavity Post primary tb Pneumonia 28 - vesicular rash Checken pox 29- most common cause of pediatric falure to thrive Cystic fibrosis Psychosocial Protein &Milk - intolerance 30- pateint with pustule arround the mouth the organism is herpes simplex what is the treatmen Oral ab Topical ab | P a g e

Acyclovir Steroid ( cant remeber if topical or oral 31- true about gastric lavage Not helpfull after 6 hours of aspirin ingestion 8 hours after .. Ma athkr 32 - drug cause gout Hydrochlorothizide Furosmide 33- mechanism of OCP Inhibit estrogen spur in mid .. & ovulation 34- rt lung has 2 pulm veins 35- scinario of cholecystitis what is the most theraputic procedure ERCP Cholecystectomy 36- pt with hoarsness of voice . Next step Laryngoscope 38- high pitch diastolic murmur Ms Mr Mvp 39- pateint came after RTA with heavy bleeding upper limb ABC Call orthopedic Press the bleding site Take to OR 40- sickler pateint u treated with antibiotic for UTI u will discharge him with Pencillin .. 41- on going treatment for sicklers iron therapy Pencillin with immunization 42- lab values all r normal except Na ( hyponitremia ) treatment | P a g e

NS with kcl at 20 cc / hour NS with kcl at 80 cc\ hour 1/2 ns ... 43- mechanism of cushing syndrome Increase ACTH from pituitary adenoma Increase ACTH from adrenal ..... 44- pt diabitic retenopathy the most u will deal with HTN with smoking TRUE 45 - 27 years old with DM 2 she already wears glasses u will follow up her after : 6 months 12 months 46- drug used in treatment of CHF which decrease the mortality B blocker Verapamil Nitrates Digoxin 47- CHF with pulm edema . Treat with Furosmide Thizide ... 48- child with hematuria 15 RBC whatnext urine cytology Repeat urine for rbc and protien Renal biopsy Cystoscope 49- pregnant lady with low back pain .. All gynecologic causes ruled out what to give : Paracetamol Profen

SLE Exam
| P a g e

28/2/2012 By : dr Eyad Al-Hudaithi


: Q1/True about dermatomyositis associated with inflammatory bowl dz - indicate underlying malignancy - present as distal muscle weakness - : Q2/ newborn presented with conjunctivitis and O.M , whats the treatment i guess this is a case of infection with chlamyedia intrauterine , they asked about several AB /: there is no dyoxycyclin nor erythromycin ( Q3/ Differences Btw dementia and delirium (read about it !! i picked Amnesia Q4/what kind of questionnaire you would use to differentiate btw central obstructive sleep apnea and snoring %$@@#%$# Michigan Questionnaire - connor questionnaire - Q5/ about shoulder that is Adducted and internally rotated (what is the mechanism of (dislocation Anterior subclavicular- Ant - Post- .POst - : Q6/ celiac dz . all should be avoided exept wheat - oat - rice - Q7/female has primary amonarhea , webbed neck , low hair line Turner = Q8/anteriolateral placenta , term pregnancy , can't be felt when examiner admit his
| P a g e

: finger through the Cervix Low set placenta - Marginal - normal - complete placenta previa- .Q9/Tinea capitis RX start Nystatin - (wood's lamp (true - : Q10/nodule don't do anything so you don't rupture it ( cryotherapy (true by luck : Q11/Rosacea case (redness patch on face with talangectasia ) what is the ttt Dyoxycyclin Q12/ child smile at at birth - month - months - months - columnar cell surrounded by Sq << Q13/ pt has GERD for 5 years , now EGD reveals cell Sq .c.c - Adenocarcnoma - barret esophgous- : Q14/ old pt , has loin pain , U/S reveals bilateral hydroneprosis , whats the cause prostate cancer - bladder cancer - urethral stricture - Q15 / infertile women for 3 years with dysparunia endometritis - Salpengitis - (endometriosis (True -
| P a g e

Q16/ pt has Lt lower Abdominal pain , Fever , constipation CT reveals thickened loop :and little perianal fat , whats appropriate to do start AB - call the surgeon for immediate OP - give laxative - barium enema - : Q17/ Typical scnario about migraine and pt doesn't want DAILY medication bio feedback - BB - CCB - inhaled ergometrin - : OCP protective Against / Breast ca - Ovarian Ca - endometrial - ( H.influnza .. (READ Abt Rx << Culture / : leukemia case .. lab (pancytopenia , lukocytosis , +ve myeloperoxidase ) Dx is / ALL - AML- - - hematology case prophral blood smear reveals target cell =SCD / : difficult consultation/ use medical term - open ended Q - close Ended Q = True - - lactating mom recently dx to have epilepsy on Phenobarbital , her child is 10 / months now , whats appropriate to tell her stop immediately - wean him for 2 weeks - give after 8 hrs of -
| P a g e

BF as much the baby and mother want - ( True about Mallory-Weiss sx / MCC of GI bleeding during pregnancy - resolved spountounsly - cases of GI bleeding is d/t this Dz / - - . old pt with progressive weakness of hand grip , dysphagia / MG dz - Mysthenia Gravis

Breast made easy


self breast exam = 2-3 days after period (monthly ) MC tumor in those <35 = fibroadenoma if related to menses = Fibrocystic changes younger than 50 , has boldly discharge , no masses = Intraductal Papiloma MC uni breast cancer = infiltration ductal ca .. bi = lobular young had painfull lump that increase with menses = Fibrocystic changes menses pain = NSAID recurrence of breast cancer = # of LN painless Breast lump ,, US reveals cystic changes , FNA reveals clear fluid = reassure

| P a g e

SLE EXAM Aseel Al-Osaimi Al-Tamimi

| P a g e

Alsalam 3laikm, I had my exam !! Which unfortunate was difficult for me I count the repeated Q's it was about 18-20 only Elmhm now I didnt used to remember Q's , I'll try my best though :) w sam7oni 3al89oor

- pt had stroke , after that he lost vision in the left eye , where is the lesion : A. Frontal lobe *** B. Occipital lobe C. Parietal lobe D. Temporal lobe

2- pt with blephiritis , with hx of acne rosecia but with no sign of keratitis , wt you will give him : A. Topical chlormphencol B. Oral doxcyclin C. Topical gentamicin

3- pic of optic nerve cupping A. Gluocomatous cupping *** B. Optitis C. Optic nerve atrophy

4- not palpable in normal head and neck exam.


| P a g e

A- lymph node *** B. Thyroid C. Hyoid bone D. Parotid E. Submandibular gland

5- pt known case of stable angina for 2 years , came c/o palpitation , Holtis monitor showed 1.2mm ST depression for 1 to 2 minutes in 5-10 minutes wt your Dx A. Myocardial ischemia B. Sinus erythmia C. Normal variant

6- drug shouldn't be prescribed in compination A. Digoxin and levodopa B. Tetracycline and Almenium hydroxide Sorry I don't remember the rest

7- after Stinning bite a 7 year old boy came c/o abdominal pain , fever , diarrhea ,maculopapular rash over the palm and soles head and abdomen , the Dx A. Lyme disease B. HSP Sorry I dont remember the rest

| P a g e

Abdullah sultan SLE, jeddah


26/2/2012

1- True regarding hiatal hernia: a. Morning vomitin. b. Increased with pregnancy. c. Dark skin pigmentation d. I dont remember e. I dont remember. 2- 6 y/o boy, mother HbsAg +ve , and he is HbsAg +ve , take only BCG , what vaccine to give him: a. DTP, HBV , MMR, OPV. b. DTP, MMR, OPV, Hib. c. Td, HBV, MMR, OPV, Hib. d. Td, MMR,OPV. 3- Acute loss of body fluid into abdominal cavity: a. Sepsis. b. Hypovolemic shock. c.
| P a g e

Cardiogenic shock. d. Neurogenic shock. e. Emesis.

4- School boy, obese , mocked at school, he DESIRES to take pill to sleep and never wake up again, what to do: a. Refer him immediately to mental professional. b. Give fast working antidepressant. c. Tell him he will grow ( and something not related). 5- For malaria prevention: a. Clothing disinfection. 6- Old bed ridden pt, with fever , blood culture reveal enteroccus , what it the source for it: a. UTI ( CORRECT). 7- Best method for eradication of entameba histolytica: a. Boiling of water.

| P a g e

8- TTT of H.pylori infection: a. Omeprazol 2 weeks, clrithromycin and amoxicillin 1 week ( correct). b. Ranitidine , erythromycin, metronidazole for 2 weeks. 9- Boy felt down on his elboe , x-ray : a. Posterior fat bad ( correct). 10- Non medical TTT of osteoarthritis: a. Muscle excersie ( correct). b. Spine manuplation. c. Analgesic cream local. 11- Female with migrane, dont want medical therapy: a. Biofeedback. 12- Old pt, knee pain, mild tenderness and swelling, on xray, narrow space joint with sclerosis : a. Osteoarthritis. 13- Old pt , right iliac fossa pain, fever for 2 days, diarrhea, on CT thickness of intestinal wall , what to do : a. Urgent surgical referral . b. Antibiotic. c. Barium enema. d. Colonscopy.

14- 9 day old infant , presented to well baby clinic, with mild jaundice and yellow scaling on face and chest, otherwise examination normal , on breast feeding, doing well according to mother, what is the cause of his condition: a. Brest milk jaundice. b. Occult infection. c. Hemolysis of hematoma for birth trauma. 15- What is the role of VIT C in wound healing:
| P a g e

a. Collagen synthesis ( correct). 16- Greatest risk factor for CVA : a. HTN.

| P a g e

17- Young female , with typical feature of polycystic ovary sydrom, want to get pregnant , what to do: a. Diagnose as PCO , and weight reduction. b. Diagnose as adrenal hyperfunction, and start supreesion therapy ( correct). 18- 5 y/o girl , presented with sore throat, and serosangious vaginal discharge: a. Foreign body. b. Chalmydia. c. Gonnorhea. d. Streptococcus infection ( correct). 19- Colon cancer stage 3, when to give chemo: a. ASAP. 20- 50 y/o female, operated for ovarian cancer, come to clinic for follow up , abdominal xray show scissor, what to do: a. Inform and refer to surgical. b. Inform and tell her it will resolve alone. c. Call attorney. d. Dont inform. 21- Old pt , k/c of COPD, low Po2 , high CO 2, what is the dervitive for respiration: a. Hypoxia. 22- Acute eye pain, decreas vision, conjunctival injection, constricted pupil, opaque lens with keratinzation, cells in aques humor : a. Anterior uveitis. 23- Ttt of the previous question: a. Steroid and cyclopentolate. 24- Blow out fracture : a. Parasthesia in superior orpital ridge.. b. Exoptlamos. c. Diplopis and upward gaze ( correct). d. Air fluid level in maxillary sinus. 25- Pregnant, 38 weekes, presented with abdominal pain and vaginal bleeding: a. Abruption placentea.
| P a g e

26- Pregnant full term , in delivery, 7 cm dilation, show late deceleration, what to do: a. O2 and change position. 27- Post partum , bleeding for more that 2 hours, vitals non stable, what to do: a. Ergotamine. b. Blood and iv fuid. c. A drug ( I remember like oxytocin) + IVF.

| P a g e

28- 32 Y/O female, c/o irregular cycle , sometimes every 2 month, and sometimes twice a month, with heavy bleeding when it comes, 15 bads in the day: a. Menorrhagia. b. Polymenorrhea. c. Hypomenorrha. d. Metrorrhagia e. Menometrorrhagia. 29- Your female , c/o of very fast period , twice a month: a. Polymenorrha. b. Hypomenorrhe. c. Olgomeorrhea. 30- 38 y/o pregnant, 8 weekes, want non invasive screening: a. Amniocentesis. b. 1st trimester screening. c. 2nd trimester screening. 31- A question about the method of taking pap smear !! a. Vaginal sample. b. 3 samples. c. 1 sample from os. d. 1 sample from endocervics. 32- A question about ECG , very long, each answer is a case !!

33- ( I dont remember ) Likelyhood ration of 0.3 : a. High increase. b.


| P a g e

Little increase. c. No increase. 34- Pic of psoriasis, how to prevent flares: a. Avoid sunburn. b. Avoid trauma

| P a g e

35- Young male , in cold weather, have a single patch in his chest, whitish, when came to hot weather, it became hypopigmented : a. Tenea versicolor. b. Other option. c. Other option. 36- Old with rectal bleeding, external hemorrohide, what to do: a. Remove. b. Colonscopy. c. Follow up after 6 month. d. Rigid sigmoidoscopy then remove. 37- Young boy with rectal bleeding, pale, anemic, how to investigate: a. Isotope scan

38- A case of cholycystitis , how to confirm: a. US. b. Something scan. c. Xray. d. Ct. 39- k/c of SCA, have URTI, then suddenly have chest pain, lopar infiltrate, WBC 18000, Hg 7 , fever , what is the cause for his condition: a. PE. b. Strepto infection. c. Acute chest syndrome.
| P a g e

40- Young boy , k/c of SCA, had UTI and ttt well, what to give prophylaxis : a. Amoxicilline. 41- A case of cushing syndrome, to diagnose, we do ACTH challenge test, what it the pathophysiology of this test !!!!! 42- Young male, diagnosed with MITRAL REGURGE by auscultation , want to do dental , what to do: a. Give amoxicilline. b. Give augmentine. c. Do ECG. d. Do ECHO. ( THERE was no option for DO NOTHING). 43- Male m diagnosed with mitral prolapsed, echo free, want to do dental work , what to do: a. Nothing . 44- Computer programmer, a case of carpet tunnel syndrome, how to splint: a. Dorsiflexion ( correct) 45- Most common cause of non traumatic fracture in osteoporosis: a. Verterbral fracture. 46- Female , on fast food diet , what to gave: a. Ca + folic acid b. Vit C + folic acid c. Zinc + folic acid d. Zinc + Vit C

47- Lactating mother, given phenoparpitone for epilepsy, what to do: a. Stop lactating. b. Lactation after 8 hours of medication. c. Continue. 48- Using gastric lavage : a. Useless after 8 hours of ASA ingestion b. No benefit after 6 hours of TCA ingestion
| P a g e

c. Patient should be in the right lateral position .

49- Young aged male presented to ER after blunt trauma to Abdomen, CT scan shows intramural hematoma: your management is a. Lapratomy with evacuation of the hematoma b. Dissection of duodenum c. Observation. 50- Gastrictomy post-op 1 day. He have temperature 38.8 & pulse 112. What is the most common cause ? a. wound infection. b. inflammatory mediator in the circulation. c. UTI d. normal

51- Patient after accident, the left ribcage move inward during inspiration and outward during expiration: a. Flial chest. 52- In cervical LNs there are well differentiated thyroid cells, during operation you find no lesion on thyroid what will you do next a. Total thyroidectomy b. Total thyoidectomy + radical cervical LNs dissection c. Total thyroidectomy + specific LNs dissection d. Thyoid lobectomy with ----

| P a g e

53- Old male with neck stiffness, numbness and parasthesia in the little finger and ring finger and positive raised hand test, diagnosis is: a. Thoracic outlet syndrome b. Impingement syndrome c. Ulnar artery thrombosis d. Do CT scan for Cervical spine 54- 18 months old came with bite by her brother, what you will do: a. Give augminten b. Give titunus toxoid c. Suture

| P a g e

55- Mother has baby with cleft palate and asks you what is the chance of having a second baby with cleft palate or cleft lip : a. 4 % b. 25 % c. 50 % d. 1 %.

56- Old patient male, presented with acute hematuria, passing red clots and RT testicular pain and flank pain : a) Testicular Ca b) RCC ( renal cell carcinoma ) c) Cystitis d) Epidimorchitis. e) Prostitis. 57- man use saldinafil (Viagra), to prevent hypotension you should not use a-nitrate b-B blocker cACIE d-CCB 58- pt his MBI = 24 kg he is a. noramal weight b. over weight c. morbid weight d. mild weight. 59- Adolescent female counseling on fast food. What you should give her: a. Ca + folic acid b. Vit C + folic acid c. Zinc + folic acid d. Zinc + Vit C

| P a g e

60- Female patient did urine analysis shows epithelial cells in urine, it comes from: a. Vulva b. Cervix c. Urethra d. Uerteral stone.

61- Newly married woman complain of no pregnancy for 3 month with unprotective sexual intercourse : a. Try more ( infertility is defined as no pregnancy for one year) 62- Which of the following is the best treatment for Giardiasis: a. Metronidazole 63- Adult polycystic kidney disease is inherited as: A. Autosomal dominant . B. Autosomal recessive . C. X linked .

| P a g e

64- child with aspirin intake overdose ...what kind of acid base balance: a. metabolic alkalosis wt respiratory b. metabolic acidosis wt respiratory alkalosis c. respiratory alkalosis with metabolic acidosid d. respiratory acidosis with metabolic alkalosis 65- Younger diabetic patient came with abdominal pain, vomiting and ketones smelled from his mouth. What is frequent pathophysiology: a. Insulin overdose. b. Insulin missd. c. Oral drugs over dose. d. Oral drugs missd. 66- Old female with recurrent fracture , Vitamen D insufeciency and smoker . which exogenous factor has the gretest exogenous side effect on osteoporosis. a. Old age b. Smoking c. . Vit D insufeciency d. Continue smoking e. Recurrent fracture 67- child with moderate persistant BA On bronch.dilat inhaler. Presented with acute exacerbation what will you add in ttt: a. Corticosteroid inhaler b. Ipratropum bromide inhaler. 68- Child had chest tightness and cough when exposed to cold and excersise, what to give for prophylaxis : a. B2 inhaled agonist, b. Steroid inhaler. c. Tehyophillin.

| P a g e

d. Oral steroid. 69- Most common cause of intracerberal hemorrhage: a. Hypertensive angiopathy b. annurysm c. AV malformation 70- Pt presented to ER with substernal chest pain.3 month ago, pt had complete physical examination, and was normal , ECG normal, only high LDL in which he started low fat diet and medication for it. What is the factor the doctor will take into considerations as a risk factor: a. Previous normal physical examination. b. Previous normal ECG. c. Previous LDL level. d. Current LDL level. e. Current symptom. 71- The useful exercise for osteoarthritis in old age: a. to maintain muscle and bone Low resistance and high repetion weight training 72- In IV canula and fluid: a.Site of entry of cannula is a common site of infection. 73- year old male found to have hepatitis b surface antibodies : a. Previous vaccination b. Previous infection c. Active infection. 74- Elderly patient presented by SOB , rales in auscultation , orthopnea, PND, exertional dyspnea, what is the main pathophysiology : A. Left ventricular dilatation . B. Right ventricular dilatation . C. Aortic regurgitation. D. Tricuscpid regurgitation .

75- Child with URTI is complaining of bleeding from nose, gum and bruising the diagnosis is:
| P a g e

a. Hemophilia A b. ITT. 76- compilication of rapid correction of hypernatremia : a.brain edema

| P a g e

77- a case of child drink corrosive material , hypotensive, pale , drooling, what to do: a. establish airway. b. 2 cups of milk. c. Gastric lavage 78- A question about allopurniol : not to use in acute attack. 79- Old male c/o sudden chest pain, decreased chest wall movement, hemoptysis , ECG changes of S1 Q3 T3 , what is most common diagnosis: a. Acute MI. b. Polmunary embolism. c. Severe pneumonia. 80- Q about drug of choice in general anxiety disorder ( they used names, not classification, I dont remember). 81- A boy who was bitten by his brother .. and received tetanus shot 6 month ago and his laceration was 1 cm and you cleaned his wound next you will: a. give augmentin b. suture the wound c. give tetanus shot d. send home with close observation and return in 48 hr 82- Child his mother let him to go to bathroom befor sleeping and avoid drinking befor sleep this mangment of: a. Enuresis 83- Most common cause of meternal mortality from the following: a. Syphilis. b. Herps. c. CMV. d. Toxoplasmosis. 84- Most common cause of immediate death in burn:

| P a g e

a. Inhalational injury. b. Septic shock. c. Hypovolemic shock. d. Other injury. 85- Case of old male, heavy smoker, on CXR there is a mass , have hyponatermia and hyperosmolar urine , what is the cause: a. Inappropriate secretion of ADH. b. Pituitary failure.

| P a g e

c. Other options I dont remember. 86- Young male, have seasonal sneezing, rhinorhhea, conjunctivitis, what to give: a. Antihistamine. b. Decongestant. c. Local steroid. 87- perinatal mortality: a. Still birth and neonatal death within 6 week not sure b. Neonatal death in within 1 week. c. Number of still birth and death in the 1 st week of life. 88- Old pt, his wife died, depreesed , loss of intrest , loss of appetite, for 6 weeks , and feeling guilty , because he didnt take her to a doctor before her sudden death, and thinking of he is the responsible for her death : a. Breavment. b. Depressive disorder. c. Adjustment disorder with depression.

| P a g e

New in 4 Edition
A)Sure
st

th

and 3 Editions Done by : Dr.Afnan Rajeh

rd

,2

nd

| P a g e

In Name of allah I wrote the Questions asconcise way ,, I Forgot a lot of choices so ,, forgive me

1-1-2012
1-carpenter 72 yrs old loss one of his family ( death due to heart attack) came to U to do some investigation he well and fit. He Denied any history of chest pain Or S.O.B . O/E everything is normal except mid systolic ejection murmur at Lt sternal area without radiation to carotid what is your diagnosis 1 aortic stenosis 2- aortic sclerosis 3- flow murmur 4- Hypertrophic Subaortic Stenosis

2- which drug should be avoided in pregnancy ( I think Pt have UTI) 1- amoxicillin 2- cephalosporin 3- I forget it something like nutro 4- quinolone 3-most common cause of postpartum hemorrhage>>> I forget choices but I choose >>> uterine a tony

4- Pt with DVT and Inferior venous obstruction what is your diagnosis 1- chrismas disease 2- lung cancer 3- niphrotic syndrome 4- SLE
| P a g e

5- pt fall down on fully extended hand what is the fracture I am sure it is colle's fracture ( coz my orthopedic Degree 3 out of 3) 6- treatment of alzahimar disease 7- pt with recurrent inflammatory arthritis ( migratory) and in past she had mouth ulcers now c/o abdominal pain what is the diagnosis I forget all choices 8- study on population of 10000 they found 2000 have DM at end of study increase 1000 what is incidence of DM 1- 10% 2- 12% 3- 24% 9- what is initial sign of increase intracranial pressure 10- case of acute cholycystitis 11- case of appendicitis 12- case of endometriosis in alQasim note 13 child with miosis diarrhea ( case of organophosphroupoisning) 14- anti depressant cause HTN crisis 15- case of hyperthyrdisim 16-pt with migraine and HTN best TTT>>>propnalol 17- mother has child with cleft plate ( percentage) same Q in alqasim note 18- long scenario of child with intassuptioninvestigation_ barium anema 19- twins male and female u will tell family about puberty 9 same Q in alqasim note 20- why we do slow correction of hyponatremia>> to avoid brain edema ( also same Q in alqasim note

| P a g e

21- haplessness predictor of >>>( also I forget all choices but I choose suicidal behavior 22- schizophrenia best predictor>>sme Q in alqasim note 23 pregnant has glucosuria also by GTT confirmed that she has gestational diabetes what ushold do a- repeat GTT b- Take a1c hemoglobin c- take fasting blood glucose d- do insulin tolerance test 24- ibuprofen contraindication >>> PUD 25-treatment of acnerosea 26- treatment of Chlamydia 27- hip pain in walking awake her from sleep what is diagnosis >> same Q in alqasim note 28- same Q in alqasim about exogenous factor for osteoporosis 29- Q about body response to increase lactic acid in body 30- best non medical treatment f osteoartharitis same q in alqasim note 31- best exercise for increase muscle strength and bone density ( also in alqasim note) 32- best treatment of HTN pt this HTN is secondary for hyperaldostronisim 33- progressive vision loss O/E opacifications>> I think it is cataract 34- same q in alqasim about phenoparpition during lactation 35- pt with S.O.B and cough during exercise also in night now it worse What is the prophylactic of asthma given to him as treat ment 36- best investigation for Giant Cell Arteritis 37- pt hase nose congestion with conjunctivitis ( allergic _ same in alqasim note 38- pt with conjunctivitis of sunlight ( same in alqasim not the choices are pollen- ultraviolet .) 39- urine analysis show cast or I think cells what is the origin >> I think something related to urethra 40- case of tension headache 41- case of common bile duct >>> I
| P a g e

a- ERCP b- PTCA I forget it but read about CBD obstruction 42- Q in alqasim about vaccine for baby +ve for HBV >>>ptd,mmr,hib 43- q about body mass index 32 >> underweight _ obese- morbid obesity normal 44- definition of epidemiology 45- pt with pulmonary embolism confirmed by CT scan what is initial therapy a- I.V heparin b_ I.V warferin c- embloectomy 46- which of flowing is indication of fetal distress>> it is b/w early or late deceleration 47- pt with black heads on forhead>>( go for surgery) 48- vitb1 deficiency 49- posterior pad sign << also in alqasimhhh 50- vaginal dryness>> estrogen cream 51- what is false about coartication of aota( congental_ make upper limb pressure- ) 52- aortic syncope >>> also in alqasim 53- palpable gallbladder>>> ca of head of pancreas 54- spontinouspneumo thorax>> tube thoracotomy in alqasim note also 55- eruption not scaly resistant to mecanozole>> also for time 1000 in alqasim note 56-best investigation for cystic breast mass 66- treatment of otitisexterna 67- treatment of otitis media 68- clavicular fracture in newborns 69- half life of flextine( antideppresant) 70- stupid scenario about child with blood per rectum and foul smeel black stool what we will do 71- pt with increase JVP PND>> also in alqasim collection 72- diarrhea after party something like that what is organism >> sorry but it is in alqasimw note 73- case of GERD
| P a g e

74 4th degree hemorrhoid treatment 75- drug contraindication in sub aortic stenosis hypertrophy:digoxin Oral and DOBUTamine 76- also q about pink pastule( measls or rubella in alqasim ) 77- pt with hepatosplenomegaly with cervical lymphadenopathy what u wil do a- cervical lymph node biopsy b- liver biopsy c- bone marrow biopsy d- EBV serology 78- what is true about malaria 79- what is true about hepatocellulat carcinoma>>liver us and a fetoprotine 80- whoch is trurpreiparuim>> I choose DVT is common 81- best way to ask pt >>>open ended q 82- choose example for open ended q >>>point the site of pain in your chest 83- preianal mass fluctuant red hot treatment ( in alqasim>> drainage) 84- which is preventive against cancer>>fibers 85- treatment of trigeminal neuralgia: Carbamazepine 86- trauma pt ( in alqasim not).. air way sure ( abc0 87-forgin body in eye ( inalqasime note) >> local antibiotic 88-back hemangioma treatment 89- AVN treatment in child ( alsoooooooooooo in alqasim collection) 90 whatis one of the following not happened in 100% o2 therapy a- chest pain b- deppression c- seizures in general read about side effect of 100% o2 91- child change his school become inactive depressed >>> I think it is adjustment disorders 92- stress fracture in alqasime note als00>>>>VD supplementation 93- baby walk around fluorinate }} 10 months 94- treatment of gastric ulcer with out H. pylori?? 95- cause of death in Inalqasim>>>>smoke inhalation

| P a g e

8-1-2012
1- A copy of dr- Mona Edwan Exam that was posted in the group 8-1-2012 2- In active increase transaminase which of the following drugs contraindicated rinatidine infidipine vastatin

-which of the hollowing drug cause , hpertennsive crisis Clonidine

-side effect of silver sulphazidine: a-leukopenia b-skin pigmentation c-acidosis d- electrolyte impalnce

-side effect of prolong 100% oxygen except: a-retrosternal chest painting b-sizure c- depression

-prenatal mortality mean: a-number of stll birth<20 WEEK gestational age


| P a g e

b- number of stillbirth + first week neonate c-numer of deaths /1000

-stage IIIcolon cancer start chemo therapy: a-As soon as possible b-if lab reasult normalize c-according the pt psychology d- if pt >60 y age

-Eating diordermangement: a-cognetive and behavioral therapy b-pharmacology -which of the following treatment should be give in mentinance bipolar: a- valporate b-lithium c-olanzapine

-scenario for child transfer from city to another city >>>>and he go to school he is not good psychology (i miss what he have ( what is the DX: Adjustment disorder

-pt came with painful rectal spasm , diaphorisis , tachycardia especially at night , DX: a-thrombosed hemorrhoid b- proctalgiafagux (answer
| P a g e

c- ???syndrome

- what is the finding in anemia of chronic illnes: a- decrease iron and increase TIBS b-decrease iron and decrease TIBS C-increase iron

-secondary amenorrhea

a-due to gonadal agenesis -b-sheehan's syndrome c-It is always pathological

-polythcemiavera also associated with: a-ms weakness b- splenomegaly

-case scenarion pt came with chest pain , radiate to jaw , increase with exersize ,decrease with rest DX: a-unstable angina b-stable angina c-prenzmetal angina

-child ,found meningits in blood >>>i think and he is assymptomatic ,what well you do: a-oral penicillin
| P a g e

b-oral rifampacin c-IV ceftriaxone

-female pt , with RTA ,she has bilateral femur fracture >>>like this scenarion , systolic blood pressure 70 >>>what will you do: a-Iv fluid b- blood tranfusion

-female pt ,KCO rheumatic heart , diastolic murmur ,complain of aphasia and hemiplegia what will you do to find the >>>etiology<<< of this stroke: a-MR angiography b-Non-contrast CT c-ECHO D-ECG E-carotid doppler -patinet improve with antidepressant , suicide risk: a-great b- less c- same

-pt diagnose papillary carcinoma , Mangement: a- surgical resection b

- which of the following take wiht analgesic to decrease side effect


| P a g e

.. a- cimitidine b-psudoephidrine c- >>>another type of anti histaminic H1 BLOCKER

!!!!!!!! NOT SURE answer A may be H2 blocker can protect stomach from analgesic like NSAID

- child with hematuria 15 RBC /hPF , all examination normal ,what is next: a-urine cytology b-cystoscopy c-renal biopsy d- repeat urine for RBCs and protein - scenario i think for TB pt , with upper lung fibrosis , he live in crowded area :what will give to the contacts a-Himophilus influenza vaccine b-immunoglubulin c-meinongicoccalvccine d- BCG

-PT, with herpes in vagina , what is ture: a-pap smear every 3 year b-CS delivary if infection 2 weeks before delivary

| P a g e

<<<<< answer B

-ORTHOPEDIC : child came withor Toeing-In , set in W shape when walk both feet and knee inward with 20 degree , both femur inwarr rotation 70 degree <<like this""" what the Dx : a- metatarsus adductus b- femoral anteversion (femoral torsion( - -- -SENARION : old pt , e hx of MI 2 weeks back and discharge from hospital hrs prior to his presentation >>>came with sudden lower limb pain and numbness ,on ex the limb pale cold>>the other limb normal what is the DX: a- Acute artery thrombosis b- acute artery embolus c- DVT D -

NOT sure betb,c myanwer is c but i think wrong

-sickler pt came with painful crisis what is the RX: a- mangement outpatient + analgesic
| P a g e

b-hospitalization +analgesic c-reffer to 3ry center

-PTH high ,Ca low ,creatinine high ,vit d nomal DX: a- vit d deficiency b-chronic renal failure

-pt with leg or knee swelling >>>>i think last month have big toe swelling and recive NSAID , and improved Q About gout: a-due to septic deosite b- deopsite due to highsaturated

-Allopurinol : a- use in acute phase b-it is uricosuric c-contraindication in chronic renal disease d-decrese uric acid renal stone

-With asprin overdose a- metabolic acidosis with respiratory alkalosis

- parent came with child vomit ahter every feed , normal growth parameter ,what will y do: reassure the parent
| P a g e

-case scenario plural effusion , cardiac effucsion e low pretein, LDH <<<<<< i forget THE nomner<<< what is the cause a-Tuberculosis b-heart failure

answer b

-normal child ,he want to walking , he have brother dead after walking what of the following must be excluded before walking a-PDA b-VSD C-hypertrophic cardiomyopathy!!!

-old pt ,she have MI and complicated with ventricula tachycardia then from that time reciveBuspirone he came with fatige>>>>normotinsive , pulse was 65 what INX must to be done

a- thyroid function b- liver and thyroid

-at which age child sopke few words a-12m b-24m


| P a g e

c-36m

3-Another Exam Posted by dr- Ibrahim Qaid : ( )


(1/6) The best ttt for binge eating disorder: - cognitive - behavioral therapy - problem - solving therapy - interpersonal therapy

(2/6) young pt with mild intermittent asthma, attacks once to twice a weak, what's best 4him as prophylaxis: - inhaled short acting B agonist - inhaled steroid

(3/6) drug useful for pts with idiopathic anovulation: - chlomophine

(4/6) bad breath smell with seek like structure, no dental caries & Ix are normal, what's the likely cause: - cryptic tonsillitis - Sojreen's synd.

(5/6) old aged female with atypical squamous cells of undetermined significance
| P a g e

(ASCUS) on pap smear, started 30 day ttt with estrogen & told her 2come back after 1 weak, & still +ve again on pap smear, what's next: - vaginal biopsy - endometrial biopsy - syphilis serology

(6/6) 24 y. Female with new Dx of DM2, she weared glasses for 10 years, you will advice her to follow ophthalmic clinic every: - 6 months - 12 months - 5 years - 10 years

salam to all.my exam was today.it went fine.most of questions were new and unseen.i am going to share some new qustions.hope they will be helpful for future candidates....

18-1-2012Posted By dr- Sana Sohail

1.patient with hypopigmentedmacules.loss of sensation.thickendnerves.diagnosis was leprosy.which type tuberculoid lepromatous borderline 2.female presented to er with HCL burn on her face there was partial thickness burn.management
| P a g e

irrigation with water irrigation with soda bi carb immidiate debridement 3.young female with left sided abdominal pain.no dysuria or change in bowel habit.history of hysterectomy 4yrs back but ovaries and tubes were preserved.on ex abd tender but no guarding.inv show leucocytosisand few pus cells in urine.there was also history of unprotected coitus with multiple partners. ididnot get the scenario well but i think it was salpingitis. management consult surgeon oral antibiotics diagnose as ulcerative colitis 4.chickpeas.kidney beans and lentils contain which element of following bromide chromium iron selenium 5.a case of clamydial eye infection 6.a scenario of otosclrosis 7.a picture of JVP graph to diagnose.patient had low volpulse,low resting bp.no murmr.pedal edema. constrictive pericarditis tricuspid regurg tricuspid stenosis pulmonary hypertension 8.treatment of psoriasis 10.case of viral gastroenteritis 11.16 yr old female with primary ammenorrhea.scarse pubic and axillary hair but proper breast dovelopment diagnosis: Complete androgen insensitivity

| P a g e

50 years old female have DM well controlled on metfromin ! now c\o diplopia RT side eye lis ptosis and loss of adduction of the eyes and up word and out word gauz !! reacting pupil no loss of visual field Something like that !! The option : Faisal palsy Oculomator palsy of the rt side Masynia gravies !! 2 -The coz of high mortality in pregnant female !: Sphillis Toxoplasmosis Phenocchromctoma -------- forgat ! ANSWER : toxo but am not sure !

Ummm a child have drink corrosive material and came to the er look not well drooling What u r management : Give 2 cup of milk Lavage Establish airway Ask about the crosive material it alkli or acidic ! ANSWE : NO FREAKIN IDEA !!

4 )What the best method for prevention dieses :


| P a g e

Immunization Teaching individual how to protect them self Shi zy keda !! :s Answer : -_Bed reidden pt he have confusion and fever blood culter shows enerocoucas From where : Pneumonia Uti I forgot the rest wallah ! I answer : uti not sure Which tocxisiaty u will rash to the baby to hospital A.S.A.P : Tac toxisoaty Qunini toxsiaty W other I dont recall it :s

Young femal she have irration valve she goes to here doctor and advise her to shange the soup she using ! but still she have this irration It was waxy with grayish "something" : Atopic dermtisist Conact dermtisiis Linch sipmplex Linch . Something :'\ Answer : who knows !!!!!!!!!

There is also other derama q was as hell !! I dont know any of them !
| P a g e

Lucman test : in ortho Testing for what ligment ! Ttt of h. pylory : I pick Omoprazol , clinadmycin , w other :s I dont recall the answer wallahi ! Pt have travel to kinnia and he resived blood transfusion there now he c\o sore throt and generalized lymphonapathy and tender spleen and hairly lucoplika ! HIV Lypomna No EBV from the answers And I think its hiv !

But over all u should read all the QS from qassim uni collection and check the answers

28/1/2012 all try to answer here from Eyad Al-Hudaithi Qs


Q1/True about dermatomyositis : 1-associated with inflammatory bowl dz 2-indicate underlying malignancy 3-present as distal muscle weakness >>in some cases Q2/ newborn presented with conjunctivitis and O.M , whats the treatment : i guess this is a case of infection with chlamyedia intrauterine , they asked about several AB there is no dyoxycyclin nor erythromycin :/ Q3/ Differences Btw dementia and delirium (read about it )
| P a g e

i picked Amnesia !! Q4/what kind of questionnaire you would use to differentiate btw central ?#$%#@@$%?????? obstructive sleep apnea and snoring 1- Michigan Questionnaire 2- Epworth Sleepiness Scale Questionnaire Q5/ about shoulder that is Adducted and internally rotated (what is the mechanism of dislocation) -Postsubacromial posterior dislocation Q6/ celiac dz . all should be avoided exept : 1-wheat 2-oat 3-rice Q7/female has primary amonarhea , webbed neck , low hair line = Turner Q8/anteriolateral placenta , term pregnancy , can't be felt when examiner admit his finger through the Cervix : 1-Low set placenta 2-Marginal 3-normal 4-complete placenta previa Q9/Tineacapitis RX. 1-start Nystatin 2-wood's lamp (true) Q10/nodule : don't do anything so you don't rupture it cryotherapy (true by luck ) Q11/Rosacea case (redness patch on face with talangectasia ) what is the ttt : Dyoxycyclin Q12/ child smile at 1- at birth 2- 1month 3-2 months 4- 6 months Q13/ pt has GERD for 5 years , now EGD reveals >> columnar cell surrounded by
| P a g e

Sq cell 1- Sq .c.c 2-Adenocarcnoma 3-barret esophgous

Q14/ old pt , has loin pain , U/S reveals bilateral hydroneprosis , whats the cause : 1-prostate cancer 2- bladder cancer 3- urethral stricture Q15 / infertile women for 3 years with dysparunia 1- endometritis 2- Salpengitis 3-endometriosis (True) Q16/ pt has Lt lower Abdominal pain , Fever , constipation CT reveals thickened loop and little perianal fat , whats appropriate to do: 1- start AB 2- call the surgeon for immediate OP 3-give laxative 4- barium enema Q17/ Typical scnario about migraine and pt doesn't want DAILY medication : 1- bio feedback 2- BB 3- CCB 4-inhaled ergometrin 18/ OCP protective Against : 1- Breast ca 2- Ovarian Ca 3- endometrial 19 / Culture >>H.influnza .. (READ AbtRx ) 20 / leukemia case .. lab (pancytopenia , lukocytosis , +ve myeloperoxidase ) Dx is : 1- ALL 2-AML

| P a g e

21/ hematology case prophral blood smear reveals target cell =SCD 22/difficult consultation : 1- use medical term 2- open ended Q 3-close Ended Q = True 423/ lactating mom recently dx to have epilepsy on Phenobarbital , her child is 10 months now , whats appropriate to tell her 1- stop immediately 2- wean him for 2 weeks 3- give after 8 hrs of 4- BF as much the baby and mother want 24/ True about Mallory-Weiss sx ) 1- MCC of GI bleeding during pregnancy 2- resolved spountounsly 3- 1/3 cases of GI bleeding is d/t this Dz 425/ old pt with progressive weakness of hand grip , dysphagia , . 1-MG dz 2- Mysthenia gravis sx 26/RTA pt with femur fx , he has laceration of the femoral artery .. What to do : 1- end to end anastomosis 2- prosthetic graft 3-arterial graft 4- venous graft

30-1-2012 Dr- reem al ahmari

| P a g e

-2-What is the most effective method to prevent the brucellosis infection: -a-Treat the infected people Immunize the farmers & those who deal with the animals -b Get rid of all the infected animals-c -d-Pastralization of the diary products (my answer Which drug conta indication in pregnant women in uti: -3Floro qinelon -4-Pregnancy 36 w her blood pressure 140/90 , no lower limp edema first thing :: -aRepeat measure of blood pressure-( my ans) b-cs c-give anti hypertension medication

-5-Child with large periorbital hemangioma , if this hemangioma cause obstruction to vision , when will be permenant decrease in visual acuity a. After obstruction by one day b. By 1 week( my answer c. By 3 months d. By 6 months 6-a Corneal Ulcer, Abrasion other investigation a-fisial felid measurement b-slit limp c- florescen .day ( my answer)

-7-Drug induce urticarea Hydralizin ( my answer_)


| P a g e

8- old female complain from rash then developed disne and lethargy What is the cause Subheretic dermatitis Urea depositin ( my answer) >>

9- pat dig hypertension obese high NA intak .the cause of hypertention in this pat.. a-high sodiam intak b-obesity

10- child obese what is adevice a-decrease kaloic intak -b-multi factorial interaction-( my answer)

11-patient has atreal fevrelation(af) risk: a-cva( my ans) b-mi

12- patient complain of headache for long time best for treatment: a-betablocher b-biofedback

13- Adult Polycystic kidney mode of inheritance: Autosomal dominant


| P a g e

14-old patint with depression take ant sycatric medication Take time 2 week 3-4 week( mu ans)

-15-Burnt death du to a-Gas inhalation( my ans) b-Septic shock

16- old female ( ostoprosis) Fear from desk compression best treatment: -aDecreas the wight -b-Take vitamin d and calcium ( my answer)

18- breat feding contra indication in; -a-Tp for 3 month -b-Asyptiomatic hiv ( my ans)

19- verecella ( give second dose only)

19- hal dm type 1 on


| P a g e

Dr3 Dr4( my answer)

20 the dx:

picture of large nodule in neck, O/E moce with degilution, what is

a-lymphoma b-goiter c-thyoglaal cyst ( my answer) -21-Patient with COPD, Which of the following increase surveillance? a) O2 home therapy( my answer) b) Steroids c) Ipratropium

-22- After removing foreign body from the eye apply local: A) Antibiotics B) Steroids( my answer) I thing is wrong

-23-Newly married woman complain of no pregnancy for 3 month with unprotective sexual intercourse : Try more ) -24-New marred female has vaginal discharge colorless no order no painful: What is this discharge?? Normal after intercore ( my answer)_ -25) condition not associated with increase alpha feto protein breech presentation Down syndrome( my ans)
| P a g e

Q26) pregnant never did check up before , her baby born with hepatosplenomegaly and jaundice : a-Rubella b-CMV( my ans) c-HSV Toxoplasmosis Q27-) old pt complaining of back pain on walking on examination there was stiffness of the muscle and there was some finding on the x-ray , best effective ttt is : Physiotherapy NSAID Surgery

-28-pregnant pt want to take varicella vaccine, what you will tell her ? That is a live vaccine It is ok to take it

-29-Female pt came to you post ovarian cancer surgery one month ago, you did X-Ray for her and you found metallic piece, what you will do ?

Call the surgeon and ask him what to do Tell her and refer her to surgery Call attorney and ask about legal action Tell her that is one of possible complications of operation Don't tell her what you found

| P a g e

-30-hypertensive pt using sildenafil , in his case it is contraindication to take :

CCB B blocker Nitrate Diuretics

-31-pt K/C of uncontrolled asthma moderate persistent on bronchodilator came with exacerbation and he is now ok, what you will give him to control his asthma :

Systemic steroid Inhaler steroid(my answer) Ipratropium

++32-patient epegastric mass show by upper gi Investigation: Endoscopy( my ans) Fall blood test Paruim enema

Case of hypothirodism ( cold intleranse _+ wight gain -| P a g e

34-_osteoporosis depend on a-age ( my answer) b-stage Gender

-35-after aspiration of abreast cyst, which of the following indicate that the cyst is benign: Aspiration is clear & the cyst not refill The aspiration is bloody Cytology study shows hyperchromatic changes Cytology study shows fibrocystic disease

-36-diagram about osteoporosis shows that from age 70-74 10% has osteoporosis, from age 75-79 40% has osteoporosis, age above 80 70% has osteoporosis, which is true: Women over 80 y has the highest risk for the osteoporosis( my ans) Women from age 70-74 , 10% will develop osteoporosis

37-pt with tingling of the little finger, atrophy of the hypothener, limitation of the neck movement, X-ray shows degenerative cervicitis, EMG study shows ulnar nerve compression, what will you do: Surgical cubital decompression(my ans) Cervical CT scan Nsad Phisotherapy
| P a g e

38-child with drowling saleva, stridor, what is the dx: a-croup b-epigoltites ( my ans)

croup

39--witch is true regarding peretonieal levarge a- freech blood on inspiration 2 ml Rbc 1000 b-wbc 50 d blood 2 ml in pregnancy -40-Testicular pain pain in groin region in examination ther is tenderness no organo megaly A-reffer to surgry B-referr to urologu C- do parim enema -41-Complication of apendecites A-smoll bowel obstruction B- illus paralytic true

42-* most commen of bleeding on postmenoposal women A-carvical plup B- utrine atony tru 100 %

| P a g e

Bmi 29.5- A- obese B- over Wight

44---- whomen came to clinic for follow up for pap smear 3 time negative and has gistory of wart from 7 years and now Atyoical Squamus tissue groing Next step a-repate pap after 1 years b-Hiv smear c-Resection loop d-hestroctomy

-45-Women have unilateral syst

-46-Patint have sizer and use 20 mg from diazepam but not improve A- increase the dose of diazepam to 40mg my ans B- add phyintion C- add phenoparpital C- add carpamizepam

47-patent plunt on hes chest coplane of cyanosis and resonance on one side
| P a g e

First step a-o2 ( my answer) b-intunation c-needle

.. -48-Patient with pnemothoax and respiratory distreas Firest do: a-intubation9 my answer) b-needile

-49-In cpr: Open the ear way and give to breath Give to breath for 2min and then chest compression -50-cpr CPR; a-2breath increase the chest ( rise chest) b--30-40% c-_ do DC 3 TIME

-51-BREAST feading in neonate A- as soon as posipile B- after 2 w

-52-Phudo goute:
| P a g e

Phosphatea--a Calssum-b Floride -c

53-Case of ostoartherites!!

-54-Nesseria goneria treatment: Ceftroxone true

55-Pre anal absses treatment: Incision and drange

56-Chronic fatige syndrome: Anti sycatric treatment( my ans) Releve by rest

57-Headech thromping in the eye band like Sinisites headech Strees Migraine

58-plast
| P a g e

AML ALL CML CLL

-59-PATENT ppd test positive for t.b before anti tb treatment

a-repate ppd test b-do mantox test( my ans) -60- smoking with drowel 1-3 2-4 d( my answer)

61-Major depression disorder treatment A- escitalprim true

!!!

Secondary to :62 A-disneses( my ans) B- phopia


| P a g e

C-apdominal pain D-tachecardia Chest pain

63-Mechanism of DKA -Hop glucose increase insulin increase keton

64-Appendcites proflaxx Metronidazole Ceftroxone( my answer) Cefroxon

65-Pt with hx of stroke for 6 hour, what medication you will give him: Aspirin (my answer) t- PA colpidogril heparin

66-Case control study:

67-Patint IHD and obese + bmi=28+>>>> Decrease weight and exercise benefit
| P a g e

68-If we draw a line through the the long axis of the radius it will pass through the capitalum Anterior pad signs Posterior pad signs(sure )

_-69-anal fissure most commen site Posterior( my ans) Anterior

--70-patient prolonged period defecation painful + bloodA- anal fissure( my answer) B-hemoroid

71-Female dx recently with epilepsy & you gave her phenoparp., she lactate her 10 month old child 3time/day, what will be your advice: stop lactation immediately (my answer) stop lactation over three weeks Lactate only 8 hours after each dose Continue the feeding

| P a g e

72-Qs about apgar score: Heart rate is significant (my answer)

73-Lactating mother with mastitis trratment: Doxcycyclin Ceftroxone Cefoxine Metronidazle

74-child malnutrition low protin+ no edema a- kwasherker b-muresmus

75-Heavy smoker came to you asking about other cancer, not Lung cancer, that smoking increase its risk: Colon Bladder( my answer) Liver

| P a g e

76-A 5 year old child came with earache on examination there is fluid in middle ear and adenoid hypertrophy. Beside adenoidectomy on management, which also you should do: Myringotomy Grommet tube insertion( MY answer) Mastidectomy Tonsillectomy

77-there is outbreak of difteria and tetanus in community , regaring to pregnant woman: contraindication to give DT vaccine if exposed , terminate pregnancy immediately if exposed , terminate after 72 hour give DT vaccine anyway

78-Pt with scoliosis, you need to refer him to the ortho when the degree is: 5 10 15 20( my ans)

79-Most common cause of intra cerebral hemorrhage Av malformation( my ans) Pre exicting anurezem
| P a g e

-80-All are primary prevention of anemia exept: A-iron and folic acid in pregnancy and postnasal( my answer) B-iron food in children C-limitation of caw milk D- genetic screen for herdateriy anemia

81-Old female with itching of vulva , by examination there is pale and thin vagina , no discharge . what is management a. Estrogen cream

82-Rebound phenomenon definition??? ...

83-Holding breath holding Generalized convultion

-84-serious otites media

85-otites externa or media ( tympanic membrane intact and discharge in urical)


| P a g e

86-female have itching in valve and thigh Contact dermatitis And other not remember

87-All in hypokalemia exept: Hyper osmolar coma Phention toxicity Musle paralysis

88-In paracetamol toxicity: Pencelinemia N acetyl>>>true 100% K intake Dexoamin..

89-_ chilled with throat congestion and mutable Ebv Hsv Adenovirus

| P a g e

90-Child rach spreed qiuqly +fever+drwesnes .....fever Rubella Measles fever....

Case 91-pericardites Pain in chest increase with movement.. sudden Best investigation Ecg my answer Cardiac enzyme

92-Case patient complain mi on treatment after 5 day patien have short of breat + crepetation on both lung a-pulmonary embolism b-pnemonia( my answer) c-mi reg d-orta reg

case cardiac 93-canion a wave fistula:)(


| P a g e

BaD Q

94-Uric acid in body how the body removed by a-increase co2 execreation of uric acid in urin b-increase metabolism of uric acid in liver c-execretion of uric acid by lung Q kwashecor syndrom 95-Child runny then developed pain in thigh no redness or tenderness Best thing to do A- elevated the leg and cold compression( my anse) B- splint C-surgry -96- ) ( a-obsession b-post partum sycosis( my answer)

female late deceleration 38 week : change the position and give o2


| P a g e

97-Kawasaky syndrome; Sterbery tong

98-Coarregation of orta all true except

f- Skeletal deformity g- Upper limp hypertion h- Systolic murmur on all pericardium

1-2-2012
1-Diagnostic peritoneal lavage : 2 ml of blood initial aspiration 2 ml of blood in pregnant WBC in cc/ Can't remember 2 - varicilla vaccine what true Not given in first trimester preg Contain live attenuated bacteria 3- painless lump in neck in child Hodgkin lymphoma Pharhngitis Infectious mononucleosis 4- surgery should done immediately in croons dis when : Fistula Intestinal obs Abdominal mass Intes bacterial overgrowth 5- child with bilious vomiting with yellow stool ,, abdominal dissension He passed stool immediately after birth . Harsh sprung dis Mid gut volvus
| P a g e

Can't remember

6 - 3 months infant with tachpnea .Respdistress .x- ray shows lower and mid lobe infiltration , opaque right lung and shifted trachea to left .. Responsible organism : H inf Pneumococcus ... 7 - acute diarrhea with epithelial infiltration E- coli Salmonella Cholera Rota virus Shigella 8 - old pat with bilatralhydronephrosis and loin pain : Pelvic cancer Prostatic hyper trophy Bladder .. Tumor ??

9- old pat with pain after walking no edema Claudication 10- old pat with tAchcardia pulse 150 otherwise normal TSH Stress ECG 11- to deffrentiate bet sinus tachycardia from atrial futter Carotid massage ... Artery massage Can't recall 12 - mitral stenosis LA hyper trophy with decrease plum ..
| P a g e

Left atrial hypertrophy and champerdilitation

13- Patient with panic attack .. Something related to secondary mechanism not symptom Epigastric pain Chest pain Dizziness TAchcardia 14- child with vaginal discharge green .. Bad odor , pelvic exam normal Foreign body Trichomonas 15- emprical treatment of peptic ulcer h. Pylori Omperazole , clarithro , antihero 16- anterio- latral located placenta not palpated by pv Marginal Low lying Normal lying Partial low lying 17- first sign of increase ICP hTN Decrease Level of consciousness 18 - increase igG in CSf Multiple sclerosis Duchine dystrophy 19 - pregnant 38 gestation bp 140/90 Cs Induction Observe bp reading Give anti hypertensive
| P a g e

20- patient was treated for glaucoma now presented with SOB , ... The drug reasons able for these symptoms : Timlol Pilocarpine 21 - pt live near industries came with attack of SOB the prophylactic B agonist. Oral steroid inhailed corticosteroid 22- polygonal rash flat topped : Lichen planus - sure 23 -Most important to instruct pt about Lyme dis : Kill ?? Insect Wear long fiber clothes ... 24- child with ear pain red tympanic mem and -ve insulation test Acute otitis media TRUE Secretory otitis media Chronic om 25- scenario of panic attack .. Treatment Benzodizepine SSRI .. 26 - young lady with emphysema A1 anti-trypsin def 27- pt with hemoptysis , night sweat . Loss appetite .. X- ray apical cavity Post primary tb Pneumonia 28 - vesicular rash
| P a g e

Checken pox 29- most common cause of pediatric falure to thrive Cystic fibrosis Psychosocial Protein &Milk - intolerance 30- pateint with pustule arround the mouth the organism is herpes simplex what is the treatmen Oral ab Topical ab Acyclovir Steroid ( cantremeber if topical or oral 31- true about gastric lavage Not helpfull after 6 hours of aspirin ingestion 8 hours after .. Ma athkr 32 - drug cause gout Hydrochlorothizide Furosmide 33- mechanism of OCP Inhibit estrogen spur in mid ..&ovulation 34- rt lung has 2 pulm veins 35- scinario of cholecystitis what is the most theraputic procedure ERCP Cholecystectomy 36- pt with hoarsness of voice . Next step Laryngoscope 38- high pitch diastolic murmur Ms Mr
| P a g e

Mvp 39- pateint came after RTA with heavy bleeding upper limb ABC Call orthopedic Press the bleding site Take to OR 40- sicklerpateint u treated with antibiotic for UTI u will discharge him with Pencillin .. 41- on going treatment for sicklers iron therapy Pencillin with immunization 42- lab values all r normal except Na ( hyponitremia ) treatment NS with kcl at 20 cc / hour NS with kcl at 80 cc\ hour 1/2 ns ... 43- mechanism of cushing syndrome Increase ACTH from pituitary adenoma Increase ACTH from adrenal ..... 44- ptdiabiticretenopathy the most u will deal with HTN with smoking TRUE 45 - 27 years old with DM 2 she already wears glasses u will follow up her after : 6 months 12 months 46- drug used in treatment of CHF which decrease the mortality B blocker Verapamil Nitrates Digoxin
| P a g e

47- CHF with pulm edema . Treat with Furosmide Thizide ... 48- child with hematuria 15 RBC whatnext urine cytology Repeat urine for rbc and protien Renal biopsy Cystoscope 49- pregnant lady with low back pain .. All gynecologic causes ruled out what to give : Acetamenophine Paracetamol Profen ..

SLE exam: 04 - 02 - 2012


1) All cause ear pain except: a- Acute otitis media b- Dental caries c- Vestibular neuritis (the answr) d- Temromandibular joint arthritis 2) Newborn has vomiting after every meal intake. The examination revealed mild dehydration. No other clinical signs. No tests ordered yet. What is your next step? a- Order abdominal CT b- Reassure the pt c- Refer to GS d- Discharge on ORS 3) Pt presented with severe hypothyroidism & serum sodium = 108. What do u
| P a g e

do? a- Intubate, give 3% sodoium then treat hypothyroidism status b- treat hypothyroidim& monitor S.NA level every 6 hours c- Give 3% sodium, hydrocortisone & treat hypothyroidism status d4) Which breast disease is Bilateral: Lobular carcinoma 5) About Methyl-dopa in pregnancy 6) A mother is lactating and she wants to take MMR vaccine. What do you tell her? a- MMR vaccine has live attenuated bacteria. b- D/C breast feeding for 72 hours after the vaccination. c- MMR vaccine can be taken safely while breast feeding d- MMR vaccine will harm your baby. 7) Regarding Varicella vaccine: a- 2 doses 6 weeks apart b- 2 doses 2 weeks apart c- 1 dose only d- 3 doses 1 week apart 8) TTT of Gonorrhea: CEFTRIAXONE AND CEFEXIME 9) Regadring dx of GERD: a- HX only b- Hx& Barium enema c- Hx& UGI endoscopy d- Barium enema & colonoscopy

10) Regarding pterygium: a- of systemic cause b- causes blindness c- due to avitemenosis A


| P a g e

d- needs surgical intervention 11) Pt presented to the ER after RTA. He was concsious , vitally stable. There was decreased air entery bilaterally & tracheal deviation to the opposite site. What is your next step? a- Order CXR STAT b- Insert a needle in the 2nd ICS MCL c- Insert a needle in the 5th ICS MCL d- Insert a chest tube in the 5th ICS MCL 12) A 10 YO was diagnosed with rheumatic fever without any defect to the heart. You will tell his parents that he needs to take prophylactic antibiotics for how many years? a- 5 months b- 3 years c- 6 years d- 15 years Psychiatry: 2 senarios about generalized anxiety disorder 13- A k/c/o Alzheimers, with psychotic manifestations. How do you treat? Haloperidol

SLE on 6th of Feb 2012 Written by Noura Al-mutlaq

1-patient with past of hx of endocarditis came to dental to do dental procedure , what antiobiotic u will give as prophylaxis : -amoxcilline 2 mg before the surgery
| P a g e

-amoxcilline 1 mg after the surgery -clindamycine 2 mg before surgery -clindamycine 1 mg after surgery

2- the marker for ovarian ca : CA 125 3- TRUE about congenital hip dislocation : -abducting an infants flexed hip causes palpable click ortalani test 4-patient known to have CHF , came with fatigue & muscle weakness , Na :123 , K: 3 , how ull manage ? -normal saline with rate 80 /hour +40 mickcl -normal saline with rate 20/hour +5mic kcl -1/2normal saline with rate 80 /hour +40 mickcl -1/2normal saline with rate 20/hour +5mic kcl 5-what is the attributable risk : -number of cases of a disease attributable to one risk factor 6- patient came with retrosternalchestpain , increase with laying dawn &sleeping , ECG and cardiac enzyme were within NL -give PPI 7-what is the most specific diagnostic for PE : -V/Q scan -pulmonary angiogram -chest x-ray 7-child with DM type 1 ass with -HLA DR4 8-most common cause of conductive hearing loss :
| P a g e

-acute otistis media -Meniere disease - perforated tympanic membrane 9-patient came to ER with 2ndry degree burns involving the face and neck , how to manage ? Silver sulfadiazine, sterile gauze, IV fluid and admit to hospital Repeated qs but the choices werent the same :l 10-SE of sulfodizene : Leucopenia 11- 40 year old male , not known to have any medical illnesses , complaining of central obesity, acne ,weakness , buffalo hump , hypertension : -cushings disease -psuedocushing induced by alcohol intake -adrenal adenoma -adrenal ca 12- elderly patient complaining of LLQ abdominal pain with fever , diarrhea -diverticulitis 13-most common feature ass with chronic diarrhea : -metabolic alkalosis 14-typical scenario about primary bleary cirrhosis 15-acute fluid loss in the abdomen cavity what it will cause : -cardiogenic shock -neurogenic shock -septic shock -hypovolemic shock 16-scenario about fibroid in pregnancy whats true :
| P a g e

-fibroid will regress after pregnancy 17- couple after marriage , came after 6 months , complaining of failure to conceive , what ull do : -continue to try -prolactine level -TSH 18- drug for induction of ovulation : Clomophine 19-42 pt came with DUP what ulldo : -OCP -D & C -hysterectomy 20-pt came with hx of 3 weeks amnerhea , with abdominal pain , laproscopy done and found to have blood in the boutch of duglas : Rupture of ectopic pregnancy 21-qs about vesicocele U should know if its repair of upper ant of the vaginal wall so the defect is vesicocele, if its lower anterior urethrocele , if upper postenterocele , if lower post rectocele 22-pt asking u why instead of doing self breast exam. Every month not to do mamogaraphyyearly , what ull say : -mamography only detect deep tumor -mamography and self exam. Are complementary -self breast exam are better bcz it detect early tumor - mammography are only for palpable masses 23-child came with sever anemia ,theyre suspecting thalacemia , whats the best diagnostic to confirm : -genetic test -iron study
| P a g e

24- scenario about heamophilia , whats the defect : Clotting factor 25-typical scenario about essential HTN 26-20 yr male ptn came with couph , chest pain , fever , what antibiotic u should prescripe : -amoxicilline -ceftriaxone 27- pt came with caf au late spots , what other things ull look for : -axially freckling 28-qs about obstructed labor 29-drug are contraindicated in ttt of glaucoma : Timolol Pilocarpine NSAID 2 new drugs I was confused , I dont remember their names :/ 30-qs abt osteoarthritis 31-pt came unable to do thump opposition : Median nerve injury 32-pt with scoliosis u will refer if degree more than : -20 33- baby at 6 months , what he can do ? -sitting without support -role from supine to prone position - role from prone position to supine 34-What feature of schizophrenia suggest good prognosis ? -family hx of scz
| P a g e

-no precipitating factors -presence of affecting symptoms -early onset 35-qs about etiology of gout , read about it :D , all the choises were not easy 36-why SSRI are the 1st line ttt o major depression ? -less expensive -most tolerable and effective -to differentiate between psychosis and depression 37-elderly pt came with hx of coma and hypotension , before the coma she complained of epigastric pain , most likely due to -AAA small leakage 38-von well brand disease how to treat: -fresh frozen plasma -factor VIII replacement 39-drug used in smoking scessation c/I in pt : -hx of seizure 40-ptn with breast mass after FNA , u will leave it alone if : -clear fluid and not refill again - Fibrocystic change on histological examination 41-prophylactic of plague rodent eradication

42- 2-3 qs about broncholitis , revise the mx v well 43-pt female with sever hip pain , increase with walking , after busy day , awake her almost all the night , ass with morning stiffness : -osteoartheritis -osteoprosis
| P a g e

44-pt with 32 BMI : Obese 45- qs abut EBV 46-qs about COPD mx 47-snellen chart , same qs in malzamah / 20/70 48- qs about pneumococcal vaccine 49-the most effective way in health education : Mass media Group discussion Individual approach 50- child with white yellow mouth lip n erythrematous base with gingivitis : HSV EBV CMV 51-breech presentation came at 34 wks , what ull do : -wait until 36 -do ECV 52-qs ptn with arthiritis , urethral discharge , culture of discharge came ve for gonnorhea and chalamedia : -Rieters syndrome -gonorhea 53- pt with skin rash , diarrhea , dementia Plegra 54- qs about asthma 55- typical scenario abt retinal detachment

My exam 18-2-2012
| P a g e

Sakinah Al-saeed
The new Qs: 1-What is contraindication for giving welbutrin in smoking sessation ? History of seizure Hemolytic anemia

2-Acyanotic middle age man radiologycally come with prominent pulmonary arteries and vascular marking ,most likely Dx? VSD ASD Coarticarion of the aorta Truncus arteriosis Pulmonary valvular stenosis

3-Patient with eye pain not relieved by patching when he came you find red eye with sclera injection with cloudy anterior chamber,DX Retinitis Uvietis 4-Patient come you find perforated tympanic membrane with foul withish discharge dX? Otoseclerosis Otitis externa Cholestitoma 5-Patient with nose truma with pain ,x-ray shows undisplaced nasal fracture ,your management Refer to ENT surgon Give analgesic
| P a g e

Anterior nasal packing 6-Patient 42 years with 5 days history of skin eruption involving the hand and soles (no other information)dx? Erythema mutiforme Fixed drug eruption Pytriasis rosea 7-Toddler with sever skin itching involving the abdomen hand and face papulvesicular Chicken pox Dermatitis herpitiform 8-Child shows spiral fracture of arm management Refer to orthopedic Open reduction and internal fixation(my answer) 9-Eye screening in DMI Now and annually Now and every 10 years After 5 years and annually (my answer) 10-Sodium content in normal saline (0.9) 50 70 90 155 (the correct answer) 200 11-Patient pregnant in her 8th month had vaginal bleeding .past history of hypertention Come now with abdominal pain dx Placenta previa Ectopic pregnancy
| P a g e

Abrubtioplacenta(my answer) 12-patient complain of irregular periodand excessive fasial hair .her mother had the same. BMI 36 normal estrogen increase testerone increase LH and decreased FSH,and her urine shows 17 hydroxysteroid Dx Chushing Polycystic ovary Adrenal adenoma(the correct answer) 13-The most important sign the physician should look in primary autonomic insufficiency ? Orthostatic hypotension (the correct answer) Sinus arrhythmia Horner 14-Patient work outside in hot weather 42C came to ER with muscle pain and cramps of the lower limb ,on examination he is alert ,cooperative ,temp 38 ,Managment Oral electrolyte replacement (my answer) Internal cold water Warm intravenous fluid tepid water 15-patien work in hot weather come with clammy cold skin ,hypotensive tachycardiac heat stroke(my answer) heat exhusion 16-patient with red blood cell disorder ,with family hx of thalassimia to confirm the dx increase the level of A2 gentic
| P a g e

17-patient came with osteoprotic thorasic vertebral fracture t score for vertebra -2.6 z score the hip -1.6 and z score 0.9 according to who classification the patient has osteopenia osteoporosis established osteoporosis(my answer) normal bone mass WHO classification : (Normal boneT-score greater than -1 OsteopeniaT-score between -1 and -2.5O steoporosisT-score less than -2.5 Severe (established) osteoporosisT-score less than -2.5 and 1+ osteoporotic fractures)

18-Patient presented with chest pain for 2 hour With anterolaterl lead shows st elevation, providing no tPCI in the hospital Management Streptokinase ,nitroglysrin ,ASA,beta blocker(my answer) Nitroglysren ,ASA ,heparin beta blocker Nitroglysren ,ASA,beta blocker Alteplase , Nitroglysren , ,heparin betablocker 19-Patient came with dysphagia interferer with daily life ,past history of hymphoma treated with chemotherapy and radiation 2 years back and he did not follow in the last year,Face congested dx : Thorasic aortic anuresm Abdominal aortic aneurism Svc obstruction (my answer)
| P a g e

IVC obstruction 20-Increase the survival in COPD patient Contious oxgen inhaled brochodilater steroid 21- most common organism causing pneumonia in adult streptococcus pneumonia legionella hemophillus inflanza type b

22-what is questioneer used to diffrentiet between sleep apnea and snoring? Mitchigan Epworth (the correct ) Cooner

19-2-2012 Maryam Abdulaziz


| P a g e

Dammam university SLE

1-Female with Hx of PID and treated with ABs she came later with fever and pain on examination there was a mass, fluectuent (they mean abcess ) in a cul-de sac !! what is ur next step? colpotomy laparotomy laparoscopy Pelvic US

2-a young girl who become very stressed during exams and she pull her hair till a patches of alopecia appear how to ttt: a) Olanzepin b) Fluxitin I don't remember the rest

3-18 weeks pregnant women her blood pressure was 160/..(high) a week after her BP was 150/..(high also) what is the Dx: a) Gestation HTN C) Chronic HTN <20 weeks b) Preeclampsia

4-an Old man , not known to have any medical illness who presented with mid back pain , he's taking only aspirin , Ca, multivitamines. He's not taking dairy products and on examination he have tendrness in the mid back with mild kyphosis! and X-ray show compression Fracture in the vertebra in ?? and ?? levels what is your Dx?? Osteopenia
| P a g e

Osteoporosis Osteomalacia

5-pt with HTN presented with edema, azotemia,GFR: 44 (not sure about the digits ) what is the cause of her Kidney diseae: a) bilateral renal artery stenoss b) diabetic nephropathy c) Reflux...?? d) Renal tubular acidosis

6-what's true about antipsychotics ? a) predominantelymetabloized in the liver b) Carbamazepin as a single dose os better than divided doses

7-a baby who fall down from stairs and came with multipe contusions some of them were old and X-ray show fracture in radius how to manage : a) Splinter for his hand b) Hospitalization and call social worker

8-In the Time of TB outbreak what will you give as a prophylaxis a) BCG b) Rifampicin .. mg PO

9-45 yr old female G0P0 not know to have any medical illness presented to ER with sever viginalbleeeding on examination there was blood in the viginalos her Pulse was 90 and BP 110 / 80 and on standing her P: 100 , BP :122/90 ( ) How to manage : a) 2 units of blood b) US .. 10-Al hydroxide+ Mg hydroxide inhibits the intestinal absorption of which drug?
| P a g e

a) Tetracycline sorry I don't remember the rest 11-in epidemiological investigation best thing to do 1st: a) good sample b) count those who have the disease C)verifying diagnosis

12-A man who is thinking that there is Aliens in his yard although that he knows that Aliens are not existing but he's still having these thoughts .. Dx a) Obessions b)Dellusions

13-if the liklihood ratio is 0.3 what does that mean??

1) Mnire's disease is a disorder of the inner ear that can affect hearing and balance to a varying degree. It is characterized by episodes of vertigo, low pitched tinnitus, and hearing loss 2 q) hiv pt. have white patch in oral cavity and skin . what is the ttt: oral antibiotic loacal antibiotic local steroid chemo & radio theraby 3 15 y/o . ( table with hight and wight ) and they said : BMI= 24.4 A) normal wight B) over C) obese 4 1) there is outbreak of difteria and tetanus in community , regaring to pregnant woman: a. contraindication to give DT vaccine
| P a g e

b. if exposed , terminate pregnancy immediately c. if exposed , terminate after 72 hour d. give DT vaccine anyway -The correct answer is d. 5 49) Most common cause of intra cerebral hemorrhage: a. ruptured aneurysm b. Hypertension c. Trauma d. -The correct answer is b. 6 Q) swelling under right lateral mandible . recurrent pain . espicialy with eating..inve?? A) x-ray b) u s c) ct d) mri 7 49) Man with sudden onset of scrotal pain , also had Hx of vomiting, on examination tender scrotom and there is tender 4 cm mass over right groin, what you will do: a. Consult surgeon b. Consult urologist c. Do sonogram d. Elective surgery -The correct answer is b , if expected tortion of tests , don't wast time on radiological study 8 q) spiral fraction in child > open reduction and interna fixation 9 85) Female presented with vaginal discharge, itching, and on microscope showed mycoleous cells and spores. This medical condition is most likely to be associated with: a. TB b. Diabetes c. Rheumatoid Arthritis The correct answer is b Vaginal thrush is a common infection caused by a yeast called Candida albicans. Vulvovaginal candidiasis is usually secondary to
| P a g e

overgrowth of normal flora Candida species in the vagina. Conditions that interrupt the balance of normal vaginal flora include: antibiotic use, oral contraceptives, contraceptive devices, high estrogen levels, and immunocompromised states such as diabetes mellitus and HIV. Women are prone to vaginal thrush between puberty and the menopause because, under the influence of the hormone estrogen, the cells lining the vagina produce a sugar and yeasts which Candida albicans are attracted to. That is why thrush is rare before puberty. 9 86) Primigravida in her 8th week of gestation, presented to your clinic wanting to do genetic screening, she declined invasive procedure . the best in this situation is a. Amniocentesis b. 1st trimester screening c. 2nd trimester screening d. Ultrasound The correct answer is b 10 87) Female presented with thirst and polyurea.. all medical history is negative and she is not know to have medical issues.. .she gave history of being diagnosed as Bipolar and on Lithium but her Cr and BUN is normal. What is the cause of her presentation a. Adverse affect of lithium b. Nephrogenic DI c. Central DI The correct answer is a 11 101) Pediatric came to you in ER with wheezing, dyspnea, muscle contraction ( most probably asthma), best to give initially is : a. theophillin b. Albuterol neubelizors c. oral steroids the correct answer is b 12 117) mother gave bitrh of baby with cleft lip and palate, she want to get pergnant again what is the percentage of recurrence a. 1% b. 4% c. 15% The correct answer is b 13 asking about duke criteria for diagnosis of infective endocarditis > what i remember a) +ve culture + mitral valve regurg
| P a g e

it was 2 major . i choose it 14 136) 55 y complain of dyspnea, PND with past history of mitral valve disease diagnosis is a-LT side HF b- RT side HF c- pnemothrax d-P.E The correct answer is a

15 139) 15y boy with unilateral gyncomastia your advice is a- my resolve spontiniously b-there is variation from person to person c-decrease use of soda oil or fish oil d-??????????? the correct answer is a - uni- or bilateral gynecomastia occur normally in newborn & at puberty 16 CA125 is a tumor marker mostly used for ovarian Ca 17 prevent malaria what i remember vector eradicate+ prevent bite 18 pic of face with scale in nasal fold and around mouth ( red in color ) i choose seborrheic dermatitis i got 2/3 in derma ----------------------------19 168) man use saldinafil (Viagra), to prevent hypotension you should not use a-nitrate b-B blocker c-ACIE d-CCB the correct answer is a 20 fishy viginal discharge > bacteria vaginosis 21 178) sickling pt after acute attack , discharge on a- penicillin b-iron
| P a g e

c-vitamin d??????????? the correct answer is a prophylactic 22 what type of edema in CHD a) alveolar b) interstial( my and) c) 23 rubella infection during pregnancy what will do a) no tttt b) vaccination c) immunoglubulin 24 226) In the appendisits the histology is: a. leukocyte in muscle b. layer of lymphoid c. tumor d. plasma cell the correct answer is a in appendicitis : neutrophil exudation throughout mucus, submucus, and muscularis 25 259) 6 years child was born to HBS positive mother is HBS positive , he was only vaccinated by BCG after birth , what you will give him now : a. HBV + oral polio + DTP + hib b. HBV + oral polio + dt + MMR +hib c. HBV + oral polio + Dt + MMR d. polip+ mmr+ dtp+ hib ( my ans) 26 287) Unilateral worsening headach , nausea , excacerbeted by movement and aggrevated by light in 17 old girl. a. Migraine (Photophobia, vomiting) b. Cluster the correct answer is a 27 osteoporosis risk factor > age no mention exogenous 28 305) which vitamin is given to new born to stop bleeding a. vit. A b. vit. D c. vit. K
| P a g e

d. vit E e. vit C the correct answer c 29 308) child with low grade fever and congested throat, negative ASO and positive EBV. he has a. infectous mononucleosis b. URTI the correct answer is a 30 the most common cause of intracerebral bleeding is. a. Hypertensive angiopathy( my ans) b. annurysm 31 most common cause of secondary HTN is a. Renal disease 32 exercise recommended for patients with CAD. is a. isometric b. isotonic c. yoga 33 old Patient wil LLQ pain, vomiting, fever, high WBC (17.000), tenderness and rebound tenderness a. Diverticulitis ( my ans) b. Sigmoid volvulus c. Appendicitis d. Toxic enteritis 34 case about abdominal aortic aneurysm 35 , lesion in brain for taste a. Temporal lobe b. Cerebellum c. Parietal lobe d. Occipital lobe 35 brain cell death in alzehimer disease ( not recognized his wife and fighting whit her ) a. Temporal lobe b. Cerebellum c. Parietal lobe d. Occipital lobe
| P a g e

36 366) Seldinfil is contraindicated with: a. Nitrate b. Methyldopa c. Gabapentine The correct answer is a 37 373) Young patient with pharyngitis, inflammation of oral mucosa and lips that has whitish cover and erythmatous base, febrile, splenomegaly. Dx: (this is infectious mono) a. Scarlet fever b. EBV c. HZV The correct answer is b 38 swlling inside lip ?? i choose gingyval cyst no idea 39 441) 50 year old Man presented to ER with sudden headach, blurred of vision and eye pain. The diagnosis is: a. Acute glaucoma b. Acute conjunctivitis c. Corneal ulcer d. ----The correct answer is a 40 max hight 12month 24 month 36month 41 panic attack . palpitation. sever anorexia a. Ssri( my and) b. tricyclic d. MAOI

42 pt going to surgery . afraid of DVT What we give ( cost effective . and .and) LMWH UNFR. HEPARIN
| P a g e

ASPRIN WARFARIN 43 50 year old Man presented to ER with sudden headache, blurred of vision, and eye pain. The diagnosis is: a. Acute glaucoma b. Acute conjunctivitis c. Corneal ulcer d. -----The correct answer is a. 44 BREAST tenderness ,fluctuant. Axillary l node enlarged ABSESS 45 509) Pregnant women has fibroid with of the following is True: a. Presented with severe anemia b. Likely to regress after Pregnancy c. Surgery immediately d. Presented with Antepartum He 46 514) A boy felt down on his elbow , the lateral x-ray shows: a. Anterior Pad sign b. Posterior pad sign c. Anterior line of humerous intersecting the cubilium d. Radial line forming 90 degree with cubilium the correct answer is b 47 about iron def anemia table with low mcv and low mch. athors normal 48-49 2q about psychiatry ans was SSRI I GOT 5/5 50 T score of bone densometry = ( -3,5) diagnosis is osteoporosis 51 ectopic pregnancy in fallopian tube what you well do wait and observe laborotomy albroscopy 52
| P a g e

570) Patient with left bundle branch block will go for dental procedure ,regarding endocarditis prophylaxis: a. No need b. Before procedure. c. After the procedure. d. The correct answer is a 53 572) most common vaginal bleeding : a. cervical polyps b. menstruation 55 good prognosis for schezophrenia: 56 acute panceraititis / feeding a) TPN B) NJ TUBE 57 601) An adult was presented by sore throat , congestion , fatigue , petechia in soft palate , tender spleen , and liver , what is the most likely diagnosis : A. EBV . 58 table aboute low bicarb normal urine ph another electrolye i do not remember ( k, ca , na ,) respriatory alkalosis reps acedosis meta alkalosis meal acidosis 59 smoking > pt desire 60 tactic to talking history from pt no talking direct Q 61 MD with albuminurea tttt ACEI ( MY ANS) 62 PT blood group a , given blood group b what type of reaction 63 most common cause of sub arachnid hemorrhage
| P a g e

aneurism av malformation htn 64 622) Benign tumors of stomach represent almost : A. 7 % B. 21 % C. 50 % D. 90 % The correct answer is a 65 705) Cardiac syncope: a) Gradual onset b) Fast recovery c) Neurological sequence after d) ? The correct answer is b 66-67 2Q ABOUT testicular sever acute pain one what is the diagnosis , and the other one what to do > urology refferal 68 test ( fece the waal . lean down ) dignosis for sciliosis

69 688) Child with atopic dermatitis at night has stridor plus barking cough on and off from time to time, diagnosis is: a) BA b) Croup c) Spasmadic Croup d) ? The correct answer is c Spasmadic croup : recurrent sudden upper airway obstruction which present as sridor and cough . approximately 50% of children have atopic disease. 70 697) A vaccination for pregnant lad with DT a) Give vaccine and delivery within 24 hrs b) Contraindicated in pregnancy
| P a g e

c) Not contraindicated in pregnancy d) ? The correct answer is c 71 702) 10 years old child with rheumatic fever treated early, no cardiac complication. Best to advice the family to continue prophylaxis for: a) 1 month b) 3 ys c) 4 ys d) 15 ys The correct answer is d ( I'm not sure , if there is 11 years choose it ) American heart associated has recommended all patient with history rheumatic fever be placed on longe term penicillin prophylaxic. Duration of prophylaxis depends on presence or absent of carditis, but for children without carditis , duration minimum 5 years or at age 21, whichever is longer. 72 Female patient did urine analysis shows epithelial cells in urine, what is the diagnosis: a)uti b) renal stone 73 Child came with his father and high BMI and look older than other children with same age, on exam child has >95th percentile of weight and tall, management is: a) Observe and appoint b) Life style change c) Give program to decrease the weight d) life style change 74 752) As doctor if you see patient and you face difficulty to get accurate information from him the best tactic to do it is: a) Ask direct question b) Ask open question c) Control way of discussion d) ? The correct answer is a 75 767) A child is complaining of severe headache which is unilateral, throbbing and aggravated by light, diagnosis is: a) Migraine b) Cluster Headache c) Stress Headache
| P a g e

d) ? The correct answer is a 76 768) The most important factor for smoker to quit is : a) Patient desire b) Give nicotine pills c) Give programmed plan d) Change life style The correct answer is a 78 central line > then sepsis in child what is the cause e coli group b strept h. inf

79 819) 60 y/o male Dx to have acute pancreatitis ( there was values for ranson criteria that I couldnt memorize ) what is the appropriate nutrition : a- TPN b- Regular diet with low sugar c- High protein ,high ca , low sugar d- Naso-jujenal tube The correct answer is d 80 821) Female with Hx of lt flank pain radiating to groin , symptoms of UTI & N+V what is DX: a- Appendicitis b- Diverteculitis c- Renal colic d- .. The correct answer is c 81 938) A very very long scenario about a female patient with vaginal discharge malodorous watery in character with pH of 6 & +ve clue cells but there is no branching pseudohyphe. (He is telling you the diagnosis is vaginosis & there is no fungal infection) Then he asks about which of the following drug regimens should NOT be used in this paitent: a. Metronidazole (PO 500 gm for 7 days) b. Metronidazole (PO 2 large dose tablets for 1 or 2 days) c. Metronidazole (IV or IM ..)
| P a g e

d. Miconazole ( PO ..) e. Clindamycin (PO ..) The correct answer is d This case is " bacteria vaginosis" and treatment by : metradinazole or clindamycine . avoid miconazole because it is antifungal 82 Provelaxix

//

Q1- 4 y/o child awake from sleep because a croup , which one shuld be in you DDx ; A/ foreign body ( my answer ) B/ broncholitis C/cystic fibrosis D/ congenital heart diseas there was no B. A in MCQs

Q2- 18 Y/o girl NOT sexually active .came with vaginal bleeding ,the doctors cant exam her due to the pain , what is the NEXT step ? a/ reassure her that it is normal in her age , and follow after three month if bleeding dont stop .

| P a g e

b/ urine pregnancy test c/ ultrasound d/ refrer to OBGyne I dont know

Q3- female patient came with fatigue and Jaundice. her CBC shows WBC =9 HGB= 9.5 ,PLT= 200 and his LFT show totalbilirubint =3 , direct = 0,9 ... wahat is the most liklyDx : a/Dubin Johnson syndrome b/Gilberts syndrome C/primary scelerosing cholangitis D/criglernajjar syndrome type 1

Q-4-8 y/o child with BMI= 30 and his height is more than 95 % for his age ... the next step ?senario not complete because the rest not important ? A/ observation and follow after 12 month b/ surgical intervention c/ obesity medication d/ life style modification

Q5- which one of the following is the best managment for 1 degree burn A/ debdidment b/warm wash and remove the material C/ water and ice d/ keep the affected area in cool area

| P a g e

Q6- what is the most reliable laboratory to estabilishe diagnosis of Acute gloerulonephritis ? A/ RBC cast in urine B/ increase WBC in urine C/low HGB with normal RBC D/ small shrunk kidney by ultrasound Q7- which one of the following anti TB medication is consider as drug induce SLE A/ ethambutol b/INH c/streptomycin d/rifampin Q8- endemic means a/ spread of disesas in incidence all the time b/it cause by virulent pathologic organism c/ spread of disesas from country to country by carrier d/rapid spread of disesase e/ there is very low incidence Q9 - before 14 d the child was bite ,nowdevelope lip swelling eryrhema ... , what type of hypersensitivity ? a/type 1 b/type 2 c/thype 3 d/type 4

Q10- 46 y/o male came to ER with abdominal pain but not that sever. He is hypedyslipidemic ,smoking ,HTN , not follow his medication very well , vitaly stable ,, o/E tall obese pt . mid line abdomen tendernes , DX A/ marfan syndrome B/aortic anyrsm

| P a g e

Q11- 17 y/o boy came with andominal pain and vomiting and leg cramp blood test was done and randome glucose = 23 {{ pic. of DKA , what is the most important next step a/ abdominal ulratasound b/ABG c/ urine analysis by dipstick d/chest x- ray Q12- elderly patient K/c of HTN and BPH , which one of the following drug Is potentially recommended as such case : ... a/ atenolol b/terazosin c/ losartan Q13- Gualine-Barrie syndrome is closlyassocited with which one of the following : a/ descending paralysis start from upper limb b/normal CSF c/ascending paralysis start from the lower limb d/need ECG Q14- the physicain will look in patient with idiopathic autonomic insufficiency for which one of the followig ... A/ absent sinus tachycardia B/ muscle wasting c/ orthostatic hypotension D/horner syndrome Q15- student in college .complin of sever itching in ankle and between finger ,, it was first attak ,, well demarcated .. Dx scabis tiaene Q16- ptsustaine RTA his bp 70/90 HR=140 RR=40 cold skin}} sign of hypvolemicshock ,, clinically there is bilateral pelvic fracture , Wht is the Appropriate NEXT step ... A/ IV replacement b/ blood transfusion c/ splint fracture
| P a g e

as ATLS guidline maximum of 2 litter IVF then blood transfusion shoud be consider , Q17- which on of the folowwing is a MINOR crteria for rheumatic fever ? a/ arthritis b/erythermamarginutum c/chorea d/ fever

Q18- regarding mitral stenosis which one is true the answer is mid diastolic murmer low pith 100 % suuure

Q19- 24 y/o healthy male complain of RT testes swelling , O/E there is larger rtmass not tender ,and the patient told you that its growin by the time , What is the most Approprite step in this situation : A/ refere to open scrotm and take a biobsy B/ refere to genealsurgey for ultrasound and take opinion C/ observation and foloow up next month Q20- healthy femal came to your office complain of lesion in her vagina that stared sice just 24 h . O/E there is cystic mass lesion non tener measure 3 cm on her labia , what is the the most likely Dx : ... A/ bartholincysy B/Vaginal adenosis C/ schic cyst D / hygroma Q21- which one of the following factors MOSTLY determiner the recurrence of colorectal cancer : A/ age B/stage ( my answer ) C/family history D/gender

| P a g e

Q22- adult healthy male came with tender red swelling on right hand up to forarm and you found black head and large pore skin , he said it happen after trauma to his hand 1 week back , the management shuld be : A/ topcical antibiotic B/ topical Antfungal C/cryosurger D/ Oral antibiotic acyually I dagnose this case as Hidraadenitissuppurativa but I am not suure and as i know the ttt= mild = cold compressor ( was not in MCQ) moderater =AB sever= surgery Q23- male, presented with pain in the posterior aspect of the thigh, he was running long distance felt a pop in his thigh, on exam, tenderness, erythema, and swelling, no defect what is the best treatment: a- Surgery b- Ice, rest, bandages, and elevation of the limb c- Bandages only. d- Splint. Q24-What is the most ACCURATE diagnosis for Ectopic pregnancy : ... A/ serial B-HCG B/ ultrasound C/laproscopy D/progesteron

Q25- what is the best management for binge eating disorder: A/ cognitive behavioral theraby( my answer ) B/ problem solving theraby C/ interpersonal theraby Q26- Which congenital heart condition is the most common associated with endocarditis A/ VSD B/ ASD C/PDA D/TOF ( most likely )

| P a g e

Q27- A man who is thinking that there is Aliens in his yard although that he knows that Aliens are not existing but he's still having these thoughts especially when he is out of home he is afraid to be die due to that ..Dx A/obsession B/ delusion C/ hallucination D/ illusion Confuse Q28/ patient diagnose withDm type 2 and he is in your office to discusse with him the plane to reduce his weight , you will told him to : A/decrease calori intake in day time B/decrease calori and increase fat C/decrease by 500 kcal/kg per week( my answer correct ) D/ decrease 800 per day

Q29- Old patient with asthma and urine retention due to prostatic enlargement, hypertensive (BP: 180/100) what's the most appropriate drug to control hypertension? A/ Labetalol B/Phenalamine C/Propanolol Same Q before but her with BA and different drug

Q30 Which one of the following dug consider as drug induce urticarial A/ Azithroycin B/ hydralazine C/ cortison D/ Penicilline( my answer ) Q31 - regardingBoutonnere deformity which one is true A/ flexion of PIP &hyperextension of DIP.(my answer most likely )
| P a g e

. B) flexion of PIP & flexion of DIP C) extenion of PIP & flexion of DIP. D) extension of PIP & extension of DIP Q32 preganant lady 38 wks GA with placenta previa marginal with mild bleeding , the cevix is dilated cervix 2 cm How to manage ; A/ CS B/spontiusdelvery C/forceps delivery( most likely ) D/ do amniotomy Q33 -whaich one of the following is true regarding the weight gain in pregnancy: A/Pregnant woman should consume an average calorie 300-500 per day( my answer most likely ) B/ Regardless her BMI or body weight she should gain from 1.5 3 lb which represent the baby's growth

Q34- 9 years old female presented to ER after ingestion almost 20 tablets of OCP and 3 tablets of another medication..She is clinically stable and there was no signs and symptoms...What will you do: a.refer her to gynecologist. b.refer her to psychiatrist. c.toxicology study.( my answer ,, because we dont know which other drug even ifshe stable now and then we can referee ) confuse d.no need for intervention. Q35-Lichen planus most commonly found in : A/Scalp B/ Knee C/ Buttocks D/Mouth( my answer and most likely ) Q36- regardingright lung anatomy which one is true : A/one fissure
| P a g e

B/ pulmonary segment C/ no relation with azygus vein D/ 2 pulmonary veins ( my answer ) E/ no sibson's fascia Q37-Patient on Amitriptyline 30 mg before bed time, wakes up with severe headache and confusion, what's the appropriate action? A/Shift him to SSRI's( myanswer , not sure ) B/Change the dose to 10 mg 3 times dail C/ continue on the same

Q38- Old pt complaining of back pain on walking on examination there was stiffness of the muscle and there was some finding on the X-Ray of MODERATEspondyloarthropathy best effective ttt: A/ Physiotherapy ( my answer , not sure if Moderate different ) B/ NSAID C/ Surgery D/ bed rest Q39-17 years old with type I DM,he is mostly has association with HLA: A/ DR 4.( my answer correct) B/ DR 5. C/ DR 7. D/ DR 9. Q40 - 28 years old diabetic female who is married and wants to become pregnant.her blood glucose is well controlled and she is asking about when she must control her metabolic state to decrease risk of having congenital anomalies: A/ before conception.( my answer ) B/ 1st trimester. C/ 2nd trimester. D/ .3rd trimester. Q41-All can cause gastric ulcer except: A/ Tricyclic antidepressant.( my answer ) B/ Delay gastric emptying. C/ Sepsis. D/ Salicylates. E/Gastric outlet incompetent
| P a g e

Q42-Likelihood ratio of a disease incidence is 0.3 mean: A/.large increase B/small increase C/no change D/ small decrease ( my answer ) E/large decrease Q43-8 months child with 3 days fever 40 , vomiting , convulsion , poor feeding & sleep , OE dehydrated , depressed ant frontanell, red ears ,no neck stiffness , his 3 year old sibling asymptomatic , whichof the following will give the defenitiveDx : a- CXR b- CBC with deferential c- blood culture ( my answer not sure , I think about sepsis ) d- CSF analysis e- suprapubic urine analysis Q44-in PHC, from 50 child 10 got the disease on the 1st week, another 30 on the subsequent 2 weeks, what is the incidence of the disease in that PHC? a- 20% b- 40% c- 60% d- 80% ( my answer ) , 10+30 /50 e- 90% Q45-the name of Questionnaire that differenctite between the primary and secondryapnea : Straing name BUT the correct answer is horchover Q46-which one is true Regarding appendicitis in the elderle: A/ If the patient is afebrile this rules out appendicitis B/WBC is often normal( my answer not sure but was most likely for me ) C/Rupture is not common D/anemia is coming finding Q47- A middle age man presented with severe headache after heavy lifting objects. His BP was high. He was fully conscious. Examination was otherwise normal. the most likely diagnosis is: a) Subarachnoid hemorrhage b) Central HTN c) Tension headache
| P a g e

d) Migraine e) Intracerebral hemorrhage Q48-patient HIV have white patch in oral cavity and how could you manage A/oral antibiotic B/loacal antibiotic C/local steroid D/chemo & radio theraby ( my answer , there was no antifungal so not candida, I diagnose it as Kaposi ) Q49- middle aged pt with ataxia , multiple skin pigmentation and decrease hearing , one of the family member has the same condition , what is the most likely DX : a- Malignant melanoma b- neurofibromatosis ( my answer ) c- hemochromatosis d- measles e- nevi Q50 -5 y.o child with history of fever and swelling of the face ant to the both ears (parotid gland enlargement) what is the most common complication at this age group : A/meningitis B/ labrynthitis C/ orchitis

Q51- what is the treatment for common mastalgia : A/ tamoxifen( my answer ) l B/ caffeine C/ OCP NO in MCQ danazolnorcromocriptine
| P a g e

Q52-4 y/o child with diarrhea for 2 days is complaining of anal discomfort. Your advice to the mother is: A/Wash with soap and water after each episode of diarrhea. B/Wash with cotton in warm water.- ( my answer ) C/Put a clean napkin in the underwear.D/Change the underwear to a highly absorbent diaper Q53-pt taking lasix having CHF and his electrolytes showed hypokalmia3 ,hyonatrima 123, hyperglycemia , hypchloerima and high urea and he had muscle cramps and weakness u will give A/ NS with 5 KCl In 20cc/hr -B/ NS with 40 KCL in 80cc/hr ( my answer ) C//2Ns with 5kcl in 20cc/hr D/2 NS with 40 kcl in 80cc/ hr Q54 4 Y/O Baby with scenario of ADHD, what is the best treatment in addition to behavioral therapy: a- Atomoxetine( my answer correct ) b- Imiramine Q55- 9- The most cause of tinnitus: A/ vitiligo b- Sensory neural deafness c- acute otits media d- noising induce tinnitus. ( my answer ) Q56- which one of the following anti TB medication consider drug induce SLE The answer is INH Q57- which on of the following OCP cause hyperkalemia : The answer is ethynyl estradiol

| P a g e

salma abubaker ,, alkhobar my SLE exam on 27/2/2012

-Child presented with gum and nose bleeding and bruising all over the body after an episode of URTI. Dx: 5) 6) 7) 8) Henoch Scholein Purpura Idiopathic thrombocytopenic purpura(sure) Vitamin K deficiency Hemophilia

2-the most a.alopecia b. wt c.hypotention d.constipation

common gain(

side correct

effect ,,

of got

antipsycotic 5/5)

3-year female has atypical squamous cells of undetermined significance on pap smear, past hx reveald 3 -ve smears, last one was 7 years ago she also geve hx of viginal wart, next step is: colposcopy hystrectomy follow up after 1 year excision by

| P a g e

4-which drug economical use twice a day a.ibuprofen b.Piroxicam C.Indomethacin d.Naproxen

5- 2 y/o child presented with painful swelling on the dorsum of both hands and feet,, he was jaundiced with Tbilli 3 Dbilli .9 ,, hgb 9 and retics 7,, what u will do as ongoing managment a.steroid b.NSAID c.penicillin and immunization(sure) d.paracetmol

6- 26 y/o pt. k/c of depression taking (citalopram)for depression ,, presented with ingestion of unknown drug ,,, on investigation she was found to have metabolic acidosis and anion gab 18 ,,, what is the most likely drug she ingested?? a. paracetamol b. asprin c. citalopram d. amitriptyline

7- 19y/o not known to have any medical illness ,presented with fever, arthritis, and rash mainly in the palms and soles ,,, he gave hx of illegal relationship ,,, mostly he is having?? a.chancroid b. 2ndry syphilis c. chalmedia trachomatis

| P a g e

8- 60 y/o pt. presented with decrease vision bilt , specially to bright light on exam he was having cupping with wedge shaped opacities ... he is having?? a.lentis sublexation b.catract (my answer and i got 3/3) c.open angle glucoma

9- 65 y/o pt. presented with hepatosplenomegaly and lymphadenopathy ...bone marrow bx confirm dx of CLL,, the pt gave hx of breast cancer 5 yrs ago and was treated with chemotherapy since then ,, the pt is also smoker what is greatest risk for developing CLL?? a. hx of radiation b. smooking c. previous cancer d. age

10- 25 y/o presented with ear pain and hearing loss in the rt. ear ,, on exam there was ear drum swelling and obscured tympanic membrane with fluid behind the membrane dx is?? a, otitis media b, tympanic cellulitis c,Chondrodermatitis

11-female pt c/o sever migraine that affecting HER twice weekly, she dont want regular medication best ttt you give - triptan - beta bloker - amitrptalyin - bio feedback

| P a g e

12-A child is about to be given flu vaccine, what allergy should be excluded before giving the vaccine? 4) Chicken 5) Egg(sure) 6) Fish 13-. Patient with greenish nasal discharge, was treated before with antibiotic but with no benefit. Management: C. Steroids D. abx

14-Young patient with unremarkable medical history presented with SOB, wheeze, long expiratory phase. Initial management: 5) 6) 7) 8) Short acting B agonist inhaler Ipratropium Steroids Diuretic

15- 38 week pregnant lady came to ER in labor, cervix 4.5 cm dilated, marginal placenta previa. Management: 1) Wait and evaluate fetus 2) SVD 3) C/S 4) Forceps 5) Rupture membrane

16- mother gave bitrh of baby with cleft lip and palate, she want to get pergnant again what is the percentage of recurrence
| P a g e

a. 1% b. 4% c. 15%

17- In the appendisits the histology is: a. leukocyte in muscle b. layer of lymphoid c. tumor d. plasma cell

18-old Patient wil LLQ pain, vomiting, fever, high WBC (17.000), tenderness and rebound tenderness a. Diverticulitis ( my ans) b. Sigmoid volvulus c. Appendicitis d. Toxic enteritis

19-Young patient with pharyngitis, inflammation of oral mucosa and lips that has whitish cover and erythmatous base, febrile, splenomegaly.
| P a g e

Dx: a. more common in children less than 14 yrs b. EBV c. HZV

20- 10 years old child with rheumatic fever treated early, no cardiac complication. Best to advice the family to continue prophylaxis for: a) 1 month b) 3 ys c) 6 ys d) 15 ys

21-The most important factor for smoker to quit is : a) Patient desire b) Give nicotine pills c) Give programmed plan d) Change life style

| P a g e

24- 50 y/o female, operated for ovarian cancer, come to clinic for follow up , abdominal xray show scissor, what to do: a. Inform and refer to surgical. b. Inform and tell her it will resolve alone. c. Call attorney. d. Dont inform.

25- Pic of psoriasis, pink scaly lesion on the elbow,knees and scalp how to prevent flares: a. Avoid sun exposure. b. Avoid trauma c. use steroid

26- Computer programmer, a case of carpet tunnel syndrome, how to splint: f. Dorsiflexion( sure)

27- Gastrictomy post-op 1 day. He have temperature 38.8 & pulse 112. What is the most common cause ? a. wound infection. b. inflammatory mediator in the circulation. c. UTI
| P a g e

d. normal

Child had chest tightness and cough when exposed to cold and excersise, - : what to give for prophylaxis a. B2 inhaled agonist .b. Steroid inhaler .c. Tehyophillin d. Oral steroid

ttt of acute gouty arthritis - Allopurinol a. b.

(Indometathin(sure Pencillamin Steroid c.

d.

| P a g e

30-Most common cause of immediate death in flam burn: a. Inhalational injury. b. Septic shock. c. Hypovolemic shock. d. Other injury.

31- what is the most reliable laboratory to estabilishe diagnosis of Acute gloerulonephritis ? A/ RBC cast in urine B/ increase WBC in urine C/low HGB with normal RBC D/ small shrunk kidney by ultrasound

32- Gualine-Barrie syndrome is closlyassocited with which one of the following : a/ descending paralysis start from upper limb b/normal CSF c/ascending paralysis start from the lower limb ( sure ) d/need ECG
| P a g e

34- man walking in street and saying bad words to stranger , he is not aware of his conditiond , what is the description : flight of idea Deprivation(got 5/5) insertion of idea loosening of association

34-OCP increase risk of which of the following?? 5) 6) 7) 8) Ovarian cancer Breast cancer Endometrial cancer Thrompoembolism

35- Female com with lump in breast which one of the following make you leave him without appointment Cystic lesion with seruse fluid that not refill again?? Blood on aspiration Solid Fibrocystic change on histological

36-Holding breath spell or holding ..which of the following is true A)mostly occurs between age 5-10 B)increase risk of epilepsy C)a known precipitant cuz of generalized convulsion my answer D)diazepam may decrease the attack e) can occur in absence of emotional upset
| P a g e

37-An old woman complaining of hip pain that increases by walking and is peaks by the end of the day and keeps her awake at night, also morning stiffness: Osteoporosis Osteoarthritis

| P a g e

38-old pt c/o bilateral knee pain with mild joint enlargement ESR and CRP normal dx - Osteoarthritis - Rheumatoid arthritis - Gout - Osteoporosis

39- Most common organism causing cellulitis in the age 6-24 month E. Strepto coccus F. Heamophilus influ G. Staph H. ,,,,,

40-pregnant lady with hyperthyroidism what you will give her : propylthiuouracil methamazole B blocker Radioactive iodine

41-The most common cause of immediate death in flame burn victims: Inhalation of smoke. Associated injures Hypovolemic shock. Septic shock.
| P a g e

42- Pt dx to have aortic stenosis ,,, he is a teacher ,, while he was in the class he fainted,,, what is the cause?? E. Cardiac syncope F. Hypotention G. Neurogenic syncope H. ,,,,,

My SLE in 5th of March 2012

.. .. 2326 yo psychotic patient presented to the hospital after 3 houres of ingestion of 3 pins, PE : unremarkable, X ray showed 3 pins in small intestine but no intestinal dilation or air fluid level. You action will be : e- Admit the patient to the hospital for serial x-rays and abdominal examination. f- Send the patient home and give follow up appointment. g- Start antibiotics and send home. h- Admit the patient and start antibiotics. 24Most effective antibiotic to treat gonorrhea is : f- Ceftrixone g- Penicillin G. h- Pipracilline. i- Gentamycin. j- Vancomycin
| P a g e

25Which of the following true regarding Apgarscore : e- Total score 12 f- Discoloration is not important g- Heart rate significant h- Assessed in the 2nd day of life. 26Patient with hx of recurranteosphagitis in the last 5 years , biopsy shows that presence of glandular islet an columnar cells in squamus cellular zone , eosphagescopy showed finger like projection upward the squamo-columnar area . most likely diagnosis ; e- Adenocarcinoma of the esophagus f- Squamus carcinoma of the esophagus g- Barret esophagus h- Normal picture of esophagitis 27Football player injured in the lateral side of his LT knee, presented to you with sever knee pain, PE there is swelling in the medial aspect of the ltknee ,valgus test showed free mobility but lachman test and McMurray's test are negative . what's your diagnosis: f- Lateral collateral ligament injury g- Medial collateral ligament injury h- Patellar fracture i- Medial menisci injury j- Lateral menisci injury

28Regarding lung cancer: c- It's the leading cause of death in females d- Adenocarcinoma common in the proximal part

29Which one of the following is a strong indicator to do diagnostic peritoneal lavage: f- Comatose patient due to sever head trauma . g- Patient with pelvic fracture h- Patient with sever abdominal pain and distention i- Patient with BP 80/56 with abdominal distention j- should be done to every patient had RTA. 30Patient with rheumatoid arthritis came to came to you and asking about the most effective way to decrease joint disability in the future, your advice will be: c- Cold application over joint will reduce the morning stiffnesssymptoms
| P a g e

d- Disease modifying antirheumatic drugsare sufficient alone 31Most common cause of recurrent tonsillitis : f- Group B streptococcus g- EBV h- Bacteriod i- Rhino virus j- Parainflunza virus. 32A 10 YO was diagnosed with rheumatic fever without any defect to the heart. You will tell his parents that he needs to take prophylactic antibiotics for how many years? a- 5 months b- 3 years c- 6 years d- 15 years 33Child withhx of URTI 1 week ago now he c/o arthalgia , fever and fatigability , what's your diagnosis: d- Rheumatoid arthritis. e- Rheumatic fever. f- .

34While you do head and neck exam , which one of the following is NOT palpable normally: F- Thyroid gland G- Submandibular gland H- Parotid gland. I- Lymph nodes J- Hyoid bone 35Which of the following finding suggesting anemia of chronic disease: e- Increase serum iron and increase TIBC. f- Decrease serum iron and increase TIBC. g- Decrease serum iron and decrease TIBC. h- Increase serum iron and decrease TIBC. 36Yong man predict that he is going to have a seizure , then he became rigid for 15 sec the developed generalized tonic clonic convulsion for 45 sec. you initial ER action in future attacks will be : d- insert airway device. e- Apply physical splint or protection.
| P a g e

f37about vareciall vaccine in adult , which is true ; e- 2 vacceineabart of 1 month f- 2vaccine abart of 6 month g- 2 vacceineabart of 2 month h- 3 vaccine abart of 6 month 38The useful excurcise for osteoarthritis in old ageto maintain muscle and bone: d- Low resistance and high repetion weight training: e- Conditioning and low repetion weight training f- Walking and weight exercise 39Pregnant lady 18 wks, her TFT showed : high TBG, high level of activated T4 , normal T4 and TSH . what is the most common cause of this result: e- Pregnancy. f- Compensated euthyroidism. g- Subacute thyroiditis. h- . 40Patient with disc prolaps will have: e- Loss of ankle jerk f- Fasciculation of posterior calf muscles. g- Loss of Dorsiflexion compartment of the foot. h- Loss of the sensation of the groin and anterior aspect of the thigh. 41Child presented to the ER after bee sting with SOB, anxiety and wheezing.PE : BP 75/54 , HR 120 and RR 20. Your action will be: d- Start IVF , IM epinephrine and antihistamine. e- Reassure the patient and tell him that everything gonna be OK after antihistamine injection. f- . 42Patient loss his wife in the last 4 months , he looks sad cannot sleep in the last 2 days, which medication can help him: e- Lorazepam( sure .. b/c it's short acting ) f- Diazepam g- SSRI. h- . 43Wha's true about Malaria : the most common cases is caused by Plasmodium falciparum.
| P a g e

44Pt work most of the time on the computer came withwrist pain ,positive tinel sign you will do cast for thehand so the hand position should be in e- Dorsxiflexion sure 100% f- Planter flexion g- Ulnar deviation h- Extension

7 March 2012 my SLE ! : ... many thanx to study group for your help ,support and encouragement..all of you ,,iddnt had the chance to thank you may GOD bless you Abo Yasser 77. ((((There is only one way to do any exam and that is once!)))))

1. 6 month child , difficulty in breast feeding , active pericardium, pansystolicmurmer s1 , loud s2 ASD large VSD MR AR PDA 78. 46 Y male , c/o early ejaculation , unability to sustained eriction , he belive his 26 yrmarige is alright , his wife ok but unorganized , obese . doctor confirm no organic cause. He look thin .sad face ,whatsttt: SSRI Sublingual nitrate 6 h befor testesteron injection 79. DM obese lady , newly discovered type 2 , compliance with diet and exercise , when start medication she felt dizziness ,dry mouth , which drug cause her symptoms: sulfonurea
| P a g e

80. Lady with 2 day hx of fever , lower abd and suprapubic tenderness , viginal discharge & tenderness Dx: acute salpingitis chronic salpingities acute appendicitis 81. Last trimester pregnant lady develop sudden left leg swilling .extend from left inguinal down to whole left leg , ttt venogame,bedrest,heparin duplex,bed rest ,heparin pleosongraphy,bed rest, cavalfelter duplex ,bed rest ,warfarine 82. Secondary prevention: seat bealt influenza vaccine for elderly DPT vaccine for children coronary bypass 83. Child with iron toxcicity several hours ago , investigation show iron conc. 700 mg/dl ,ttt: gastric lavage activated charcoal iv deferoxamine 84. Ptosteopnia in femure with increase serum alkaline phosphatase , normal serum calcium, normal phosphate ,normal vit d: ttt estrogen receptor modulator calcium regulator bisphosophnate 85. Pt respiratory problem , foul smelling , CXR bilateral lesion at base of lung, heamoptosis , finger clubbing: bronchactasis 86. Child with ear congested , opacity , recureent URTI , o/e NEED adenectomy , beside adenectomy u must do: tosilectomy maryingotomy government tube 87. Middle aged 23 y/o not known to have any medical illness apart of annular lesion in mouth with painless ulcer ,presented with fever, arthritis, and rash mainly in the palms and soles ,,, he gave hx of illegal relationship ,,, mostly he is having?? 2ndry syphilis
| P a g e

88. 60 y/o pt. presented with decrease vision bilt , specially to bright light on exam he was having cupping with wedge shaped opacities ... he is having?? a.lenssublexation b.catract (my answer and i got 3/3) c.open angle glaucoma 89. Which of the following not a live vaccine: BCG HB OPV 90. Pt came with deep injury on the wrist site, the nerve that has high risk to be injured will manifest as? Can not oppose thumb to the other finger?median nerve 91. Pt work most of the time on the computer came with wrist pain , positive tinel sign you will do cast for the hand so the hand position should be in Dorsxiflexion 92. An elderly lady presented with Swilling knee pain bilaterally that increases with activity & decreases with no history of trauma .The most likely diagnosis is: Osteoarthritis 93. 5 y.o child with h.o fever and swelling of the face ant the both ears (parotid gland enlargement) what is the most common complication >> meningitis 94. PT case of CHF , loved to eat outdoor 2-3 time weekly u advice him: - eat without any salt -eat 4 gm salt -low fat,high protein 95. known case of cervical spondylolysis , presented by parasthesis of the little finger , with atrophy of the hypothenar muscles , EMG showed cubital tunnel compression of the ulnar nerve , what is your action now? Cubital tunnel relase- decompression
| P a g e

96. Female with dysuria on examination there is epithelial cell >>>>chlymdia urethritis 97. hx of travel abroad , best in elimination of entamebahistolytica>> - boiling water 98. child 6 yo with R.FEVER , continue AB for>>>15years 99. pt known smoker 10 cigar for last 10yr, present with oral ulcer , recived antibiotic with no improvement ?? - biopsy 100. newborn given injection to reduce bleeding>>vit. K

101. definition of epidemic curve >>>A graph in which the number of new cases of a disease is plotted against an interval of time to describe a specific epidemic or outbreak. 102. commonest cause of intracerebralhaeg.: aneuresym trauma Hypertention 103. headach , which trure: - normal CT can exclude subarachnoid haeg. -clusterheadach more in male . 104. testicular fullness ,like bag of worm , positive valsava: varicocele 105. testicular mass at tip , positive valsava: indirect inguinal hernia 106. female 25yo , ask you about breast self examination when should be done: -6-7 day after cycle -5 day befor -7-10 day after -14-16 day after - after 2 day ????????????????????????/

| P a g e

107. teacher ,complain of panic , this after mistake in class room, he know it must be useful in future day , co sweting , tachycardia , tightness> - benzodiazepam -ssri -social plhobia 108. pt told you the refregator told him that all food inside poisoning: - audiatoryhalluscination -dellusion -illeusion 109. pt underwent colectomy dignose as stage B2: - NO lymph node involve -2to4 l.n - one l.n 110. 111. mastalgiattt:OCP Fibroid :regress after pregnancy

112. Ttt for menpusalwomen ,c/o bleeding , not ass with intercourse: -estrogen -progesteron 113. Picture ,Child with skin lesion at elbow , seen positive wood lamp: -fungal -bacterial 114. 115. a. b. c. d. 116. Skin rash in buttock, hamturia :HSP Indication for tonsillectomy is: Pharyngeal abscess Sleep apnea Recurrent infection Asymmetric tonsillar hypertrophy Most common problem present in primary care :coryza

117. Child with duodenal atresia, characterstic sign in imigaing: -double bubble
| P a g e

118. Firest sign in increase intracraineal pressure: -vomiting -nausea -ipsilateral pupil constrict -cotralateral pupil constrict Answer is: decreased level of consciousness The full sequence is: decreased level of consciousness, confusion, headache, projectile vomiting, unequal pupils (anisocoria), and the presence of a pronator drift or motor weakness 119. Pt with barretesophagus , risk of get malignancy: _ adenocarcinoma -sequamous 120. Most common cause of CVA, Mostly embolic resource - AF -VSD 121. Old female ,fear from desk compression and fracture : - vitd,calcium -wt. reduction -progestrone 122. Known alcoholic chronic for long time, present with lymph node in mid cervical , your action: -larygoscop -excitional biopsy -needle biopsy 123. Old lady,outcome baby with Clinical feature of down , single palmer creases , epicanthic fold, wide palepral fissure - trisomy 21 124. Ptelderly , with unilateral headach , chronic shoulder and limb pain , positive Rheumatoid factor ,and +ve ANA ttt\: - asprine -indomethacine -corticosteroid 125. Asthmatic child , how to decrease the allergy: - cover pillow and bed with impermeable material
| P a g e

-throw the rugefrome house 126. Yong male with 3 day of dusuria, anal pain , O/E perrectum boggy mass : - acute prosatities 127. Outbreak of TB , person found negative TUBECLIN : - rifimpcin -vaccination 128. Kwashikower: - low protein,high CHO 129. PICTURE CXR of pericardial effusion,TYPICAL presentation S&S

130. 80 y/o male CASE HTN on ttt with mild begnine prostatic enlargement , causes feelling of incomplete voiding - alpha blockers -surgery

131. Computer programmer, a case of carpet tunnel syndrome,positivetinnel test , how to splint: Dorsiflexion( sure) 132. Chronic gastric ulcer ,pt intake a lot of antiacid , no still complian:ttt> - H 2 antagonist -proton pump inhibitor 133. Most reliable test to diagnose Acut glomerular nephrities: - red cast in urine. 134. City with 1500persons, no of 105 birth , 5 are still birth , 4 die at first month,2 die befor age of one year , perinatal mortality? -4 -5 -6 -8 -9 135. Pt. chronic depression ,now u start tt.Paroxetin (paxil) you told the pt: - need 3 or4 week to act
| P a g e

-side effect ??? idont remember 136. Phayngitis treated with oral penciling you should : - 7 day -10 day -14 day 137. Syncope due heart : - rapid recovery 138. Most specific for diagnosis of pulmonary embolism: -EKG -Ventilation perfusion ratio (V/Q scan). -pulmonary angiogram ???? 139. Elderly pt .fever and infection by enterocucusfecalies, source of infection: - urinary -lung 140. 12 y/o Child overweight BMI=31 , +ve family history of hyperlipedemia , parents fear of child get dyslipidemic, when you should request lipid profile?????????? - upon parent request - COZ overweight 141. Pt with high total chloestrol 265mg/dl , LDL 150 , triglecride 325 , HDL 100 most single risk factor??? - low LDL -High LDL -High HDL -low HDL -high total cholesterol 142. Best secreening test for liver malignancy: -us+ liver biopsy -CT scan + Liver BIOPSY -CEA + AFP

| P a g e

143. FEMALE , analysis of urine tet ,epithelial cells:indicate - vulvar contamination -cervical tear -renal stone -UTI 144. Young female always eat fast food , you advice supplement of: -zinc +vit. C -vit. C+ folic -vit.d+ zinc -folic acid+ Ca 145. Exercise recommended for osteoporosis pt.

146. Male with collosion bicyclic motor bike , closed head injury . cant direct spoon to his muoth , site of lesion: -cerbellum -parital lobe -frontal 147. patient has history of parotid and salivary gland enlargment complains of dry eye . mouth and skin ,, lab results HLA-B8 and DR3 ANA +ve rheumatoid factoe +ve what is the course of treatment a-physostigmin b-eye drops with saliva replacemnt c-NSAID D-plenty of oral fluid 148. LACERATION IN ANTERIOR ASPECT OF WRIST: - wrist drop -median nerve injury -claw hand 149. battered women which is true: a. mostly they come from poor socioeconomic area b. usually they marry a second violent man c. mostly they come to the E/R c/o.. d. mostly they think that the husband respond like this because they still have strong feeling for them

| P a g e

150. most common physiological cause of hypoxemia - shunt -Ventilation perfusion mismatch -hypoventilation 151. arterial bleeding after injury: - red blood ,continous -red bright , spruting -dark blood 152. teacher c/o malise fever , right upper abd tenderness , two student develop same condition , eye become icterus, best CONFIRM dignose: -HBA IgG - HBA IgM - HBA core AB 153. Apgar score: Heart rate is significant

SLE 10 March
Q1/pt with BP of 180/140 you want to lower the DyastolicBP(which is true ) : a- 110-100 in 12 hrs b-110-100 in 1-2 days c- 90-80 in 12 hrs d- 90-80 in 1-2 days Q2/pt with wt loss , night sweat ,generalized lymadenopathies , diarrhea , mild spleenomegally .. has a H/O blood transfusion at kenya .. most likely Dx : a-HIV (my answer) b-Lymphoma (it has the B symptoms but there is Hx of blood transfusion ) !! c-TB D-.. Q3/child with Massive HepatosplenoM , Blue nodule , n Mass on his Lt. cervical region .. next step ?
| P a g e

1-BM aspiration 2-EBV serelogy 34Q4/pregnant with attached lab results of Thyroid Func test reveals (normal TSH , High T4,High TGB) interpretation ? 1- normal with pregnancy (my answer) 2- compansatedEuthyroid 3- toxic T3 Q5/Acute GouteMx : 1-Alluprinol (used as prophylaxis ) 2- NSAID (correct) 3-Paracetamol 4-gold salt

Q6/pt with Hx of Appendectomy . now and distention ,cramp pain vomiting , constipation ,, Dx 1-mechanical Obs of small intestine 2-paralytic ilues 3- acute cholesystits Q7/pt medically free , has snoring .. exam wise normal ur advice : 1- to loss wt 2-adenoectomy Q8/ direct Q abt TOF (all are true except ) Q9/ pt ,her husband just passed away , didn't sleep for 2 days , you will give her ? 1- fluxetin 2-imipramin 3-Lorazepam (my answer) 4-cholpramzide
| P a g e

Q10/HBV serological marker (Know what is the 1st marker that rises and what rises at the window area and what rises after 20 wks ) HBsAg : indicate carrier state. HBsAb : indicate provide immunity to HBV HBcAg: associated with core of HBV HBcAB: during widow period, HBcAb-IgMindicate recent disease

3 yrs old child ,, ingest sth 30 min bk .. looks toxic and irritable ur 1st step ? a-maintain airway (my answer) b-active Charcot c-know ist acidic or alkaline agent 4-gastric lavage 5-endoscopy

| P a g e

New in 4 Edition
Done by:
1)Dr.Hten Al-Majed Al-Hazzani 2)Dr.Raneem Alshareef 3)Dr.Manal M AL-anazi 4)Dr.Hind sindi 5)Dr-Bayan Ahmad 6)Dr.Ahmed Alfaqwi 7)Dr.Mahmoud Hijazi 8)Dr.Azzam Alkhalifah 9)Dr.Yasser Alashhab 10)Dr. HASSAN ABDELRAHMAN 11)Dr.Hassan Mortaja 12)Dr.Rusha 13)Dr..wadah khairi

th

| P a g e

Hten Al-Majed Al-Hazzani march 10th 2012

1)unstable angina dx: - least grade II and new onset less than 2 months ago. -usually there is an evidence of myocardial ischemia. -same ttt as stable angina. -discharge when the chest pain subsides. 2)patient having chest pain radiating to the back, decrease blood pressure in left arm and absent left femoral pulse with left sided pleural effusion on CXR, left ventricular hypertrophy on ECG, most proper investigation to dx: 1-aortic angiogram 2-amylase level 3-cbc 4-echo 5-? 3)pregnant in 35 week with mild preeclampsia, presented with BP 150/95 and edema in lower and upper limbs, how to manage? -diuretics -immediate delivery -maternal and fetal evaluation and hospitalization (correct) 4)well tolerated in pregnancy: mitral regurg (got 5/5 in ob) 5)best place to find gonoccal in females: urethra rectum cervix posterior fornix of vagina pharynx 6)Na high, K low, HCo3 high: primary hyperaldosteronism, addison pheochromocytoma 7)snoring + tonsillar enlargement: weight loss,
| P a g e

CPAP, adenoidectomy (correct) 8)COPD coughing greenish sputum, whats the organism? staph aureus strep pneumonia mycoplasma chlamydia h.influenza 9)scenario of a child typical of ADHD, ttt: atomoxetine 10)PMS symptoms relieved with: fluxetine 11)patient has been wearing contact lenses for the past 10 years, now has photophobia, what do you recommend? -take them off at night (correct) -saline drops 4 times a day 12)red eye with watery discharge: local antihistamine (correct) steroids antibiotics 13) pregnant + high TG and Total T4, with normal level of free T4 and TSH because of? pregnancy (correct) thyrotoxicosis I tried to remember the new ones, I'll add later if I recalled anything else, best of luck!

SLE 10 March

Q1/pt with BP of 180/140 you want to lower the Dyastolic BP(which is true ) : a- 110-100 in 12 hrs b-110100 in 1-2 days
| P a g e

c- 90-80 in 12 hrs d- 90-80 in 1-2 days Q2/pt with wt loss , night sweat ,generalized lymadenopathies , diarrhea , mild spleenomegally .. has a H/O blood transfusion at kenya .. most likely Dx : a-HIV (my answer) b-Lymphoma (it has the B symptoms but there is Hx of blood transfusion ) !! c-TB D-.. Q3/child with Massive HepatosplenoM , Blue nodule , n Mass on his Lt. cervical region .. next step ? 1-BM aspiration 2-EBV serelogy 34Q4/pregnant with attached lab results of Thyroid Func test reveals (normal TSH , High T4,High TGB) interpretation ? 1- normal with pregnancy (my answer) 2- compansated Euthyroid 3- toxic T3 4Q5/Acute Goute Mx : 1-Alluprinol (used as prophylaxis ) 2- NSAID (correct) 3-Paracetamol 4-gold salt Q6/pt with Hx of Appendectomy . now and distention ,cramp pain vomiting , constipation ,, Dx 1-mechanical Obs of small intestine 2-paralytic ilues 3- acute cholesystits Q7/pt medically free , has snoring .. exam wise normal ur advice : 1- to loss wt 2-adenoectomy 34Q8/ direct Q abt TOF (all are true except )
| P a g e

Q9/ pt ,her husband just passed away , didn't sleep for 2 days , you will give her ? 1- fluxetin 2-imipramin 3-Lorazepam (my answer) 4-cholpramzide 5Q10/HBV serological marker (Know what is the 1st marker that rises and what rises at the window area and what rises after 20 wks )

Alaa H. Albassami MY EXAM : 10TH OF MARCH 2012:

OPTHALMOLOGY 3/3 : 1- PICTURE OF AN EYE : NO HX OT DISCHARGE ONLY TEARY EYE AND REDNESS ETC : DX WAS VIRAL CONJUNCTIVITIS. 2- HTN LADY WITH HIGH LEVEL OF PB HEADACHE , OPTHALMOLOGIC EXAMINATION SHOWED CUPPING AND EXTRA FININGS WHICH I DONT RECALL IT : MOST APROPRIAT MANAGEMENT WAS : URGENT REFERRAL TO OPHTHALMOLOGIST. 3- DRY EYE , PRESCRIPE THE DOSE OF THE LUPRICANT : 1 DROP IN THE LOWER ..ETC.
| P a g e

Raneem Alshareef
..
-Treatment of leishmaniasis -- antimonial (SURE -Most important physiological process of hypoxia ventilation/perfusion mismatch (SURE( -patient post-MI 5 weeks,c/o chest pain,fever,and arthralagia: a- dressler's syndrome b- meigs syndrome > not sure about the spelling c- costochondritis d-MI e- PE -patient with chest pain,x-ray revealed pleural effusion,high protein & high hdl: a-TB (SURE( b-CHF c-hypothyroidism d-hypoprotienemia -Treatment of bacteroides fragilis :clindamycin (SURE(
| P a g e

-Initial Tx of psoriasis with 15% body involvement : Topical steroid (SURE( -Asthmatic pt with scales on face and forhead & antecubital fossa Atopic dermatitis (SURE( -Ricotan tx for ace can cause :birth defect (SURE( -Main symptom of AOM is :pain (SURE( -Nasal obstruction in one nostril ttt: Steroid (SURE( -Most common cause of epistaxis in children:self induced trauma (SURE( -Most common cause of postmenopausal benign bleeding: a- cervical polyp b- cervical something c- vaginitis d- endometrial hyperplasia -Drug used for mastalgia: a-OCP(SURE( b-BENZODIAZEPINE c-beta blocker d-caffiene -child with baking cough,stridor,and mild fever 38 Dx: croup (SURE( -child with cough,runny nose and fever,O/E:tonsillitis ttt: paracetamol and throat swab (SURE( -child with blab la bla x-ray(steeple sign ): croup (SURE( -Mitral stenosis murmur : diastolic low pitched,rumbling ( SURE( -child with URTI then arthralagia and fever Dx: rheumatic fever(SURE( -OCP can cause changes in : cervical mucosa (SURE( -pt with 18 months amenorrhea,high FSH,divorced: a-pregnancy b-premature ovarian failure c- hypothalamic amenorrhea
| P a g e

d- pituitary microadenoma -pt with chronic heartburn,treated with antacids,no improvement,wt next action: a- another antacids b- h2 blockers c-PPIs d- prokinetic agents

-drug used in systolic dysfunction heart failure: nifidepinedeltiazm -ACEI -B-blocker -Congenital heart disease with greatest risk of endocarditis: TOF (SURE( -Obese child with BMI=30,blab la bla lifestyle change (SURE( -Young drug abuser sould screen him for HIV,hepB,staph >DUNNO THE ANSWER WISH U ALL THE BEST,PRAY FOR ME PLZZZZZZZ

Manal M AL-anazi Dammam university 12-3-2012


-which one non-pharmacological is the most appropriate in hypertension 1-whight loss((my answer)) 2-low-deit salt 3- decrease alcohol 4- stop smoking
| P a g e

-Old patient after taking bath start to develop pruritus and weakness lab showing polycythemia , the mechainsim of action 1-increase hitamine sensitive 2-relaese abnormal histamine ((my answer )) -Youg pt present with exceesive fulid intake +polyuria , lab result showing Fasting blood suger 6.8 mmol/l what is the diagnosis 1-DM 2-DI 3-impaired fasting blood suger ((my answer ))

-Different between DMII AND DM type I 1-acute onset 2-herditary factor 3-DR4 , DR4

-In which medical condition Methylergometrine (methergine) Which use in postpartum hemorrhage is contraindication 1-maternal asthma 2-maternal hypertension -Female 47 yrs old complian of bleeding discharge from the breast 1-ductal ectisa 2-papilloma 3-fibroadenoma

-Symptom of reflux esophagitis


| P a g e

1-minor the risk of MI 2-not effected by alkali 3-increase by standing 4-can be distinguish between it and duodenal ulcer -6 yrs old child came to you he only had his BCG vaccine, HbsAg +ve (mother also +ve) wt to give: -DTP,OPV,HiB,HepB,MMR -DTP,OPV,HiB,MMR -male singer with colon cancer stage B2 ; which of the following correct ? a- no lymph node metastases b-one lymph node metastasis c-2-4 lymph node -pt with did colectomy after colon cancer , now lymph node showing micro????((mean met to lymph )) 1-it is sensitive to chemotherapy -In cystic fibrosis the genetic defect in 1-short arm of human chromosome 7. 2-long arm of human chromosome 7.((my answer )) 3-short arm of human chromosome 17 -Best view to see the rib fracture 1-posterior-anterior x-ray (my answer )) 2- anterior-posterior x-ray

-wound at end inflammatory phase which of the following correct : Epithelial tissue formation -angiogenesis Wound sterile((my answer )) eschar formation

| P a g e

-most common risk factor in intra-crinal hemmorge is 1-truma 2-vascular hypertension((my answer )) 3- rupture aneurysm -An elderly lady presented with chronic knee pain bilaterally that increases with activity & decreases with rest. The most likely diagnosis is: a) Osteoarthritis ( true ) b) Rheumatoid arthritis c) Septic arthritis -A 34 year old lady presented with pelvic pain and menorrhagia. There is history of infertility. On examinations the uterus was of normal size & retroverted. She had multiple small tender nodules palpable in the uterosacral ligament. The most likely diagnsosis is: a) endomytritis b) Endometriosis ( true ) c) Adenomyosis d) PID -Patient after accident, there was a part on his left chest moving inward during inspiration and outward during expiration Dx -1 Pneumothorax 2- Rib fx 3- Flail chest ( my answer ) 4- Rib dislocation -Most common cause of meningitis 1-N-meningitis -pt taking bupropion to quit smoking what is SE can happen a. Arrythmia b. Seizure ((my answer buz word can happen )) c. xerostomia d. Headache????

- pregnant with insulin dependant with good control, so to decrease risk of congenital disease
| P a g e

a-good metabolic control before pregnancy b-"""""""""""""""""""""""1st trimester c-""""""""""""""""""""""""2nd """"" d-""""""""""""""""""""""""3rd """"""" - Infant born with hemangioma on the right eyelid what is appropriate time to operate to prevent amylopia: a. 1 day b. 1 week c. 3 months d. 9 months

-male old patient has S&S of facial palsy ( LMNL) ; which of the following correct about it ; A- almost most of the cases start to improve in 2ed weeks b- it need ttt by antibiotic and anti viral c- contraindicated to give corticosteroid d- usually about 25 % of the cases has permanent affection -in cachectic patient, the body utilize the proteins of the muscles >> to provide Amino acid and protein synthesis

-5 y.o child with h.o fever and swelling of the face ant to the both ears (parotid gland enlargement) what is the diagnosis 1-mumps 2-parotid tumor -girl with band like headache increase with stress and periorbital , twice / week >> tension headache migrin cluster -Patient is known case of cervical spondylolysis , presented by parasthesis of the little finger , with atrophy of the hypothenar muscles , EMG showed cubital tunnel compression of the ulnar nerve , what is your action now?
| P a g e

Ulnar nerve decompression Steroid injection CT scan of the spine -Pt has Hx of URTI , came complain from vertigo Most likely diagnosis is 1-austic neuroma 2- meniere's disease 3- vestibular neuritis ((my answer )) 4- Benign positional vertigo -80 yr old in his normal state of health presented with decrease visual acuity bilaterally without any defect in visual field his VA Rt eye= 20/100 VA Lt eye=20/160 fundoscopic exam showed early signs of cataract and drusen with irregular pigmentations. No macular edema or neovasculirization. The appropriate action beside antioxidants and Zn is: a. Refer the pt for emergency laser therapy b. Refere the pt for cataract surgery c. See the patient next month -Primigravida with whitish discharge the microscopic finding showed pseudohyphae the treatment is: a. Meconazole cream applied locally b. Tetracycline c. Metronidazole d. Cephtriaxone -you would like to do: a. Renal function test b. Urine microscopic sedimentation c. Renal ultrasound -Pt with sudden cardiac arrest the ECG showed no electrical activities with oscillation of QRS with different shapes. The underlying process is: a. Atrial dysfunction b. Ventricular dysfunction c. Toxic ingestion d. Metabolic cause
| P a g e

10 year old had an episode of rheumatic fever without any defect to the heart. The patient need to take the antibiotic prophylaxis for how long: a. 5 months b. 6 years c. 15 years Rebound phenomena It cause by over use of vasoconstriction medication

-Pt with adult respiratory distress syndrome.. he got tension pneomothorax.. what is the propable cause: a-severe lung injury b-Negative pressure c-central venous line

-The heart increase its blood supply by - Pulmonary resistnace - Dilate coronary artery - Constrict aortic artery -HIV pt. with skin lesion show (spindle cells) Diagnosis: - Kaposi sarcoma -Young patient with pharyngitis, inflammation of oral mucosa and lips that has whitish cover and erythmatous base, febrile, splenomegaly. Dx: a. Scarlet fever b. EBV (my answer) c. HZV

-if the likelihood ratio .3 what does that mean ? 1- No change 2- Small increase 3- Large decrease 4- Moderate decrease ((my answer)
| P a g e

No choose of small decrease

- study done on 10,000 people for about 3 years in the beginning of the study 3,000 developed the disease and 1,000 on the end of the study what is the incidence: a. 10.3% b. 12.5% c. 20%

-Pap smear :: One collection from os of cervix 3 collection from the endocervical canal One collection from vagina -Patient with pain in the anatomical snuffbox, he most likely has: a-Boxer's fracture b-Colle's fracture c-Scaphoid fracture((my answer)) - newborn with fracture mid clavicle what is true: a. Most cases cause serious complication b. Arm sling or figure 8 sling used c. Most patient heal without complications

-Wha's true about Malaria : the most common cases is caused by Plasmodium falciparum

patient having chest pain radiating to the back, decrease blood pressure in left arm and absent left femoral pulse with left sided pleural effusion on CXR, left ventricular hypertrophy on ECG, most proper investigation to dx:
| P a g e

1-aortic angiogram((my answer)) 2-amylase level 3-cbc 4-echo

13/3/2012 Hind sindi , Jeddah , SLE The question not 100% exactly as the exam and not all choices written.. I wrote what I remember
1-True regarding perths disease Affect girl > boy Common age 11-16 Always unilateral Painless 2- This picture ,,, fluoresce a coral red colorin wood lamp << what is the diagnosis: Erythrasma. Candida Psoriasis 3- mother breast fed her baby each 3 hours,, she id taking phenobarb for seizure.. what should she do regarding it: Stop medication immediately Feed baby 8 hours after medication intake Baby weaning within 3 weeks Continue medication and breast feeding 4-case o painless late trimester vaginal bleeding > placenta previa 5-defention of delusion:
| P a g e

6-abdominal pain, nausea, vomiting, low grade fever, increase neutophil, after appendectomy appendix will show: Neutophill in the muscular layer check the video minute 4 http://www.youtube.com/watch?v=RcJL-xnm3tM Lymphoid hyperplasia with giant cell infiltration Dilated lumen filled with mucocele 7-left Iliac fossa pain, rebound tendresness, nausea , vomiting, fever 38.2 diagnosis is : Diverticulitis 8-abdominal pain, distention, vomting, cant pass flatuse, medically free, hx of appendectomy 7 months ago diagnosis: Cholycystits Mechanical intestinal obstruction 9-hx of long standing abdominal pain improve with peptic ulcer medication,, present with abdominal pain,distention, forcfull vomiting, emesis contain morning food .. diagnosis: Gastroparesis Gastric outlet obstruction " not sure" Dialted cardia Esophageal reflux 10-6 month old came with sign and symptom or RD " fever, tachyepnia, intercostals ressesion, expiratory wheez, nazal flare".. best intial management : Oxygen Erythromycin Bronchodilator 11-young pt admitted because of URTI and BP 120/90 7 days after she develop acute abdomen , tenderness on examination , pt become pale ,sweaty, BP 90/60 what will you do : Anterior abdomen CT
| P a g e

IV fluid and observation << I dont remember if there was antibiotic Gastroscope A double-contrast barium 12- 6 year old boy received only BCG at birth ,, his mother and he is HeB +ve what should he receive: DPT,HBV,Hib, OPV,MMR dT,HBV, MMR, OPV dT, HBV, MMR, OPV, Hib DPT, MMR, OPV, Hib 13-fall on left elbow, fracture on x-ray seen as: 14- youn boy run for long distance "3 killometrr I think" pt complain of persist pain on examination there is knee swelling, x-ray of knee releveal nothing .. what the diagnosis: Ligament tear Tebial fracture I dont remember the rest maybe the correct answer is not one of these two 15- after CPR for a childe with a systole give him: Atropine Epinephren 16-cause of death in flame burn: Airway affection Hypovolemic shock 17- which role used to calculate burn surface area in case of burn: Nine Seven 18- girl with hypokalemia, weight loss, erosion of tooth enamel: Bulimia nervosa
| P a g e

Anorexia nervosa 20-young lady with oligomenorrhea, acne, increase hair, 60 kg her weight diagnosis: Hypothyroididm Polycystic ovary disease 21-smoking directly related to which cancer: Colon Liver 22-old man with rectal bleeding and picture of anemia,,, most commen cause of this anemia: External hemorrhoid Colon cance 23-kid with dark urine, dark brown stool, positive occult test.. what to do: Isotope scan Abdomen US XRay 24-old pt with neck pain on eating, examination reveal submandiblar mass how to investigation: MRI, XRay,CT 25-how to prevent lyme disease: Protective clothes 26- case of meningitis "neckstifness, phtophbia" how to treat contact: 27- common type of non traumatic fracture in osteoporosis: Compressed vertebral fracture 28-common cause of intracranial hemorrhage: Hypertension
| P a g e

29- DM1 HLA linked disease associated with which DR: ,4,5,6,7. 30-pregnant young lady with high TBG, high Total T4, Normal TSH, normal Free T4 .. what is the cause. 31-pt with hx of URTI now having post glumeriolonephritis symptom most diagnostic test : Low Complement 32-diabtic pt for 20 years,, eye examination reveal vetriouse hemorrhage, neovasculrizaton.. How to manage: Strict diet referral to ophthalmologist name of medication 33-vasomotot rhinitis: Decongestion Antihistamine 34- pt came to ER decreased level of consciousness and pinpoint pupil >> opiate over dose 35-likhood ration of disease is 0.3 mean what? 36- nerve injury in deep laceration of anterior aspect of wrist lead to: Claw hand Wrist drop 37- gullin barre syndrome: ascending coarse 40-berberi = B1 deficiency 41- Case scenario of pellagra (diarrhea, dermatitis, and dementia, glossitis) 42- Old pt with bilateral knee swelling, pain, normal ESR: Gout Ostioarthrirts RA
| P a g e

43- lady 4 month ago did CS ,, medically free, complain of wrist pain, phalen test ve, Finkelsteins test positive,, tenderness distal to radial styloid>> I think it's a case of DeQuervain's tenosynovitis: Volar splint Entrapment release"sugery" Thumb splint Drug I dont remember the name 45-clavical fracture in infant: Usually heal without complication Usually associated with nerve injury Need figure of 8 Need internal fixation 46- tobacco withdraw symptom peake 2-4 5-7 day 47-young patient complain of watery diarrhea, abdominal pain.. with a previous hx of mucus diarrhea. Symptom improve when sleep Chrons UC IBS 48- ttt of erectile dysfunction because of stress: sildenafil, avoid stress 49-Scoliosis: if 20 degree refer to ortho 50-stap wound in triceps.. days late green wound discharge.. on wound examination no resistance ,, what is the organism: streptococcal gangrene fournier ,
| P a g e

Clostredia gangreen 51- lower limb edema, congested neck vein.> rt heart failure 52-case of lion pain, Dysuria ,obstructive urinary symptome, fever, on PR boggy prostate and tender on palpation: Prostatitis Cystitis 53-youn bou woke up with ear pain, symptom of facial palsy.. true regarding it: Healing usually occure in the 2nd week Need antiviral More than 25% will not heal 54- IDDM case sinario of DKA.. what is the pathophysiology: Missing insulin lead to release of Free fatty acide and form keton body 55-tow cases of ectopic 56- infant born with eyelid hemangioma what appropriate time for trament: 1 day 1 week

Yasser Alashhab My SLE 21/3/2012 good luck for all ..


1-Best exercise to increase the muscle strength and bone density in aging process: A- Low resistance, high repetition muscle training B- Conditioning, low repetition muscle training C- Walking and endurance muscle training D- Low resistance and conditioning muscle training 2-What is the initial management for a middle age patient newly diagnosed knee osteoarthritis. a. Intra-articular corticosteroid. b. Reduce weight.
| P a g e

c. Exercise. d. Strengthening of quadriceps muscle. 3-which of the following cause hirsutism 1/ anirexia 2/ hypothyrodism 3/ clomiphin citrate 4/ OCP 6 mnth pregnant lady came complaining of sever abd pain on exmenation there is utraim fibroid and life fetuse what to do? A-Give drug for pain B-Myoctomy C-Termenat pregnancy 4- pregnant never did check up before , her baby born with hepatosplenomegaly and jaundice : a-Rubella b-CMV c-HSV d-HIV 5-Pt presented to the ER after RTA. He was concsious , vitally stable. There was decreased air entery bilaterally & tracheal deviation to the opposite site. What is your next step? a- Order CXR STAT b- Insert a needle in the 2nd ICS MCL c- Insert a needle in the 5th ICS MCL d- Insert a chest tube in the 5th ICS MCL

6- Which breast disease is Bilateral: Lobular carcinoma 7-Old patient with bilateral enlarged knee , no history of trauma , no tenderness , normal ESR and C-reactive proteins . the diagnosis is a. Osteoarthritis b. Gout c. Infectous arthritis 8-pt female with sever hip pain , increase with walking , after busy day , awake her almost all the night , ass with morning stiffness : A-osteoartheritis B-osteoprosis 9-Young patient with red, tender, swollen big left toe 1st metatarsal , tender
| P a g e

swollen foot and tender whole left leg. His tep 38 Diagnosis is: a.Cellulitis b.Vasculitis c.Gout Arthritis 10-Case of gout: A- Ca pyrophosphate B- Na urate 11-Facial nerve when it exits the tempromandibular joint and enter parotid gland it passes: a) Superficial to retromandibular vein and ext. carotid artery b) deep to ex. Carotid c) deep to R vein d) between retrmandibular vein and external carotid artery 12-What's the most common area in women gonorrhea affects ? a- Cevix b- Urethra c- Poterior fornix of vagina 13-Regarding group A strepto how lead to rhumatic fever A-Blood dissemention B-By causing pharngitis/tonslitis C- Joint invasion 14-what is true about puerperium: a.lochia stays red for 4 weeks (color dcrease with time) b.epidural analgesia cause urinary retention c.abdominal uterus is not felt after one week (within 2 wks) 15-unilateral headache, exaggerated by exercise and light , Dx : a.migraine b.cluster headach c.stress headache 16-4years old child what can he do 1.Copy square and triangle 2.Speak in sentences 17- Pt with hematural .. uremia .. and bilateral loin mass dx : a-amyloedosis b-sarcidosis
| P a g e

c-Polycystic kidney d-Metastatic hypernephroma 18-baby can sit without support, walk by holding furniture. Pincer grasp, pull to stand how old is he a.8 months b.10 months c.12 month d.18 month 19-trauma by tennis ball with blood in ant chamber .. u must r/o : a-conjunctivitis a-keratitis c-penetrating FB d-belphritis 20-ttt of dacrocyctitis : a-topical antibiotic b-oral antibiotic c-oral steroid d-oral antiviral 21-month old child brought to you for delayed speech, he can only say "baba, mama" what's your first step in evaluating him? a-Physical examination b-Delevelopmental assessment. c=Head CT d-Hearing test. 22-one non-pharmacological is the most appropriate in hypertension a-whight loss b- decrease alcohol c- stop smoking 23-Female with dysurea, urgency and small amount of urine passed .. she received several courses of AB over the last months but no improvement .. all investigations done urine analysis and culture with cbc are normal .. you should consider: a) interstitial cystitis b) DM c) Cervical erosion d) Candida albicans 24- what proven to reduce incidence of cancer ?
| P a g e

1-fiber 2-vit c 3- vit D 4- ca 5- folic acid 25-pt with DM and obese ,plane to reduce his wt is : a.decrease calori intake in day time b.decrease calori and increase fat c.decrease by 500 kcal/kg per week d.decrease 800 per day 26-assement the progression of labor : a- the force of uterine contraction b- the frequency of uterine contraction c- the descend of the presenting part 27-how do u treat a unilaterla sweeling in nose ?? a-decongestant b-antihistamen c-corticosteroids 28-0 year old farmer with lesion in his hand , elevating mass dome shape and there is keratin DX: A-melanoma B-keratoacanthoma C-BCC D-SCC 29-which of the following take with analgesic to decrease side effect ?? a- cimitidine b-psudoephidrine c-metaclopromide 30-6 years child was born to HBS positive mother is HBS positive , he was only vaccinated by BCG after birth , what you will give him now : a. HBV + oral polio + DTP + hib b. HBV + oral polio + dt + MMR +hib c. oral polio + Dtp + MMR+ hib 31-ttt of foliculitis a- oral steroid b- topical steroid c -oral antibiotics
| P a g e

32-12 yo boy brought by his parent for routine evaluation , his is obese but otherwise healthy , his parents want to measure his cholesterol level , what is the best indicator of measuring this child cholesterol : a. His parent desire b. Family hx of early cva c. High bmi 33-mechanism of OCP Inhibit estrogen spur in mid .. & ovulation

New SLE 2012 ,March2012- Hassan Abdelrhman


1) 37 old patient came to the clinic complaining of chest pain chronicly ,the pain is burning over his chest, & increasing when he is lying in bed associated with cough, WHAT IS THE MOST LIKELY DIAGNOSIS? a) Ischemic pain b) GERD c) Bronchitis 2) Young male,very thin complaining of cough, x ray of the chest shows b.lateral hailar white apperance in the upper part of the lungs, he is living in very crowded area,, what the to give him? a) BCG b) .. c) .. 3) Child sqagueled for elective surgery his wieght is 22 kg,, what is the fluid deficit to give? a) 37ml/h b) 65ml/h c) 90ml/h d) 88ml/h 4) Young female complaining of severe diharrea, weight loss, vomiting, abdomenal pain,has been diagnosed to have crhon diseased, what is etiology mechanism of crhon disease? a) Female more affected b) Something granulomatose c) Diabetic d) Unknown 5) Old patient around 70 years I think, complaining of ulcerating lesion 3*4 cm just below his nustreles(question with picture),the lesion is increasing after he
| P a g e

was retired from work 10 years back, he was in continuse expausre to sun light,DIAGNOSIS? a) Squameus cell carcinoma b) Adenocarinoma c) keratoxanthoma 6) why SSRI medication had showed the best theraputic effect among other antidepressant? a) Because of low price and so available. b) Best theraputic + less side effect & tolerable medication c) 7) Celly question about control study in 2010 for 100 epliptic patients. 8) Question about hip with chart similar.,read about it.

9) Long cenario about women with anexity dissorder (asking about the diagnosis) 10)Young female ,complaining of severe headaches over long period, now she starting to avoid alchol, not to smoking, doing heathy happits, and she notes that she had improved over her last prgnancy,, what you think about her condition? a)biofeedback b) she was on b-blocker c)alchol caseation 11) question about mechanism of open angel glaucoma (read about it)

| P a g e

12) depressed patient has injestion big quantity of Aspirin 6 hours ago,, came to ER complaining of nauesa ,vomiting, increase respiration, investigatin showed highly elevated level of ASA, what is your action? a)urine acidity something b) charcoal c)haemodyalisis 13)picture of infant with brown to black lesion in his abdomen about 4*5 ,,painless, not itchy, not presented at birth, slowly in growing, he is otherwise healthy, the parents are wory,? a)FNA b)reussurance c) biopsy and consult neurologist 14) cenario about postpartum haemorrage,, they telling about management,,and one of these steps is to give strange name of medication, the question about the relative contraindication for maternal site for that drug? a)matrenal asthma b)maternal DM C)maternal HTN d)maternal crhon disese 15) 17 years old patient admitted to the hospital because of iv drug abuse,and investigated, so he is in risk of which of the folowing? a) hepatitis B b) HIV c) cardiac valve prolapsing( confuse) my answer was hep B. 16) patient with complain of calf tender and swellen,,diagnosed to have DVT,,what is the rule of LOW MOLECULAR WIEGH HEPARIN in DVT treatment as comparing to something heparin? a) LMWH is less effictive
| P a g e

b)LMWH is prone to more bleeding c)LMWH is safe and no need to regular monitor the PTT. 17)young lady just joined new job after getting her last pregnancy a cuople of months peviously, in this new job she dont have to get pregnant for 3 years as rule, she came to you telling that I dont want to pregnant,I dont want to use OCP, or IUD, you recommended for her transdermal device,what you should tell her more about this? a)it is more likely to form more clots around the area b) it can be forgetable by time c)its safe to use for long time 18) patient had burned by hotty oil in the right side of his arm and leg,came to you in clinic. So you will refferd him to burn clinician specialist if? a)10 cm painful area with no blusters b)5 cm painful area with blusters c)5 cm paineless area with no blusters (third degree ,full thickness) 19) female patient with painful red eyes(b.lat), blurred of vision for 24 hourse,,behind the otptic disc is intact and one more something, I remember very poor finding was given) Dx? a)neurosyphilis b)DM c)HTN d)multiple sclerosis 20) patient Diabetic ,age 39 ,,has diagnosed to have DM when he was 30, came to your clinic complaining of blurred vision,redness,irritable eye, on fundoscop there is new vassels growing (angiogenisis) Dx? a) background retinopathy b)proliferative retinopathy c)
| P a g e

21) old age complaining of diaharrea, leftsided abdomenal pain, fever,vomiting, on palpation there is tender mass in left iliac fossa,for two days, also given lap data for wbc:elevated+ c reactive protien: mild elevated and ESR, whats the DIAGNOSIS? a)crhon disease b)appendicites c)diverticulitis 22) female complaining of suprapupic abdomenal pain, fever, vaginal discharge, foul smelling, for one week,,she was niggative for gonorrhea,chalymidia, whats is the possible causetive organism? a) something vaginosis,( sorry I dont remember the choices,but you got the picture) 23) child complaining of severe abdominal pain , foul gressy stool, vomiting, constipation on/off, his belly is distended, what is the investigation of choice to confirm your DIAGNOSIS? a)barrium enema b)colonscopy c)barrium meal 24) male patient complaining of abdomenal pain sever, constipation, decrease in bowel motion,he had abdomenal surgery 2 years back,, (they showed abdomenal x-ray with entire enlarged distended colon,hausterations,invoving segmental parts of small bowel), what is the best correction? a) surgical colostomy b)removal of obustructed colonal part c). 25) 2 months infant with severe gastrointritis, vomiting, diaharrea, increase of the skin trigor, depressed antriorfontanel,pale,dry mucos membrane,crying but no tears, what is your management? a) agrrisive oral rehydration therapy
| P a g e

b)iv salaine c)O.R.S solution given to mother to rehydrate the infant d).. 26) male middle age brought to the emergency department after involving in RTA (road traffic accident) ,on arrival GCS 12/15,,mild confuse, tacchycardic 113beat pir m, tachypnic 32 breath pir m, Bp 80/60, with mild traumatic lesion in his chest,,your action? a)thoracotomy b)iv fluid c) CT scan d)ultrosound 27) patient complaining of fever , enlarged parotid gland and weakness, la la la,,,what is the complications? a)encepholitis b)miningitis c) 28) child has sore throat and enlarged tonsils for the past week,,fever,, bodyach,,enlarged spleen..what is the causetive orgnism? a) staph aurse b) a b_strept coccus c) H.influanze ( no EBV in the choices) 29) what is the treatment of mild to severe depression? Given for choices all are antypsychotic just one of them is TCA which was the answer.() 30) 20 years old girl,,complaining of dysuria, suprapupic pain, fever,,flank tender for 6 days,urine analysis reveals epithelia cells,,the aperance of these cells indicate? a) urethral injury(this is the answer)
| P a g e

b) c). 31) 18 years old boy complaining of fever 38c, flank pain, pain during urination(dysuria) for 4 days,urine analysis showed WBC 50 to 60,,,your action ? a) ciprofolxacine 500 for 2 days and to came back to clinic b)pinicilin for two weeks & to be seen in the clinic for reussurance. c)admition to hospital and iv antibiotics d) was celly choice 32)postpartum one,, came to clinic and telling that during pregnancy she was taking iron suppliment, and now she is complain of fatigue, dizziness,, weakness after mild effort,,lap investigation Hb=7,8 MCV=60,,Dx? a)iron deficency anemia b)thalasemia 33) which one of the following below is at risk to commet socide? a) 20 year college boy who had big conflict with his girlfrend b)60 years women who is taking antidepressant and newly diagnosed to have ostioprosis. c) old male I dont remember,he was sick but not that to commet socide. 34)very loooong case about girl multisexual partners, had complained of hottness right knee joint,which resolved by taking ibu-brufen, now is multiple hot and tender joints with fever,,abd pain,weakness, what is the DIAGNOSIS? a)gonorrhea b)septic arthritis c)PID 35) 6m baby with mild viral diarrhea , ttt by ORS as a-100ml/kg for 4 hour then 50 ml/kg /day after b-50>>>>>>>>>>>>>>>>.50>>>>>>>>>> c-100>>>>>>>>>>>>>>>>100>>>>>>>> d-50>>>>>>>>>>>>>>>>>100>>>>>>
| P a g e

36)58 years old male came complaining of shortness of breath,for 3 days ,x-ray of the chest showed cardiomegaly ,plueral effusion,,the analysis of the effusion showed mild protien and moderate HDL, what is the common cause of the effusion? a)TB b)bronchopneumonia c)heart failure 37) one case about Kawasaki,,they gave symptoms and asked for Diagnosis,,,fever ,pericarditis, la la la 38) clear cut case about coarctition of Aorta,,,given symptoms various puls rate & Bp+ substernal notch asking for Dx. 39) patient known asthmatic,the mechanism to prevent reccurent asthma attack is? a) medication to form something to allergin b) to prevent airway inflamation (this is the answer) 40) pregnant women in labor,,suffer from sever pain,dialated cervix,all the manifestation within normal,the type of analgesia? a)epidural b)spinal c)general 41)pregnant lady 16 weeks GA, caomplaining of vaginal bleeding,,la la lain the question mentioned (snowstorm) Dx? a)complete haydatifom mole b)partial mole c)

.... .

| P a g e

Dr-Bayan Ahmad My SLE 2/4/2012 .. Bayan*

1) 35 year old smoker , on examination shown white patch on the tongue, management: (leucoplakia) excision biopsy 2) 8 years BMI = 30 weight and hight above 95 percentile , next step : Life style modification 3) case scenario ptn come to ER c/o colicky abd pain after meal, other in Hx & Ex ve : a. U/S of Abd b. Abd X-ray 4) y.o child with history of fever and swelling of the face ant to the both ears (parotid gland enlargement) what is the most common complication at this age group : meningitis 5) pt taking bupropion to quit smoking what is SE : Seizure

6) old pt c/o bilateral knee pain with mild joint enlargement ESR and CRP normal dx : Osteoarthritis 7) pt with dysphagia to solid and liquid by barium there is non peristalsis dilatation of osophagus and air fluid level and tapering end dx: Achalasia 8) What the MCC of pnempthorax in ARDS ptn : a. central line insertion b. lung damage 9) Pregnant w s/s of hyperthyrodism best treatment : Propylthiourocil. 10) What is true about Propylthiourocil : block thyroid hormones.
| P a g e

11) computer user came with wrist pain, need cast in which position : Dorsiflexion. 12) What is true about Peritonitis : a. chemical irritation can cause it. b. Associated with abdominal rigidity which increase as the Paralytic illeus develops. 13) old pt complaining of back pain on walking on examination there was stiffness of the muscle and there was some finding on the x-ray , best effective ttt is : a. Physical therapy b. bio feedback. 14) Most common cause of immediate death in burn: a. Inhalational injury. b. Septic shock. c. Hypovolemic shock. d. Associated injury. 15) sildenafil , which is contraindication to take : Nitrate. 16) Chronic fatige syndrome : Anti sycatric treatment. 17 + 18) 2 Q about secondary dysmenorrhea & 2nd amenorrhea : I'm confused btw choices & forget sorry, but I think my ans in one of them is Sheehan's syndrome . 19) MCC of bloody vaginal discharge : a. menstruation. b. mischarge. 20) case scenario young female abd & back pain every month, headache & I think fatigue (unspecific symptoms) all s/s started from years w menstrual cycle & progressively worse : premenstrual syndrome. 21) case scenario ptn came w ant. MI + premature ventricular ectopy that indicate pulmonary edema, give digoxin + dirutics + after-load reducer, what add? a. amiodarone.
| P a g e

b. propranolol. 22) case scenario DM : DR-4. 23) case scenario old ptn female came w osteoporotic thoracic #, T & Z score of spine & .. what is classification depend on WHO : a. osteoporosis. b. osteopenia. c. severe osteopenia. d. established osteoporosis. e. I forget. 24) case scenario sorry I forget it : Bulimia nervosa !!! 25) stroke caused by embolic cause : AF. 26) Adult SCA, CNS complication: Cerebral infaction. Ataxia. Seizure. 27) epidemic curve : >_< Bad choices. 28) positive predictive value : 29) case scenario to increase CO, by left atrium pressure which : a. Lt ventricular hypertrophy & chamber constriction. b. Rt ventricular hypertrophy & chamber dilatation. c. Rt ventricular hypertrophy & chamber dilatation. d. Rt ventricular hypertrophy & chamber constriction. 30) case scenario ptn in labor, baby in late deceleration, what u will do in this case : a. change position & give O2. b. give Mg sulfate. 31) Marsmus : Retarded growth & reduced weight. 32) MCC of non traumatic SAH :
| P a g e

a. anuresm. b. bridge vein. c. MMA. 33) Rt side submandibular swelling & pain associated w eating, induration in floor of mouth : a. CT. b. MRI chest. c. chest X-ray. d. ECG. 34) case scenario gastric ulcer, pathology -ve neoplasm, or H.pylori : a. table show MCV, iron in serum, decrease & binding iron capacity increase : a. IDA. b. Pernicious anemia. 35) case scenario infantile beriberi : a. Vit B1. b. Vit C. c. Vit E. d. Niacin. 36) case scenario pregnant, exposed to truma, gush of blood from the vagina what is the Dx : a. Abrupto placenta. b. placenta brevia. c. utrine contusion. 37) case scenario pregnant in 9th month, c/o small amount of brown dark blood w no abd pain : a. Abrupto placenta. b. placenta brevia. 38) case scenario RUQ abd pain, N/V, bilirubin, Alp, & WBC high : a. Acute cholecystitis. b. chronic cholecystitis. c. appendicitis. 39) case scenario LLQ abd pain, x-ray show sigmoid thickening, pericoloic fat decrease what ttt : a. Abx. b. 40) case scenario hepatomegaly, KayserFleischer rings what ttt : penicillamine.
| P a g e

41) case scenario hepatosplenomegaly, bluish skin nodule, lat neck swelling Ix to make Dx : a. EBV serology. b. CXR. c. CT. 42) case scenario meningococcal infection ??!! 43) case scenario pharyngitis group a streptococcus ttt : Don't start ttt before serology result w streptococcus. 44) case scenario unilateral knee & leg swelling w creptius in knee : doppler ultrasound

Aida Badry From my SLE exam 2/4/2012


1) Long scenario abuot pt having epigastric pain radiate to the back increase with lying and decrease when standind ass fever tachycardia. It is typical with acute pancreatitis .. what is the next diagnostic step: a-abdominal CT b- abdominal Xray c-ERCP d-seum amilase and lipase. 2) pt with episodes of pain started in the mid left abdomen radiate to the back no nausea vomiting or diarrhoea not releaved by antacid not related to mael on Ex: non remarkable.dx: a-chronic pancreatitis b-duodenal ulcer c-gastric ulcer d-mesentric thrombosis 3) pt has non complicated varicose vein which is not changed sience its occurance in her last pregnancy she wear stocks and elevate her legs she asked about furthur cosmotic option you will tald her: a-nothing can be done more. b-stripping will make it worse. c-coagulation therapy d-somethings about saphenous ligation.??
| P a g e

4) Pt with nephrotic syndrom on ACEi taking rich protien food what do you suspect the result: a- Increase serum albumin b- Decrease serum albumin c- Increase triglyceride d- Decrease triglyceride. 5) Pt undergone sunburn causing erythema and burning pain on wide areas of his body he is hypertensive and on hydrothiazide despite your manegment you will: a- Stop hydrochlorlthiazie and follow the blood pressure. b- Sorry I forget the remaining cause I select (a). 6) Pt walking fot relativly long time on ice whene she was in vacation (somewhere in cold area) her feet is pale with marked decrease in pain sensation but the pulse is palpable over dorsalis pedis what is the appropriate thing to do: a-immedate heat with warm air b-put her feet in worm water. c- I forget the rest but it is not apropriate 7) Picture showing tinea corporis (he give hx that a 15 year old child develop this lesion in mid leg no other symptoms hes hx of short trip. 8) Q about pt has irrigular cycle and low estrogen level he ask how can low estrogen cause endometrial proliferation and save the bone density??? a- Amenorrhea and osteoprosis b- Galactorrhea and osteoprosis (I forget the rest sorry) read about 9) Female about 50 doing regular exercise and in good heath sceening show mild degree of osteoporosis and her mother fall and get fracture of wrist what will you advise her: a-wear safety devise and training exercise b-ca ,vit D and biphosphate c- 10) In vellage total population 2500 Number of birth . Number of stilbirth Number of death . What is the crude death rate?
| P a g e

11) Pt presented with desmenorrhea, dysparunia .. pelvic Ex reveal retroverted uterus with mild tenderness dx a-endometriosis b-endometrial lieomyoma c-.

/
1. a man travelled to some country , there is endemic of onchocerciasis ,he stays there for 1 wk .his ability to get this disease is A -HIGH B-SEVERE C-MINIMUM D-NON EXISTANT ***

2. SNELLEN CHART ,,, there is a chart ,, old man cums with decrease in vision ,dr check his vision by snellen chart he is able to read upto 3rd line ,, so his vision is A-20/70*** B20/100 C-20/50 D-20/40

3. Child come with complain of "barking" cough, stridor, hoarseness, and difficult breathing which usually worsens at night.The stridor is worsened by agitation or crying ..what is the diagnosis ,,, A-epiglottitis B- airway foreign body C-subglottic stenosis Dangioedema E- laryngeotracheobronchitis ****

4. mother having ANENCEPHALY in her first baby ,, the chance to have same condition in 2nd baby is ,,,,, A- 2%*** B-10% C-25% D- 50%

5. pt have vericose vein in her last pregnancy which is not changed , she wear stocks and elevate her legs she asked about furthur cosmetic options you will told her Anothing can be done more B-stripping will make it worse C-coagulation therapy Dsaphenous vein laser treatment ***

6. a young pt comes with complaint of painful night sleep and back pain , on investigation there is spinal disc herniation ,, the treatment is ,,, A- surgery*** B-epidural steroid injection C- spinal annalyzing D-Spinal manipulation

| P a g e

7. child having pain in the night esp calf muscles ,pain is very severe in the night that child is not able to sleep ,, it is also associated with tingling and burning sensation , in the day time he is alright ,,, most probable diagnosis is A-idiopathic leg syndrome Bcompartment syndrome C-restless leg syndrome*** D-functional disease

8. cause of bleeding after D& C is A-asherman syndrome B-missed disease C-Perforated uterus*** D-infection

9. picture of an old man having red eye of left side , between the two eyes above the nose there is small papular lesions ,for which he is using acyclovir cream , it is characterized by a prodrome of fever, malaise, nausea, vomiting, and severe pain and skin lesions between eyes...treatment is A-topical antibiotic B-topical antihistamine C-topical steroids *** D-topical congestants

10. old pt having HTN for long time and taking beta blockers and hydrochlothiazide ,,, now pt complain of sunburn causing erythmia and burning which pass on wide areas of body I FORGET THE OPTIONS ,,,????

11. PIC of child having ulcer near angle of the mouth,, bright red in colour , 1.5 cm size Afungal infection B-impetigo C-atopic dermatitis*** D-Angular chelitis

12. pt is hypersensitive having all allergic sypmtoms like sneezing ,flu congestion and sensitive to sunlight , cause is hypersensitive to A-stress and sunlight*** B-pollen and dust C- cold D-infection

13. young male pt , norma physic , BP 120/80 mmhg , RR 18 /min ,HEART RATE 210 ,no chest pain ,no discomfart ,no cynosis , having cmplaint of palpitation ,,, ur next step is A-prolong PR interval B-holter*** C- Vasodilator

14. pt complaint of polydypsea and polyurea with increase osmolarity ,,, FBS is 105 ,, the cause is A-ADH related*** B- due to hyperglycemia C-pituitary tumor Dphenylketonuria

| P a g e

15. young girl athlet ,, she is alright before , now complaint of acne ,long hairs on the face etc A-Adrenal & ovarian abnormality*** B-Pituitary adenoma C-heridetary disorder Dchromosomal abnormality

16. massive spleenomegaly other wise all blood readings are normal ,, the cause is A-IDA B-THALLASEMIA*** C- LEUKEMIA D-??

17. pt having leukemia ,, there is a long chart with multiple values ,, acc to that WBC increase ,RBC decrease and thrombocytopenia, circulating leukemic blasts,positive myeloperoxidase ...the diagnosis is A-MYELOGENOUS LEUKEMIA*** B-MYELOBLAST LEUKEMIA C-LYMPHOCYTIC LEUKEMIA D- LYMPHOBLASTIC LEUKEMIA

18. pt come in emergency with complaint of HCL burn on her ,, the skin of the pt is burnt ,, now emergency treatment is A-NAHCO3 B- DEBRIDGEMENT C- WATER IRRIGATION*** D-???

19. pt come in emergency with frozen foot , FIRST AID treatment is A-HEAT AND WARM AIR B-IN WARM WATER*** C-GIVE COFFE AND TEA D- RUBBING THE FOOT

20. male young pt , having mass in the scrotum which increase in size ,,, painless , no lucency with light ,, how to manage A- refer to surgery B-refer to urology*** C-Refer to radiology D-send him home

21. young male pt having only complaint of gross hematuria otherwise normal , on examination normal , on investigation US normal ,urine culture normal ,, now whats ur investigation of choice A-RENAL BIOPSY B-URINE ANALYSIS C-cystoscopy*** D-RENAL ANGIOGRAPHY

22. Young male pt having pain in the abdomen,, pain is veryyy verry severe that pt is in fetal position and not able to straight having sign and symptoms of peritonitis ,, now first step to investigate is A-US B-CBC WITH DIFFERENTIALS C-X RAY*** D-parasentisis
| P a g e

23. 6months postpartum having hallucination ,dellusion ,disorganized thinking and speech ,having social and emotional difficulty , having history of child death 3 months ,,, all of the following should be the possibility except A- SCHIZOPHRENIA B-SHIZOPHRENIFORM DISORDER C-BRIEF PSYCHOTIC DISORDER *** D-SCHIZOEFFECTIVE DISORDER

24. PT having elevated mood state characterized by inappropriate elation, increased irritability, severe insomnia, increased speed and volume of speech, disconnected and racing thoughts, increased sexual desire, markedly increased energy and activity level, poor judgment, and inappropriate social behavior ,,, associated with above pt should have one more symptom to fit on a diagnosis A- HALLUCINATION B- DELLUCION C-GRANDIOSITY*** D-DELLIRIUM

25. pt having glaucoma and taking treatment for it presents with shortness of breath ,, which of the drug is he taking A-pilocarpine*** B-TIMOLOL C-BETAXOL DACETAZOLAMIDE

26. pt having otitis media ,sinusitis, laryngitis and bronchitis and septic arthritis ,,, organism is gram negative diplococci A- Moraxella catarrhalis*** B- Neisseria gonorrhoeae CNeisseria meningitidis D- strept pneumonia

27. 12 yr old boy presents with headache and neck stiffness associated with fever, confusion or altered consciousness, vomiting, and an inability to tolerate light.. other than this there are rapidly spreading petechial rash. The rash consists of numerous small, irregular purple or red spots on the trunk, lower extremities .. treatment is A-PENICILLINE*** B-AMPICILLINE C-VANCOMYCIN D-AMINOGLYCOSIDE

28. 15 YR boy comes Blood underneath the nail of his finger and having lines in front of her teeth ,,,there is intense pain in his finger ,,,Pressure generated between the nail and the nailbed, where the blood collects ,,, management is A- apply ice directly to skin B-refer to surgery C-lidocaine is injected at the bottom of finger and evacuation*** Dpainkillers

| P a g e

29. old pt have swollen knees and patella ballotment and fluid +ve ,,, what is the next step A-MRI B-X RAY*** C-INCISION AND DRAINAGE D-???

30. 35 YR old lady comes with complaint of swelling in the neck , swelling become firm, large, and lobulated ,,,pt complaints of psychosis, weight gain, depression, sensitivity to heat and cold, fatigue, bradycardia, constipation, migraines, muscle weakness, cramps and hair loss...during investigations TSH INCREASE & T4 DECREASE ,,diagnosis is A-Addison disease B-Hashimoto thyroiditis*** C-Idiopathic hypoparathyroidism DHypopituitarism

31. old pt , smoker,COPD , having cough and shortness of breath in day time not at night ,, how to treat him A-THEOPHILINE B-IPRATROPIUM*** C- LONG ACTING CORTICOSTEROIDS D- ???

32. Pt having major depression and taking medicine for it ,, after taking medicine she is complaining of insomnia and irretable ,which med she is taking A-SSRI*** B-TCA CMAO D-ECT

-A malnourished child with pedal edema and distended abdomen, an enlarged liver with fatty infiltrates, thinning hair, loss of teeth, skin depigmentation anddermatitis.eyes are also very dry with wrinkled cornea and in anterior chamber there are cells ,, diagnosis is A-Marasmus B-kwashirkor*** C-cachexia D-water intoxication

33. pt complaints of abdominal pain and joint pains ,,The abdominal pain is colicky in character, and accompanied by nausea, vomiting and diarrhea. There is blood and mucus in the stools. The pain in joints involved in the ankles and knees, ,, ,, on examination there is purpura appear on the legs and buttocks ,,, A-Meningococcal Infections B-Rocky Mountain Spotted Fever C-Systemic Lupus Erythematosus DHenoch sconlein purpura***

34. pt having infection with flavi virus ,, prevention from the disease to contacts is A-isolate the pt B-separate his cloths C- if vaccinated then contact will never get the disease Ddo nothing ***

| P a g e

35. long scenerio ,, bone mineral density ,having T score - 3.5 ,, so diagnosis is AOSTEOPENIA B-OSTEOPOROSIS*** C-NORMAL D-RICKETS DISEASE

36. Menopause women with decrease estrogen cause A-HOT FLUSHES B-OSTEOPAROSIS*** C-ATHEROSCLEROSIS D-INCREASE LIBIDO

37. A boy came with parents for cholesterol level evaluation ,,indication is A-family history of cardiac disease*** B-high BMI 33 C-fatty diet

38. child having scabies ... telling the possibilities to mother in infecting the other children in the house ,it transmit through A-personal contact*** B-Blood C-air contaminated Dwater

39. pt taking antidepressant drugs works in an office ,, next day when he came ,he told you that he have planned a sucide plan ,, ur action is A-counceling B-admit to hospital*** C-call to police D-take it as a joke

40. ABOUT DPT a senerio ,, A- DPT is not contraindicated during pregnancy*** B-DPT is not contraindicated during breast feeding c- DPT is not contraindicated in school going D?? 41. 50 yr male PT smoker having ulcer on lateral side of the tongue ,1.5 cm adhere with the skin ,,,, diagnosis is A-Dysplasia of cells*** B-lichen planus C-oral thrush Dseconadary syphilis

42. 20 yr old girl with decrease BMI =16 ,, history of anorexia nervosa comes in clinic with complaint of multiple fractures ,, her bones are so fragile that they oftenly break ,, AOSTEOPAROSIS B-HYPOVITAMINOSIS OSTEOPENIA*** C-OSTEOGENESIS IMPERFACTA DOSTEOMALACIA

43. Child having vomitting ,nystagmus and difficulty in walking ... cause is A-dry beriberi*** B-wet Beriberi C-plegra D-VIT A DEF

44. Female want to know about her height ,, you told her that her height will stop after A-24 MONTHS B-36 MONTHS* C-48 MONTHS D-72 MONTHS
| P a g e

45. Old pt had hemi colectomy after colorectal carcinoma ,,, you advice him to have colonoscopy every A-6 MONTHS*** B- 12 MONTHS C- 2 YRS D- 5 YRS /

46. 48 YR old pt having hysterectomy , after which she complaints of unwanted urine leakage and incomplete emptying of the bladder ,, there is urination with coughs, sneezes, laughs, or moves in any way that puts pressure on the bladder,,,treatment is A-KEGEL EXERCISE B-SURGERY*** C-REASSURANCE D-???

47. Post partum women complaint of passage of flatus and stool through the vagina, diagnosis is A-perineal tear B- rectovaginal fistula*** C- vaginal cancer D-??

48. pt having HIV want to take TB DRUGS ,, A- antibiotics containing rifampin, isoniazid, pyrazinamide and ethambutol for the first two months and just rifampin and isoniazid for the last four months B- treatment with at least four effective antibiotics for 18-24 month is recommended Crifampin, isoniazid, pyrazinamide and ethambutol for 1 yr*** D-no treatment only Surgery on the lungs may be indicated 49. Pt with hypopigmented macules loss of sensation.thickened nerves diagnosis was leprosy which type A-Tuberculoid *** B-Lepromatous C- Borderline D-??

50. HIV PT having negative pap smear , follow up A-first 3 months than 6 months*** Bannualy C-every 3 months D- every month

51. In irritable bowel S. the following mechanism is contraction and slow wave myoelectricity seen in A.Constipation*** B- Diarrhea C-Obstruction D-gases

52. indirect inguinal hernia ,what is the treatment A-elective surgery*** B-emergency surgery C-Reassurance D-No need for any surgery

53. child take an unknown medicine and presents in emergency with decreased level of consciousness ,pinpoint pupil , urination, diarrhea, diaphoresis, lacrimation, excitation, and salivation .... treatment is A-gastric lavage B-activated charcoal C-atropine*** Dnalaxone

| P a g e

54. pt get septicemia from the venous line , the source is A-from the skin*** B-from hand hygiene C-from the IV line D-from the hospital

//
**MMR vaccine safe for breast feeding not harm infant **End stage of COPD ( : ERYTHROCYTOSIS( HIGH Ca ( low K MY answer : 1 ** **RBBB : LONG S wave in lead 1 and V6 & LONG R in VI **FLU vaccineegg **drug for acan is retinoic acid **Rx chronic pain.acetaminophen **month face skin lesion improve with hydrocortisol cream.eczyma **cholesteatomawhite pathes at tympanic membrane defect with bad smell **confirmative Dx of rotavirus,,,,,Ag of virus in stool. **picture of impetigo in face **DRUG RIGEMEN for E. histol .Metronidazole 800 mg 8 hourly fror 5days **hIGH BP, AZOTEM, GFR 44 <<<< RENAL ARTERY STENOSIS **CUSHING DISEASE <<<< WAS VERY CLEAR " **AWARE " method in Rx of anxiety dis " .A " REPRESENT **BMI 30 ,, High Na intake which one play more role to cause essential hypertension.. I select High Na intake **drug that not indicated in maintenance Mx of opoid dependence. **common site of ectopic pregnancy <<<<<<F. TUBE **Seasonal affective disorder .. read about which people more prone to develop. **Common tests used to diagnose an aortic dissection include a CT scan of the chest with iodinated contrast material **TO DECREASE UTI ) <<<<decrease PH & DECREASE UREA &INCREASE OSMOLARITY**(

Mahmoud Hijazi Praise be to God, in yesterday's I pass the SLE exam in makkah 10/6/2012 the best luck for all this some Qs
| P a g e

1-Pt . heavy somking for 30 yrs complaining of dysphagia endoscope done show picture (protrusion lesion) : Sequamus cell carcinoma ( my answer ) Polyp other selection I forget that 2 -3-4 3 questions of derma about herpes zoster all is clear 2 Q about Dx one for face and another for back and the 3rd q for treatment _____ acyclovir my ans other selection 2 antiboitic and steroid 5- which of following drugs not use in WHO treatment of leprosy : dapsone , clofazimine , rifampicin , holperidol( my answer) 6- pt with asthma use short acting beta agonist and systemic corticosteroid < classification of treatment : Mild intermittent Mild persistent Moderate " Sever " (my answer) 7- question about asthma response to allergy and give 4 graph A,B,C,D for allergic phase ..

8- pt with HTN and use medication for that , come complain of pain and swelling of big toe (MTJ) on light of recent complain which of following drug must be change : Thiazid my ans ..other drug of antiHTN (SIDE EFFECT OF THIAZID IS GOUT) 9- 12 yrs old complain of LL , UL and face odema and other cardic sym. Dx: Wet beriberi (my answer) Dry beriberi Vit. A deficiency 10- pt heavy alcohol drinking C\O forceful vomiting and retching , then vomiting with blood , Then blood stop Mx : Hydration my ans Surgery
| P a g e

11- pt. complain of Rt. Hypochondrial pain and fever , he have past H\O bloody diarrhea and + Ent. Hystoltica in stool < he done aspiration for liver ____ anchovy sauce as result Dx: amoebic liver abscess pyogenic liver abscess 12- female pt with Chlamydia , HSV type 2 and he underwent cervical cerculage She diagnosed as cervical dysplasia ,the most likely cause of cervical dysplasia is: Human paplioma virus HSV 2 my ans Chlamydia cervical cerculage 13- case about secondary syphilis Rx 14- pt with Rt upper quadrant pain , nausea and vomiting pain radiating to back , on examination Grey-Turner's sign and Cullen's sign Dx : Acute pancreatitis (my answer) Acut chlocystitis 15- pt with sever pain in Rt upper quadrant pain ( colicky ) , there is past H\O same attack the most appropriate test is : U\S my answer not sure CTscan MRCP I forget the 4th selection 16- case about post menopause Mx every thing is ok but her mother have breast cancer when she in 60 yrs Estrogen (I thing this false because estrogen increase risk of breast cancer ) Evaluate bone density and start osteoarthritis prevention (my answer) She not need any intervention now .. 17-18- 2 question about vertigo (Read about the causes and C\P of vertigo and Mx of benign paroxysmal positional vertigo ) () 19- picture of pelvic x-ray Dx: (I saw that the picture is normal but there is decrease in bone density ) The x-ray is normal Osteoporosis (my answer) ankylosing spondylitis .. 20- pt . C\O vertigo, vomiting ,nausea and hearing loss( sensorineural type ) Dx: Mnire's disease( I think this is the answer)
| P a g e

21- Q about study for 2 group on group exposure to risk factor or drug I forget but other group exposure to placebo wt this study : Cohort prospective study (my answer) I am not sure Cohort retrospective Case control study Cross sectional study I forget other questions now but inshalla if i remembered any question, I'm leaving in this post as comment sorry for delayed .

NEW SLE 16/06/2012 AD


1) 18 years old female missed her menstruation for 2 cycles , no sexual activity since 3 months when her housband travel abroad, she now complaining of heavy menstruation, what is the diagnosis - endometrits - chronic endometriosis - anovulatory cyle - cancer

2) Patient complaing of eye itching due to flying of foreing body in his eye, after removal the foreign body what you will do - topical antibiotic - oral antibiotic - topical steroid - oral steroid

3) (picture of hand with red finger) Patient came with redness of finger, you give augmentin for one week but no improvement, so what you will do now - incision and drainage under general anestheisa
| P a g e

- incision and drainage under local anestheisa - give augmentin for another week - change antibiotic

4) Which true about alzehiemer - brain atrophy is not unusual generalized - arterioscelerosing is most common cause

5) Which is true about pre eclampsia - will change to eclampsia

6) Patient coming after road traffic accident while distal small intestine and proximal large intestine remove, he complain of , what is the cause - vitamin B 12 deficiency - folate deficiency

7) Patient with DM type 1, present with kussmal breathing and acetone smelling, what is pathophysiology for acetone smelling - insulin deficiency which lead to utilize fatty acid and produce ketone - missed hypoglycemic medications which lead to utilize protein and produce ketone

8) Sickle cell patient with 11 months years old, what is true about pneumoccal vaccine - not recommended for healthy people - not necessary for patient whom their age is under 2 years
| P a g e

9) Facial nerver after pass mystoid will cross parotid - superfacial to parotid

10) Eldery patient known case of AF came with abdominal pain , and bloody stool, What is the diagnosis - ischemic mesentry

11) Child came with hypertropic right atrial , what is the congenital anomalies lead to this condition - ASD - VSD

12) Female child came with short stature, lossing of breast pad, short neck, what is the diagnosis - Turner syndrome

13) What is the first step in mild burn - wash by water with room temperature - place an ice - put a butter

14) Table with lung volume measurement (I could not remember the numbers) Patient was smoker , and stop smoking for 10 years, now complaining of dyspnea, which type of pulmonary disease he has - restictive only - obstructive and restrictve
| P a g e

- emphysema

15) 30 years old male , he is healthy , coming with suddenly shortness of breath with left side chest pain, on examination there is resonant on left side, what is the diagnosis - spontaneous pneumothorax - pulmonary embolism - pneumonia

16) Smoker coming with painless mass of lateral side of tongue, what is the diagnosis - leukoplakia - sequamous cell carcinoma

17) When aspirate breast cyst, what is that good prognosis - when the cyst not filled again by fluid - when it is solid - when there is fibrocytic changes

18) Young male healthy , come for routine examination he is normal except enlarge thyroid gland without any symptoms, what is the next step - CT - MRI - US - Iodine study

| P a g e

19) Elderly came with sudden loss of vision in right eye with headache, investigation show high CRP and high ESR, what is the diagnosis - temporal arterities

20) What is true about cloniphine - stimulate ovulation

21) What is necessary condition to do abdominal lavage in RTA - comatose patient with hypotension - conscious patient with sever abdominal pain - patient with pelvic fracture

22) What is the more prognostic factor for Chronic graneulocytic leukemia - stage - bone marrow involvement - age at discover

23) Elderly patient know case of IHD , you give him PRBC , but after that he suffer from fever with 38.5 temperature, what you will do - decrease rate of transfusion - stop transfusion and treat patient with acetamiophin only - stop transfusion and treat patient with mannitol and acetamiophin

24) In out break of TB , what is the best way to prevent it - give BCG

| P a g e

25) Patient came comatose to ER with ingestion of many sleep pills, the doctor notice he is only grasp breath. Doctor do breath by mask, but nothing happen , what you will do - continue one breath every 5 seconds - put him on recovery position - intubation - do nothing till whole medical team coming

26) Best way to promote health in populations - environment modification - promote personnel hygiene 27) Know case of DM 2 with poor controlling, coming with right knee pain and ballottement, what you will do - incision and drainage 28) Patient came with pitting edema grade 1, where is fluid will accumulate - arteriole - veniole - interstitial - capillary 29) Patient came with read eye and itching with discharge , what is the diagnosis - conjunctivitis - iritis 30) Adult want to take varcilla vaccine , how you will give - 2 dose, 2 week apart - 2 dose, 6 week apart - 2 dose, 6 month apart
| P a g e

- 3 dose, during 6 months 31) Lady want to come pregnant and want to take varcilla vaccine, what you will tell her - varcilla vaccine will not protect pregnant lady - she should wait 1 - 3 months before coming pregnant - it is a live attenuated bacterial 32) What is true about treatment of streptococcus phayngitis - decrease incidence of streptococcus glomerunephritis 33) Treatment of gonorrhea - ceftriaxone - pencillin - gentamycin 34) Elderly patient have mitral valve prolapse , will go under dental procedure , what you will give for prophylaxis - nothing 35) 22 years old patient newly diagnosed with DM type 1 , when you will check his eye for diabetic retinopathy - now , then annually - After 3 years then annually - after 5 years then annually

36) Which one will decrease risk factor for colon cancer - folic acid - vitamin D

| P a g e

36) Patient with mild asthma, he want to join sport team, what is the question you will ask the patient to know the severity of activity on his asthma - do you cough at night - do you use your salbutamol inhalor more frequent

37) Ulcer reach to involve muscle, what is the stage Stage I Stage II Stage III Stage IV 38) Patient complain of scaly itching lesion on posterior side of knee and anterior side of elbow, the diagnosis is - contact dermatitis - scabies - eczema 39) patient complaining of vertigo, vomiting ,nausea and hearing loss (sensorineural type), what is the diagnosis - Mnire's disease - osteosclerosing ------------------------------------------------------------------------------------------------------------

17june2012
2```Q1...clear case of cystic fibrosis ..pt whc repeated resp. infection...foul smell stool ,chloride is increase Q2.Dm type 1 normal vision..how to follow him to check any change 1)now and thn annually 2)now and after 3 yr 3) every 5 yr

| P a g e

Q3.guess it is repeated from al qaseem ..qs was Dm pt with pain in knee joint O/E knee was red and swelling what wl u do next 1)XRAy 2)MRI 3) arthocentesis for culture (my ans) 4)incision and drainage (i think fluid was there nt sure) Q4..pt of depression taken dg whc cause neutropenia, ecgs change etc 1) SSRI 2)clonazpine (my ans) Q5.. pt of anxiety wht is dg for RAPID releif of her symptoms 1)benzodizipine (my ans )2)barbiturates 3) SSRI 4)bupro.. q6..and Q7 was abt Q8.antidepressent hw it works 1) increase serotonin 2) decrease serotonin ( repeated qs) Q9) mechanism by whc glacoma produce 1) iris obstruct the flow of aqueous (my ans)..i forgot other options Q10..colon cancer stage 1 prognosis 1)more thn 90% (my ans )2) 70% 3) 40% 4)? Q11..pic with clear case of seborrheic dermatitis Q12..BMI chart with qs female with wt of 24.5 1)underwt 2) over wt (my ans) 3)obese

Q13..pic of ecg with a qs pt with no pulse 1) vent. tachycardia 2) atrial tachycardia 3) wolff-parkinson-white syndrome 4)tardive ..??? Q14 and Q15.. abt leprosy forgot bt easy it was abt diagnose (read leprosy) and other how to prevent leprosy 1)insect repellent (my ans ) Q16.. flavi virus mode of transmission and vector 1) sand fly plus ?2) mosquito plus ?(my ans)

| P a g e

Q 17..pt with sore and red tongue ...lab values B12 was low cause 1) pernicious anemia (my ans)2)hemolytic anemia Q18..bundle branch block causes 1)arotic stenosis 2) pulmonary stenosis 3) mitral ?? 4)cardiomyopathy (my ans ..bt not sure ) Q19..Dm pt go for hernia surgery how to give insulin dose 1) one dose at morning one on raising 2) omit the both dose 3) as previous schedule 4)?/ Q 20..dg cause gastric ulcer 1) ibupropen q21..pt with PPH ...try massage ,oxytocine ,ergometrine bt stiil bleed ..wht you do next 1)hystrectome 2) ligate internal iliac artery (my ans) Q22..old female pt with osteoprosis what is EXOGENOUS cause 1) age 2) dec vit D (my ans )

| P a g e

Azzam Alkhalifah 20 /6/2012 Assalam 3lekom my exam was on 11th of June It was mixed of easy and hard Qs Time was more than enough (3 hours for 100 Qs) Each Q presented alone then you pass to the next by the end of the Qs you can revised the Qs which are marked or not answered and here are some new and doubtful questions
1- 40 years old Pt. known to have crohn's Disease, came with fevers, hip and back pain, blood positive brown stool. on Examination, soft abdomen, normal bowel sounds, nprmal range of motion of hip. what is the best radiological diagnosis? a- Abd. US b- Abd. CT c- Hip CT d- IV venogram e- Kidney US (I don't know what exactly he want to diagnose but my answer was D) 2- 60 years old post surgery (cholecystectomy) came with unilateral face swelling and tenderness. past history of measleswhen he was young. on examination moist mouth, slightly cloudy saliva with neutrophils and band cells. Culture of saliva wasn't diagnostic. what is the diagnosis ? a-Sjogren Syndrome b-Parotid cancer c-Bacterial Sialadenitis d- ?? I forgot (No choice of parotid stone :( my answer C i think its wrong) 3- Pregnant for 12 weeks, Ex. uterus as large as 16 weeks, High BHCG, US showed small fetus less than his age. Diagnosis? a-placental site trophoblastic disease b-choriocarcinoma c-Complete hydated cyst d| P a g e

4-Parents asking about Lyme disease for there children. practitional is mos correct to tell them (for prevention) : a- kill vector b- clothes of natural fibers c- antibacterial soap d(I think C is the correct) 5- Most Risk factor for stroke : a- HTN ??? b- Atrial fibrillation ??? c- DM d- Smoking ( A or B) 6- Best excercise for Ischemic heart disease patients: a- Isotonic ?? b- Isometric c- Yoga d- Anaerobic ?? 7- Pt. Obese , Smoker, High LDL, High triglycerides, Low HDL, past Hx of HTN but he didn't us his medications for the last 6 months, On Ex. BP=130/95 . for better survival correct : a- Smoking, Obesity, HDL b- Obesity, HTN, Cholesterol cd(my suspecion was between A and B) 8- 2 years old PT. was severly ill, high fever for 2 days, then develop Rashes, Low BP, Tachycardia : a- Meningococxemia b- Rubella cd(My answer was A) 9- Most specific test for PE: a- venography b- Ventilation Perfusion (V/Q) c- X-ray d(my answer was A)
| P a g e

10- Cause of death for Ludwig Angina : a- Asphaxia b- Septecimia c- Pneumonia d- Rupture free wall (My Answer was 7a8ara Ba8ara) 11- Young child, atopy, Stridor & barking cough mid night resolved spontaneously after few hours. same attack 6 months ago, your diagnosis? a- Asthma ?? b- Croup c- Spasmodic croup ?? d- Epiglottitis (My answer was C )

12- treatment of pyoderma gangrenosum? a- Steroid b- Topical antibiotics c- Oral antibiotics d- Methotrexate e- Plasma phoresis (My answer was A) 13- scally rash on face and flexor areas of the limbs? a- Atopic Dermatitis b- Contact dermatitis c- Seborrheic dermatitis d(My answer was A) 14- Pt. after his wife died had insomnia for 5 days didn't sleep for the last 2 days what drug you'll give? a- Fluoxetine b- Lorazepam c- Imipramine d- Chlorpromazine ( My answer was B , my grade in psychiatry 5/5) 15- Main defference b/w dementia and delerium ?
| P a g e

a- Memory impairment b- Level of conciousness c- aphasia d16- Before instrumental delevary, Role out: a- Cephalopelvic disproportion b- cord prolapse c- Breach presentation d17- 2 years migrain, best to diagnose ? a- MRI brain b- CT c- Full history and examination d(My answer was C ) 18- Pt Known BPH stable on medications. on examination prostate was smooth with no nodularity, He asked for PSA screening. what will you till him? a- No need for PSA b- Explain pros and cons of PAS c-&d- order other advanced Investigations (biopsy, ?? ) (Answer is A or B ) 19- Whats true about rubella ? a- cause mouth ulcer b- a cause of arthritis c- High fever on first days of presentation d20- OM treated , resolved fever and pain. after 3 weeks Pt. came still there is fluid in the middle ear without fever and pain. your action? a- Steroid b- Myringotomy c- Antibiotics d(Answer is B) 21- 56 years with papillary thyroid cancer, what to do? a- surgical resection b- Radiation c- Radioactive Iodine d| P a g e

(My answer was A) 22- Newborn with left eye purulent discharge, redness, edema. culture showed gram -ve diplococci. your TTT ? a- IV cephalosporin b- IM cephalosporin c- Oral floroquinolone d- Topical sulfonamide (My answer was D) 23- CSF in aseptic meningitis ? a- Low Protein b- High glucose c- Neutrophils d- Lymphocytes e- Esinophils (My answer was D) 24- Estrogen containing Contraceptives INCREASE risk of ? a- Breast CA ?? b- Ovary CA c- Endometrial CA ?? d- Thromboembolism ?? (My answer was A) 25- True about OCPs : a- May contain upto 0.5 ethinyl estradiol b- Change viscosity of cervix discharge c- Can delay menopause 26- True about DT vaccine : a- No benifit for pregnants b- pregnancy is not CI c- If taken, do abortion d(My answer was B) 27- Infant with sickle cell anemia, whats true about prophylaxis? a- Infants should take 23-valent vaccine b- Children above 2 years take only pentavalent vaccine c- even if vaccine taken, if there is contact with ill people child should be given prophylactic Antibiotic d- if not high risk no need for prophylaxis (My answer was C )
| P a g e

these are the new and the doubtful Qs I could remember, Other Qs were easy, mentioned before or forgotten :) . Its the matter of Luck, Let your parents pray for you ;) you need to concentrate. So, sleep well and get yourself out of the stress. Rubella and Measles are important, many Qs about them. Surgery, Ortho and psychiatry was easy. If you have time, read for Medicine and family medicine I wish you all a good luck. Slaaam

Prometric Exam,by: Dr.Hassan Mortaja ... :) ( ()


1-DM ptwent an elective surgery for hernia he is fasting form midnightconcerning his insulin you will give him: a-half dose of morning dose b-half dose of morning and half dose of midnight c-usual insulin dose d-you will let him omit the scheduled surgery dose

| P a g e

2-most common (or first line..i dont remember) Ax for OM; amoxicillin

3-pt with risk factor for developing infective endocarditis..he will underwent an urology surgery..and he is sensitive for penicillin..what you will give him? a- IV vancomycin plus IV gentamicin b-oral tetracycline?? c-no need to give

4-long scenario about obese pt and his suffering with lifethe important thing that he is snoring while he is sleepingand the doctors record that he has about 80 apnec episode to extend that po2 reach 75% no other symptoms..exam is normall..your action: a-prescribe for him nasal strip b-prescribe an oral device c-refer to ENT d-refer for hospital for CPAP and monitoring

5-pt with typical signs and symptoms of DVT..which one of the following will increase her condition: a-DIC b-Christmas disease(Haemophilia B)

6-what is the pathophysiology infection in DM:


| P a g e

(why they develop infection) a-decrease phagocytosis b-decrease immunity c-help in bacteria overgrowth

7-pt came with pnemosistis carini infection..what is your action? a-Ax and discharge b-check HIV for him

8a-immunoglobulin and one dose vaccination b-immunoglobulin c-three doses HBV vaccine

9-varicilla vaccine in adult; ab-

10white discharge if squeezed..he is asymptomatic Your action: a-core biobsy b-total excision c-cryotheraby
| P a g e

d-Ax

11-pt with DM and obese ,plane to reduce his wt is : a.decrease calori intake in day time b.decrease calori and increase fat c.decrease by 500 kcal/kg per week d.decrease 800 per day

12-case about pt with papules in the genital area with central umbalicasation (hx of unprotected sex) (Molluscum contagiosum) a-Acyclovir he mentioned names of two strange solutions(podofex -monfil??or some thin like that

13-Old pt what to do : a. Urgent surgical referral . b. Antibiotic. c. Barium enema. d. Colonscopy

14-conserning depression: a-SSRI is associated with20% risk for sexual dysfunction b-venlafaxine can ve used safely in sever HTN

15-which one of the following is true about exercise : a- exercise decrease HDL b- exercise increase C reactive protein c- not useful in central obesity
| P a g e

d- to get benifetyou have to exercise daily

16-pt with Hx of unprotected sexcam with penile discharge culture done and revealed gram negative diplococcic Associated picture of the discharge and the gram stain Your diagnosis: a-chlamydia b-gonorrhea c-strept d-staph

17 - 50 years old female with anxiety she had a Hx of an interview about one month ago when she became stressed..anxious tacycardic..dyspnicand she had to cancel it She is always try to avoid that room that she had the interview in it Diagnosis? a-GAD b-specific anxiety disorder c-panic disorder d-post traumatic disorder 18-your plan in management of Crhons disease (as long term management)is to watch for: a-lopus like disorder b-serum sickness reaction

19-40 year old female(G2 P2) with hx of heavy bleeding and bleeding between periods.no hx of taking any contraceptive method she didnt gave hx of intercourse for more than one
| P a g e

yearbecause her husband in travel I dont remember about the examination..but I think it was normal) Your diagnosis: a-anovuatory cycle b-endometrail cancer

20-pt wake up with inability to speak!!..he went to a doctor..he still couldn't speak..but he can cough when he asked to do.. (He gave you a picture of his larynx by laryngoscope..which grossly looks normal!!) Your diagnosis: a-paralysis of vocal cords b- infection c- functional aphonia

21- pt with pain in Rt iliac fossa..while you are doing your palpation he developed an vomiting and nausea !!: tour diagnosis? a-crhons disease b-appendicitis c- diverticulitis

22- best initial antidepressant: SSRI

23-typical presentation of MANIA..(he asked about the diagnosis)

| P a g e

24-Case of ENTROPION(red eye with pain inflammation due to rolling in of eye lash)

25-young female with Hx of night sweat and wt loss for about 6 month -splenomegally-reed sternberg cells in blood picture your diagnosis is : a- hodgkin's lymphoma b-non hodgkin's lymphoma

26- Goodpasture's syndrome consist of the following : Pulmonary hemorrhage and glomerulonephritis

27- pt with ARDS had pneumothoraxwhat do you think the cause? a- Lung damage b- Central line insertion c- 100% o2

28stable ..no significant pasr medical Hx (X-ray is attached to the Question showing slight decrease in pulmonary markers of left lower side of Rt lung) Your action is: a- Explain to the pt that this is due to viral pleurisy b- Refer the pt for urgent ventilation perfusion scan 29- best method to maintain airway in conscious multiple injury Pt is: a- nasopharyngeal device b- oropharangeal device c- intubation
| P a g e

30-child with Hx of sore throat 5 days fever- O/E: red enlarged tonsils with white plaque with erythematous base ..associated with gingivitis Diagnosis? a- EBV b- Adenovirus c- Herpes simplex virus

31-Pt had rheumatic episode in the past.. He developed mitral stenosis with orifice less than(mm) (sever stenosis) This will lead to : abcdLt atrial hypertrophy and dilatation Lt atrial dilitation and decreased pulmonary wedge pressure Rt atrial hypertrophy and decreased pulmonary wedge pressure Rt atrial hypertrophy and chamber constriction

32-cat bite predispose to skin infection by witch organism? a- Staph b- Strept c- Pasteurella multocida 33-best mangment in case of child with iron overdose ingestion is : a- Gastric lavage b- Ipac c- Self-induce vomiting

34- Pt in TB outbreak has negative PPD ..best prophylaxis is : a- BCG b- Chemo prophylaxis

35-Q about alcohol in pregnancy..what is true? a- Placenta is a barrier for alcohol b- Alcohol is not associated with miscarriage
| P a g e

c- Alcohol fetal syndrome is associated with mental retardation, hyperexitability , facial malformation 36-what is the drug that make Cholecystitis more worse? abcdMorphine Naloxone Phoso?? Merpidine

37- pt with rheumatoid arthritis came with swelling in the knee..he asked you about the pathophysiology of that? a- Synovial cells secretion substances b- Prostaglandin hypersensitivity

38T:-2.4 _ Z: -1.2 for vertebral bone And T:-1.2 _ Z: -9 for hip bone (almost the exact Numbers in the question) Your diagnosis: a- Osteoporosis b- Established osteoporosis c- Osteopenia

DEXA scan and scored

39- what is the organism that cause skin rash in children( I think less than 2 years ) face ..accompanied with fever : (cellulitis) a- Staph b- Strept c- H.Influenza 40-Pt taking isotretinoin for Acnethe true thing you have to say to him about the drug is: a- it cause oily skin b- it cause hypersensitive skin for the sun c- it cause enlargement in breast tissue
| P a g e

41-Pt came to your clinic for check -up- O/E: you noticed Exophthalmos That she were not aware about it..how do you can measure or know the degree of this abnormality? a- Ask family members b- Ask for old photo c- Measuresomething?

42- Known case of allergic conjunctivitis ..that suffer in every spring..he is a Gardner and cannot avoid allergic substanceswhat do you advice him to reduce th symptoms in the night? a- Sleep in air conditioned room b- Eye drops c- Apply cold compressors

43-A old pt came to your clinic to chick for a macule on his back with typical characteristic of MALIGNANT MELANOMA (irregular borders ,asymmetric ,more than .7mm,brown-black colure) Revise the ABCD mnemonic of melanoma

44- diabetic women with Hx of fetal full term fetal demise in last pregnancy, what is your recommendation for current gestation? ab- C/S in 38 week

45- when you prescribe wellburtin for smokers to help them to quit ,you have to ask them about what? Hx of seizures

| P a g e

46- child with erythema and itching and scaling in front of both elbows, behind knees , face ..your diagnosis? a- Contact dermatitis b- Scabies c- Eczema (he didnt mentioned seborrhoid dermatitis)

47- pt with rhomatoid arthritis ..asking you about permanent loss of joints .how to prevent it what is the true : a- Oil fish can help b- Alternative medicine has no benefit c- DRAMADs is sufficient

48-Q about relative risk what does it equals (schedules is attached to the Q)

49- what is first step to conduct the epidemiological curve? a- Collecting samples b- Verifying diagnosis c- To know the incubation period

50- one of the fallowing is one of the characteristics of randomized control study?? (I dont remember the stupid choices )

51- child with typical Hx pain during the examination suddenly he became pale and hypotensivewhat is your action?(splenic rupture) a- IV fluids, Ax , observation b- IV fluids , urgent CT scan
| P a g e

52- pt with typical Hx of viral conjunctivitis in Rt eye..what is your action? a- Add topical steroid b- Add topical antiviral c- Add topical antibacterial

53- Pt came to you asking about why should we take influenza vaccine annually??what true thing you will tell him? Because : a- Antibacterial prophylaxis b- Change in mood of transmission c- Changings in virus structure (something like that)

54- pt with cervical spondylosis came with atrophy in Hypothenar muscle and decreased sensation in ulnar nerve distribution.. studies showed alertness in ulnar nerve function in elbow ..tour action is : a- Physiotherapy b- Cubital tunel decompression c- Bla bla bla

55- pt came with osteoarthritis and swelling in distal interphalangeal joint what is the name of this swelling ?? a- Bouchard nodes b- Heberden's nodes

56- what is boutonnire deformity in RA? a- PIP flexion with DIP hyperextension b- PIP flexion with DIP extension c- PIP extension with DIP flexion
| P a g e

57- A women G1 P1 came to your clinic complaining of amenorrhea ..she is breast feeding for her last child 4 month old.. urine pregnancy test is negativewhat is next step? a- Prolactin level b- TSH level c- CT scan

58- post C/S pt .. forth day ..started to develop dyspnea ..your action is : a- Supportive therapy b- IV heparin.. arrange for urgent ventilation perfusion scan 59-child with swelling in his Rt thigh with erythema and pain.. no significant past history .. movement still possible .. knee is not swelled .. next step? a- Blood culture b- ASO titer c- X- ray 60- man fall down from ladder .. O/E:he almost not breathing ..cyanosed , no breath sound, although Rt side of his chest in hyperresnoant.. your action now is : abcdRt pneuoectomy Intubation Tube thoracotomy Lung pleurodisis NB; no choice like needle aspiration in second intercostal space 61- Old Pt was coughing then he suddenly developed pneumothorax best management: abcdRt pneuoectomy Intubation Tube thoracotomy Lung pleurodisis NB; no choice like needle aspiration in second intercostal space

Rusha SLE 2012


| P a g e

1-Mcc of post partum hemorrhage uterus atony multigravida 2- LACERATION IN ANTERIOR ASPECT OF WRIST: -median nerve injury cannot appose

3- Elderly pt . fever and infection by enterocucus fecalies, source of infection: - urinary -lung 4- City with 1500persons, no of 105 birth , 5 are still birth , 4 die at first month,2 die before age of one year , perinatal mortality? 10.3 4- Most reliable test to diagnose acute glomerular nephrities: red cast in urine

5- Computer programmer, a case of carpet tunnel syndrome,positive tinnel test , how to splint: Dorsiflexion 6- Which of the following not a live vaccine: HB 7- 6 yrs old child came to you he only had his BCG vaccine, HbsAg +ve (mother also +ve) wt to give: -DTP,OPV,HiB,HepB,MMR -DTP,OPV,HiB,MMR 8- In cystic fibrosis the genetic defect in 2-long arm of human chromosome 7
| P a g e

9- most common risk factor in intra-crinal hemorrhage is 2-vascular hypertension 10- Primigravida with whitish discharge the microscopic finding show dpseudohyphae the treatment is a. Meconazole cream applied locally 11- The heart increase its blood supply by Dilate coronary artery

12- newborn with fracture mid clavicle what is true Most patient heal without complications 13- patient having chest pain radiating to the back, decrease blood pressure in left arm and absent left femoral pulse with left sided pleural effusion on CXR, left ventricular hypertrophy on ECG, most proper investigation to dx: 1-aortic angiogram

14- defention of Epidemic 15- side affect of nitrate sexual dysfunction bradychardia hypotention throbbing headache 16- seizure drug which cause hair loss ??? 17- mother complane pain when she hold her baby in her wrist.OE radiostaloid tendenss , pain when extend and abduct the thumb dx?? gamer thumb .......??

| P a g e

18- F pt G..P.. for evaluation she had 3 prevuse termination by D&C, OE she was normal dx?? asherman syndrome shehan syndrome kalman syndrom polycystic ovarian syndrom 19- tt of alcoholic withdraw?? Benzodiazepam ??? 20- retired pt complaininggg of shoulder pain cant sleep from it now he cant left any thing, OE reduse rang of motion x-ray eroion joint dx??

21- ear pain rupture of tempanic memb. Cloudy secreation TT ?? AB drop systmic AB corticosteroid I think the last one was viral drug 22- ear pain bulging of tempanic memb. With undemarcation of the eadg erethema behind it dx?? OE OM Tempanoc cellulitis

23- pt with cough, lung infertelation Hx of glucoma medication in the past , what drug that do this manifistation timelol Pilocarpine ... 24- Hx of glucoma $ COPD what tt?? 25- Qus in the ansure there is avocado ??
| P a g e

26- pt fall from a ladder came to er cyianosis, dimenished air sound rt lung ,resonant in perction what to do ?? O2 mask endotracheal tube pneomonectomy chest tube for drange serios x-rays 27- baby with emisis, diarehea, rectal bleeding x-ray show obstruction pattern what to do ?? rehydration mediate surgery 30- olecrnon bruritis cause from multiple truma in elbow cause from antibodies .......

31-Most important to instruct pt about Lyme dis Wear long fiber clothes 32- hx of chid this brother bit him 3 hr ahes havin 1cm larceration . Preveios hx of taking booster dose of tetanus tt ?? agmuntin anther dose of tetanus 33- child with hematuria 15 RBC whatnext Repeat urine for rbc and protien 34- pt 38 week g complaining of abd pain, US show fibroid in uterus baby is alive what to do ? Termenation pain killer manegmant 35- Qus about Degoxin toxicity
| P a g e

36- The useful excurcise for osteoarthritis in old age to maintain muscle and bone Low resistance and high repetion weight training

37- a pt presented with DKA & hyperkalemia & hypotension, best initial treatment 2 liters NS with insulin infusion at rate of 0.1/kg 38- Elderly patient presented by SOB , rales in auscultation , orthopnea, PND, exertional dyspnea, what is the main pathophysiology : A. Left ventricular dilatation . B. Right ventricular dilatation C. Aortic regurgitation. D. Tricuscpid regurgitation

NEW SLE 16/06/2012 AD

months when her housband travel abroad, she now complaining of heavy menstruation, what is the diagnosis - endometrits - chronic endometriosis - anovulatory cyle - cancer

foreign body what you will do


| P a g e

- topical antibiotic - oral antibiotic - topical steroid - oral steroid

for one week but no improvement, so what you will do now - incision and drainage under general anestheisa - incision and drainage under local anestheisa - give augmentin for another week - change antibiotic

4) Which true about alzehiemer - brain atrophy is not unusual generalized - arterioscelerosing is most common cause

5) Which is true about pre eclampsia - will change to eclampsia

after road traffic accident while distal small intestine and proximal large intestine remove, he complain of , what is the cause - vitamin B 12 deciency - folate deficiency

elling, what is pathophysiology for acetone smelling


| P a g e

- insulin deficiency which lead to utilize fatty acid and produce ketone - missed hypoglycemic medications which lead to utilize protein and produce ketone

d, what is true about pneumoccal vaccine - not recommended for healthy people -

- superfacial to parotid

came with abdominal pain , and bloody stool, What is the diagnosis - ischemic mesentry

11) Child came with hypertropic right atrial , what is the congenital anomalies lead to this condition - ASD - VSD

12) Female child came with short stature, lossing of breast pad, short neck, what is the diagnosis - Turner syndrome

13) What is the rst step in mild burn - wash by water with room temperature - place an ice
| P a g e

- put a butter

14) Table with lung volume measurement (I could not remember the numbers) type of pulmonary disease he has - restictive only - obstructive and restrictve - emphysema

15) 30 years old male , he is healthy , coming with suddenly shortness of breath with left side chest pain, on examination there is resonant on left side, what is the diagnosis - spontaneous pneumothorax - pulmonary embolism - pneumonia

16) Smoker coming with painless mass of lateral side of tongue, what is the diagnosis - leukoplakia - sequamous cell carcinoma

17) When aspirate breast cyst, what is that good prognosis - when the cyst not filled again by fluid - when it is solid - when there is fibrocytic changes

rmal except enlarge thyroid gland without any symptoms, what is the next step
| P a g e

- CT - MRI - US - Iodine study

high CRP and high ESR, what is the diagnosis - temporal arterities

20) What is true about cloniphine - stimulate ovulation

- comatose patient with hypotension - conscious patient with sever abdominal pain - patient with pelvic fracture

22) What is the more prognostic factor for Chronic graneulocytic leukemia - stage - bone marrow involvement - age at discover

with 38.5 temperature, what you will do - decrease rate of transfusion - stop transfusion and treat patient with acetamiophin only
| P a g e

- stop transfusion and treat patient with mannitol and acetamiophin

24) In out break of TB , what is the best way to prevent it - give BCG

o ER with ingestion of many sleep pills, the doctor notice he is only grasp breath. Doctor do breath by mask, but nothing happen , what you will do - put him on recovery position - intubation - do nothing till whole medical team coming

- environment modification - promote personnel hygiene what you will do - incision and drainage 28 - arteriole - veniole - interstitial - capillary

- conjunctivitis - iritis
| P a g e

30) Adult want to take varcilla vaccine , how you will give - 2 dose, 2 week apart - 2 dose, 6 week apart - 2 dose, 6 month apart - 3 dose, during 6 months 31) Lady want to come pregnant and want to take varcilla vaccine, what you will tell her - varcilla vaccine will not protect pregnant lady - she should wait 1 - 3 months before coming pregnant - it is a live attenuated bacterial

- decrease incidence of streptococcus glomerunephritis 33) Treatment of gonorrhea - ceftriaxone - pencillin - gentamycin give for prophylaxis - nothing diabetic retinopathy - now , then annually -

36) Which one will decrease risk factor for colon cancer
| P a g e

- folic acid - vitamin D

question you will ask the patient to know the severity of activity on his asthma - do you cough at night - do you use your salbutamol inhalor more frequent

37) Ulcer reach to involve muscle, what is the stage Stage I Stage II Stage III Stage IV

nt complain of scaly itching lesion on posterior side of knee and anterior side of elbow, the diagnosis is - contact dermatitis - scabies - eczema

what is the diagnosis - Mnire's disease - osteosclerosing

-----------------------------------------------------------------------------------------------------------| P a g e

SLE july 2012 By: dr..wadah khairi


Surgery: 1. Thyroid cancer, definitive treatment.. 2. Anal mass, definitive treatment.. 3. Hemorrhoids for 10 years, asymptomatic, what to do? 4. Breast cancer, investigation? less than 35 years= ultrasound,,, more than 35= mammography 5. RUQ pain + murphy sign positive= stone RUQ pain + fever = cholecystitis 6. Pancreatitis, most diagnostic investigation. 7. women with appendicitis best investigation? Obs &gyne: 1. methergine is contraindicated with which medical condition? hypertension/pre-eclampsia 2. treatment of cervical dyskaryosis? 3. best method of cervical screening? PAP smear 4. which method of contraception to be used in a sexually active women with multiple partners? condom 5. emergency contraception? levonorgestrel upto 72hrs 6. infertility in women, the 1st investigation to be done? semen analysis 7. if u suspect pre-eclampsia, whats the first thing to do? check BP 8. athletic women with history of infertility, has normal LH,FSH, her partner has 2 kids from his previous marriage.. she refuses to quit exercise, what are you going to give her? GRH pulsatile or continuous,,, OR,, TRH pulsatile or continuous Medicine: 1. huntington's disease? pathophysiology? 2. testosterone injection in HIV patients, why is it given? 3. anti-TB treatment (rifampicin, isoniazid, pyrazinamide, ethambutol) given to HIV patient, he came after 1 month, what is your plan? continue the same medication for another month and then give 2 medications for 4 months 4. hepatitis diagram.. window period? 5. 5 phospho-diesterase inhibitor contraindicated with which medication? nitrate 6. questions about Diabetes Emergency:
| P a g e

1. patient fell off a building, he is unconscious, 1st step in treatment? ABC 2. pneumothorax case? treatment? initial= ABC, definitive= chest drain 3. CO poisoning. treatment? 100% oxygen Pediatrics: 1. management of asthma in acute attack? salbutamol 2. management of patient with asthma not controlled with salbutamol and long acting b-agonist? steroid inhaler 3. sudden wheezing in a child? foreign body ingestion 4. child with URTI followed by petechia? 5. newly born child with constipation, after PR examination huge amounts of stool was excreted. diagnosis? hirschsprung's disease 6. child with ear infection , developed seizures, on exam he had papilledema. Investigation? CT/MRI Dermatology: 1. patient was treated for pneumonia with antibiotics, developed rash on his back extending to the trunk but not crossing the midline (picture was shown): whats the diagnosis? most probably drug induced rash 2. a picture was shown of a child with tender nodules on his legs and hands, he had a past history of throat infection, whats the diagnosis? most probably HSP 3. child with swelling over the frontal-temporal region with loss of hair on that area. whats the diagnosis? 4. tender nodules on the tibia, diagnosis? sarcoidosis 5. painless penile ulcer, with enlarged and tender inguinal lymph nodes. diagnosis? lypmhogranuloma venereum 6. treatment of syphilis? 1st stage penicillin IM single dose,, or azithromycin single dose,,,,, latent and 3rd stage= penicillinIV for 10days 7. tinea capitis.. treatment? griseofulvin..... investigation? wood's lamp opthalmology: 1. dendritic eye lesions. diagnosis? herpes simplex 2. herpes simplex keratitis treated with anti-virals what to add on top that? steroid drops 3 blue sclera in a chile? differential diagnosis? osteogenesis imperfecta and child abuse 4. whithish colour on the pupil of a new born. diagnosis? TORCH infection (cataract in newborn) 5. severe sudden pain around one eye? diagnosis? acute angle glucoma... what to do? emergency opthalmology referral ENT: 1. in an elderly what is the most common cause of conductive hearing loss? otosclerosis
| P a g e

2. common cause of nasal bleeding in children? trauma Psychiatry: 1. bupropion, side effect? seizure 2. TCA , ECG changes? prolongs QT interval 3. case of schizophrenia 4 anorexia nervosa.. electrolyte abnormal others: 1. osteoporosis diagram, describe what you see on the graph (very easy) 2. calculation of still birth from numbers given in a table

Best of Luck :)

| P a g e

New in 4 Edition

th

C)New organized Collections (divided into supspecalities) Done by: 1) Dr.Ahmed Omer Alhebshi 2)Dr.Eyed Alhudaithi 3)Dr.Samia Khan

| P a g e

Optha collection from Different SLE Exams 2012 1) After removing foreign body from the eye apply local: A)Antibiotics# B) Steroids

2) Gardener has recurrent conjunctivitis. He cant avoid exposure to environment. In order to decrease the symptoms in the evening, GP should advise him to: 1. Cold compression 2. Eye irrigation with Vinegar Solution 3. Contact lenses 4. Antihistamines#

3) A patient complains of dry eyes, a moisturizing eye drops were prescribed to him 4 times daily. What is the most appropriate method of application of these eye drops? a. 1 drop in the lower fornix# b. 2 drops in the lower fornix c. 1 drop in the upper fornix d. 2 drops in the upper fornix

4) Pt involve in RTA, develop raccon eye : - fracture of the globe


| P a g e

- fracture in base of anterior fossa -concussion -base skull fracture#

5) SNELLEN CHART ,,, there is a chart ,, old man cums with decrease in vision ,dr check his vision by snellen chart he is able to read upto 3rd line ,, so his vision is A-20/70*** B-20/100 C-20/50 D-20/40

6) pt having glaucoma and taking treatment for it presents with shortness of breath ,, which of the drug is he taking A-pilocarpine#? B-TIMOLOL C-BETAXOL D-ACETAZOLAMIDE

7) Patient with lateral and vertical diplobia, he cant abduct both eyes, the affected nerve is: 1.II 2.III 3.VI 4.V

| P a g e

8) 54 y old patient , farmer , coming complaining of dry eye , he is smoker for 20 years and smokes 2 packs/ day , your recommendation : advise him to exercise stop smoking# wear sunscreen 9) child had recent onset flu then develop red eye + lacrimation no itching dx: -viral conjunctivits # -bacterial conj -allergic conj

10) female pt with Rt eye pain and redness with watery discharge,no h.o trauma,itching,O/E there is diffuse congestion in the conjunctiva and watery discharge what you'll do: a. give Ab b. give antihistamine c. topical steroid# d. refer her to the ophthalmologist

11) patient came with complete ptosis, the eye is looking out and down , with dilated pupil , the nerve involved : 1- third only # 2- third and fourth only 3- fourth only 4- third and sixth only

12) A patient with a suspected corneal ulcer: Cotton debridment and systemic antibiotics.
| P a g e

Cotton debridment and cycloplegics. Burr debridment and (forgot the choices) Topical antibiotic, cycloplegic and refere to ophthalmologist. #

13) newborn with eye infection a- Oral antibiotic b- Oral steroid c- Topical antibiotic#

14) pt with trachoma in eye . for prevention you should a- water# b- '''''''''''''' +eradication of organism c- mass ttt

15) Pt with TB, had ocular toxicity symptoms, the drug responsible is: 1.INH 2.Ethambutol# 3.Rifampicin 4.Streptomycin

16) male came to you complaining of sudden progressive decreasing in vision of left eye over last two/three days, also pain on the same eye, on fundoscopy optic disk swelling was seen , Dx : a. central retinal artery occlusion b. central retinal vein occlusion
| P a g e

c. optic neuritis# d. macular degeneration

17) Left red eye, watery discharge, photo phobia, peri-auricular non-tender lymph nodes .. Dx 1.Bacterial conjunctivitis 2.Viral conjunctivitis#

18) Acute eye pain, decreas vision, conjunctival injection, constricted pupil, opaque lens with keratinzation, cells in aques humor : a. Anterior uveitis.

19) A man who bought a cat and now developed watery discharge from his eyes he is having: a) Allergic conjunctivitis# b) Atopic dermatitis c) cat scratch disease

20) pt came to emergency room complaing of acute pain in rt eye and watery discharge and photopobia , in slit lamp examination founded keratin layer detachment behind cornea and block aqueous miswork !!!! What is diagnosis , A- acute closed angle glucoma B- acute keratitis C- acute conjuctivitis D-cillary body dysfunctin ?? E- deposite of ...........
| P a g e

21) The most dangerous red eye that need urgent referral to ophthalnologest 1.associated with itching 2. presence of mucopurulant discharge 3.bilateral 4.associated with photophobia#

22) pateint is taking steroid eyedrops for allergic conjuctivitis for a long time, what is the side effect that you should concern about: catarct glaucoma#

23) pt with recent Hx of URTI , develop sever conj. Injection with redness, tearing , photophopia , So, what is TTT: a) Topical ABx b) Topical acyclovire ??# c) Oral acyclovire d) Topical steroid ??

24) 34-A 26 year old female complaining of headache more severe in the early morning mainly bitemporal, her past medical Hx is unremarkable. She gave Hx of OCP use for 1 year. Ophthalmoscope examination showed pappilledema but there is no other neurological findings. The most probable diagnosis is: a. Optic neuritis b. Benign intracranial hypertention
| P a g e

c. Encephalitis d. Meningitis e. Intracranial abscess

25) pt with blephiritis , with hx of acne rosecia but with no sign of keratitis , wt you will give him : A. Topical chlormphencol B. Oral doxcyclin C. Topical gentamicin

26) pic of optic nerve cupping A. Gluocomatous cupping# B. Optitis C. Optic nerve atrophy

27) pt had stroke , after that he lost vision in the left eye , where is the lesion : A. Frontal lobe B. Occipital lobe# C. Parietal lobe D. Temporal lobe

28) pt c/o pain when moving the eye ,fundoscopy normal ,, diagnosis is 1-optic neuritis 2-papilloedema

| P a g e

29) Painfull vision loss: A. Central vein thrombsis B. Central artery embolism C. Acute angle closure glaucoma#Top of Form

30) HTN pt, with decrease vision, fundal exam showed increase cupping of optic disc dx: a. Open angle glaucoma# b. Closed angle glaucoma c. Cataract d. HTN changes

31) 24 YO male with painless loss of vision ,macular degeneration and optic atropy: a-pathological myopia b-physiological myopia C.........myopia

32) 100% O2 given for prolonged periods can cause all except: a. Retrosternal Pain b. Seizures c. Depression# d. Ocular Toxicity

| P a g e

33)Eye screening in DMI Now and annually Now and every 10 years After 5 years and annually#

34) Child came to ophthalmology clinic did cover test, during eye cover , his left eye move spontaneously to left, the most complication is: a) Strabismus# b) Glaucoma c) Myobloma

35) Patient has decrease visual acuity bilateral , but more in rt side , visual field is not affected , in fundus there is irregular pigmentations and early cataract formation . what you will do 1. Refer to ophthalmologist for laser therapy# 2. Refer to ophthalmologist for cataract surgery

36) Diabetic pt. have neovasclarization and vetrous hemorrhage , next step : 1. Refer to ophthalmologist

37) 24 y/o female newly diagnosed type 2 DM, she is wearing glasses for 10 years, how frequent she should follow with ophthalmologist: a. Every 5 years. b. Annually#

| P a g e

38) man c/o of fever , vesicular rash over forehead management 1-antiviral , follow 3-5 day 2-antiviral , refer to ophthalmologist#

39) Patient came to you with small swelling under his eye , on examination he have inflammation in lacrimal duct , you refer him to ophthalmologist before that what you will give him ? a- Topical steroid b- Topical antibiotic# c- General antibioticBottom of Form

40) Verryy long scenario of old age pt with DM, HTN, hx of multiple cardiac attack, CVA, came for routine check up in PHC, u found bilateral opacification in both lenses, with decreasing of visual acuity, u will: Refer to lazer therapist# refer to cataract surgeon, refer to ophthalmologist, follow up

41) Child with large periorbital hemangioma , if this hemangioma cause obstruction to vision , when will be permenant decrease in visual acuity a. After obstruction by one day b. By 1 week#
| P a g e

c. By 3 months d. By 6 months

42) retinal detachment all of the following are true EXCEPT: a) can lead to sudden loss of vision b) more in far sighted than near sighted# c) follow cataract surgery d) if you suspect it sent for ophthalmologist e) !!

43) patient with red eyes for one day with watery discharge No itching or pain or trauma (nothing indicate allergy or bacterial infection) there is conjuctival injection visual acuity 20/20 what is next management

antihistamines topical AB No further management is needed refer to ophthalmologist ? topical steroids ?#

we discussed it before with a little difference in the scenario

44) acute angle glucoma with COPD and DM : acetazolamide#

| P a g e

45) Contraindicated in acute glaucoma management: a. Pilocarpine b. Timolol not sure c. B-blockers, CA inhibitors, NSAID, Mannitol d. Diprovin #?

46) What is the management of acute congestive glaucoma : 1.IV acetazolamide and topical pilocarpine #

47) 60yrs pt presented w decrease vision bilt, specially to bright light o/e he was having cupping w wedge shaped opacities....he is having A) lens sublexation B) cataract# C) open angle glaucoma

48) Acute angle glaucoma , with COPD and DM you give? a) Metoprolol b) Something with lol c) Acetazolamide# d)steroid

49) 70 y/o female say that she play puzzle but for a short period she can't play because as she develop headache when playing what u will exam for 1. Astigmatism#
| P a g e

2. Glaucoma

50) patient w pain in Rt. eye ass. with photophobia and redness, patient has a hx of previous uveitis in the other eye...what is ur dx? 1- acute angle glaucoma 2-uveitis#

51) 50 year old Man presented to ER with sudden headache, blurred of vision, and eye pain. The diagnosis is: a. Acute closed angle glaucoma# b. Acute conjunctivitis c. Corneal ulcer d. -----

52) Pt diabetic for 10 y.s. with vision problem on fundoscpy you found red spot on retna nevasculazition and macular enurism your diagnosis : a) Macular degeneration# b) proliferation

53) Corneal alcer due to truma :: your action a- antibiotic + analgesic + refer to ophthalmologist# 54) 80 yr old in his normal state of health presented with decrease visual acuity bilaterally without any defect in visual field his VA Rt eye= 20/100 VA Lt eye=20/160 fundoscopic exam showed early signs of cataract and drusen with irregular pigmentation. No macular edema or neovasculirization. The appropriate action beside antioxidants and Zn is: a. Refer the pt for emergency laser therapy#
| P a g e

b. Refer the pt for cataract surgery c. See the patient next month d. No need to do anything

55) old diabetic patient with mild early cataract and retinal pigmentation o Drusen formation. , u prescribed anti oxidant what to do next : - urgent ophth appointment - routine ophth referral - cataract surgery - see him after One month to detect improvement

56) patient take 3-hydroxy-3-methylglutyle coenzime A (HMC CA) what is side effect a- chronic rhinitis b- infertility c-rhabdomyelysis d-postural hypotension# e-cataract

57) What is the side effect of steroid on the eye ? a- Glaucoma . b- Cataract# c- Keratoconus

| P a g e

Psychology Collection from Different SLE Exams2012 By : dr. Ahmed Omer Alhebshi 1- Pt with hx of diarrhea, abdominal pain, agitation, headache, dizziness, weakness, pulstile thyroid, unsteady gate. Examination was normal. Dx: a. Hypochondriasis b. Somatization disorder# c. Thyroid Ca d. Anxiety

2- Old man psych pt , has halosination , aggressive bebaviour ,loss of memory ,Living without care , urinate on him self , what is next step to do for him ? a) Give antipsychotic b) Admit him at care center for elderly . mostly #

3- Concerning depression A) SSRI ass w 20% risk for sexual dysfunction # B) venlafaxine can used safely in sever HT

4- Patient loss his wife in the last 4 months , he looks sad cannot sleep in the last 2 days, which medication can help him: a- Lorazepam b- Diazepam c- SSRI#
| P a g e

5- 46yrs males/o early ejaculation, usability to sustained eriction he believe his 26 yr marriage is alright. His wife ok but unorganized , obese. Doctor confirm no organic cause. He look thin, sad face what's ttt A) SSRI# B) sublingual nitrate 6 h before C) testosterone injection

6- antideprssants associated with hypertensive crisis treatment a. SSRI b. MOAIs# c. TCA

7- patient taking antidepressent medication now complaining of insomnia what r the expected drug he is taking ? A- SSRI# B- MOA c- TCA

8-Alternative therapy for severe depression and resistance to anti-depressant medications are: 1.SSRI 2.TCA 3.ECT#

9-SSRI was prescribed to a patient with depression , the effect is suspected to be within :
| P a g e

1.One day . 2.Two weeks . 3.Three to four weeks# 10- Female had history of severe depression, many episodes, she got her remission for three months with Paroxitine ( SSRIs) .. now she is pregnant .. your advise Stop SSRi's because it cause fetal malformation Stop SSRi's because it cause premature labor Continue and monitor her depression# Stop SSRIs

11-What is the mechanism of OCD drugs: Increase availability of Serotonin# Decrease production of Serotonin Increase production of Serotonin .. Serotonin .. Serotoni

12- drug for ttt of ADHD .. (they mention typical scenario of it) is : - atomoxetine# -olanzapine

13- 3o yr old man cover the TV he said that the goverment spy him and he said God tell him that as he talk with him through the lamp , dx is: -schizophrenia#
| P a g e

14- pt taking medication and develop symptoms of toxicity : tachycardia, dry mouth, hyperreflixia, dilated pupils and divergent squint. the medication most likly: -TCA # -SSRI -ephedrin

15- major depressive disorder wt trestment>>>ssri

16- old age previous ok...long story develop agitation say some thing about dilirum the answer...

17- new marreid the wife notice her husband go out sid then came back to close the door more than 10time also when he take shwer ...for long time ...repeate praying also....the ans was OCD

18- long story...pt complain ansomnia irregular sleep....so treatment ssri my ans

19- 40 years old , thin , k/o premature ejaculation , loss of libido ,he look sad , his wife is obese , expender money unorganized , claims their marriage is alright , the examination prove no organ pathology : Wts ur action 1-ssri# 2-weekly testesrone injection

20-65 yrs old lady came to your clinic with Hx of 5 days insomnia and crying ( since her husband died ) the best Tx. For her is :
| P a g e

a- lorazipam b- floxitein# c- chlorpromazine d- haloperidol

21) Psycatric pt on antipsychotic drug most drug that lead to impotence with antipsychotic is a- proprnlol# b-NSAI c-ACEI NONE of them ... correct me if i'm wrong ?

22) 25 year theacher have fear attack and worry before enter the class ( I forgot all the scenario) what is the initial treatment: a. Selective serotonin reuptake inhibitor# b. Tricyclic depressant c. Beta blocke

23)teacher ,complain of panic , this after mistake in class room, he know it must be useful in future day , co sweting , tachycardia , tightness> - benzodiazepam -ssri -social plhobia

| P a g e

24) Which one of these drugs is not available as emergency tranquilizer in psychiatric clinics: - Haloperidol - Phenobarbital - Lorazepa

25) 50 yrs female w anxiety had an interview one month ago when she became stressed anxious, tachycardia and she had to cancel it. She always avoid that room she had the interview in it. Diagnosis A)GAD B) specific anxiety disorder# C) panic disorder D) post traumatic disorder

26) the symptom/sign that comes 2ry rather than presented symptom in panic pt. tachycardia epigastric pain chest pain phobia#

27) pt was in the lecture room, suddenly had an attack of anxiety with palpitation and SOB, after this episode she fears going back to the same place avoiding another attack
| P a g e

Panic attack# Anxiety attack Generalized anxiety disorder

28) scenario of panic attack .. Treatment Benzodizepine SSRi#

29) Patient with panic attack .. Something related to secondary mechanism not symptom Epigastric pain Chest pain Dizziness TAchcardia 30) May indicate Good prognosis for schizophrenic pt. +ve Family hxif he said, No precipitating factor # Gradual onset Apathy Marrie

31) Colzapine is used in which childhood psychiatric disease? a- Schezophrenia# b- Depression c- Enuresis
| P a g e

32) 6months postpartum having hallucination ,dellusion ,disorganized thinking and speech ,having social and emotional difficulty , having history of child death 3 months ,,, all of the following should be the possibility except A- SCHIZOPHRENIA B-SHIZOPHRENIFORM DISORDER C-BRIEF PSYCHOTIC DISORDER *** D-SCHIZOEFFECTIVE DISORDER

33) PT having elevated mood state characterized by inappropriate elation, increased irritability, severe insomnia, increased speed and volume of speech, disconnected and racing thoughts, increased sexual desire, markedly increased energy and activity level, poor judgment, and inappropriate social behavior ,,, associated with above pt should have one more symptom to fit on a diagnosis A- HALLUCINATION B- DELLUCION C-GRANDIOSITY*** D-DELLIRIUM

34) Child after his father died start to talk to himself , walk in the street naked when the family asked him he said that his father asked him to do that , he suffer from those things 3 days after that he is now completely normal and he do not remember much about what he did Dx a Schizophrenia b Schizoaffective c Schizophreniform d Psychosis

35) The best drug used in treating schizophrenia, mania and schizophreniform disorders is: Risperidone# Amitriptyline
| P a g e

Olanzapine Paroxetine

36) Obssive neurosis patients will have: a. Major depression# b. Lake of insight c. Schizophrenia

37) Pt. can't go to park , zoo and sport stadium , her problem: a- Agoraphobia# b- Schizophrenia c- Social phobia d- Panic disorders

38) man who is thinking that there is Aliens in his yard although that he knows that Aliens are not existing but he's still having these thoughts all that happen especially when he is at out of home and the patienr afraid to die because that .. Dx A/ Obsessions# # B/hallusination C/ dellusion # D/ illusion

39) Patient with echolalia, echopraxia, poor hygiene, insomnia, and weird postures. Treatment? A. Lithium#
| P a g e

40) Regarding postpartum Psychosis: Recurrences are common in subsequent pregnancies# It often progresses to frank schizophrenia It has good prognosis It has insidious onset It usually develops around the 3rd week postpartum

41) Pt w chronic depression now u start tt.paroxetin u told pt A) need 3-4week to act# B)side effect????

42) Obsessive neurosis: 1. Treatment is east 2. Clomipramine doesnt not work 3. Mostly associated with severe depression## 4. Can be cured spontaneously 43) 80 years old living in nursing home for the last 3 months. his wife died 6 months ago and he had a cornary artery disease in the last month. he is now forgetful especially of short term memory and decrease eye contact with and loss of interest. dx a. alzihiemer b. depression# c. hypothyroidis
| P a g e

44) Partner lost his wife by AMI 6 months ago , presented by loss of appetite , low mood , sense of guilt , what is the diagnosis : 1.Beverament # 2.Major depression episode.

45) A female patient on the 3rd week postpartum. She says to the physician that the frequently visualizes snakes crawling to her babys bed. She knows that it is impossible but she cannot remove the idea from her head. She says she wakes up around 50 times at night to check her baby. This problem prevents her from getting good sleep and it started to affect her marriage. What is this problem she is experiencing? a. An obsession# b. A hallucination c. A postpartum psychosis d. A Delusion

SLE Rush collection , 2012 Medicine Which of the following medication if taken need to take the patient immidiatly to the hospital: b. diphenhydramine long scenario for pt came to ER after RTA splenic rupture was clear acurte sintences describe long term manamement: 1- we give pneumococcal vaccine for high risky peoples just
| P a g e

What it is the most common congenital heart disease come VSD the most common regimen in TTT of uncomplicated community acquired pneumonia ; a-azithromycine OCP increase the risk of : c- thromboembolism Which of the following is a criteria of brain stem death (read about them) Absence of doll eye movement in both sides Drug use in CHF with systolic dysfunction ? Nifidepine* deltiazm* and two drugs from ACEI I forget their names * the 5th choice is one of B blocker* pt with asthma on daily steroid inhaler and short acting B2 agonist what category: b- Mildpersistent

Male patient complain of excruciating headache, awaken him from sleep every night with burning sensation behind left eye, lacrimation and nasal congestion. What is effective in treating him : Ergonavine Sumatriptan SC Methylprednisolone NSAID Pt with active hepatitis what medication should not to give c- Atrovastsin the best investigation for acute diverticulitis: c-CT pt with COPD came with couph , wheezing and greenish sputum The causative organism: H.influanza The useful excurcise for osteoarthritis in old age to maintain muscle and bone - Low resistance and high repetion weight training
| P a g e

excercise in OA: Isometric exercises for joints with minimal range of motion >isometric is type of resistant exercises a pt presented with DKA & hyperkalemia & hypotension, best initial treatment : 2 liters NS with insulin infusion at rate of 0.1/kg Pt with hypotension & bradycardia and with this lab results (Na:130) low , (K:7) high , (Cl:116) high , (Urea: I forget but it was high) What is this patient have? b. Hyperkalemia

Man with history of alcohol association with: High MCV question about 24 years old female came to you with tachycardia you did ECG found narroe complex tachycardia 140bpm and other thing is normal what is true . - amidarone should be use >> I choose this don't know - class IV antiarrythmic contraindicated - cardiversion contraindicated - re- alter... haapen >> don't know what is it - this condition with not recur.

senario about leukemia she have anemia , throbocytopenia and high WBC with prominant blat cell with prminant large neuclus and 1-2 small neculi and +ve myeloperoxidase and -ve tyrsinase ( i think) what type of leukemia ? - lymphocytic - Myelogenous - monocytic - ........ middle age pt complaining og abdominal pain and he think he has gastric cancer he went to 6 gastroentrologisit he didi 1 ct 1 barium enema and sereis
| P a g e

of invX all normaaaaaaaaaaaaaaal diagnosis hypochondriasis

Adult pt received a vaccine. After that he complains of icthing, tachycardia, and SOB. Whats the treatment? 2) SC epinephrine In pt with moderately sever acne valgarus best ttt >> Oral isotretinoin /

| P a g e

old male with stroke , after 9 day he loss left eye vision , what are the affect structure ; a-frontal lobe bpartial c-occipital d-temporal Mass in the upper back .. with punctum and releasing white frothy material a- It's likely to be infected and Antibiotic must be given before anything b- Steroid will decrease its size c- It can be treated with cryotherapy d- It must be removed as a whole to keep the dermis intac dont remove sebaceous cyst while inflammed wait for the subside of the inflammation ( with the aid of ABx and I&D) then remove it entirely to decrease the chance of recurrence Benign tumors of stomach represent almost : A. 7 % B. 21 % C. 50 % D. 90 % which of the following take with analgesic to decrease side effect ?? a- cimitidine b-psudoephidrine c- another type of anti histaminic H1 BLOCKER mechanism of esseintial HTN A-increase of prephiral resistance B-salt water retention

- patient with hypopigmented macules.loss of sensation.thickend nerves.diagnosis was leprosy.which type tubercul oid lepromat
| P a g e

ous borderline

Intermediate Leprosy (IL) This is the earliest and mildest form of the disease. Few numbers of hypopigmented macules (cutaneous lesions) may occur. Loss of sensation is rare. Most cases progress into a later form, although patients with strong immunity may either clear the infection on their own or persist in this form without progressing. Tuberculloid Leprosy (TT) This form usually presents with large lesions (hypopigmented and erythematous macules) which are anesthetic. Infected nerves often thicken and loose function. Progression can occur leading to borderline-type leprosy and, in rare instances when the patient goes untreated for many years, the lepromatous form can develop. Borderline borderline Leprosy (BB) In this form cutaneous lesions are also present but now they are numerous and less well defined than those in the tuberculloid form. Anesthesis is less severe than TT. In this form, the disease may regress, improve or stay the same. Borderline Lepromatous Leprosy (BL) As with BB, lesions (macule type) are numerous, however now they may also consist of papules, plaques, and nodules.Punched-outappearing lesions that look like inverted saucers are common. Anesthesia is often absent. As with BB leprosy, the disease may remain in this stage, improve, or regress. Lepromatous Leprosy (LL)
| P a g e

Early on, cutaneous lesions are small, diffuse and symmetric (consisting mainly of pale macules). Later, larger and deeper lesions form and these contain many bacilli. At this point, the skin texture does not change, and little or no loss of sensation occurs. The nerves are not thickened. Loss of eyebrows occurs and then spreads to the eyelashes and then the trunk, however, scalp hair remains (see picture). Eye involvement occurs, causing pain, sensitivity to light, decreased visual acuity, glaucoma, and blindness. Testicular atrophy can occur, resulting in sterility. If the larynx becomes involved hoarseness will result. Nasal infiltration can cause a saddle-nose deformity (see picture). Swelling (edema) of the legs is sometimes a late finding. Unlike the other types of leprosy, LL cannot convert back to the less severe borderline or tuberculoid types of disease. Pt. has DM and renal impairment when he had diabetic nephropathy:there is curve for albumin a. 5y b. 10y c. 20y d. 25y

old pt , e hx of MI 2 weeks back and discharge from hospital 24 hrs prior to his presentation <<<came with sudden lower limb pain and numbness ,on ex the limb pale , cold >>the other limb normal what is the DX : a- Acute artery thrombosis b- acute artery embolus c- DVT pt has Lt lower Abdominal pain , Fever , constipation CT reveals thickened loop and little perianal fat , whats appropriate to do: 1- start AB 2- call the surgeon for immediate OP 3-give laxative 4- barium enema patient with hypopigmented skin macule on the trunk, become more obvious when expose to sun light, what you give him : Topical steroid Topical antibiotic
| P a g e

Systemic antibiotic Selenium Sulfate pt take cephalexin after tooth extraction for days. After that he develop profusre , green foul smilling diarrhea with low grad fever . He has tachycardia and mild abdominal dist. Sigmoioscopy showed white mucosal patches , what is the most ttt for this condition? A-Clarythromycine B-Vancomycine C-Cephalosporine D-Lineozides

Pt known case of hypothyrodisim , and you start levothyroxine but she come after 1 wk with cold intolerance, and bradycardia, THS INREASED ; Continue and check after 1-2 month/ decrese the dose/ stope until tsh is become normal Same the above case but : Incrase dos and after 3 wk/ incresea and follow after 6 wk Old patient presented with Ear pain ,headache , hem paresis>> most likely cause: Epi dural abscess Spinal abscess Subdural hematoma

All are 1ry prevention of anemia except: _ health education about food rish in iron _ iron fortified food in childhood _ limitation of cow milk before 12 month of age _ genetic screening for hereditary anemia _ iron,folic acid supp. In pregnancy and postnatal

| P a g e

Patient is presented with hand cellulitis and red streaks in the hand and tender axillary lymphadenopathy. This condition is more likely to be associated with: a. Malignancy b. Pyoderma c. Neuropathy d. Lymphangitis 45 male pt, will undergo dental procedure, he had a Hx of LBBB, but there is no structural abnormality, and ejection friction rate in normal range, no hx of arrhythmia, the good next step: Avoid hard dental maneuvers , give Abs before, give Ab after noneed for prophylaxis???. there is an interaction betweeen CARVIDILOL and: a:warfari n b:digoxi n c:thiazide Patient with chest pain that aggravated by coughing, there is added sound on left sternal border .in ecg you will find St changes Pr prolongation Hypervoltage pt with recent Hx of URTI , develop sever conj. Injection with redness, tearing , photophopia , So, what is TTT ? a)Topical ABx b) Topical acyclovire c)Oral acyclovire d) Topical steroid this is a viral infection treatment is supportive and u give Abx to prevent
| P a g e

superinfection drugs are CI in HCM?? hypertrophic cardiomyopathy 1.digoxin. 2.b-blocker. 3.ASA.

regarding paracetamol toxicity: - Not toxic if dose exceed 150-180 mg - Cause vomiting and neuropathy ?? - Therapeutic effect after 4 hours - Use Deferoxamine which of the following medication s associated wid QT prolongation?? A:chloropromazone b:clozapine: c:helopridol d:ziprasidon e which of the following is the most imp prognostic factors in CML : 1- stage 2-Age 3-lymphocytic doubling time 4-involvement of bone marrow A young girl pt had URTI 1 week ago & received septra (trimethoprime + sulphamethoxazole). She came with crampy abdominal pain & proximal muscle weakness. The diagnosis is: Polymyositis Gullian parre syndrome Intermittent porphyria Periodic hypokalemic paralysis Neuritis 33 year old make C/O of pain in his lip and right check . Pain was stabbing like
| P a g e

triggered with touch. O/E, cranial nerves were intact , The best Rx Is: aOxcrazepine b- Propanol cErgotamine d- Lithium Med 2 Pic of psoriasis, how to prevent flares . Avoid trauma The ACTH stimulation test is a medical test usually ordered and interpreted by endocrinologists to assess the functioning of the adrenal glands stress response by measuring the adrenal response toadrenocorticotropic hormone (ACTH)

Young male, diagnosed with MITRAL REGURGE by auscultation , want to do dental , what to do : ECHO Male m diagnosed with mitral prolapsed, echo free, want to do dental work , what to do: Nothing man use saldinafil (Viagra), to prevent hypotension you should not use a-nitrate b-B blocker c-ACIE d- CCB Which of the following is the best treatment for Giardiasis: a.Metronidazole Pt presented to ER with substernal chest pain.3 month ago, pt had complete physical examination, and was normal , ECG normal, only high LDL in which

| P a g e

he started low fat diet and medication for it. What is the factor the doctor will take into considerations as a risk factor: a. Previous normal physical examination. b. Previous normal ECG. c. Previous LDL level. d. Current LDL level. e. Current symptom. Elderly patient presented by SOB , rales in auscultation , orthopnea, PND, exertional dyspnea, what is the main pathophysiology : A. Left ventricular dilatation . B.Right ventriculardilatation . C. Aortic regurgitation. D. Tricuscpid regurgitation . Old male c/o sudden chest pain, decreased chest wall movement, hemoptysis , ECG changes of S1 Q3 T3 , what is most common diagnosis: b. Polmunary embolism Most common cause of immediate death in burn:--> Inhalational injury.

Case of old male, heavy smoker, on CXR there is a mass , have hyponatermia and hyperosmolar urine , what is the cause: a. Inappropriate secretion of ADH. b. Pituitary failure. Young male, have seasonal sneezing, rhinorhhea, conjunctivitis, what to give:--> . Antihistamine

| P a g e

Elderly patient presented by SOB , rales in auscultation , high JVP , +2 lower limb edema , what is the main pathophysiology : Right ventricular dilatation . ptn with chest pain and SOB , decreased by leaning forword . O/E friction rub and increased JVP >>>> (a case of pericardial effusion) ECG will show: ST changes first one appear Lowvoltage which prevent or decrease incidence of getting post herpetic neuralgia Varicella vaccination 50y female with breast cancer and CA125 elevate. So elevation due to a-breast cancer b-associatewithovarian cancer c-due to old age d-normal variation what is the effective half life of fluxetin ? a- 2 hours b- 18 hours c- 2 days d- 6 days e8 days patient having chest pain radiating to the back, decrease blood pressure in left arm and absent left femoral pulse with left sided pleural effusion on CXR, left ventricular hypertrophy on ECG, most proper investigation to dx: aortic angiogram((my answer)) 59 y/o presented with new onset supraventicular tachycardia with palpitation,no Hx of SO Or chest pain ,chest examination normal , oxygen sat in room air = 98% no peripheral edemaOthers normal, the best initial investigation: Thyroid stimulating hormone a young girl experienced crampy abdominal pain & proximal muscular weakness but normal reflexes after receiving septra (trimethoprim sulfamethoxazole) :--> polymyositis
| P a g e

which drug can cause hirsutism danazol which prevent and decrease incidence of getting post herpatic neuralgia acyclovir

Patient has symptoms of infection, desquamation of hands and feet, BP 170\110 dx: Scarlet fever Free fluid accumulate in abdominal cavity cause: Hypovolemic shock an 69 year old non diabetic. with mild hypertension and no hx of Coronary heart desease. the best drug in treatment is. thiazides before 14 d the child was bite ,now develope lip swelling eryrhema ... , what type of hypersensitivity ? a/type 1 b/type 2 c/thype 3 d/type 4 female patient came with fatigue and Jaundice. her CBC shows WBC =9 HGB= 9.5 ,PLT= 200 and his LFT show totoalbilirubint =3 , direct = 0,9 ... wahat is the most liklyDx : a/Dubin Johnson syndrome b/Gilberts syndrome C/primary scelerosing cholangitis D/criglernajjar syndrome type 1 Pt with sever asthma , silent chest what is next step ? Iv theiophilen Scenario of trauma , on face examination there is shifted mouth angle, loss of sensation of anterior third of tongue, which CN is affected: Facial nerve
| P a g e

Most commonest site of lichen planus: extensor area 40 yr old male presented to ER with 6 hr hx of severe epigastric pain, radiating to the back like a band , associated with nausea . No vomiting , diarrhea. No fever . On examination he was in severe pain & epigastric tenderness. ECG was normal, serum amylase was 900 u/l, AST & ALT elevated double the normal. Which of the following is the LEAST likely precipitating factor for this patient: Hypercalcemia chronic active hepatitis chronic alcohol ingestion hyperlipidemia cholelithesis --------------------------------------------------------------------------------------------Pediatric - Child with fever and runny nose, conjunctivitis and cough then he developed Maculopapular rash started in his face and descend to involve the rest of the body: a. EBV b. Cocxaci virus c. Rubella virus d. Vaccini virus child with asthma use betamethazone, most common side effect is a-increase intraocular pressure b-epilepsy c-growth retardation baby with streptococcal pharyngitis: - Ttt after 9 days carries no risk of GN - Ttt effective in prevention of GN - Clindamycin effective against gram ve organisms Infant with bright blood, black stool and foul smelling stool. Best way to know the diagnosis:
| P a g e

a) US b) Radio Isotop scan c) Angiogram d) Barium meal This is a case of melena (black foul smelling stool \ digested blood ) along with fresh blood or multiple sites bleeding of GIT so THE BEST is to do localization either by radio isotop scan ( the preffered in this case) or angiogram .. Radio isotop scan used in case of slow blood loss .5ml\min Angiogram for rapid blood loss 1-2 ml\min .. Stool analysis never to be the best , it will detect the presence of RBCs but not its source OR the presence of WBCs or antigens BUT based on the scenario IT IS NOT LIKELY TO BE AN INFECTION and even no diarrhea

child with hyperemia and pulging of tym mem had previous history of treated impetigo so ttt is: - Cefuroxime - Amoxicillin - Erythromycin - Ceftriaxone - Cephalexine

malaria in a child: a- crescent shape gametocyte of vivex is diagnostic in the stool b- the immediate ttt primquine for 3 d c- 72h tt t of malaria is suffeceint d- the most common cause is falciparum

11- scaly purpule lesions in the face of a child the cause a- staf. Aureus b- beta haemolytic srept.coci c- H. influenza
| P a g e

12- child >90% of the normal . < persentile hight with sever bowing of legs what help u for diagnosis: a- lower extremeties x-ray b-pelvicx-ray c- cbc d- alkaline phosphatase

Child with wheezing couph dyspnea with recurrent symptoms presented this time with same symptoms plus hemoptysis chest bilateral infiltration and sputum analysis show blood recently shifted from breast feeding to cow milk hx of dermatitis immediate management : Sodium cormoclgate Corticosteroid Antibiotic Milk free diet Heiner syndrome (HS) is a food hypersensitivity pulmonary disease that affects primarily infants, and is mostly caused by cow's milk (CM). The symptoms were in the form of cough , wheezing, hemoptysis, nasal congestion, dyspnea, recurrent otitis media, recurrent fever, anorexia, vomiting, colic or diarrhea, hematochezia, and failure to thrive. All had radiologic evidence of pulmonary infiltrates. Main treatment avoidence of trigger 6 years old child presents with straddling gait and in ability to stand or walk without support, he is irritable with vomiting 3 times, he has a history of chickenpox 3 weeks ago. O/E all are normal except resistance when trying to flex the neck, what is the most likely diagnosis: Fradrich's ataxia Acute cerebellar ataxia Meningioecephalitis Gullian Barre syndrome

2 months old infant ,presented with vomiting after each meal ,50% percentile growth ,labs normal, management?? a- Reassurance + follow up b- Surgical referral c- Try PPIs d- CT abdomen
| P a g e

Baby presented with cellulitis in his face ,what is the most common pathogen causing cellulitis in age (6-24 months)? a- Staph aurus bSterptococu ss c-h influenza 8 years old boy , has a height of a 6 year old and a bone scan of a 5.5 years. DX? a) Steroids b) Genetic (constitutional) c) Hypochonroplasia d) Hypothyroidism ?? normal child ,he want to walking , he have brother dead after walking , what of the following must be excluded before walking ??? a-PDA b-VSD C-hypertrophic cardiomyopathy !!! D-!!! child >90% of the normal . < persentile hight with sever bowing of legs what help u for diagnosis: a- lower extremeties x-ray b- pelvic x-ray c- cbc d- alkaline phosphatase 4 y/o boy felt down his mother pulled him by his arm & since then kept his arm in pronation position what is your management: A) Splint B) Do x-ray for the arm before any intervention C) Orthopedic surgery -child with hematuria 15 RBC /hPF , all examination normal ,what is next : a-urine cytology b-cystoscopy c-renal biopsy d- repeat urine for RBCs and protein
| P a g e

9year old boy cam to PHC with URTI and swap was taken and sent home, after 5 days the result was Group A streptococcus and then you called the family and they told you the boy is fine and no symptoms whats you next step: A- Give Ceftixim IM one dose B- Penicillin for 7 days C- Penicillin for 10 Days D- Do Nothing Ped 2 9 day old infant , presented to well baby clinic, with mild jaundice and yellow scaling on face and chest, otherwise examination normal , on breast feeding, doing well according to mother, what is the cause of his condition: Brest milk jaundice 5 y/o girl , presented with sore throat, and serosangious vaginal discharge: Streptococcus infection k/c of SCA, have URTI, then suddenly have chest pain, lopar infiltrate, WBC 18000, Hg 7 , fever what is the cause for his condition: PE. Young boy , k/c of SCA, had UTI and ttt well, what to give prophylaxis : Amoxicilline Child had chest tightness and cough when exposed to cold and excersise, what to give for prophylaxis : a. B2 inhaled agonist, b. Steroid inhaler. c. Tehyophillin. d. Oral steroid.

a case of child drink corrosive material , hypotensive, pale , drooling, what to do: establish airway.
| P a g e

Child his mother let him to go to bathroom befor sleeping and avoid drinking befor sleep this mangment of: Enuresis What is the most common treatment for juvenile rheumatoid arthritis Paracetamol after bite, pediatric patient presented with abdominal pain and vomiting , stool occult blood , rash over buttock and lower limbs , edema of hands and soles , urine function was normal but microscopic hematurea was seen: a. Lyme b. Henoch-Schonlein Purpura

Child is ill with fever, abdominal pain & pass bloody mucus, obstructive pattern, next? barium enema Child fell on her elbow and had abrasion, now swelling is more, tenderness, redness, swelling is demarcated (they gave dimensions) child has fever. Dx: a. Gonoccal arthritis b. Synovitis c. Cellulitis of elbow 3 years old presented with shortness of breath and cough at night which resolved by itself in 2 days. he has Hx of rash on his hands and allergic rhinitis. he most likely had bronchial asthama What is true about rubella arthritis On examination of newborn the skin show papules or (pastules) over erythema base: erythema toxicum neonatorum
| P a g e

After doing CPR on child and the showing asystole: Adrenaline central line complicated by sepsis in child , what is the causative organism group B stept

Surgary 22 yr old with sudden shortness of breath with trachea deviation the firt step: a. Needle decompression in the 2nd intercostal space midaxillary line b. Needle decompression in the 2nd intercostal space anterior-axillary line c. Needle decompression in the 5th intercostal space midaxillary line d. Needle decompression in the 5th intercostal space anterior-axillary line pt Intubated ,the most reliable method to make sure for tube proper position: - 5 point auscultation bilaterally breathing heard - CXR RTA pt with femur fx , he has laceration of the femoral artery .. What to do : 1- end to end anastomosis 2- prosthetic graft 3-arterial graft 4- venous graft breast cyst which is green coulored on aspiration, what is the next step in management? throw the fluid away ,surgical excision of the cyst ,send for cytological examination lady with 3 cm breast mass, solid nontender, mobile, persist durimg menses, slightly increased n size: Fibroadenoma Fibrocustic Ductal carcinoma Papilloma A 70 YO woman presented with a 3 days hx of perforated duodenal ulcer.
| P a g e

She was febrile , semi comatosed and dehydrated on admission. the BEST ttt is: a) Blood transfusion, Rehydrate , perform Vagotomy & drainage urgently. ?? b) NGT suction, Rehydrate , systemic AB & observe. c) NGT suction, Rehydrate , systemic AB & perform Plication of the perforation. d) Rehydrate, Blood transfusion , systemic AB & perform hemigastrectomy. e) non of the above

which is true regarding peretonieal lavage: a- fresh blood on inspiration 2 ml. b - Rbc 1000. c-wbc 50. d blood 2 ml in pregnancy

55 ys old male pt, presented with just mild hoarseness, on examination: there was a mid cervical mass, the BEST investigation is: 1) Indirect laryngioscopy 2) CT brain 3) CT neck A 42 year old woman presented with a painful breast mass about 4 cm in the upper lateral quadrant. It increases in size with the menstrual period. Examination showed a tender nodularity of both breasts. What is the management: Hormonal treatment with oral contraceptive pills Hormonal treatment with danazol Lumpectomy Observation for 6 months

Surgary Old pt , right iliac fossa pain, fever for 2 days, diarrhea, on CT thickness of intestinal wall , what to do : colonoscopy Blow out fracture : Diplopis and upward gaze
| P a g e

Old with rectal bleeding, external hemorrohide, what to do: Colonscopy. Young aged male presented to ER after blunt trauma to Abdomen, CT scan shows intramural hematoma: your management is--> Observation Gastrictomy post-op 1 day. He have temperature 38.8 & pulse 112. What is the most common cause inflammatory mediator in the circulation In cervical LNs there are well differentiated thyroid cells, during operation you find no lesion on thyroid what will you do next--> Total thyroidectomy + specific LNs dissection Old patient male, presented with acute hematuria, passing red clots and RT testicular pain and flank pain : RCC ( renal cell carcinoma ) a wound stays in it's primary inflammation untill a. Escher formation b.epitheliazation c. after 24 hours d. wound cleaning Right upper quadrant pain and tenderness , fever, high WBC , jaundice, normal hepatic marker . Acute cholecyctitis

Obs & Gyne During the third trimester of pregnancy , all of the following changes occur normally except Decrease paco2 Decrease in wbcs Reduced gastric emptying rate Diminshed residual lung volume Diminshed pelvic ligament tension Pregnancy in the final month and labor may be associated with increased WBC levels.
| P a g e

all true about engagement except: - 2/5 fetus felt in the abdomen - Head reach the ischeal spine - Biparital diameter pass the pelvic inlet - Crowing is present A diabetic pregnant with HX of fetal demise .. now is having a fetus who is healthy and her DM is very well controlled .. you will allow her for: a) C/S at 38 weeks b) induction at 36 weeks c) allow SVD hepatitis can be confirmed in pregnant lady by elevation of : ESR ALP WB C SGOT Pt had spontaneous abortion what is the correct answer ? 1-Must do cervical exam to confirm . 2- common cause of infertility . 3- occur mostly in 2nd trimester Which of the following is true regarding antepartum (third trimester ) hemorrhage : a- Can be caused by polyhydrominos b- Rare to be associated with hypofibrogenemia c- Cervical problems are a major cause 38 yrs old female came to you at your office and her pap smear report was unsatisfactory for evaluation .. the best action is a- consider it normal & D/C the pt. b- Repeat it immediately c- Repeat it as soon as possible d- Repeat it after 6 months if

| P a g e

considered low risk e- Repeat it after 1 year if no risk A placenta that's positioned on the antero-lateral wall of the uterus, can't be reached by finger through cervical examination: Low lying placenta Normal lying placenta Marginal placenta previa Partial placenta previa Pt G3 P3 all her deliveries were normal except after the second one she did D&C,All of the examination normal even the utrus, labs all normal except : high FSH, high LH, low estrogen DX : a- Asherman syndrome b- Ovarian failure c- Turner syndrome a 38 yrs old female came to you at your office and her pap smear report was unsatisfactory for evaluation .. the best action is a- consider it normal & D/C the pt. b- Repeat it immediately c- Repeat it as soon as possible d- Repeat it after 6 months if considered low risk e- Repeat it after 1 year if no risk

| P a g e

female c/o colorless itching vagina , her partner c/p urethral disch. . Cervical examination : shows strawbery spots meconazole cream estrogen cream progesteron cream douch A 54 YO female with chronic pelvic pain is found to have a right sided ovarian mass. After the initial evaluation, surgery is planned to remove the mass. To avoid excessive bleeding during the surgery , the surgeon should ligate which of the following structures? A) Round ligament B) Suspensory ligament C) Ovarian ligament D) Transverse Cervical ligament E) Mesosalpinx pregnant has glucosuria also by GTT confirmed that she has gestational diabetes what u shold do a- repeat GTT? b- Take a1c hemoglobin c- take fasting blood glucose

SLE Recall , By Dr. : Eyad Alhudaithi -KKU


Basicsa Poor wound healing with diabetics is d/t : immunityy
| P a g e

Structure not palpaple = LN .9% NS has 155 Na Rubella incubation 12-21 d streptococcal pharyngitis = TTT to prevent RF(penicillin for 10 days ) ,, no TTT to prevent GN ! leshmania vector = Sand fly antimitochondorial antibodies = primary biliary cirrhosis G-ve diplcocci = n gonnerhea carpopedal spasm, which was reproducible by inflating a blood-pressure cuff placed on the patient's arm. Chvostek's sign, the twitching of the circumoral muscles with tapping lightly over the facial nerve = HYPOCALCEMIA celiac dz .. has nothing to do with = rice and corn brainstem damage = increase pco2 (reps depression) + no doll eye mov loss of rugeal folds at stomach and infiltration of B lymphocyte = H.pylori Medicine Hyperthyrodism comp >> bone scan to r/o osteoprosis Hypothyrodism A/W = elevated cholestrol Smell = fronatal lobe Dendritic cell = HSV CSF shows high igG with band like = MS True abt malaria = MC is falciparum Initial sign of ICP = decrease conscious level Rapid and best for complete gastric evacuation = charcol Pt with so many risk factors ( MI , HTN , DM LDL ) the worst is = LDL Increase COPD survival = oxygen ( if smoking not present ) COPD not responding to bronchodilator , what to add = aminophylin COPD MCC of pneumonia = H.I pneumonia MC place = Right middle lobe Most specific test for syphlis = TPHA ... Sensetive = VDRL Fever + PAINFULL HSmegally = viseral lishmania Dysphagia + diplopia + fasiculation = motor nuron dz Pt wit polycythemia vera has pruritis after bath = d/t histamin release
| P a g e

Pt with hemoptysis and has rosy cheeks = Mitral stenosis Most difficult mode of transmetion to be prevented = airway RA pt >> applying cold comp is the best Status epi >> give diazipam no responce repeat no responce phyntoin IV Premature VC = decrease O2 supply to the muscle Bi pneumonia + high grade fever but normal WBC = - Chlamydia pneumonae ptosis = 3rd N P Pt with Urethritis then arthritis = Riter dz pt with H/O RF now is having sore throat .. chance of reinfection with RF = 50% acute knee pain and swelling = Arthrocentasis ( always aspirate if acute ) most specific test to diagnose pt with post-strept acute GN= low C3 Aortic stenosis >>> valve replacement is ttt of choice Saddel nose , hemptysis , hematouria = Wegners G Most serious comp of CO poisoning = Arrythmia pt was treated as pneumonia after starting abx become complete opacification "hydeothorax" .. causative organism = Staph a organism invade intestine n LN n causes bacteremia = shigella facial palsy ( LMNL) = start to improve in 2ed week idiopathic autonomic insuffeciency = orthostatic hypotension Erosive gastritis >> happen within 24hrs of injury Post MI now with abd pain and bloody diarrhea = Ischemic colitis scenario of HEART F .. most common cause of heart failure = CORNARY ARTERY DZ MCC of
| P a g e

excessive day time sleepiness = sleep apnea quastioneer to dfr btw central and obst sleep apnea = Epworth OA TTT .. Initially = Quadriceps strengthening exercise pain and swelling at first metatarsophalyngeal joint = Na urate crystals IV abusr came with splinter Hge, koplic spot on retina = EI LMWH (Enoxaparin ) used for DVT and PE speacially for obese and pregnant Hiv wit skin lesion = Kaposi Drug that delay need of surgery with AR = nifdipin only live vaccine for HIV pt = MMR ASA +clopidigrol for = Acute MI UC . dx by (colonscopy) MC extraintestinal manifestation = osteoprosis Hemochromatosis = Tranferrin EBV Rx=supportive ... Diabetic with black nasal discharge = mycomyosis increase show size = Somato anti TB (ISONIAZIDE) >> numbness = pyridoxine (B6) infection, desquamation of hands and feet, BP 170\110 = Toxic shock syndrome (s.aurus ) or scarlet fever !! hodgkin =red strunberg chromosm 16 = POLY CYCTIC KIDNEY Adult polycystic kidney disease is inherited = AD oral+genital ulcer +arthritis = Bahcet dz when to give FFP = DIC hemochromatosis = tranferin S/S = DM , liver

| P a g e

kawasaki = itis + strwebarry tounge Rx = ASA , IVIG CF = Long arm chromosome 7 progressive weakness of eye and swallowing = myasthenia gravis (antibody against acetylcholine receptors) SEIZURE AND ELECTROLYTE =ALL hypo exempt NA both hypo n hyper serum Ferritin is the best screening test of = IDA and iron overload (hemochromatosis) Coxsackie A virus = Hand -foot-mouth sx proximal muscle weakness = polio , hypothyrodism , Polymyalgia rheumatica best excercise with CAD = anaerobic complication of sleep apnea = CHF male with thrombosis = nephrotic sx .... Female = antiphospholipid " Brain stem death = no doll eye mov. + Resp F BUlemia = hypokalmemia and met alkalosis pseudo gout = CACO3) diet distribution (50%carbs , 30%fat 10%protein Alkalosis = Hypo K & Hypo Ca++ . acidosis = opp Hypothyrodism = elevated cholestrol if Mg low = low K & low Ca . correct Mg first MCC of gastroenteritis in Adult = Norwalk virus MCC of secretory diarrhea = E.coli Dysentry & HUS = Ecoli coarctation of the aorta = upper limp HTN Hoarsness of voice and stridor can occure with mid facial injury = Head n neck injury bullimeia has (Hypokalemia , Met Alk Rt lung has 3 lobes & 10 segments (medial basal segment only in inf. Lobe of Rt lung) Lft lung has 2 lobes & 9 segments (Lingular segments in lft lung)
| P a g e

psudobulbur = LMNL (DM) CAD best excersize = Anaerobic isometric = muscle strengthening and increases bone density was for osteoporosis RBC casts =GN epithelial cells = Infection (Kidney) in case of ischemic stroke <3 hrs = TPA up to 48hrs = ASA legionella : grame negative rod growth on charcoal agar Active form of TFT = T3 hyper ca in hospital = malignancy ambulatory = hyyper PTH Dxa scan of osteoprosis = -2.5 Metaphysis = osteomylitis fecal fat +ve = chronic pancritits (malabsorption ) Ankylosin spondylitis = uvitis honey comb crust = staph aurus , imptigo ! Drugs Givin with iron = vit C best Rx from chronic pain mx = acetaminophen Used to decrese hot flush post M (non hormonal ) = Paroxitin Mythesegide = Migrain headache Prophylaxis .. Migrain Rx = sumatriptan .. HTN and migrane ttt = BB drug that causes HTN crises if not stopped gradually = clonidine L-dopa S/e = Dyskinesia psychotic pt on medication developed rigidity and uprolling eyes , afebrile = tradive dyskinasia fluxtin half life = 7-9 days rx of dementia = galantamine applying of eye drops = 1 at lower fornix Rosacea = deoxyxyclin .. scabies = permethrin .. tenia= azole .. acne = retnoic acid GOUT ; acute attack = NSAID = colchcin prophylaxis = alluprinol Any carrier state ( S.A ,, meningitis , H.influnza ) = rifampin
| P a g e

SVT = Adenosin salisalate = Asprin = NSAID = cycloxgenase = RESP ALK + MET Acidosis Trigeminal Neuralgia = Carbamazepine silver sulvazide SE = leukopenia Clonidine = HTN crises Zolpidem drug for insomnia Isoniazid causes pyridoxine(vitaminB6) deficiency = priphral nueopathy Vitamin A is a known teratogen. (ACNE) ASA SE on kidney = Renal papillary necrosis organophesphorus toxity (antidot = Atropin HTN .. wt reduction is the most import an factor in lowering BP OCP SE = obst juandice cimitidine = protect stomach from analgesics SE as NSAID Don't give LEvedopa with digoxin Clonidine = decrease Levedopa

| P a g e

lithium don't affect liver Anti coagulant // MI for 6 wks DVT for 3monts ,, recurrent for 6monts galantamine = demntia magnisium sulfate toxicity = loss of deep tendon reflex mashrom toxity = halusitnation iron overdose = Gastric lavage phenobarbital and lactation = continue and monitor baby (to avoid withdrawl symptoms ) MOA of Asprin = inhibit cycloxgenase Warfarin ..... >7 stop for one day ... >10 stop < 7 monitor and adjust saldinafil (viagra ) = avoid with other vasodilator as nitrate Heparin = RX DVT ASA & Calopidogril = AMI chlamydia = dyocycyclin MCC of vit K def = antibiotics which anti epiliptic causes alopecia = na valporate Silver Sulfadiazine SE = neutropenia thiazide SE = hyper urecemia (gout ) alluprinol = Reduce the chance of uric acid stone ! prophylaxis (contact ) of mengntits = Rifampcin ... but for pregnant give Ceftriaxone more than 50% of patients taking anticholinergic have side effects : dry mouth, blurry vision, constipation and urinary retention. bacteroid =clindamycin drug C/I with hypertrophic obstructive cardiomyopathy = Digoxin Yellow disc of vision= digitalis toxity Aluminum salt will decrease absorption of = Tetracyclin iron overdose Rx = desferrioxamine dilated pupil , hyperreflexia , convulsions , strabismus (which toxity ) = TCA zolpidem = DOC for insomnia , if not there diazepam
| P a g e

pt developed Fever , tacky , rigidity , he's taking odansetron (anti emetic ) = Nueroliptic malignant Sx (FARM) drug cause hypertensive crises when used with tyramine = MAOI What drug reverses the effect of Benzodiazepines ( as in Resp depression ) = Flumazanil Anticholenrgic s/e = blurred vesion Propylthiouracil= inhibit Thioperoxidase SVT = adenosin pE >> heparin .. DVT prophylaxis = LMWH (Enoxaparin) ASA + clopidigrol = after cath Gonorrhea Rx = ceftriaxon Giardia >> metro , salmonella >> ampicillin , shigella >> tri/meth ,, vibrio >> tetracyclin , c.dif >> metro or vancomycin Hiccup Rx = cholpramzide Antihistamin prophylaxis is to = decrease inflammation Chlamydia Rx = doxycyclin ( if preg give erythromycin ) cold induce urticaria = diphenhydramine (Antihistamin) what drug causes heat-stroke as it inhibits sweating = Hyoscamine Sulfate phenobarbitone SE = hair growth Valporate = alopecia antiviral drug causes fever and muscle pain = Interferon asthma Ex with silent chest >> aminophyline drug increase incidence of reflux oesophagitis = Thiophylline which BB has alpha blocking affect = labetalol and carvidlol drug C/I in preg = Glicpnamide (sulfanyuryase ) Cotrimox ( trimeth /sulfa) , dyoxycyclin , Na valporate , phynetoin , warfarin ,
| P a g e

Surgery Irrigular border and color = melanoma Papillary thyroid ca >> surgery Burned pt die d/t = smoke inhalation Mc position for hand bite inj = fist bilateral hydronephrosis d/t = Prostate enlargement 1st sign of transplant rej = Fever appendicits best to dx by = WBC C/I in acute cholysystits = morphin MCC of chronic intermittent rectal bleeding = homorrhoid face suture to be removed >> 3-5 days Best inv to visualize the cystic breast masses = US POst OP fever .. if day 1 = atelecsis Parkland formula = r*KG*BSA% (1/2 at first 8 hrs ) OPen frx Rx= 1st G.C + Aminoglycoside(gentamycin) + Metro + tenitus DVT = Anticoagulant(LMWH=enoxaparin) for 6months AAA = CT (G.S) undescnded testes = surgery 6-18m

| P a g e

corkscrew appearance = Diffuse esophageal spasm bird peak = Achlasia SAH = ruptured aneurysmS/S : worst headache ever .. intracranial = HTN .. truma = epidural (cresent ) subdural (LEns ) Aortic Anyurism = CT indirect inguinal hernia = sac lies Anteromedial to cord medial leg ulcer = Venous ,, Mx. compression Black head = Surgery acute pancreastits = naso j tube pnetrated wound = unstable (lapratomy ) stable (CT) painful rectal spasm , diaphorisis , tachycardia especially at night , DX coloctomy .. when to F/U = after 3 m CT reveals Intramural hematoma after blunt abdominal truma = observation OB/GYN Girl , no female ccc , amonarhea = lyding cell tumor For pregnant with pre existing HTN dont use ACEI ... Use methyldopa a2 methyl progesterone C/I in = pregnant with HTN Cervical Ca >> Cone Bx Cyst on US 4-8cm = follicular cyst screening for down = triple test OCP MOA is = inhibit mid cycle gonadotropins and so ovulation OCP = Decrease ovarian Ca hx of amonarrhea + Fluid in Pouch of Douglas = rupture ectopic pregnancy pregnant with UTI = nitrofurantoin MCC of maternal mortality = Toxoplasm ? pregnant developed a sudden massive swelling of the left lower extremity = Impedance plethysmography, bed rest, heparin most dang of Post Menop = osteoporosis pregnant with fibroid has abd pain >> pain mx. Mg sulfate toxity = depressed deep tendon reflex
| P a g e

Pre eclampsia = protein Uria could accompany preexisting HTN or gestational HTN (>20wks) preg 16wks with high blood p = chronic HTN 1st thing when investigating couple with infertility - semen analysis Lactational mastaitis = doxycycline female with recurrent UTI >> increase fluid intake OCP that causes hyperkalemia = drospirenone Acanthosis Nigricans a/w PCO cardiac drug C/I in preg (warfarin-ACEI-statin-Amidarone-Na nitrate ) metronidazole (use with caution in preg ) >> preterm labor Flucanazol >> C/I in preg gonorrhea isolation of organism from = cervix Yeast ( candida albican ) (vulvovaginitis ) ( Diabetic ) = flucanazol PO , Nystatin vaginal discharge , KOH test , fish smell = Bacterial vaginosis ... RX=metro Vaginal trichomoniasis = protozoal "trichomonas vaginalis" = pruritis , treat her couple , green discharge , strawberry vagina pregnant has chlaymedia .. terato = conjuctivitis + otitis M .. Rx. (erythromycin ) if not pregnant = dyoxycyclin female with H/O ovarian cancer .. now has palpitation = strum ovari HIv positive pt (pap smear ) = if it comes negative at first screen after 6 months then after one year acyclovir safe at prgnancy 48 , Female , condom , dark cervix , missed 2 cycle >> B HCG postpartum Psychosis = Recurrences are common in subsequent pregnancies mode of delivery C/S = HIV & HSV snowstorm on US = Complete molar which drug can cause hirsutism = OCP ! highest maternal mortality in pregnancy = hyperbilirubnemia (HELLP sx) 100 300 Kcal / day , 500 Kcal / day in breastfeeding
| P a g e

Wt. gain : 1 1.5 kg / month , 11 16 kg gain during pregnancy. hysterectomy : MC comp = ureTral inj M serious = Hge chlamydia = Doxycycline -- pregnant give erythromycin ALP can rise in pregnancy also ESR while High BUN and Cr = renal F at ischial spines = station 0 = engaged above ( 5 1) or cm below (+1 +5) we say head IS Engaged if 2/5 of head is palpable in the abdomen Cr ,BUN , GFR increase normally Breast
| P a g e

self breast exam = 2-3 days after period (monthly ) MC tumor in those <35 = fibroadenoma if related to menses = Fibrocystic changes younger than 50 , has boldly discharge , no masses = Intraductal Papiloma MC uni breast cancer = infiltration ductal ca .. bi = lobular young had painfull lump that increase with menses = Fibrocystic changes menses pain = NSAID recurrence of breast cancer = # of LN painless Breast lump ,, US reveals cystic changes , FNA reveals clear fluid = reassure Pedia infant = 1y .. newborn = 1wk .. neonate=28days bk pain wake kid from sleep = Kyphosis breath holding (spells ) = RF for genrlized convulsions hemangiomas obstructing his vision = surgery. after 1 week Gout milk = folate def Spasmadic Croup = Croup = Atopc dermatitis precocious puberty MCC = idiopathic child has unilateral flank mass + HTN = willms spells >> Convulsions PDA = MC congenital Heart dz in premature give indomethacin (PGE) to keep it patent ACyanotic H dz = VSD , ASD , PDA Cyanotic = MC (TOF) : pulmonary stenosis , Rt.V.hypertrophy , VSD, over riding of Aorta .. on Xray ( boat shape) least congenital heart dz to have RF = ASD "secundum " MCC of microangiopathic anemia , thrombocytopenia and renal failure in chilldren = Ecoli
| P a g e

Blood sugar in DM type 1 is best controlled by = Basal and bolus insulin . Mimic croup = FB Duchenne Muscle Bx muscular dystrophy scenario with positive Gower's sign >> Infant just introduced to cow milk now has pulmo signs = hiner dz 1st few words = 18 months DM1 , hypoglycemic and comatosed >> IM Glucagone intussusception >> Barium enema apgar score 3 >> ventilation ( has to be above 7 ) Child with recurrent UTI how to counsel him = increase fluid intake best inv to exclude UTI comp = Kidney US don't give DPT if pt has >> Unusual cry Preterm at NICU >> give Glucose infusion child HIV positive .. C/I vaccine is = Polio Hx of URTI ttt with septra(Trimethoprim and Sulfamethoxazole) then came with and pain and px muscle weakness = intermittien porphyria tebial tubercle pain .. in 13 y/o boy having growth spurt = Osgood frx (same Q )boy after running for hours , has pain in knee and mass on upper surface of tibia a.= Osgood scatter disease Derma Big ulcer .. Red .. Raised edge >> bx Lesion clear at center red in p at leg = tinea corpora Erythema multiform : has target cell , itching , appears at night Maculopapular reash then vesicles = HZV = varicella = shingles ( all the same ) Acne vulgaris rx = oral isotritoin fever + desquamation, one of the most distinctive features of = scarlet fever. hypopigmented macules at chest and arm "Pityriasis versicolor " RX = Na selinum scabies (Burrow ,
| P a g e

itching ) = permethrin dewdrops , vissicle & Crust = cknpox rash that appears pink on wood light = Erythesma satellite lesions = candida honey comb crust lesion. Culture showed staph aureus. The diagnosis is= impetigo Ophtha Pteregyium = surgery UVITIS Rx. = cycloplegic + steroid After removing a fly or FB = topical Ab subconj Hge >> reassure Retinal pigmentation = Retinal V thrombosis RD = more in far sighted DIabetic retinopathy = control HTN and Obesity (Smoking not related ) painless loss of vision + cherry spot = retinal artery occlusionn pilocarpine = Glucoma vertical Diplopia = 4th neve (trochelar ) DIabetic retinopathy = control HTN and Obesity (Smoking not related ) Ortho Inj at L4 = inable to dorseflex Child , his legs twisted n fell down , Dont want to walk = spiral tibial fx +ve leckman = ACL injury pain in anatomical sniph box = scavoid fracture

| P a g e

fracture

| P a g e

+ve valgus stress test = MCL tear index finger in flexed position (DIPJ)= mullet finger posterior hip dislocation = the hip is flexed ,internal rotation,adducted radial styloid tenosynovitis,Finkelstein's test all means De Quervain syndrome .. >>> Thumb spike Psychiatry Mania A/w granducity Onco Hodgkin L Reed-Sternberg = nodulur sclerosis (always the scenarios is act this type ) Thyroid ca a/w = euthyroid All penia but leukocytosis (leukemia)+ blast cell (acute ) +ve peroxidase = AML high risk CLL = Age malignant tumor of parotid in childern = mucoepidermoid ca PSA is the only tumor marker used for screening (prostate) MC histo in BReast ca = invasive ductal ca. Tamoxafin = Rx of Breast ca CEA = F/U colon ca .. b4 and after surgery MC brain tumor of adult = glioma child = cerebellar astrocytoma cancer with 8th nerve pals = acoustic neuroma "shwanoma" thyroid cancer : TFT (best is TSH ) . hot nodule >> not cancer >> U/s & FNA Adenocarcinoma = MC primary cancer in lung
| P a g e

tumor that causes Cushing (ACTH) and hypo NA (SIADH) is = Lung ca AFP and BhCG rise in which tumor = testicle cancer prevented b vaccin = HCC MC cancer of paranasal sinus = maxillary PSA is the only tumor marker used for screening (prostate) High risk for colon ca = Low fiber high fat And gardner syn. young male and colon cancer = Gardner sx genetic predisposing of colon cancer = give folic acid MC cancer in children = ALL MV intra abdominal = willms

Family Specifity = negative of disease, and test is negative seNsetivity = Positive in dz . sPecifity = -ve in health Cohort (prospective ) future . Case control (cross sectional ) = ( retrospective pandemic= worldwide endemic = constant presence of dz at specific area epidemic = outbreak of dz at specific area ENT
| P a g e

16 y/o female become deaf suddenly.. her mother was deaf when she was 30.. Dx: = Otosclerosis (AD) Maxillary sinus best Dx = CT perforated tympanic membrane ttt = Systemic ABX Waking up from sleep..cant talk, no fever, can cough, normal vocal cords = Functional aphonia which one causes conductive hearing loss = OM OM exept = no HIgh fever difficult breathing from one nose , erythtmotous swelling = Steroid MCC of ear infection = meningitis or Mastoditis !! Ludwig's angina MCC of death >> Sudden asphyxiation ( &%# $ " ! + & %-./ /! ) Studying SLE together

| P a g e

| P a g e

SLE Collection from a different SLE Exams 2012,collected by : Dr.Samia Khan


Medicine 1- a 58 yrs. old male pt. came with HX of fever, cough with purulent foul smelling sputum and CXR showed : fluid filled cavity the most likely diagnosis is : a- abscess b- TB c- bronchieactesis 2- One of the major factors causing physiological hypoxemia is: ventilation-perfusion mismatch. decreased diffusional capacity of the alveolar membrane. increase in the level of 2,3 DPG. 3- 55 years old male patient presented with cough for 10 years which did not bother him much, it is productive of mucoid and purulent sputum alternatively, Hx of excessive smoking for 23 years. He is obese 123 kg. He was wheezing during talking with you. On examination you find rhonchi allover his chest, the most probable diagnosis is: a. Chronic bronchitis. b. Emphysema. c. Pneumothorax. d. Cystic fibrosis. e. Bronchiactasis. - 55-year-old male presented to your office for assessment of chronic cough. He stated that he has been coughing for the last 10 years but the cough is
| P a g e

becoming more bothersome lately. Cough productive of mucoid sputum, occasionally becomes purulent. Past history: 35 years history smoking 2 packs per day. On examination: 124 kg, wheezes while talking. Auscultation: wheezes allover the lungs. The most likely diagnosis is: A. Smokers cough B. Bronchiectasis C. Emphysema D. Chronic bronchitis E. Fibrosing alveolitis 4- Forced vital capacity: a. Volume of gas that can be expelled after inspiration in one minute. b. Volume of gas that can be expelled in the 1st second. c. Volume of gas that can be expelled after maximal inspiration. d. Maximal air flow rate in FVC. e. Maximal air flow in 1 second. 5- Asthma after 40 years old. What is true?: a. Could be psychological. b. Eosinophiles are increased significantly. c. Peak expiratory value change from night to day. d. Oral steroid change the peak expiratory value significantly. 6- Air Bronchogram is characteristic feature of: -Pulmonary edema. -HMD= Hyaline membrane disease. -Lobar Pneumonia. -Lung Granuloma. 7-ln moderate to severe asthmatic patient you find all except:
| P a g e

a. Decrease Po2 <60 b. PCO2 >60 c. HCO3 decreased d. IV hydrocortisone relief after few hours e. Dehydration - In moderate to sever asthmatic patient, you will find all the following except: a- P02 < 60 b-PCO2>60 c- low HCO3 d- IV hydrocortisone will relieve the symptoms after few hours. e- dehydration. - Regarding moderately severe asthma, all true except: - PO2<6OmmHg - PCO2 > 60 mm Hg ,early in the attack - Pulsus Paradoixcus - I.V cortisone help in few hours 8- The most specific investigation for pulmonary embolism is: a. Perfusion scan b. X-ray chest C. Ventilation scan d. Pulmonary angiography 9-A 30 year old male presented with Hx of left sided chest pain & shortness of breath BP 80/50. On examination left sided chest hyper-resonanse. The most likely diagnosis is: a- pneumonia with pleural effusion. b- MI. C. Spontaneous pneumothorax 10-history of recurrent pneumonia, foul smelling sputum with blood, clubbing: a. Bronchiactasis
| P a g e

11-Patient in ER: dyspnea, Rt sided chest pain , engourged neck viens and weak heart sounds , absent air entry over Rt lung Plan of Treatment for this patient: a) IVF, Pain killer, 02 b) Aspiration of Pericardium c) Respiratory Stimulus d) Intubation e) Immediate needle aspiration chest tub 12-Which of the following radiological features is a characteristic of miliary tuberculosis: A) Sparing of the lung apices B) Pleural effusion C) Septal lines D) Absence of glandular enlargement E) Presence of a small cavity -Radiological features of Miliary TB? a- Sparing lung apices. b -------------------------c- Septal line. d- No glandular enlargement e- Small cavity. 13- Which one of the following regimens is the recommended initial treatment for most adults with active tuberculosis? 1. A two-drug regimen consisting of isoniazid (INH) and rifampin (R ifad in). 2. A three-drug regimen consisting of isoniazid, rifampin, and ethumbutol MyambutoI). 3. A four-drug regimen consisting of isoniazid, refimpin, pyarazinamide and ethumbutol. 4. No treatment for most patients until infection is confirmed by culture.
| P a g e

5. A five-drug regimen consisting of Isoniazid, rifampin, pyrazinamide, ethu mbutol and ciprofloxacin 14- A 24-year-old woman develops wheezing and shortness of breath when she is exposed to cold air or when she is exercising. These symptoms are becoming worse.Which of the following is the prophylactic agent of choice for the treatment of asthma in these circumstances? A. Inhaled 2 agonists. B. Oral aminophylline. C. Inhaled anticholinergics. D. Oral antihistamines. E. Oral corlicosteroids. 15- 25-year-old man had fixation of fractured right femur. two days later he became dyspnic, chest pain and hemoptysis. ABG: pH 7.5 po2: 65 pCo2: 25 initial treatment is: A. Furosemide B. Hydrocortisone C. Bronchoscopy D. Heparin E. Warfarin

16- Which one shifts oxyheamoglobin dissociation curve to the left : a) Hypoxia. b) Acidosis. c) High altitude. d) None of the above.
| P a g e

17- All of the following are true about pulmonary embolism, except: a) Normal ABG. b) Sinus tachycardia is the most common ECG finding. c) Low plasma D-dimer is highly predictive for excluding PE. d) Spiral CT is the investigation of choice for diagnosis. e) Heparin should be given to all pts with high clinical suspicion of PE. 18- In mycoplasma pneumonia, there will be: a) A (+ve) cold agglutinin titer. b) Lobar consolidation. 19- The treatment of community acquired pneumonia is: a) First generation cephalosporin. b) Penicillin G + second generation cephalosporin. c) Erythromycin. d) Erythromycin + Gentamycin. 20- All indicate severity of bronchial asthma , except: a) Intercostal and supraclavicular retraction b) Exhaustion c) PO2 <6OmmHg d) P02< 60 mmHg +PCO2> 45 mmHg e) Pulsus paradoxis > 2OmmHg 21- a 25 year old pregnant presented with fever and sore throat (in flue season) then she developed non productive cough and dyspnea, she was extremely hypoxic, the most likely Dx: a) Staph. Pneumonia b) Strep. Pharyngitis c) Pneumococcal pneumonia d) Viral pneumonia
| P a g e

22-interstitial lung disease all of the following are true except: a) Insidious onset exertional dyspnea b) Bibasilar inspiratory crepitation in physical examination c) Haemoptysis is an early symptom d) Total lung capacity volume is reduced - Interstitial lung disease All true except: A-Insidious onset exertional dyspnea. B-Bibasilar inspiratory crepitations in physical examination. C-Haemoptysis is an early sign. D-Total lung volume is reduced. 22- Primary TB: a) Usually involves upper lobe of lung. b) Normal X-ray. c) +ve PPD test. d) None of the above. e) All of the above. 1- Pre-Renal Failure: a. Casts b. Urine Osm <400 C. Urine Na <20 mmol/L d. Decreased water excretion 2-Polycystic renal disease is: a. AD b. AS C. Sex-linked dominant d. Sex-linked recessive e. Autosomal dominant combined with... 3-Patient with Hx of severe hypertension, normal creatinine, 4g protein 24 hrs. right kidney 16cm & left kidney 7cm with... suggesting of left renal artery stenosis. Next investigation:
| P a g e

a. Bilateral ... renal angiography b. Right percutanious biopsy C. Left precutanious biopsy d. Right open surgical biopsy e. Bilateral renal vein determination 4 -Urine analysis will show all except: a) Handling phosphate. b) Specific gravity. c) Concentrating capacity. d) Protein in urine. 5- In acute renal failure, all is true except: a) Phosphatemia. b) Uremia. c) Acid phosphate increases. d) K+ increases. 1- When lactic acid accumulates, body will respond by: a) Decrease production of bicarbonate b) Excrete C02 from the lungs c) Excrete Chloride from the kidneys d) Metabolize lactic acid in the liver 2- What is the initial management of acute hypercalcemia? a) Correction of exter-cellular fluid (by adequate rehydration) - The first step in the management of acute hypercalcemia should be: A. Correction of deficit of Extra Cellular Fluid volume. B. Hemodialysis. C. Administration of furosemide. D. Administration of mithramycin. E. Parathyroidectomy.

| P a g e

3- All of the following signs or symptoms are characteristics of an extracellular fluid volume deficit except: means dehydration A. Dry, sticky oral mucous membranes. B. Decreased body temperature. C. Decreased skin turgor. D. Apathy. E. Tachycardia. 4- Blood pH a) high after diarrhea b) low after vomiting c) more in Rt atrium than Lt atrium d) lower in Rt atrium than Lt ventricle e) lower in renal vein than renal artery 6- All cause recent loss of weight, except: AIDS Cancer Nephritic syndrome Kwashiorkor 5 -In a patient with weight loss, all can be a cause except: a) Thyrotoxicosis. b) Nephrotic syndrome. c) TB. d) AS. 6- The most common cause of hypercalcaemia in a hospitalized patient is: a. Dietary, such as milk-alkali syndrome. b. Drug related, such as the use of thiazide diuretics. c. Granulomatous disease. d. Cancer. e. Dehydration 7- Hyperkalemia is characterized by all of the following except: a) nausea and vomiting.
| P a g e

b) Peaked T-waves. c) Widened QRS complex. d) Positive Chvostek sign. e) Cardiac arrest in diastole. 8- Normal daily caloric intake is: a) 0.3 kcal/kg b) 1.3kcal/kg c) 2.Okcal/kg d) 3.Skcal/kg e) 35kcallkg 9-Hypokalemia occurs with all except: a) Metabolic alkalosis. b) Acute tubular acidosis. c) Chronic diarrhea. d) Hyperaldosteronism. e) Furosemide. 1- a 24 yrs old pt. came for check up after a promiscuous relation 1 month ago .. he was clinically unremarkable, VDRL : 1/128 he was allergic 2 penicillin other line of management is : doxycycline 2- a 25 yrs old Saudi man presented with Hx of mild icterus , otherwise ok .. hepatitis screen : HBsAg +ve , HBeAg +ve , anti-HBcAg +ve , the diagnosis : a- acute hepatitis B b- convalescent stage of hep. B c- recovery with seroconversion Hep . B d- Hep B carrier e- chronic active Hep. B

| P a g e

3-What is the least effective AB of the following to staph. aureus: clindamycin. erythromycin. amoxicillin. Vancomycin. Fewer than 5% of staph. aureus isolates are sensitive to penicillin. - Which of the following antibiotics has the least activity against S. aureus? a. Erythromycin. b. Clindamycin. c. Vancomycin. d. Dicloxicillin. e. First generation cephalosporins. 4- regarding protective measures of malaria, all true except: - infestation occur more in day than night : - using insect repellant is useful 5- Regarding typhoid fever, all are true except: fever and red spots appear on the same time. can be completely eradicated even in the presence of gall stones. transmitted by food, milk and water. can be treated by quinolones. 6- Within 6 hours after attending a dinner party, 10 participants developed sever N/V, abdominal cramp and diarrhea. Most of them resolved spontaneously. Few were admitted for correction of dehydration. The most probable cause is: a. Salmonella food poisoning. b. Botulism. c. Staphylococcal food poisoning. d. Giardiasis.
| P a g e

e. Clostridium perfringes food poisoning. -A family went to a dinner party after 6 hours they all had symptoms of abdominal pain,nausea, vomiting and dehydration. Some of them recovered while others needed hospitilzation. Whats the most likely organism? a) giardia b) staph aurens c) salmonella d) c . perfiringis e) c.boyulism 7- The following are characteristics features of brucellosis except: a. Lymphadenopathy b. Hepatomegly. c. Splenomegaly. d. Diarrhea. e. Backache. - All symptoms and signs of brucellosis, except: Back pain. Lymphadenopathy Splenomegaly Hepatomegaly Gastroenteritis 8- in brucellosis, all of the following are true except: a) brucella abortus cause more severe form than B. melitansis in children . b) human to human is rarely documented . c) human can be infected through inhalation . d) brucella species are small, non motile gram -ve coccobacilli . e) pt with high titer can show false -ve .

| P a g e

9- 82 y/o female presented to ER in confusion with hypotension. BP was 70/20, P=l6O/min, rectal T= 37.7C. The most likely of the following would suggest sepsis as a cause of hypotension is: a- Low systemic vascular resistance & high cardiac output. b- High systemic vascular resistance & low cardiac output. c- Pulmonary capillary wedge pressure less than 26. d-PH is less than 7.2. e- Serum lactate dehydrogenase more than 22. 10-A 55 years old male with COPD , complaining of 1 wk fever and productive cough on CXR showed Left upper pneumonia, sputum culture +ve for H.inf, most drug effective is a. Pencillin b. Doxycycline c. Cefuroxime d. Gentamycin e. Carbinacillin - A 55 years old man known case of COPD. Now complaining of 1 week fever, productive cough. CXR showed left upper lobe pneumonia. Sputum culture positive H.influenza. what are you going to give him? a- penicillin. b- doxycyclin. c- cefuroxirne. d- gentamycin. e- carbincillin 11-Which one of the following diseases is not transmitted by mosquitoes?
| P a g e

a)rift valley fever b)yellow fever c)relapsing fever d)filariasis e)dengue fever 12- Pt presents with fever swelling is felt,Ant.lymph node swelling warm, tender & fluctuant Dx: a) viral infection . b) bacterial lymphadenitis . c) Hodgkin L. d) ALL . 13- Patient with H/O fever, peripheral blood film +ve for malaria: a) Banana shaped erythrocyte is seen in P. vivax b) Mostly due to P. falciparium c) Treated immediately by primaquin 10mg for 3 days d) Response to Rx will take 72 hr to appear 14-The following drugs can be used in prophylaxis of malaria in chlorquine-resistant area, except: a) Mefloquin . b) Doxycycline . c) Proguanil . d) Chlorquine + dapsone + pyrimethamine . - the following can be used in prophylaxis in malaria in chlorquine resistant area except: a-mefloquine b-doxycycline c-chlorquine with proguanil e-dapsone 15- Hepatitis most commonly transferred by blood is: a) HBV.
| P a g e

b) HAV. c) HCV (previously known as non a non b). d) None of the above. 16- The greatest risk of developing chronic hepatitis and cirrhosis occurs after: a. Hepatitis A infection. b. Hepatitis B infection. c. Hepatitis C infection. d. Hepatitis D infection. e. Hepatitis E infection. 17- In a gram-negative bacterial septicemia: a) pseudomonas is the most common organism involved. b) Many of the advese changes can be accounted for by endotoxin. c) The cardiac index is low d) Central venous pressure is high. e) Endotoxin is mainly a long-chain peptide. 18- In septic shock: a) The mortality rate is 10 to 20%. b) Gram-negative organisms are involved exclusively c) The majority of patients are elderly d) The most common source of infection is alimentary tract. e) Two or more organisms are responsible in the majority of cases. 21- HSV type 1 infection of the oral cavity, all true except: a- Is the commonest viral infection in the oral cavity b- Can give gingivostomatitis c- In primary infection, there is systemic involvement d- May present with tonsillitis without oral lesion
| P a g e

32- A 40 year old white male is transferred to your institution in septic shock less than 24 hours after onset of symptoms of a non-specific illness. He underwent a splenectomy for trauma 5 years ago. Antibiotic coverage must be directed against: a. Streptococcus, group A. b. Klebsiella pneumoniae. c. Staphylococcus aureus. d. Escherichia coli. e. Streptococcus pneumoniae. 1-60 year old male was refer to you after stabilization investigation show Hgb 18,5 g/dl , hect. 64% , RBC 7.8 , WBC 15.3 & Plt. 570 Diagnosis : a- iron def. Anemia b- Hgb pathy c- CLL d- 2ry polycythemia e- Polycythemia rubra Vera 2- 55 y/o female presented to ER because her family noticed skin discoloration. Has Hx of 5 kg loss over 3 weeks. Her medical Hx is -ve apart from vitiligo. Her examination is within normal except for scleral icterus & skin jaundice. Her lx : WBC 2500 ,Plt 70000 (165415 x 1000/mm3) , MCV 106, Hct 17, Retics count 15, T.bil 3, which of the following test will be more associated with the syndrome she has: microcytic anemia a- Chromosomal kariotype of bone marrow. b- Antiparietal cells antibodies. c- Extrahepatic biliarv obstruction d- Decrease gastric fluid
| P a g e

3- 32-year-old Saudi man from Eastern province came to you for routine pre-employment physical exam. He has always been healthy and his examination is normal. Lab: HCT 35% (38.846.4) , MCV: 63fL (7993.3 fL) , WBC: 6800/ml ,retics: 4000/ml (0.7%) (0.82.3% red cells) Platelet: 27000/u1 his stool: -ve for occult blood The most direct way to confirm suspected diagnosis: A. Peripheral smear B. Measure Hb A2 level C. G6PD screening D. Measure iron, TIBC and ferritin level E. Bone marrow stain for iron 4- 26-year-old man presented with headache and fatigue. Investigations revealed: Hb 8 g/dI , MCV 85 fL retics 10% ,All the following investigations are useful except : A. Coombs test B. Sickling test C. Serum bilirubin D. Serum iron as it is normocytic anemia with reticulocytosis (hemolysis or hemorrhage) E. Hb electrophoresis 5- Increased bleeding time is seen in all of the following except: a) Hemophilia. b) Scurvy. c) VwD (Von-Willebrand disease).

| P a g e

6- critical count of platelets which lead to spontaneous bleeding is: a) 1000 b) 50.000 c) 75.000 d) 100.000 e) 20.000 7- Serum ferritin reflects: a) Total iron stores. b) Serum iron. c) Bone marrow iron. d) None of the above. 8-In a patient with Hb = 8, MCV = 82, retic = 10%, all is needed except: a) Hb electrophoresis. b) Coombs test. c) Serum iron level. d) Serum bilirubin level 9- pt with fever, pallor petechei, echemosis, CBC as WBC 2,800 Imm3 ,Hb 6 & plt 2900. next step of investigation: a)bone marrow aspiration 10- A 23 year old white female is diagnosed as having chronic ITP . Which of the following will best predict a favorable remission after splenectomy: a. Presence of antiplatelet antibodies. b. Increased bone marrow megakaryocytes. c. Absence of splenomegaly.
| P a g e

d. Platelet count of 170000/mm3 on corticosteroids. e. Complement on platelet surfaces. 11- Which of the following would most likely indicate a hemolytic transfusion reaction in an anesthetized patent? a) shaking chills and muscle spasm b) fever and oliguria c) heperpyrexia and hypotention d) tachycardia and cynosis e) bleeding and hypotention 12- Which of the following organs is likely to receive a proportionately greater increase in blood flow? a) kidneys b) liver c) heart d) skin e) none of the above 1-a 28 year old lady presented with history of increased bowel motion in the last 8 months.About 34 motions/day .Examination was normal.Stool analysis showed: Cyst,yeast: nil ,Mucus ve Culture:no growth, Whats the most likely diagnosis? a)inflammatory bowel disease b)irritable bowel disease c)diverticulitis 2- Fecal leukocytes come with all except: a) Shigellosis. b) Clindamycin induced colitis. c) Idiopathic ulcerative colitis. 3- Which of the following is the most potent known stimulator of gastric acid secretion: a. Pepsinogen.
| P a g e

b. Gastrin. c. Acetylcholine B. d. Enterogastrone. e. Cholecystokinin 4-In acute pancreatitis the chief adverse factor is: a. Hypercalcaemia (> 12 mg/dl). b. Age above 40 years. c. Hypoxia. d. Hyperamylasemia (> 600 units). e. Gallstones. 5. Complications following pancreatitis may include all of the following except: a. Pulmonary atelectasis. b. Altered mental status. c. Shock. d. Afferent loop syndrome. e. Sepsis. 6- All are true in black hairy tongue, except: a) Hydrocortisone can be used. b) Advice pt not brush his tongue. - All true about black hairy tongue, except:- Advice patient not to brush his tongue - It is caused by bacterial over growth - Bad oral hygiene is a potential cause - Antibiotics may be prescribed if refractory 7-regarding aphthous ulceration in the mouth all are true except: a-there is no treatment for acute ulcer b-tetracycline suspension helps in healing c-there is immunological role in its development d-mostly idiopathic in origin 8- Diarrhea can occur in all the following, except: - Hypothyroidism - Hyperthyroidism
| P a g e

1- 42 y/o female presented with 5 months Hx of N, V & malaise. Na is low, Cr is high, Glucose is 2.7 mmol/L & HCo3 is 10. a- hypothyroidism. b- Addisons disease. c- hypervolemia because of the vomiting. d- pheochromocytoma. e- SIADH. -43 yo female presented wIt 6 mnth history of malaise , N,V. Lab results: Na = 127, K 4.9, urea 15, Ceriatinine = 135, HCO3 = 13 ,glucose 2.7. Most likely Dx is: a. Hypothyroidism b. Pheochromocytoma c. Hypovolemia due to vomiting d. SIADH e. Addisons disease 2- In a patient with elevated serum level of calcium without hypocalciuria, which of the following tests is almost always diagnostic of primary hyperparathyroidism: a. Elevated serum level of ionized calcium. b. Elevated serum level of chloride and decreased serum phosphorus. c. Elevated serum level of intact parathyroid hormone (PTH). d. Elevated 24-hour urine calcium clearance. e. Elevated urinary level of cyclic AMP.

| P a g e

Primary hyperparathyroidism is confirmed by demonstration of an inappropriately high PTH level for the degree of hypercalcemia. 3- Some patients develop hypoparathyroidism after thyroid or parathyroid operations. What is the treatment for hypoparathyroidism: a. Oral 1,25-vitamin D and calcium. b. Transplantation of fetal parathyroid tissue. c. Intramuscular PTH injection. d. Reoperation to remove the thymus. e. Oral phosphate binders.

1- The mechanism of action of ASA: inhibition of the platlet cyclo-oxygenase. decrease the lipids. 2- Anticoagulant effect of heparin based on: A. Alteration of thrombin levels B. Potentiation of antithrombin III C. Activation of plasmin into plasminogen D. Inactivation of ionized calcium E. Reduction of available factor VII - Heparin anticoagulant action depend on: a) potentiation of antithrombin three . b) change plasmin to plasminogen . c) affect prothrombin . d) affect ionized Ca++ . - The mechanism of action of heparin: activation of antithrombin III.
| P a g e

3- Heparinization includes all except: -increase PT -increase PTT -Anti inflammatory. - open potential collateral vessels 4- Which drug can be givin to G6PD patient? a- ASA b- Sulphonamide. c- Nitrofurantoin. d- Chloroquine. - All of the following drugs contraindicated in G6PD deficiency, except: - Aspirin - Nitrofurantoin - Chlorquine - Sulphonamide - Gentamycin 5- One of the following combination of drugs should be avoided: a) cephaloridine and paracetamol b) penicillin and probenecid c) digoxin and levodopa d) sulphamethoxazole and trimethoprim e) tetracycline and aluminium hydroxide 6- All of the following drugs advised to be given to elderly pt, except: a) cimitidine b) thyroxin c) digoxin d) chloropropamide
| P a g e

7- Furosemide increase excretion of: a) Na+ b) K+ c) phosph. d) non of the above 8-All of the following are side effects of furasomide except: a. Hyperkalemia b. Hypoglycemia C. Bronchospasm d. Haemolytic anemia e. Pre-renal azotemia 9- Digoxin toxicity: a) tinnitus b) pleural effusion c) nausea d) all of the above e) none of the above 10- Which of the following is not associated with phenytoin toxicity? - hirsutism - osteomalacia - ataxia - osteoprosis 11- Complications of long term phenytoin therapy include the following except: A. Hisutism. B. Osteoporosis. C. Osteomalacia. D. Macrocytosis.
| P a g e

E. Ataxia. -Which of the following side effect is not associated with phenytoin: a) Hirsutism b) Macrocytic anemia c) Osteomalacia. d) Ataxia e) Osteoporosis 12-All of them are renal complications of NSAIDs except: a. Acute renal failure b. Tubular acidosis C. Interstitial nephritis d. Upper GI bleeding - NSAIDs cause all except: a. Acute Renal failure b. Tubular necrosis c. Hypokalemia d. Interstitial nephritis

-all the following regarding NSAID are true except: A-acute renal failure b-acute tubular necrosis c-interstisial nephritis d- hypokalemia 13- Which one of these drugs is administered orally: a) Amikacin. b) Neomycin. c) Gentamycin.
| P a g e

d) Streptomycin. e) Tobramycin. 14- Chronic use of steroids will give: a) Osteomalacia. b) Myopathies of pelvic girdle. c) Increased risk of breast Ca. d) Hypoglycemia. 15- All of the following cause gastric irritation, except: a) Erythromycin. b) NSAIDS. c) Sucralfate. Antiulcer agent; gastric mucosa protectant. d) Diclofenac. e) Penicillins. 16- All of the following are anti-arrhythmic drugs, except: a) Xylocaine. b) Digoxin. c) Quinidine. d) Amiodarone. e) Procainamide. Group IA : Quinidine sulfate PO ,Quinidine gluconate PO ,Procainamide IV PO , Disopyramide PO Group IB : Lidocaine IV , Mexiletine PO Group IC : Flecainide PO , Propafenone PO Group II : Metroprolol IV , Esmolol IV Group III : Amiodarone PO IV , Ibutilide IV , Dofetilide PO , Sotalol PO Group IV : Verapamil IV PO , Diltiazem IV Other : Digoxin IV, PO , Adenosine IV

| P a g e

17- All can be used for the treatment of acute gout except: a) Allopurinol. b) Penicillamine. c) Gold salt. In severe inflammation d) Paracetamol. e) Indomethacin. 18- pt on chemotherapy presented with fever, all should be done, except: a) blood culture b) urine culture c) aspirin is effective d) broad spectrum antibiotics 19- Nitroglycerine cause all of the following, except: a) increase coronary blood flow. b) Methemoglobinemia. c) Venous pooling of blood. d) Efficient for 5 mm if taken sublingual. e) Lowers arterial blood pressure - Nitroglycerine cause all the following except: a- Lowers arterial blood pressure b- Increase coronary blood flow c- Effect for 5 min if taken sublingually d- Causes venous pooling of blood e- Can produce methemoglobinemia 20- Allopurinol, one is true: b) Effective in acute attack of gout. c) decreases the chance of uric acid stone formation in kidneys d) Salicylates antagonize its action. - Regarding Allopurinol: - is a uricouric agent - decrease the development of uric acid stones
| P a g e

- useful in acute attack of gout 21- Oral anticoagulants: a) can be given to pregnants during 1st trimester b) can be reversed within 6 hours c) are enhanced by barbiturates d) can not cross blood brain barrier e) none of the above 22- which drug does not cross the placenta: a) Heparin b) Aspirin c) Warfarin d) Tetracycline e) Diazepam 23- all the following are side effect of thiazide diuretics except: a-has diabetogenic effect b-cause hypocalcemia c-cause hypomagnesimia d-flat curve response e-cause hypokalemia 24 - regarding 2 blocker all are true except: a-morning dose effective more than evening dose. b-long term maintance therapy should be avoided 25- All true about cephalosporin use, except: - The most common side-effect is allergy - There is a skin test for cephalosporin sensitivity Attempts to develop a skin test for allergy to cephalosporins have been unsuccessful 26-which of the following combination is safe: a-alcohol and metronidazole b-digoxin and amiodarone c-warafrin and propanolol d-furosemide and gentamycin

| P a g e

12- All the following are differentials of acute abdomen except: a-pleurisy b-MI c-herpes zoster d-polyarteritis nodsa e-pancreatitis

Cardio
1-Pt had arthritis in two large joint & pansystolic murmur ( carditis ) ,Hx of URTI the most important next step: a- ESR b- ASO titre c- Blood culture 2-The following are features of rheumatic fever, except: a)restless, involuntary abnormal movements. b)subcutaneous nodules. c)rashes over trunk and extremities. d)short PR interval on ECG. e)migratory arthritis - The following are features of rheumatic heart disease except: 1) Restless involuntary abnormal movement 2) Rashes over trunk and extremities 3) Short P-R interval on ECG 4) Migratory arthritis

| P a g e

2- years old lady on .., feels dizzy on standing, resolves after 10-15 minutes on sitting, decrease on standing, most likely she is having : a- orthostatic hypotension 3- what is the most appropriate treatment for the above patient : a- antiemetic b- antihistamine c- change the antidepressant to SSRI d- thiazide diuretics e- audiometry 4- High output HF causes includes all except: anemia. Mitral Regurgitation (MR) AV fistula. pagets disease 12- Premature ventricular contraction (PVC),all are true except: a) If anti-arrhythmic given after MI for protection of PVC --> decrease chance of sudden death. b) Use of anti-arrhythmic type I increase mortality. c) PVC in normal population doesnt increase risk of sudden death. - Premature ventricular contracture (PVC), all are true except:- Use antiarrhythmic post MI improve prognosis - Use of antiarrythmic type 1 increase mortality 5- MI with premature ventricular contractions, the best Tx:
| P a g e

digoxin. lidocaine. quinidine. - A patients ECG showed anterior wall Ml with PVCs hes on digoxin, warfarrin , lasix what treatment would you like to add: - patient had anterior wall MI and will he was transferred to ICU the nurse notice that he has PVC .... 20 per minute. He is on digoxin, diuretic. What do you want to add? a- propanolol. b- amiodarone. c- moxillin. d- nothing. -A 60 year old male presented with Hx of 2hrs chest pain ECG showed ST elevation on V1-V4 with multiple PVC & ventricular tachycardia. The management is: a. Digoxin b. Lidocane c. Plavix & morphine d. Amidarone

6- A 48 years old female with long standing infection present with bradycardia, your management will be: a. IV. fluid b. Atropine. c. Dopamine

| P a g e

7- Indication for valve replacement in infective endocarditis include all except: a. Viral endocarditis. b. Resistant bacterial endocarditis. c. Fungal endocarditis. d. Aortic valve regurgitation. 8- A 61-year-old man with known ischemic heart disease and peripheral vascular disease is started on an ACE inhibitor by his GP for hypertension. Three weeks later he is admitted with increasing confusion and vomiting. Investigations reveal: CBC Hb 14.9 g/dI, MCV 88 fI, WBC 13.6 x 109/L; U & Es: Na+ 131 mmol/L, K+ 7.3 moI/L, urea 37.8 mmol/L, Cr 858 umol/L. The patient is suffering from: A. Diabetic nephropathy. B. Phaeochromocytoma. C. Polycystic kidney disease. D. Raised intracranial pressure. E. Renal artery stenosis. 9- Which of the following is the least likely to cause infective endocarditis: a. ASD b. VSD C. Tetrology of Fallot d. PDA 10-Which of the following is believed to reduce mortality in CHF? a. Digitalis b. Furosamide C. Enalopril
| P a g e

d. Procainamide e. Asprin 11-All may cause increased BP except: a. Obesity b. High alcohol intake c. Smoking d. Gout e. NSAID use 13-All can cause secondary hyperlipidemia except: a. Hypothyroidism b. Alcoholism c. Nephrotic syndrome d. Estrogen therapy e. Hypertension - Causes of Secondary hyperlipidemia are all except: a. HTN b. Nephritic syndrome c. Hypothyroidism d. Obesity 14-Coarctation of the aorta in commonly associated with which of the following syndromes? a)down b)turner c)patau d)edward e)holtorain 15-a patient came to you & you found his BP to be 160/100,he isnt on any medication yet. Lab
| P a g e

investigations showed: Creatinine (normal) ,Na 145 (135-145) ,K 3.2(3.5-5.1) ,HCO3 30(22-30) What is the diagnosis? a)essential hypertension b)pheochromocytoma c)addisons disease d)primary hyperaldosteronism 16- 35-year-old woman presented with exertional dyspnea. Precordial examination revealed loud S1 and rumbling mid diastolic murmer at apex. Possible complications of this condition can be all the following except: A. Atrial fibrillation B. Systemic embolization C. Left ventricular failure D. Pulmonary edema E. Pulmonary hypertension 17- A 70-year-old male was brought to the emergency with sudden onset of pain in his left lower limb. The pain was severe with numbness. He had an acute myocardial infarction 2 weeks previously and was discharged 24 hours prior to his presentation. The left leg was cold and pale, right leg was normal. The most likely diagnosis is: A. Acute arterial thrombosis. B. Acute arterial embolus. C. Deep vein thrombosis. D. Ruptured disc at L4-5 with radiating pain E. Dissecting thoraco-abdominal aneurysm.

| P a g e

18- 70-year-old woman has had Ml. 2 days after admission she developed abdominal pain and diarrhea with passage of blood. Abdomen x-ray showed distended intestine with no fluid level. Serum amylase level slightly elevated with mild fever. The diagnosis is: A. Ulcerative colitis B. Acute pancereatitis C. lschemic colitis D. Diverticulitis E. Phenindione-induced colitis (*phenindione is an anticoagulant) 19- Blood pressure, all of the following are true except: a) if 2/3 of cuff -* false high BP b) internal cuff must cover 80% of arm c) follow circadian vary 4 late night high BP d) high BP* 3 standard deviation away from normal e) you have to have more than one reading to Dx high BP 20- Percentage (%) of reinfarction for pt undergoing non-cardiac surgery: a) 5%, 3 months after the infarct b) 15%, 3 months after the infarct c) 35%, 3 months after the infarct d) 5%, 3-6 months after the infarct e) 35%, 3-6 months after the infarct 21- All of the following causes secondary HTN, except: a) Pheochrmocytoma. b) Addisons disease.
| P a g e

c) Hyperaldosteronism (conns disease) d) Renal disease. e) Pregnancy. f) Primary hypothyroidism. 22- Most important complication of systemic hypertension: Pontine infarction Renal artery stenosis Subdural hemorrhage Intracerebral hemorrhage -complications of systemic hypertension are all except: a) Intracerebellar haemorrhaqe b)Renal artery stenosis.(this causes HTN) 23- S3 can be auscultated in all, except: Mitral stenosis Mitral regurgitation Lt sided heart failure Thyrotoxicosis -S3 occur in all of the following except: a) Tricuspid regurgitations. b) young athelete. c) LV failure. d) mitral stenosis. 24- Pt with chronic atrial fibrillation more than 6 months, all can be given except: Cardioversion Heparin
| P a g e

Digoxin -Treatment of chronic atrial fibrillation all, except: a) cardioversion b) digoxin c ) warfarin 26-Treatment of unstable angina include all except: a) heparin b) nitroglycerin c) -blocker d) aspirin e).. 27-The first symptom of left heart failure is: a) orthopnea b) edema c) dyspnea on exertion - The first symptom in pt with Lt heart failure: a) orthopnea. b)PND. c) Pedal edema. d) Exertional dyspnea. ?? - The first symptoms of LHF is 1) Orthopnea 2) Dyspnea on exertion 3) Oedema 29- pt with low grade fever and arthalgia for 5 days, presented with pansystolic murmer at the apex.H/O

| P a g e

difficulty in swallowing with fever 3 wks back. Most likely diagnosis: a) bacterial endocarditis b) viral myocarditis c) acute rheumatic fever d) pericarditis 31- In atrial fibrillation and stroke, all are true , except: a) Aspirin can be given in AF for prevention of stroke. b) Warfarin can be given in AF for prevention of stroke. c) Non valvular AF can cause stroke. - One of the following is not useful in patient with atrial fibrillation AF and Stroke: a- Aspirin and AF b- Warfarin and AF c- Valvular heart disease can lead to CVA in young patient d- AF in elderly is predisposing factor 33-the following murmer can be accentuated by postioning of the patient: a-aortic regurgitation by sitting b- venous hum by lying down c-pericardial rub by sitting d-outflow innocent murmer by sitting 34- ECG stress test is indicated in the following except: -routine (yearly) test in asymptomatic patients - in high risk jobs - 40 year old patient before starting exercise program
| P a g e

35- Pulmonary stenosis: a. Commonest lesion in turners syndrome. b. P2 is loud indicates sever stenosis. c. Ejection click means it is valvar. d. All of the above. 1- 25-year-old student presented to your office complaining of sudden and severe headache for 4 hours. History revealed mild headache attacks during the last few days. On examination: agitated and restless. What Dx must be considered this case? A. Severe migraine attack B. Cluster headache C. Subarachnoid hemorrhage D. Hypertensive encephalopathy E. encephalitis 2- Greatest risk for stroke: a- DM. b- family history of stroke. c- high blood pressure. d- hyperlipidemia. e- cigarette smoking. -the most important factor predisposing to stroke is: a)DM b) HTN C) Hyperlipidemia d) Cholesterol -one of the following is the single most important cause of stroke: a-D.M. b-HTN c-family history
| P a g e

d- hyperlipidemia e-hypercholesteremia 3- Pt come within 3 hrs C/O Lt side weakness, examination revealed Lt side hemiparesis, pulse 120/min irregular with diastolic murmur at mitral area. 1st step of management: a) heparin b) digoxin e) EEG d) carotid angiography e) echo 4- Patient suspected of having brain abscess, the most important q. in the history is: frontal sinusitis. ear discharge. head injury. bronchioctasis. Hx. of vomiting. -which of following mostly occur in a patient with intracranial abscess a)cough b)vomiting c)ear discharge d)frontal sinusitis 1-a patient ( known case of DM ) presented to u with diabetic foot ( infection) the antibiotic Antibiotics Appropriate monotherapy for cellulitis includes cefazolin or clindamycin. Although gram-negative organisms are the unusual causes of cellulitis, even in diabetes, if they are suspected, a fluoroquinolone (eg, levofloxacin) may be used in conjunction with clindamycin.

2- In DKA, all the following are true except: a. Increase of corticosteroid, glucagon and GH.
| P a g e

b. Body water deficit is 4 to 6 L 3- A 14 years old boy with type 1 D.M. presented in coma. His blood glucose level is 33 mmol/l. Na is 142 mmol/l, K is 5.5 mmol/l, bicarb is 10 mmol/l. the following are true except: a. The initial Rx. Should be l.V. normal saline 3 L/hour for 1-2 hours b. IV. insulin loading dose 1 u/kg is necessary. (0.1 U/kg/h is the true) c. IV. Na bicarbonate could be given if pH is 7 or less. d. Hyprephosphatemia can occur during treament. e. Hyperchloremia can occur during treatment 4-A 45 year old presented with polyurea, urine analysis showed glucosurea & -ve ketone FBS l4mmoI. What is the best management of this patient? a. Intermediate IM insulin till stable b. NPH or Lent insulin 30mg then diet C. Sulphonylurea d. Diabetic diet only e. Metformin -A 36 years old female with FBS = 14 mmol & glucosuria, without ketones in urine, the treatment is: a) Intermittent I.M. insulin NPH. b) Salphonylurea + diabetic diet. c) Diabetic diet only. d) Metformin. - A 36 years old man, obese. Recently, developed polyuria, polydepsia and weight loss. Urine analysis
| P a g e

showed glucosuria and ve ketone. FBS is 280 mg/dl. The best initial therapy is: a. Intermitted l.M. insulin iniection till stabe. b. NPH or Lenti insulin 30 units daily + diabetic diet. c. Sulphonylurea + D.D. d. Metformin - A 30 y male came to ER with polyuria but -ve keton. Random blood sugar 280 mg/dl. management: Nothing done only observe Insulin 30 U NPH+ diet control Diet and exercise Oral hypoglycemic

5- A 30 years old teacher complaining of excessive water drinking and frequency of urination, 0/B Normal. You suspect DM and request FBS = 6.8 .the Dx is: a. DM b. DI c. Impaired fasting glucose d. NLbloodsugar e. Impaired glucose tolerance

6- 60 years old male complaining of decreased lipido , decreased ejaculation, FBS = 6.5 mmol, increased prolactin, Normal FSH and LH, your opinion is a. Measure Testosterone level b. He has DM c. Do CT of head
| P a g e

d. He has Normal Fasting Blood sugar 7- A 46-year-old man, a known case of diabetes for the last 5 months. He is maintained on Metformin 850 mg Po TID, diet control and used to walk daily for 30 minutes. On examination : unremarkable. Some investigations show the following: FBS 7.4 mmol/L ,2 hr PP 8.6 mmol/L ,HbA1c 6.6% ,Total Cholesterol 5.98 mmol/L ,HDLC 0.92 mmol/L ,LDLC 3.88 mmolIL , Triglycerides 2.84 mmolIL (0.34-2.27) ,Based on evidence, the following concerning his management is true: 1. The goal of management is to lower the triglycerides first. 2. The goal of management is to reduce the HbA1c. 3. The drug of choice to reach the goal is Fibrates. 4. The goal of management is LDLC 2.6 mmol/L. 5. The goal of management is total cholesterol 5.2 mmol/L.

According to guidelines of the ADA and the American Heart Association, the target lipid values in diabetic individuals (age >40 years) without cardiovascular disease should be: 1) LDL < 2.6 mmol/L (100 mg/dL); 2) HDL > 1.1 mmol/L (40 mg/dL) in men and >1.38 mmol/L (50 mg/dL) in women; and 3) triglycerides < 1.7 mmol/L (150 mg/dL). Fibrates have some efficacy and should be considered when the HDL is low in the setting of a mild elevation of the LDL.
| P a g e

8- Regarding the criteria of the diagnosis of diabetes mellitus, the following are true except: 1. Symptomatic patient plus casual plasma glucose 7.6 mmol/L is diagnostic of diabetes mellitus. 2. FPG 7.0 mmol/L plus 2 h-post 75 gm glucose 11.1 mmol/L is diagnostic of diabetes mellitus. 3. FPG 5.5 mmol/L = normal fasting glucose. 4. FPG 7.0 mmol/L = provisional diagnosis of diabetes mellitus and must be confirmed in another setting in asymptomatic patient. 5. 2-h post 75 gm glucose 7.6 mmol/L and < 11.1 mmol/L = impaired glucose tolerance. Impaired glucose tolerance (IGT) is defined as plasma glucose levels between 7.8 and 11.1 mmol/L (140 and 199 mg/dL) 9- In DKA, use: both wrong a) Short and intermediate acting insulin. b) Long acting insulin. 12- a 70 yr old male, suddenly felt down & he is diabetic, it could be: a) May be the ptnn is hypertensive and he developed sudden rising BP b) He might forgot his oral hypoglycemic agent dose c) Sudden ICH which rise his ICP -A 70 year Saudi diabetic male suddenly fell down, this could be: A-Maybe the patient is hypertensive and he developed a sudden rise in BP. B-He might had forgot his oral hypoglycemic drug. C-Sudden ICH which raise his ICP.
| P a g e

Autonomic Neuropathy

Surgery 1- a 27 yrs. old female C/O abdominal pain initially periumbilical then moved to Rt. Lower quadrant she was C/O anorexia,nausea and vomiting as well ..O/E : temp.38c , cough , tenderness in Rt lower quadrant but no rebound tenderness.Investigations : slight elevation of WBC's otherwise insignificant ..The best way of management is: a- go to home and come after 24 hours b- admission and observation c- further lab investigations d- start wide spectrum antibiotic e- paracetamol 2- what is the most likely diagnosis for the above patient ? a- mesenteric lymph adenitits b- acute appendicitis c- peptic ulcer 3-All are signs & symptoms suggestive of acute appendicitis except: a. Vomiting b. Anorexia
| P a g e

c. Paraumbalical pain shifting to right lower quadrent d. Temp 38.IC e. Sitting & leaning forward

-All suggest acute appendicitis except: a. Fever 38.1 b. Anorexia c. Vomiting d. Umbilical pain shifting to the Rt lower Quadrant e. Pain improving with sitting and leaning forward 4- Appendicitis most diagnostic: a) fever b) diarrhea c) urinary symptoms d) leukocytosis e) tender Rt lower quadrant with rebound 5- acute appendicitis in children all false except: a) leukocytosis is diagnostic b) rarely perforated if it is not well treated c) can cause intestinal obstruction d) need ABC before surgery for every child 6-the following is true in suspected appendicitis in a 70 yr old person: a) Perforation is less likely than usual b) Rigidity is more marked than usual c) Abdominal X-ray is not useful for exclusion of obstruction acute

useful

| P a g e

d) Out look is relatively good e) Intestinal obstructin may be mimicked -Appendicitis in elderly: a) less risk of perforation. b) more rigidity. c) can mimic intestinal obstruction.

-about appendicitis in elderly: a-perforation is not common b-gives more rigitiy than usual c-can mimic obstruction - The following is true about suspected acute appendicitis in a 70 year old man: A-Perforation is less likely than usual. B-Rigidity is more marked than usual. C-Abdominal x-ray is not useful. D-Outlook is relatively good. E-lntestinal obstruction maybe mimicked. 7- The most sensitive test for defining the presence of an inflammatory focus in appendicitis is: a. The white blood count. b. The patients temperature. c. The white blood cell differentiaL d. The sedimentation rate. e. The eosinophil count. 8- The peak incidence of acute appendicitis is between:
| P a g e

a. One and two years. b. Two and five years. c. Sixand 11 years. d. 12 and 18 years e. 19 and 25years. 9- Acute appendicitis: a. Occurs equally among men and women. b. With perforation will show fecoliths in 10% of cases. c. Without perforation will show fecoliths in fewer than 2% of cases. d. Has decreased in frequency during the past 20 years. e. Presents with vomiting in 25% of cases. 10- The mortality rate from acute appendicitis in the general population is: a. 4 per 100. b. 4 per 1000. 1 : 1000 c. 4per 10000. d. 4 per 100000. e. 4per l000000. 11- A 17 year old boy presents with pain over the umbilicus 10 hours prior to admission. During transport to the hospital the pain was mainly in the hypogastrium and right iliac fossa. He has tenderness on deep palpation in the right iliac fossa. The most likely diagnosis is: a. Mesenteric adenitis. b. Acute appendicitis. c. Torsion of the testis. d. Cystitis. e. Ureteric colic.

| P a g e

1- a 29 yrs. Old female has a breast lump in the upper outer quadrant of the left breast , firm , 2 cm. in size but no L.N involvement what is the most likely diagnosis ? a- fibroadenoma 2- What is the management for the above patient? a- mammogram (true if patient > 35 years) b- excisional biopsy c- FNA Fine-needle aspiration (FNA) cytology d- breast US e- follow up in 6 months

3- 45 years old lady presents with bloody nipple discharge. Most likely Dx: - Breast ca. - Fibroadenoma - Ductal Papilloma. - Ductectasia. -A 45 year old female came with nipple discharge containing blood. The most likely cause is: a. Duct papilloma b. Duct ectasia C. Breast abcess d. Fibroadinoma a. Fat necrosis of breast

| P a g e

- A 35 years old female with bloody discharge from the nipple, on examination there is cystic swelling near areola, the most likely diagnosis is: a) Duct ectasia. b) Intra-ductal papilloma. c) Fibroadenoma. - A 45 y.o. lady presented with nipple discharge that contains blood. What is the most likely diagnosis? a- duct papilloma. b- duct ectasia. c- breast abscesss. d- fibroadenoma. e- fat necrosis of breast 4-Which of the following indicates that a breast lump is safe to leave after aspiration? a) a cyst that doesnt refill b) solid rather that cyst c) cytology showed fibrocystic disease e) minimum blood in aspiration fluid 5- A 23-year-old female consulted her physician because of breast mass; the mass is mobile, firm, and approximately 1 cm in diameter. It is located in the upper outer quadrant of the right breast. No axillary lymph nodes are present. What is the treatment of choice for this condition? A. Modified radical mastectomy. B. Lumpectomy. C. Biopsy. D. Radical mastectomy. E. Watchful waiting 6- A 30-year-old female presented with painless breast lump. Ultrasound showed a cystic lesion.
| P a g e

Aspiration of the whole lump content was done and was a clear fluid. Your next step is: A. Do nothing and no follow-up. B. Send the aspirated content for cytology and if abnormal do mastectomy. C. Reassure the patient that this lump is a cyst and reassess her in 4 weeks. D. Book the patient for mastectomy as this cyst may change to cancer. E. Put the patient on contraceptive pills and send her home. 7- in breast CA, all true except: a) 2 cm mass with free axilla is stage I b) Chemotherapy is must for pre-menopausal with +ve axilla c) Radical mastectomy is the choice of surgery d) Yearly mammogram for contra-lateral breast 8- which one will give bilateral breast CA: a) lobular breast ca (ILC) b) intraductal breast ca (IDC) c) mucinous breast ca d) paget disease e) medullary breast ca f) tubular breast ca

10- Factors associated with an increased relative risk of breast cancer include all of the following except: a. Nulliparity. b. Menopause before age 40. c. A biopsy showing fibrocystic disease with a proliferative epithelial component. d. First term pregnancy after age 35.
| P a g e

e. Early menarche. 11- The following statements about adjuvant multiagent cytotoxic chemotherapy for invasive breast cancer are correct except: a. Increases the survival of node-positive premenopausal women. b. Increases the survival of node-negative premenopausal women. c. Increases the survival of node-positive postmenopausal women. d. Is usually given in cycles every 3 to 4 weeks for a total period of 6 months or less. e. Has a greater impact in reducing breast cancer deaths in the first 5 years after treatment than in the second 5 years after treatment. 12- Concerning the treatment of breast cancer, which of the following statement is false? a) patients who are estrogen-receptor-negative are unlikely to respond to anti-estrogen therapy. b) The treatment of choice for stage I disease is modified mastectomy without radiotherapy. c) Patients receiving radiotherapy have a much lower incidence of distant metastases . d) Antiestrogen substances result in remission in 60% of patients who are estrogen-receptor-positive. e) A transverse mastectomy incision simplifies reconstruction. 13- What is the most important predisposing factor to the development of an acute breast infection? a) trauma b) breast feeding
| P a g e

c) pregnancy d) poor hygiene e) diabetes mellitus 14- A 46-year-old female wrestler H presents with a painful mass I x2 cm in the upper outer quadrant of the left breast. There are areas of ecchymosis laterally on both breasts. There is skin retraction overlying the left breast mass. What is the most likely diagnosis? a) fat necrosis b) thrombophlebitis c) hematoma d) intraductal carcinoma e) sclerosing adenosis 15- Clear aspirated fluid from breast cyst will be: a) sent to cytology b) thrown away c) sent to biochemical analysis d) combined with biopsy 16- Cause of giant breast includes these statements : a) diffuse hypertrophy b) cystosarcoma phylloids c) giant fibroadenoma d) all of the above e) none of the above

17- breast cancer in female under 35 yr. all of the following are true EXCEPT:
| P a g e

a) Diagnosis and treatment are delayed due to the enlarged number of benign disease b) The sensitivity of the mammogram alone is not enough for Dx c) Family history of benign or malignant disease is predictive of Dx d) All discrete breast lumps need fine needle aspiration dominant mass only -Breast cancer in a female that is less than 35 year of age.. .all true except: A-Diagnosis and treatment are delayed due to the enlarged percentage of benign. B-The sensitivity of the mammogram alone is not enough for the diagnosis. C- Family history of benign or malignant disease is predictive of the diagnosis. D-All discrete breast lumps need fine needle aspiration.

| P a g e

1- ? year old woman her menstrual period has stopped since 7 weeks. She was complaining of light bleedidng & discomfort. She had +ve pregnancy test at home. The best test for her now is: a. B-HCG b. Human prolactin C. Progesterone d. Esteriol a Prolactine 2- A 20 year old married lady presented with HX of severe left lower abdominal pain amenorrhea for 6 weeks. The most appropriate investigation to rule out serious DDx is: (means ectopic Pregnancy) a. CBC b. ESR C. Pelvic US d. Abdominal x-ray e. Vaginal swab for culture & sensitivity

| P a g e

- A 20 y.o. married lady presented with history of left lower abdominal pain & amenorrhea for 6 weeks. The most appropriate investigation to role out serious diagnosis is: a- CBC. b- ESR. c- pelvic US. d- abdominal XR e- vaginal swab for culture & sensitivity 3- A women came to Antenatal Care ANC at 8th week of gestation. Diagnosed as case of cervical incompetence, which of the following is the appropriate management? a) Insert a suture in the same week. b) insert suture at 14-16 wks gestation c) Confirm the Dx by inserting Hegars Dilator. d) Admit the patient throught the Pregnancy time in the hospital for observation. e) Give Beta-mimetic agent (Ritodrine) 4- Recurrent abortion: a) Genetic abnormality b) Uterine abnormality c) Thyroid dysfunction d) DM e) Increased prolactin 5- A 25 year old G3P1 present to the emergency room complaining of lower abdominal crampy pain 6 wk from her last normal period .She has had significant vaginal bleeding but no passage of tissue. -the pts most likely diagnosis is: a- Incomplete abortion. b- Complete abortion.
| P a g e

c-Missed abortion. d- Threatened abortion. e- Ectopic pregnancy. -the most important step in this pts evaluation should be: a- Sonography. b- Physical exam. c-CBC. d- Quantitative B-hCG. May be after sonography e- Detailed menstrual history. - Transvaginal ultrasonography would most likely reveal: a- Fetal heart motion. b- An intact gestational sac. c- A discrete yolk sac. d- A thickened endometrium with no gestational sac. e- Fetal heart motion in the adnexae. 1- If a pregnant eating well balanced diet, one of the following should be supplied: a) Ca++ b) phosph. c) vit. C d) none of the above 2- Normal pregnancy in the 2nd trimester is characterized by all of the following except : a. Elevated fasting plasma glucose b. Decreased fasting plasma glucose c. Elevated postprandial plasma insulin d. Elevated postprandial plasma glucose e. Elevated plasma triglycerides
| P a g e

1- a 38 yrs old female came to you at your office and her pap smear report was unsatisfactory for evaluation .. the best action is : a- consider it normal & D/C the pt. b- Repeat it immediately c- Repeat it as soon as possible d- Repeat it after 6 months if considered low risk e- Repeat it after 1 year if no risk 2- 32 yo wth malodorous discharge and itching . Strawberry spots on cervix most appropriate Rx is: a) Metronidazole. b) Estrogen cream c) Progesterone cream d) Vinegar cream e) Salphonamide cream 1-Pregnant women G4P3+1 on GA 10 wk came to you with IUCD inserted & the string is out from O.S what is the most important measure : a- leave the IUCD & give A.B b- leave the IUCD & send to Ob/ Gynaecologist to remove c- leave the IUCD d- do laparoscopy to see if there is ectopic preg. e- Reassurance the pt 2- Action of O.C.P: -Inhibition of estrogen then ovulation. -Inhibition of prolactin then ovulation -Inhibition of protozoa by changing the cervical mucosa. -Inhibition of mid cycle gonadotropin then ovulation -Inhibition of implantation of the embryo. - Action of contraceptive pills: a) Inhibition of estrogen and then ovulation
| P a g e

b) Inhibition of prolactin then ovulation c) Inhibition of protozoa by change in cervical mucosa d) Inhibition of midcycle gonadotropins then ovulation e) Inhibition of implantation of the embryo - action of oral contraceptive pills: a) Inhibition of estrogen then ovulation b) Inhibition of prolactin then ovulation c) Inhibition of protozoa by changing the cervical mucosa d) Inhibition of midcycle gonadotrophins then ovulation e) Inhibition of implantation of the embryo - Primary mechanism of contraceptive pills: a) Cause changes in cervical mucosa b) Cause changes in endometrium preventing implantatioin c) Inhibit relase of estrogen from follicle d) Inhibit gonadtropin surge (no ovulation) 3- A 26 year old female complaining of headache more severe in the early morning mainly bitemporal, her past medical Hx is unremarkable. She gave Hx of OCP use for 1 year. Qphthalmoscope examination showed pappilledema but there is no other neurological findings. The most probable diagnosis is: a. Optic neuritis b. Benign intracranial hypertention c. Encephalitis d. Meningitis e. Intracranial abscess change

| P a g e

-A 26 yr old female, hx of 6 month bilateral temporal headache increasing in the morning with hx of OCP for the last 1 yr. O/E BP = 120/80, fundoscopy showed bilateral papilledema .Dx is: a. Encephalitis b. Meningitis c. Optic neuritis d. BIH = Benign intracranial Hypertention e. Intracerebrall abscess 5-post pill amenorrhea, all true except: a) Need full investigation if persist >6 months b) Pregnancy should be considered c) Prolonged use of contraceptive pill will increase risk of post pill amenorrhea d) More common in women who had irregular periods -post pill amenorrhea, all of the following are true EXCEPT: a) more in Pt with history of menstrual irregularity b) the more usage the higher the risk c) more than 6 months need investigations d) non of the above - The most important mechanism of action of combined oral contraceptive is: a. Inhibition of implantation b. Inhibition of fertilization c. Alteration of tubal motility d. Alteration of cervical mucous e. Ovulation suppression 7- Possible mechanisms of action of intrauterine contraceptive devices: a. Inhibition of implantation
| P a g e

b. Alteration of endometrium c. Suppression of ovulation d. (a) and (b) e. (a) and (c) 8- Non-contraceptive use of combined oral contraception include a. Menorrhagia b. Primary dysmenorrhea c. Functional small ovarian cyst d. All of the above e. None of the above 9- Intrauterine contraceptive devices are associated with a. Decreases menstrual loss b. Septic abortion c. Cervical dyplasia d. Decrease risk of pelvic infection e. Unchanged rate of ectopic pregnancy 10- Absolute contraindication to the insertion IUCD is a. History of genital herpes b. History of molar pregnancy c. Positive pregnancy test d. History of ectopic pregnancy e. None of the above 11- Progestin only contraceptive pills do all except: a. Suppress ovulation b. Increase cervical mucous c. Associated with increased incidence of breakthrough bleeding d. May cause Menorhagia 4- Actions of oral contraceptive pills: A-Inhibition of prolactin then ovulation
| P a g e

B- Inhibition of estrogen then ovulation C- Inhibition of progesterone then ovulation D- Inhibition of mid-cycle gonadotrophin then ovulation E-l n hibition of implantation of embryo. 16- Post-pill amenorrhea.. ..all except: A-More in patients with history of menstrual irregularity. B-The more you use the higher the risk. C-More than 6 months needs investigation. D-lnvestigate the patient for pregnancy. E-None of above 1-Pregnancy test +ve after : a- one day post coital b- 10 day after loss menstrual cycle ?? c- One wk after loss menstrual cycle = 1 week after the lost menstrual cycle 2- Which of the following tests is mandatory for all pregnant woman? a) HIV b) Hepatitis B surface antigen c) VDRL (venereal disease research laboratory) d) all of them are mandatory 3- A healthy 28 years old lady P1+0 presented to you with 6 months amenorrhea. What is the most likely cause for her amenorrhea? a) Pregnancy (the most common cause of 2 amenorrhea is pregnancy) b) Turner syndrome 4- Sign and symptoms of normal pregnancy, except : a) Hyperemesis b) Hegar sign
| P a g e

c) Chadwicks sign d) Amenorrhea 5- A serum progesterone value <5 ng/mI can exclude the diagnosis of a viable pregnancy with a certainty of: a- 20%. b- 40%. c- 60%. d- 80%. e- 100%. 6- In normal pregnancy, the value of B-hCG doubles every a- 2 days. b- 4 days. c- 8 days. d- 10 days. e- 14 days. Diseases of Pregnancy 1- a young pregnant lady (Primigravida) , 32 weeks of gestation came to you C/O : lower limbs swelling for two weeks duration .. she went to another hospital and she was prescribed ( thiazide & loop diuretic ) .. O/E : BP : 120/70 , mild edema , urine dipstick : -ve and otherwise normal. The best action is : a- continue thiazide & stop loop diuretic b- cont. loop diuretic & stop thiazide c- stop both d- continue both and add potassium sparing diuretic e- cont. both & add potassium supplement

| P a g e

2- 8 wk Primigravida came to you with nausea & vomiting choose the statement that guide you to hyperemmesis gravidarum : a- ketonuria b- ECG evidence of hypokalemia c- Metabolic acidosis d- Elevated liver enzyme e- Jaundice 3-35 years prime 16 wk gestation PMH coming for her 1st check up she is excited about her pregnancy no hx of any previous disease. Her B/P after since rest 160/100 after one wk her B/P is 154/96 Most likely diagnosis : a- Pre eclempsia b- Chronic HTN c- Labile HTN d- Chronic HPT with superimposed pre eclampsia e- Transit HPT?? - A35 years old primi 16 weeks gestation coming for her first check up. She is excited about her pregnancy. No history of any previous disease. Her blood pressure after a rest was 160/100. after one week her BP was 154/96. what is the most likely diagnosis? a- pre-eclampsia. b- chronic HTN. c- labile HTN. d- chronic HTN with superimposed pre-eclampsia. e- transient HTN.

| P a g e

4- 16 wk GA with (++) glycosuria, FBS 4.4, 1 hr PB 8, 2 hr PB 7.2 renal glycosuria. GDM. K.M syndrome.???

5- 35 wk GA PG with pre-eclampsia BP is high with ankle edema, the best to be done is: diuretics. low salt diet. labetolol. immediate delivery. maternal-fetal monitoring with continuous hospitalization. . 6- One of the following is a known cause of polyhydraminos: maternal diabetes insipidus. duodenal atresia. renal agenesis. increased fetal urine production or decreased amniotic fluid absorption, either due to impaired swallowing or impaired absorption at the amnionic interface with the uterus - One of the following increases the amniotic fluid: a. Patient of diabetes insipidus b. Duodenal atresia C. Renal agenesis d. CId primigravida
| P a g e

- The most common cause of polyhydramnios is a. Immune hydrops b. Nonimmune hydrops c. Diabetes d. Factors which impair fetal swallowing e. Idiopathic 7- Which of the following doesnt cause IUGR? -Toxoplasmosis -CMV. -Rubella. -Syphilis. -HSV2. 11- All of the following is true about IUGR except: a) Asymmetric IUGR is usually due to congenital anomalies b) IUGR babies are more prone to meconium aspiration and asphyxia c) Inaccurate dating can cause misdiagnosed IUGR postterm infant is more prone to meconium aspiration 8- Management of pre-eclampsia includes all except: a. Bed rest b. Diuretics c. Hospitalization d. Non-stress test e. NSAID

| P a g e

Treatment Prehospital Care Oxygen via facemask 9- Pregnant Teacher in her 20th week reported 2 of her student developed meningitis. Prophylactic Treatment: a) Observe for the sign of meningitis b) Meningitis Polysaccharide vaccine c) Ceftriaxone 500mg P0 once one IM dose of ceftriaxone (250 mg). d) Cefuroxime 250 mg IM or IV once e)Rifampicine 600 mg BD for 2 days 10- in a vesicular mole: a)B-hCG is lower than normal b)fundal height in lower than normal c)fetal heart can be detected d)ovarian cyst is a common association e)hypothyroid symptoms may occur

11- diagnosis of hydatidiform mole can be made accurately on the basis of: a- Elevated B-hCG. b- Pelvic U/S. c- Pelvic exam. d- Chest radiograph. e- Absence of fetal heart tones in a 16 weeks size uterus. - definitive therapy for hydatidiform mole is most commonly: a- Evacuation.
| P a g e

b- Abdominal hysterectomy. c- Evacuation followed by methotrexate therapy. d- Evacuation followed by hysterectomy. e- Radiation. - evacuation of hydatidiform mole may be complicated by: a- Hemorrhage necessitating transfusion. b- Acute respiratory distress. c- Both. d- Neither. - Following evacuation of a molar pregnancy, B-hCG titers will fall to undetectable levels in about 90% of pt within: a-2wk. b-4wk. c-8wk. d- 10 wk. e- 12-16 wk. - After the B-hCG titer become undetectable, the pt treated for hydatidiform mole should be followed with monthly titers for a period of: a- 3 months. b- 6 months. C- 1 yr. d-2 yr. e-5 yr. 12- Anti D Ig not given to a pregnant if: a) 25-28wk b) antiDAb titer of 1:8 already the mother sensitized against Rh + blood
| P a g e

c) after amniocentesis d) after antepartum hemorrhage e) after chorion villi biopsy

13- Cord prollapse caused by all of the following except: Premature labor Malpresentation PROM Twin pregnancy Oligohydramnious -A cord prolapse occurs in all except: a) Premature rupture of membranes. b) Preterm delivery with rupture of membranes. c) Oligohydramnios. d) Head high in pelvis. 14- -fetoprotein increase in all except: Myelomeningocele Spina bifida Encephalitis Breach presentation 15-The following are risk factors of puerperal infection except: a) endometriosis b) cervical laceration c) haemorrhage d ) anemia
| P a g e

e) retained placenta -All of following can increase risk of puerperal infection, except : a) Hemorrhage b) RPOC ?? (maybe EPOC evac. Product of concepts) c) Endometriosis d) Anemia 16- All the following drugs should be avoided in pregnancy except: a) Na Valproate. b) Glibenclamide. c) Keflex. d) Septrin. e) Warfarin. 17 -In diabetes in pregnancy: a) Oligohydramnios occurs. b) Hypoglycemia occurs in the baby after delivery. c) Hypercalcemia occurs in the baby. 18- Ectopic pregnancy,all true except: a) Occur ovarian in 20% b) Empty uterus by u/s with high beta-HCG before 12 wks c) Beta -HCG double of normal 19- DIC occur in all , except: a) Abruptio placenta b) Fetal death c) DM d) Pre-eclampsia

| P a g e

20- Pregnancy induced HTN, all true except: a) Ankle edema b) Polyuria c) Exaggerated reflex d) RUQ pain 21- Pyelonephritis in pregnancy, all true except: a) Gentamycin is drug of choice b) Abruptio placenta should be ruled out may be as it cause backpain c) E .coli common organism d) Should be treated even for asymptomatic

22- The following drug can be used safely during pregnancy: a) Septrin b) Cephalexin c) Tetracycline d) Aminoglycoside e) Cotrimoxazol 23- Indication of hepatitis during pregnancy is high level of: a) WBC b) Alk phospht c) SGOT d) BUN 24- Which of the following does not cause IUGR: a) Toxoplasmosis.
| P a g e

b)CMV. c)Rubella. d) Syphilis. ?? e)HSVII. ?? 25- One drug of the following can not cross the placenta: a) Heparin. b) Warfarin. c) Aspirin.

26- A16 year old pregnant,which of the following is the least likely to be a complication of her pregnancy: a) Anemia b) Pelvic complication c) Toxemia d) Low birth wt infant e) Infant mortality 27- Regarding HSV II, all are true, except : a) Use of fetal scalp electrode increase the risk of infection. 28- Toxemia in pregnancy, all are true except : a) More in primigravida than multigravida. b) More in multiple pregnancy. c) can progress rapidly to toxemia.

29- The safest Rx of Chlamydia trachomatis during pregnancy is:


| P a g e

a) Nitrofurantoin. b) Azithromycin. c) Erythromycin base. ?? d) Tetracycline. e) Erythromycin sulfate. ??

30- An Rh- ABO incompatible mother delivers an Rh+ infant at term and does not receive Rh immune globulin. The probability of detection of anti-D antibody during her next pregnancy is about: a. 2% b. 5% c. 10% d. 16% e. 25%

31- The class of antibody responsible for hemolytic disease of the newborn is: a. lgA b. lgD c. IgM d. IgE e. IgG . 32- All of the following are seen in utero with alloimmune hydrops except : a. Anemia b. Hyperbilirubinemia
| P a g e

c. Kenicterus d. Extramedullary hematopoiesis e. Hypoxia 33- An Rh - woman married to an Rh+ man should receive Rh immunoglobulin under which of the following conditions? a. Ectopic pregnancy b. External cephalic version c. Both In any condition she should receive Rh immunoglobulin (Prophylactic) d. Neither 34- Generally accepted cutoff values for plasma glucose on the 100 gm, 3- hour glucose tolerance test in pregnancy (according to the National Diabetes Group) include all of the following except : a. fasting glucose > 90 mg/dl b. fasting glucose 105 mg/dl c. 1 hour value 190 mg/dl d. 2 hour value 165 mg/dl e. 3 hour value 145 mg/dl

35- The prevalence of gestational diabetes in the general population is about: a. 2% b. 4% c. 8% d. 15% e. 20%

| P a g e

36- Gestational diabetes is associated with a. Increased risk of spontaneous abortion b. Increased risk of fetal cardiac malformation c. Increased risk of fetal CHS malformation d. Intrauterine growth restriction e. Decreased head circumference abdominal circumference ratio Look at the table

37- Infants of mothers with gestational diabetes have an increased risk of all of the following except: a. Hypoglycemia b. Hyperglycemia c. Hypocalcemia d. Hyperbilirubinemia e. Polyeythemia 38- Gestational diabetes is associated with an increased risk of all of the following except : a. Cesarean section b. Shoulder dystocia c. Fetal macrosomia d. Intrauterine fetal death e. Intrauterine growth restriction 39- Infants of mothers with gestational diabetes are at increased risk of becoming a. Obese adults b. Type II diabetics c. Neither d. Both

| P a g e

40- Control of gestational diabetes is accomplished with all of the following except: a. Insulin b. Diet c. Oral hypoglycemic agents d. Exercise 41- Ectopic pregnancy can be ruled out with a high degree of certainty if: a- The pt has no adnexal tenderness. b- B-hCG level is <6,000. c- The uterus measures 6 wk size on bimanual exam. d- An intrauterine gestational sac is observed. e- Tissue is observed in cervical os. 42- the most common antecedent cause of ectopic pregnancy is: a- Salpingitis. b- Congenitally anomalous tube. c- Endometriosis. d- Tubal surgery. e- Previous sterilization. 43- The majority of ectopic pregnancies occur in the: a- Ampullary tube. b- Ovary. c- lsthmic tube. d- Cervix. e- Fimbriated (distal) tube.

| P a g e

44- The most common presenting symptom of ectopic pregnancy is: a- Profuse vaginal bleeding. usually spotting not profuse b- Abdominal pain. c- Syncope. d- Dyspareunia. e- Decrease pregnancy associated symptoms. 45- Acceptable management of possible ruptured ectopic pregnancy would include all of the following except: a- Exploratory laparotomy. b- Diagnostic laparoscopy followed by observation. c- Partial salpingectomy. d- Total salpingectomy. e- Observation followed by methotrexate. 46- If the above described pt has had a previous term pregnancy prior to her current ectopic pregnancy, her chances of subsequent intrauterine pregnancy would be about: a- 80%. b- 60%. c- 40%. d- 20%. e- <10%. 47- A syndrome seen in preeclamptic women called HELLP syndrome is characterized by all of the following except: a- Elevation of liver enzymes. b- Hemolysis.
| P a g e

c- Low platelet count. d- Prolongation of the pro thrombin time. 48- The most common presenting prodromal sign or symptom in pt with eclampsia is: a- RUQ abdominal pain. b- Edema. c- Headache. d- Visual disturbance. e- Severe hypertension. 49-The most consistent finding in pt with eclampsia is: a- Hyper reflexia. b- 4+ proteinuria. c- Generalized edema. d- DBP >ll0 mmHg. e- Convulsions. 50- Appropriate responses to an initial eclamptic seizure include all of the following except: a- Attempt to abolish the seizure by administering I.M diazepam. b- Maintain adequate oxygenation. c- Administer MgSO4 by either l.M or l.V route. d- Prevent maternal injury. e- Monitor the fetal heart rate. 51- Eclampsia occurring prior to 20 wk gestation is most commonly seen in women with: a- Hx of chronic hypertension. b- Multiple gestation. c- Gestational trophoblastic disease (molar pregnancy). d- Hx of seizure disorder.
| P a g e

e- Hx of chronic renal disease.

52- Of the following, the most common complication of eclampsia is: a- Magnesium intoxication. b- Recurrent seizures following administration of magnesium sulfate??. c- Intracranial hemorrhage. d- Maternal death. e- Pulmonary edema. Recurrent seizures: About 10% of women with eclampsia will have an additional seizure after receiving magnesium sulfate. 53- If a woman with preeclampsia is not treated prophylactically to prevent eclampsia; her risk of seizure is approximately: a- 1/10 b- 1/25 c- 1/75 = 1.25 % d- 1/200 = 0.5 % e- 1/500 Eclampsia occurs in 1% of patients with preeclampsia . 54- Likely contributory mechanisms of the anticonvulsant action of MgSO4 include all of the following except: a- Neuronal calcium-channel blockade.
| P a g e

b- Peripheral neuromuscular blockade. c- Reversal of cerebral arterial vasoconstrictions. d- Inhibition of platelet aggregation. e- Release of endothelial prostacyclin. 55- All of the following antihypertensive medications are considered safe for short term use in pregnancy except: a- CaptopriL b- Methyldopa. c- Hydralazine. d- Nifedipine. e- Labetalol. 56- The reason to treat severe chronic hypertension in pregnancy is to decrease the: a- Incidence of IUGR. b- Incidence of placental abruption. c- Incidence of preeclampsia. d- Risk of maternal complication such as stroke. 57- pregnant pt with hepatitis, Dx. By: a) GOPT b) BUN body urea nitrogen c) WBC 58- risk factor for HSV II accusation in infants all of the following except : a) Cervical transmission is commoner than labial b) Maternal first episonde is of greater risk than recurrence c) Maternal antibodies against HSV I protect from HSV II d) Head electrodes increase risk of infection

| P a g e

- Risk factors for HSV2 in infants include all the following except: A-Cervical transmission is commoner than labial transmission 5-Maternal first episode is of greater risk for infants C-Maternal antibodies for HSV I protects against HSV2. D-Head electrodes increased the risk of infection.

59- PET: (Pre-eclampsia) a) Commoner in mutipara than primigravida b) Mostly in diabetic c) Hedache and blurred vision d) Progress very fast to eclampsia 60- drug avoided in pregnancy, EXCEPT: a) Cotrimox b) Cephalexin c) Glibenclaniide d) Na+ valporate e) Doxicyclin 61- One of the following drugs is safe in pregnancy a- Metronidazole is unsafe in first trimester b- Chloramphenicol in last trimester c- Erythromycin estolate is safe in all trimesters d- Nitrofurantoin 62- One of the following drugs can NOT cross the placenta: a- Reparm heparin b- Warfarin

| P a g e

63- Pregnancy induced hypertension, all true except: - Use of birth control pills increases the risk - Common in primigravida - Changing the partner increases the risk 64- Vomiting in pregnancy, all true except: - Hospital admission causes it - More in molar pregnancy - More in pregnancy induced hypertension 65-Pregnant patient with hepatitis: A-SGPT; ALT B-SCOT; AST C-BUN D-WBC 66- Which of the following drugs does not cross the placenta? A -Heparin B-Chloramphenicol C-Tetracycline D-Warfarin E-Diazepam F-Aspirin 67- IUGR can be caused by all of the following except: A-Syphilis B-CMV C-Toxoplasmosis D-Herpes simplex II E-Rubella 68-Drugs that should be avoided during pregnancy include all of the following except:
| P a g e

A-Cotrimox B-Cephaellne C-Na valproate D-Doxicyclin E-Glibenclamide 69- All of the following drugs should be avoided during pregnancy except: 1. Navalproiate. 2. Warfarin 3. Glibinclarnide 4. Septrin 5. Ceflex 70- The following are risk factor for puerperal infection, except: 1) Endometriosis 2) Cervical laceration 3) Anemia 4) Retained placenta 5) Hemorrhage 71- Preeclampsia, PET: A-Commoner in multipara B-More in diabetics C-Headache and blurred vision D-Rapidly progresses to eclampsia 72 -Compare with Type II diabetes, Type I diabetes is associated with all of the following except : a. Greater incidence of preeclampsia b. Greater incidence of preterm delivery c. Greater risk of maternal hypoglycemia d. Greater risk of maternal diabetic ketoacidosis e. Reduced risk of intrauterine growth restriction
| P a g e

73- Gestational diabetes substantially increases the mothers risk for the ultimate development of : a. Type I diabetes b. Type II diabetes c. Neither d. Both 1- A post transvaginal hysterectomy having vaginal urine dripping during micturition Dx: vesicovaginal fistula. urethrovaginal fistula. ureterovaginal fistula. - A 30 year old female with history of vaginal hysterectomy with anterior and posterior repair, is complaining of urine coming from the vagina during micturation the diagnosis is: a) Uretro-vaginal fistula b) Urethro-vaginal fistula c) Recto-vaginal fistula 2- Post D&C the most common site of perforation is the: the fundus. ant. wall of the corpus. post. wall of the corpus. lat. wall of the corpus. cervix. 3- The most frequent complication of cesarean hysterectomy is: a. Pelvic cellulites. b. Pulmonary embolism.
| P a g e

c. Bladder injury. d. Hemorrhage. e. Ureteral injuries. -The most frequent complication associated with caesarian hysterectomy is: a. Pelvic cellulites b. Pulmonary embolism c. Bladder injury d. Haemorrhage e. Uretheral injury 1- most of the causes of Postpartum infection: a) Anemia which is most probably the cause during pregnancy b) Retained placenta c) Hemorrhage during pregnancy d) Endometriosis 2- gonococcal infection: a) Less common in females with IUCD b) Causes permanent tubal blocking c) No need for laparoscopy for further diagnostic evaluation -Gonoccocal infection which is true: A-Less common in females with IUCD. B-Causes permanent tubal blocking. C-No need for laparoscopic for further evaluation 3- a 24 years old female pt. C/O : gray greenish discharge , itching .. microscopic examination of discharge showed : flagellated organism most likely diagnosis is : a- trichomoniasis ( trichomonas vaganalis )

| P a g e

4- Most common site of gonococcus infection in females in: cervix. posterior fornix. urethra. - The site most likely to yield gonococci in women is: A. The cervix. B. The urethra. C. The rectum. D. The pharynx. E. Posterior vaginal fornix - A 38 year old lady with gonorrhea. Where is the most likely site for isolation of the organism? a. Pharynx b. Cervix c. Urethra a Vagina a Posterior fornix 6- Trichomanas vaginalis all true except: a) Generally STD b) Causes cytological cervical discharge c) Common in DM d) Caused by protozoa - about vaginal trichomonosis all the following are true except : a-common in diabetics b-is protozoal infection c-diagnosed by wet smear d-treated by metronadazol - Vaginal trichomoniasis, all are true, except : a) More in diabetic.
| P a g e

b) Protozoal infection. c) Dx by microscopic examination of diluted vaginal smear. d) Rx by Metronidazole 7- All of the following are normal flora and should not treated, except: a) trichomonas b) candida c) E.coli d) fragmented bacteria 8- PID (pelvic inflammatory dis) , all true , except: a) Infertility b) Endometriosis d) Dysparunia c) Can be treated surgically 9- Vulvovaginal candidiasis a. Cause muco-purulent cervicitis b. Frequently associated with systemic symptoms c. May be diagnosed microscopically by mixing discharge with KOH d. Is treated with doxycycline e. Is one of sexually transmitted infections 10- Bacterial vaginosis a. Is a rare vaginal infection b. Is always symptomatic c. Is usually associated with profound inflammatory reaction d. Causes fishy discharge which results from bacterial amine production e. Is treated with clotrimazole 1- A 25 years old female patient who is with 2ry amenorrhea, her prolactin level is 400 ng/ml. the
| P a g e

probability to have pituitary prolactin secrecting adenoma is: a. <25. b. 25-49 c. 50-74 d. 75-85 e. >85 2- hyperprolactinemia associated with all of the following except: a. Pregnancy b. Acromegaly c. OCP d. Hypothyroidism 3- Dyspareunia caused by all of the following except: a) cervicitis b) endometriosis c) lack of lubricant d) vaginitis e) uterine prolapse 1- a 62 yrs. old female pt. a known case of osteoporosis & on 1 alpha + Ca supplement .. her lab works shows normal level of PO4, Ca & ALP her Xray shows osteopenia with SD = -3.5 . The best action is to : a- continue on same medications b- start estrogen c- start estrogen & progesterone d-add alendronate (Bisphosphonate) 2- 60 Y/O lady on OCP 21 days a month having recurrent vaginal bleeding (spotting) after the stop of estrogen, best Tx:
| P a g e

endometrial Bx. papsmear of the cervix. add progestone. stop estrogen. abdominal US. or laparoscope. 3- Regarding menopause, one of these is a major health problem: a. Cardiovascular disease b. Depression C. Osteoperosis d. Endometrial carcinoma 4- hormone replacement therapy is contraindicated in which of the following conditions: a- Chronic hypertension. b-Type I DM. c- Previous MI. d- Hypercholesterolemia. e- Recurrent deep vein thrombophlebitis. 5- Surveillance of pt on hormone replacement therapy includes all of the following except: a- Blood pressure. b- Breast examination. c- Glucose tolerance test. d- Pelvic examination. e- Endometrial sampling in the presence of abnormal bleeding. 6- Age of menopause is predominantly determined by: a- Age of menarche.
| P a g e

b- Number of ovulation. c- Body mass index. d- Socioeconomic status. e- Genetics 7- All of the following result from combined estrogenprogestin replacement therapy except: a- Decrease the risk of osteoporosis. b- Relief of vasomotor symptoms. c- Relief of dyspareunia. d- Increase the risk of coronary artery disease. d- Decrease the risk of coronary artery disease. 8- All of the following r known to increase the risk of osteoporosis in the postmenopausal women except: a- Early menopause. b- Cigarette smoking. c- Low calcium intake. d- Sedentary life style. e- Black race.

9- all of the following are characteristic changes seen in menopause except: a- Decrease body fat. b- Decrease skin thickness. c- Increase facial hair. d- Decrease collagen content in the endopelvic fascia.

4- Infertility due to endometriosis ,Rx: Progesterone Danazole


| P a g e

Radiotherapy Surgical ablation of lesions is often performed at the time of diagnostic laparoscopy. 5- A 16 yrs old female presents to your office with a chief complaint of never having had a menstrual period. She had never had a pelvic exam. Physical exam reveals the following BP 11O/7O P 72b/m wt6Okg Ht l72 cm .The patient appears her stated age. Axillary and pubic hair are scant. Breasts are tanner stage IV. External genitalia are normal female . A mass is palpable within the inguinal canal. Pelvic exam reveals an absent cervix with the vagina ending in a blind pouch. the uterus and overaies are difficult to delineate. The most likely diagnosis is: a) hypothalamic amenorrhea. b)prolactin secreting adenoma c)polycystic overian syndrome d)turner syndrome e)androgen insensitivity syndrome - confirmation of your diagnosis would be most readily obtained by ordering the following test: a)diagnostic laproscopy b)pelvic ultrasound. c)pelvic CT. d)karyotype e)MRI of pituitary -Karyotype is performed on the patients peripheral blood lymphocytes .The karyotype is most likely a)46xx b) 45x c)46xy d)46xx, 45x
| P a g e

e)47xxy - the hormone profile in this patient would include all of the following except : a)elevated LH b)elevated estradiol for a male c)normal to elevated FSH d)normal to slightly elevated testosterone for a male. e)normal testosterone for a female.

-The inguinal mass most likely represents a) a uterus b)an ovary with arteric follicles c)a testis with hyperplastic leyding cells and no evidence of spermatogenesis d)a herniated sac containing a peritoneal contents. - the most long term treatment would be: a) total abdominal hysterectomy b)estrogen replacement therapy c)androgen replacement therapy d) ooporectomy - without surgery, this patient is at risk to develop: a)gonadoblastoma. b)dysgerminoma c)neither d)both .

| P a g e

- All of the following are true about this patient except: a) H-Y antigen is present b) these patients are always sterile c)antimullerian hormone is present d)normal levels of dihydrotestosterone e)clitromegaly may develop later in life. 1- a 62 yrs. old female pt. a known case of osteoporosis & on 1 alpha + Ca supplement .. her lab works shows normal level of PO4, Ca & ALP her Xray shows osteopenia with SD = -3.5 . The best action is to : a- continue on same medications b- start estrogen c- start estrogen & progesterone d-add alendronate (Bisphosphonate) 2- 60 Y/O lady on OCP 21 days a month having recurrent vaginal bleeding (spotting) after the stop of estrogen, best Tx: endometrial Bx. papsmear of the cervix. add progestone. stop estrogen. abdominal US. or laparoscope. 3- Regarding menopause, one of these is a major health problem: a. Cardiovascular disease b. Depression C. Osteoperosis d. Endometrial carcinoma

| P a g e

4- hormone replacement therapy is contraindicated in which of the following conditions: a- Chronic hypertension. b-Type I DM. c- Previous MI. d- Hypercholesterolemia. e- Recurrent deep vein thrombophlebitis. 5- Surveillance of pt on hormone replacement therapy includes all of the following except: a- Blood pressure. b- Breast examination. c- Glucose tolerance test. d- Pelvic examination. e- Endometrial sampling in the presence of abnormal bleeding. 6- Age of menopause is predominantly determined by: a- Age of menarche. b- Number of ovulation. c- Body mass index. d- Socioeconomic status. e- Genetics 7- All of the following result from combined estrogenprogestin replacement therapy except: a- Decrease the risk of osteoporosis. b- Relief of vasomotor symptoms. c- Relief of dyspareunia. d- Increase the risk of coronary artery disease. d- Decrease the risk of coronary artery disease. 8- All of the following r known to increase the risk of osteoporosis in the postmenopausal women except:
| P a g e

a- Early menopause. b- Cigarette smoking. c- Low calcium intake. d- Sedentary life style. e- Black race.

9- all of the following are characteristic changes seen in menopause except: a- Decrease body fat. b- Decrease skin thickness. c- Increase facial hair. d- Decrease collagen content in the endopelvic fascia. 1-20 year lady come to ER with Hx of Rt severe lower abdominal pain with Hx of amenorrhea for about 6 wk the most serious diagnosis of your deff. Diagnosis could reach by: (means ectopic pregnancy) a- CBC b- ESR c- U/S of the pelvis d- Plain X-ray e- Vaginal swab for C/S 2- Secondary amenorrhea: a.is Part of Sheehan syndrome. b. Commonly associated with turner syndrome. c. Always pathological. d. May be due to gonadal agenesis. e. It is rare to be gonaal agenesis.

| P a g e

3-15 yo female, menarche at age of 13 , complaining of menstrual pain, not sexually active O/E and pelvic US: Normal , Rx is: a. Laprotomy b. Danazol c. Cervical dilatation d. NSAIDs - A 15 years old girl her menarche was at age of 13 years. She is complaining of menstrual pain. She is not sexually active. Her examination & pelvic US were normal. How are you going to manage her? a- laprotomy. b- danazol. c- cervical dilatation. d- NSAID. 4- The average length of menstrual cycle is: -22 days -25 days -28 days -35 days -38 days 5- Premenstrual tension a) more in the first half of menses b) 60% associated with edema c) associated with eating salty food d) menorrhagia 6- Primary amenorrhea due to: a) Failure of canalization of mullarian duct b) Kallmann syndrome c) Agenesis d) All of the above e) Non of the above
| P a g e

Primary amenorrhea is the failure of menses to occur by age 16 years, in the presence of normal growth and secondary sexual characteristics. If by age 13, menses has not occurred and the onset of puberty, such as breast development, is absent, a work-up for primary amenorrhea should start. 9- Amenorrhea is a feature of all, except : a) Hypothyroidism. b) Stein-Leventhal syndrome. ( PCOS Poly Cystic Ovarian Syndrme) 1- In twins all true, except : a) Dizygote more common than monozygote b) In dizygote more twin-to twin transfusion c) Physical changes double time than single form d) U/S can show twins

1- It is C/I to stop preterm delivery in the following condition: aminochoronitis. placental abruption. preeclampsia. a & b. 2- Before you start instrumental delivery it is important to check if there is: face presentation. CPD. breech presentation. cord prolapse.
| P a g e

- Before doing instrumental delivery you have to exclude: a. Cord prolapse b. Cranio-pelvic disproportion C. Breech d. Abruption placentae -Before any instrumental delivery we should rule out: a)cord prolapse b)cephalopelvic disproportion c)face presentation?? d)placental abruption 3- In occipitoposterior malpositioning of the fetal head, all of the following are true except: all true 10% if all vertex deliveries. it causes significant delay of labor duration compared to the anterior presentation. android pelvis is a predisposing factor. flexion of the head helps the rotation to the ant. position. 4-After delivery start breast feeding: a. As soon as possible d. 36hrs b. 8hrs e. 48hrs c. 24hrs 5- In lactation all true, except : a) Sucking stimulate prolactin b) Sucking cause release of oxytocin c) Milk release deacreased by overhydration 6- Not correct during management of labour a) Intensity of uterine contractions can be moniterd manually.
| P a g e

b) Maternal vital signs can vary relative to uterine contractions. c) Food & oral fluid should be withheld during active labor d) Advisable to administer enema upon admission e) IVF should be administered upon admession 1- 33 yo with multiple C sections, 6th day post op, clothes stained with copious serosangious dishcahrge: -Vesicocutaneous fistula -Entercutaneos fistula - Hematoma -Stitch abscess - Wound dehiscence. 1- Which of the following suggests enormous ovarian cyst more than ascites? a) Fluid wave b) Decrease bowel motion c) Shifting dullness d) Tympanic central, dullness lateral e) Dullness central, tympanic lateral 1- Perinatal mortality a) Includes all stillbirth after the 20th week of pregnancy b) Includes all neonatal deaths in the 1st 8 week of life c) Includes all stillbirths & 1st week neonatal deaths d) Specifically.... Neonatal Deaths. e) is usually death per 10,000 live births

| P a g e

the number of lives lost during the 5-month period from the 20th week of gestation to the 7th day after birth - Perinatal mortality: a) Includes all stillbirth after 30 weeks b) Includes all stillbirth and neonatal deaths in the 1st week per 1000 lives and stillbirths c) Includes all neonatal deaths up to 6 weeks d) Characteristically excludes post natal deaths e) It is deaths per 10,000 live birth 2- perinatal asphyxia could be caused by all except : a) Abruptio placenta b) Hyperemisis gravidium c) Pre-eclampsia 1- A 35 year old lady G4, P3+1 presented with 1 year Hx of irregular, heavy menstrual bleeding. Physical examination was normal. The most likely diagnosis is: a. Nervous uterus b. Early menopause C. Dysmenorrheal with heavy bleeding d. DUB=dysfunctional uterine bleeding e. Endometriosis - A 35 y.o female G4 P2 + I , 1 yr hx of irregular heavy bleeding, Examination within normal .Most likely Dx? a. Early menopause b. Narvouse utenis c. Dymsmennorreah d. DUB e. Endometriosis

| P a g e

1- 45 year old female complaining of itching in genitalia for certain period, a febrile, -ve PMH, living happily with here husband since 20 year ago on examination no abdominal tenderness , erythema on lower vagina , mild Gray discharge no hx of UTI . pyleonephritis Most probable diagnosis: a- Vaginitis b- Cystitis c- CA of vagina d- Urithritis ( non gonococal ) 2- Rx of bacterial vaginitis a) ampicillin b) tetracycline c) metronidazol d) erythromycin 3- atrophic vaginitis is best treated with: a- Oral metronidazole. b- Topical miconazole. c- Prolonged application of topical estrogen. d- Oral estrogen & progestin therapy. e- Topical sulfa cream. 1-Placenta previa excludes : a- Painless vaginal bleeding b- Tone increased of uterus c- Lower segmental abnormality d- Early 3rd trimester - Placenta previa, all true except : a) Shock out of proportion of bleeding b) Malpresentation c) Head not engaged d) Painless bleeding
| P a g e

2- A 34wk GA lady presented with vaginal bleeding of an amount more of that of her normal cycle. O/E utrine contracts every 4 min, bulged membrane, the cervix is 3 cm dilated, fetus is in a high transverse lie and the placenta is on the posterior fundus. US showed translucency behind the placenta and the CTG (Cardiotocography) showed FHR of 170, the best line of management is: C/S immediately. give oxytocin. do rupture of the membrane. amniocentisis. 3- PPH happens more commonly with: multiple pregnancies. anaemia. preterm delivery. antithrombin III deficiency. 4- The most common cause of Post Partum Hemorrhage is: -Uterine Atony -Multiple Pregnancy. -Pre Eclampsia. - Most important cause of immediate post partum hemorrhage: a) laceration of cervix b) laceration of vagin c) uterine atony d) placental fragment trtention - most common cause of postpartum hemorrhage: a) Uterin atony b) Retained placental fragment
| P a g e

c) Cervical laceration d) Nonofthe above 5-The most accurate Dx of Ectopic Pregnancy is: a. Culdocentesis b. Pelvic U/S c. Endometrial Biobsy d. Serial B-HCG e. Laproscopy 6-A pregnant women with anterior lateral placenta on US, examiner finger cannot reach the placenta . the placenta is: a) Low lying b) Marginal placenta Previa c) Partial Placenta Previa d) Lateral Placenta Previa 7- About antepartum hemorrhage: a) Need immediate assessment by vaginal exam b) Mother risk is more than fetal risk (bcz its one of the leading cause of maternal death) 8- U/S of pregnant lady showed posterior wall placenta. It dose not reach examining finger by vaginal exam. Which of the following is true? a) Complete placenta previa b) Normal site placenta c) Low lying placenta d) Placenta previa marginalis e) Incomplete centralis 9- Pt with post partum hemorhage & infertility, all can be found except : A) Balloning of sella turcica b) Decrease Na
| P a g e

c) Hypoglycemia d) Decreased T4 e) Decreased iodine uptake

thyroid

10- The most dangerous symptom during pregnancy is: PV bleeding Ankle swelling Hyperemesis Cramps 11- Physical exam reveals the uterus to be about 6 wk size. Vaginal bleeding is scant with no discernible tissue in the cervical os. There are no palpable adnexal masses. The uterus is mildly tender. Ultrasonographic exam does not reveal a gestational sac. Which of the following should be recommended? a- Dilatation & curettage. b- Culdocentesis. c- Observation followed by serial B-HCG determinations. d- Diagnostic laparoscopy. e- Laparotomy. 12- If the above pt presented at 8 wk gestation & pelvic exam revealed unilateral adnexal tenderness w/o discernible mass, consideration should be given: a- Observation. b- Culdocentesis. c- Laparoscopy. d- Dilatation & curettage. e- Laparotomy. 3-Most common cause of post-partum bleeding: A-Uterine atony
| P a g e

5-Laceration C-Retained placental tissue D-Uterine inversion Pediatric 1- Symptoms of cystic fibrosis in neonate: a) meconium ileus b) pneumothorax c) steatorrhea d) rectal prolapse 2- IV LR given at age of: a) 3 months b) 8 months c) 12 months d) 24 months 1- The child can walk without support in: a. 6 months b. 9 months c. 15 months d. 18 month -Children are expected to walk without support at age of: a) 6 months b) 9 monthes c) 15 monthes d) 18 monthes e) 20 monthes 2- 8 years old boy, has 6 year old height & bone scan of 5.5 years .Dx is: a. Steroid b. Genetic c. Hypochondroplasia d. Hypothyroidism
| P a g e

e.constitutional 3-12 month old baby can do all except: a. Walk with support of one hand b. Pencil grip c. Open a drawer d. Respond to calling his name e. Can play simple ball 4- Which of the following physical findings in boys is the earliest indication that the puberty has begun? a) Increasing Prostatic size b)Appearance of upper lip hair c) Increasing Penis size d) Increasing Testicular size e) Appearance of pubic hair. 5- An 18 months old baby brought by his mother. She complains that her child says only mama & baba. Otherwise the baby is completely normal. First step to evaluate this patient is: a) Physical Examiniation. b) Chromosomal Analysis c) Hearing Evaluation. d) Developmental testing. e) CT scan of the head. - 18 month-old pt ,the mother complain that pt is saying only mama baba , no other words .pt otherwise completely normal. 1 step to evaluate: a) Physical examination b) Hearing test c) Developmental test d) Test speech 6- An 18-month-old child is found to have dental decay in the upper central and lateral incisors. This is most suggestive of:
| P a g e

A. Excessive fluoride ingestion. B. Milk-bottle caries. C. Tetracycline exposure. D. Insufficient fluoride intake. E. Failure to brush the childs teeth properly. 8- All are normal in association with teething EXCEPT: a) Rhinorrhea b) Diarrhea c) Fever>39C d) Irritability 9-acute gait disturbance in children; all are true EXCEPT: a) Commonly self limited b) The usual presenting symptom is limping c) Radiological investigation can be reveal the DX d) Most often no cause can be found -Acute gait disturbance in children, all of the following are true EXCEPT:a- Commonly self limiting b- Usually the presenting complaint is limping c- Radiological investigation can reveal the Dx d- most often there is no cause can be found 10- All can cause short stature, EXCEPT: a) Hypothyroidism. b) Turner syndrome. c) Down syndrome. d) Klinefilelter syndrome. - All the following can cause small stature in children except: a-hyopthyrodism
| P a g e

b -tunner syndrome c-klinefenter syndrome d-down syndrome 11- Development in children, all are true EXCEPT: a) At 1 yr, can feed himself by spoon. - About the development of the child, all true except: - 1 year child can feed himself by spoon - Grasp smoothly by 8 months - Roll over by 6 months - Say at least 5 more words other than mama and baba by 1 year and 5 months 12- an 18 months old baby can typically: a) Feed himself by a spoon b) Say a vocabulary of approximately 10 words c) Build tower of 10 bricks d) Drinks by a cup - 18 months baby can typically do the following except: A-Have a vocabulary of 10 words B-Build a ten brick tower. C-Drink from a cup. D-Feed himself with a spoon. 1- a full term baby boy brought by his mother weighing 3.8 kg. developed jaundice at 2nd day of life .. coomb's test ve ,Hb : 18 gm/dl ,billrubin : 18.9 & indirect : 18.4 O/E : baby was healthy and feeding well .. the most likely diagnosis is : a- physiological jaundice b- ABO incompatibility c- breast milk jaundice d- undiscovered neonatal sepsis
| P a g e

- A full term infant brought by his mother weighing 3800 gm developed jaundice on the 2nd day of life. The infant appears healthy & breast-fed well. Hb 18 g/dI. Direct & indirect coombs tests are negative. T.Bili l89 mol/L (11gm/dl). Indirect bili 184 mol/L (10.7 gm/dl) . The most likely Dx is: a) Undiagnosed neonatal sepsis. b) Breast milk Jaundice. c) Physiological Jaundice. d) Jaundice due to minor blood group incompatibility. e) ABO boold group incompatibility. - Full term baby born with a birth weight of 3.8kg, developed jaundice on 2nd day of life he is on breast feeding his Hb l80 gm/L Direct Coombs test -ve Indirect Coombs test -ve Total bilirubin 11gm/dl Indirect bilirubin l84mmol/L (10.7 gm/dl) The Diagnosis is: a. Breast milk jaundice b. Undiagnosed neonatal sepsis C. Physiological iaundice d. Due to minor antigen blood group incompatibility a ABO incompatibility -A full term boy , wheighing 3.8 , developed jaundice at 2nd day of life, Coombs test is -ve, JIB 18, bilirubin 18.9 gm /dl & indirect 18.4 gm /dl O/E:baby is healthy and feeding well, the most likely Dx is: a. Physiological jaundice b. ABO compatibility c. Breast milk jaundine d. Undiscovered neonatal sepsis

| P a g e

2 - Term baby born to a mother who developed chickenpox 7 days before delivery. The baby is asymptomatic. Which is true a- give acyclovir & 15 mg 1kg l.V Q 8 hr. for 7 days immediately.?? b- give acyclovir & varicella zoster immune globulin when the baby develops symptoms. c- serologic evidence is needed before initiation of therapy. d-the mother & baby should be nursed together at their own room. e- none of the above 3- A 48 hour old newborn infant in critical care unit with respiratory distress & Jaundice.Hb 9g/dl, retic 4%. Maternal Hx of previous normal term pregnancy without transfusion, Blood typing shows hetero specificity between mother and child. Indirect Coombs test is +ve. The most probable Dx is: a) Thalassemia b) Maternal-Fetal blood group incompatibility c) Sickle cell anemia. d) Septicemia. e) Hereditary . 4-Risk factor of sudden death syndrome include all of the following, except : a) cigarette smoking during pregnancy b) old primigravida c) crowded living room d) prematurity e) small gestational age 5- To prevent tetanus in neonate: a) give anti-tetanus serum to neonate b)give immunoglobulin to mother
| P a g e

c) give tetanus toxoid to mother d) give antibiotics to mother e) give penicillin to child to kill tetanus bacilli 6- Hypothyroid in young baby usually due to: a) endocrine irresponse b) enzyme def c) drug by mother d) agenesis 7- Apgar score: a) Heart rate is an important criterion. b) Is out of 12 points. 10 points only c) Gives idea about favoribilty of vaginal delivery. d) Taken at delivery time and repeated after 5 minutes. After delivery at 1 & 5 minutes e) Respiratory rate is an important criterion. Respiratory effort not rate is true - APGAR score: a) of 12 points. b) color is not important. c) Heart rate is important 11- Neonate with APGAR score 3 after 1 min, the most important action is: a) Chest expansion. b)Warm. c) Ventilation. d)Bicarb. Inj. - child with APGAR score 3 in 1 min, the most important step is: a) Ventilation b) Chest expansion c) Volume expansion d) Drying e) Warming 8- Neonatal just delivered , term pregnancy. Developed resp. distress CXR showed multicystic
| P a g e

lesion in Lt side shifted mediastinum to the Rt, decreased bilateral breath sound & flat abdomen: a) Diaphragmatic hernia b)RDS c) Emphysema 10- In neonates, the following need emergency Rx: a) Erupted teeth. ?? b) Hydrocephalus. c) Absent femoral pulse. - in new born ,the following needs immediate treatment: a-asymptomatic hydrocele b-erupted tooth c-absent femoral pulse 12- About Kernicterus, all are true except : a) Can occur even if neonate is 10 days old. b) It causes neurologicl abnormalities, it can be reversed. c) Can cause deafness. d) All types of jaundice cause it. - About kernicterus all are true, except : - Can occur even in late neonatal age . - Can be caused by all severe types of jaundice . - Cause neurological abnormality which can be reversed by Rx . 1- A mother brought her 16-month-old baby boy to the emergency room. She said the baby was crying on and off for about 24 hours, now he is passing currently jelly stool. The most likely diagnosis is: A. Necrotizing enterocolitis. B. Duplication of the Gut. C. Intussusception.
| P a g e

D. Bowel obstruction secondary to internal hernia. E. Meckels diverticulum. 2-child has tracheoesophageal fistula, all can be used in management, except a) Insertion of chest tube b) Insertion of NGT c) Pulmonary toilet d) Gastrostomy - management of tracheo-esophageal fistula all of the following EXCEPT: a) Chest tube b) Gastrrostomy c) Pulmonary toilet d) !! e) I.V antibiotics - Management of Tracheoesophageal fistula: A-Chest tube B-Gastrostomy C-IV antibiotics D-Pulmonary toilet E-IVG sump catheter 3- Child with imperforated anus the most useful diagnostic procedure is: a) Plain X-ray of abd. with child inverted position b) Plain X-ray abdomen

| P a g e

Epidemiology
1- For health education programs to be successful all are true except : a- human behavior must be well understood b- Information should be from cultural background c- Doctors are only the health educators d- Methods include pictures and videos (mass media) e- Involve society members at early stage -For health education programme to be effective all true except : a- Human behaviour should be well understood. b- Procedures used include illustration & picture. c- Doctors should be the only educator. d- Community member should be involved in early stage. 2- Evidence based medicine: as in text book. according to department policy. according to latest published articles. according to strong scientific evidence. - Evidence based medicine: a. Practice medicine as in the book b. Practice according to the department policy c. Practice according to Available scientific evidence d. Practice according to Facility e. Practice according to Latest published data
| P a g e

3- A disease lasts 2-3 wk with fatality rate of 30 %: incidence = prevalence. incidence> prevalence. incidence < prevalence. incidence = 1/2 prevalence. has no relation. 4- The most important way to prevent disease transmission b/w patients and health workers is: wear gloves for all patients. washing hands before and after examination. wearing a mask. needle container. -what is the best method for preventing infection from one patient to another & to health care workers? a)wearing gloves when examining every patient b)hand washing before & after each patient c)wearing a mask & gown before examining an infected person d)recapping needles & put them in a sharp container e)isolation of all infected persons - Standard precautions are recommended to be practiced by all healthcare workers (HCW) to prevent spread of infections among patient and HCW. Most important measure: A. Wearing gloves when examining any patient B. Hand washing before and after examining any patient C. Wearing masks and gowns before examining infected patients D. Recapping contaminated needles and disposing it in sharp container E. Isolating all infected patient in single rooms 5- Most common source of bacterial infection in l.V canula is: A. Contamination of fluids during manufacturing B. Contamination of fluids during insertion of the canula
| P a g e

C. Contamination at site of entry through skin D. Contamination during injection of medication E. Seeding from remote site due to intermittent bacteremia - A patient having an IV line developed an infection, what is the most important source? infected IVF. infection during the insertion of the line from the skin. bactermia. during changing IVF. -which of the following is the most likely cause of infection after IV fluid through a canula? a) infection of the fluid in the factory b) infection of the fluid during passing in the canula c) infection at the site of needle insertion d) disseminated infection due to transient bacteremia 6- Important tools for listening to a patient include: using tools for asking. imagination. using similar words and expressions as the patient. a sense of humor. all of the above. 7- In PHC which statement is correct: a. Use less lab. techniques. b. Examine each patient completely. c. Write prescription as patient remained. d. Reassurance of patient only at tertiary Hospital. 8- Definition of TRUE NEGATIVE. (Review FAMCO) - Def. of true negative: negative results that come back actually representing the absence of the disease. 9- A risk of assessment test is negative if individual is not at risk for developing the disease. True negative if: - Predict individual will develop disease & actually develop
| P a g e

it. - Predict individual will develop disease & he didnt develop it. - Predict individual will not develop disease & he develop it. - Predict individual will not develop disease & he didnt develop it. - If not assessed person develop the disease. -when a person id predicted NOT to have a disease he is called (negative).then what is true negative? a)when a person is predicted to have a disease,has it b) when a person is predicted to have a disease,doesnt have it c)when a person is predicted to not to have a disease,doesnt have it d) when a person is predicted to not to have a disease,has it e)when risk cannot be assessed 10- Definition of the following. (All with its right definition) except: -prevalence is -infant mortality rate is. -Incidence is A. Incidence and prevalence 1. The incidence of a disease is the number of new cases of that disease within a population over a specific time divided by the total number of individuals in the population. For example, if 100,000 people live in a certain town, and over a 1year period 50 people in the town develop lung cancer, the incidence rate for lung cancer in this town would be 50/100,000 per year. Specific incidence more accurately describes the incidence of a disease for a particular subset of the population by calculating the number of patients with the disease within a particular age group (termed age-specific incidence) or gender (gender-specific incidence).
| P a g e

2. The prevalence of a disease is the total number of existing cases of that disease within a population at a specific point in time divided by the total number of individuals in the population. Prevalence therefore differs from incidence in that it does not take into account when the disease developed, only if the disease exists or not. The relationship between incidence and prevalence will depend on the characteristics of the disease. For example, a disease that lasts a lifetime but does not shorten life span appreciably is likely to have a much higher prevalence than incidence because as more people develop the disease each year, the total number of patients with the disease will be very large, even if only a few people get iii each year.

11. Standard deviation is.... shows how much variation or "dispersion" exists from the average (mean, or expected value). A low standard deviation indicates that the data points tend to be very close to the mean, whereas high standard deviation indicates that the data points are spread out over a large range of values.

12-2ndry prevention is least useful in which disease ? a. Breast CA b. DM c. Leukemia d. Malnutrition of children e. Toxemia in pregnancy - Secondary prevention is best effective in: a) DM. b) Leukemia. c) Pre-eclampsia. d) Malabsorption if malnutrition
| P a g e

Secondary prevention is least likely of benefit in: a) Breast cancer b) Leukemia c)DM d) Toxemia of pregnancy 6) An example of secondary prevention is: a) Detection of asymptomatic diabetic pt b) Coronary bypass graft c) Measles vacc d)rubella vaccine 13-your asked to manage an HIV patient who was involved in a car accident.You know that this patient is a drug addict & has extramarital relations. What are you going to do? a)complete isolation of the patient when he is in the hospital b)you have the right to look after the patient to protect yourself c)you will manage this emergency case with taken all the recommended precautions d)you will report him to legal authorities... . after recovery e)tell his family that he is HIV positive 14-Using the following classification: Risk factor Present Absent Total Case A C A+C Non-case Total B A+B D C+D B+D

Relative risk of those with the risk factor to those without risk factor is: a- A/A+B / c/c+d b- C/C+D c- AD/BC d-A/B Relative risk = incidence among exposed / incidence among non exposed

| P a g e

15- Regarding smoking cessation, the following are true EXCEPT: A. The most effective method of smoking control is health education. B. There is strong evidence that acupuncture is effective in smoking cessation. C. Anti smoking advice improves smoking cessation. D. Nicotine replacement therapy causes 40-50% of smokers to quit. E. The relapse rate is high within the first week of abstinence. 16- Incidence is calculated by the number of: A. Old cases during the study period. B. New cases during the study period. C. New cases at a point in time. D. Old cases at a point in time. E. Existing cases at a study period. 17-The MOST effective method of health education is: A. Mass media. B. Group discussion. C. Internal talks. D. Individual approach. E. None of the above. - The best way for health education: a) Mass media. b) Interview. c) ?? 18- Communicable diseases controlled by: a) control the source of infection b) block the causal of transmition c) protect the susciptable pt d) all of the above e) non of the above Control of infection disease a) Control source of infection b) Block channel of transmissioin
| P a g e

c) Protect the receiver of infection d)All of the above 19- Treatment of contacts is applied in all of the following except: a) Bilharisiasis. b) Malaria. c) Hook worm. d) Filariasis. 20 -Prospective Vs Retrospective studies all are true EXCEPT: a) retrospective studies have more bias than prospective studies. b) in prospective studies, those who enter the group depend whether they the disease or not. c) prospective studies are expensive.

18 -prospective and retrospective studies, all are true EXCEPT: a) retrospective are typically more biased than prospective b) retrospective are typically more quickly than prospective c) prospective allocation of person into group is based on presence or absence of disease d) prospective in more in cost than retrospective e) effect is more identifiable in prospective 18- Comparing the prospective and retrospective studies, all are true except: A-Retrospective are typically more biased than prospective B-Retrospective studies are typically quicker than prospective C-Prospective allocation of person into group depends on whether he has the disease or not. D-Prospective costs more than retrospective.

| P a g e

E-Effect is more identifiable in prospective. 38- Secondary prevention is least effective in: 1. DM 2. Pre-eclarnpsia 3. Leukemia 4. Malabsorption in children 30- In ischemic heart disease a) Prevalence is the number of case discovered yearly b) Incidence is new cases yearly c) There is association between HTN & ischemic heart disease d) Smoking is an absolute cause if IHD 28- A 68 years old businessman recently diagnosed to have hepatocellular carcinoma . Regarding telling the patient about his Dx., one of the following is true: a. Inform him directly after confirm the diagnosis regardless his wishes. b. Only the patient family should be informed. c. 50% survival rate should be calculated as recent study and discussed with the patient. d. This responsibility should be conducted by the social worker. e. Informed the patient according to his morals, knowledge and psychiatric conditions. Mr. A is 68-year-old businessman diagnosed to have hepatocellular carcinoma. One is true regarding disclosure (informing patient): A. Patient should be told immediately after confirming the diagnosis regardless of his whishes B. Only patients family should be informed C. 50% suivival rate should be calculated according to literature and discuss with the patient D. Social worker should be responsible to tell the patient E. Patient morale and understanding should be studied before telling him
| P a g e

The End
Thanks for all doctors who participated in this edition To ensure that these editions are continue updated and published , please send your collection for your exam to SLEBank1@gmail.com And because I will busy in my residency programme , This edition will be the last edition for me as Team leader, and one of my colleague will lead the work . in the future inshallh . I hope I did a good work during the last period, what was right is from Allah, and what was false is from my self and devil

. Your brother: Dr.Abdullah Saleh Alhudaib ,27/8/2012 Dr-Abdullah-26@hotmail.com

| P a g e

You might also like